You are on page 1of 371

4

5
6 A 46-year-old male sustains severe head and abdominal injuries in a motor vehicle accident. Initial
7 emergency assessment of the patient establishes his Glasgow coma scale rating of 6 out of 15, blood
8 pressure of 84/50 mm Hg, and heart rate of 1221min. The patient is successfully resuscitated and placed on
9 a respirator. Two days later, he develops tachypnea, noisy breathing, and severe hypoxemia. A chest x-ray
10 obtained at this time is shown below.
11
12
13 ,.
14
15
16
17
18
19 -
20
21
22
23
24
25
26
27
28
29
30
31
32
33
34
35
36 Which of the following most likely preceded this patient's current condition?
37
38
39
40
41 ~
2 Item: 1 of 46 II P Mark -<:I t>-
Q. Id : 6423 (388147) Previous Next
3
4
5 W hich of the follow ing most likely preceded this patient's current condition?
6
7 A. Alveolar w all destruction [40%]
8
9 B. Bronchial mucous plugging [9%]
10 C. Endothelial damage [39%]
11 D. Right ventricular infarction [4% ]
12
13 ~
x @ E. Pulmonary fibrosis [8%]
14
15
16 Explanation: User Id: 388147
17
Given this patient's history of recent trauma now complicated by diffuse symmetric pulmonary opacities and a
18
relatively normal heart size, he is most likely suffering from acute respiratory distress syndrome
19 - (ARDS). ARDS is defined as acute onset lung dysfunction, pulmonary edema, normal left atrial pressure, and
20
a Pa02/F102 of <200. A pulmonary capillary w edge pressure might be helpful in this patient to confirm that
21
the left atrial pressure is normal. Trauma itself can be a potential cause of ARDS, as can complicating
22
factors such as pulmonary contusions, infection/sepsis, and fat embolism.
23
24 ARDS is caused by injury to the endothelial cells lining the pulmonary capillaries adjacent to the alveoli. The
25 injury is primarily caused by inflammatory cytokines and neutrophils. This process is sometimes referred to
26 by its pathologic term diffuse alveolar damage (DAD). Damage to the endothelial cells allow s fluid to enter the
27 alveoli leading to respiratory compromise during the initial exudative stage. This is follow ed by a proliferative
28 stage w here the pulmonary edema resolves and there is proliferation of type II pneumocytes. In some
29 patients there is a final fibrotic phase in w hich there is diffuse pulmonary fibrosis and cyst formation .
30
31 (Choice A) Alveolar w all destruction occurs in emphysema . This finding is not associated w ith diffuse, fluffy
32 alveolar infiltrates on chest radiograph.
33
34 (Choice B) Bronchial mucous plugging can occur in a number of conditions, such as asthma . This finding is
35 not associated w ith diffuse, fluffy alveolar infiltrates on chest radiograph.
36
37 (Choice D) Isolated right ventricular infarction w ould be more expected to result in peripheral as opposed to
38 pulmonary edema.
2 Item: 1 of 46 II P Mark -<:I t>-
Q. Id : 6423 (388147) Previous Next
3
4 (AKUS). AKUS 1s det1ned as acute onset lung dystunct1on, pulmonary edema, normal lett atrial pressure, and
5 a Pa02/FI02 of <200. A pulmonary capillary w edge pressure might be helpful in this patient to confirm that
6 the left atrial pressure is normal. Trauma itself can be a potential cause of ARDS, as can complicating
7 factors such as pulmonary contusions, infection/sepsis, and fat embolism.
8
9 ARDS is caused by injury to the endothelial cells lining the pulmonary capillaries adjacent to the alveoli. The
10 injury is primarily caused by inflammatory cytokines and neutrophils. This process is sometimes referred to
11 by its pathologic term diffuse alveolar damage (DAD). Damage to the endothelial cells allow s fluid to enter the
12 alveoli leading to respiratory compromise during the initial exudative stage. This is follow ed by a proliferative
13 ~
stage w here the pulmonary edema resolves and there is proliferation of type II pneumocytes. In some
14 patients there is a final fibrotic phase in w hich there is diffuse pulmonary fibrosis and cyst formation .
15
(Choice A) Alveolar w all destruction occurs in emphysema . This finding is not associated w ith diffuse, fluffy
16
alveolar infiltrates on chest radiograph.
17
18 (Choice B) Bronchial mucous plugging can occur in a number of conditions, such as asthma . This finding is
19 - not associated w ith diffuse, fluffy alveolar infiltrates on chest radiograph .
20
21 (Choice D) Isolated right ventricular infarction w ould be more expected to result in peripheral as opposed to
22 pulmonary edema.
23
24 (Choice E) Pulmonary fibrosis can develop as a delayed complication in some patients w ho develop ARDS,
25 but 2 days follow ing a trauma the patient is still likely in the exudative phase.
26
27 Educational objective:
28 ARDS is defined as acute onset lung dysfunction, pulmonary edema, normal left atrial pressure, and a
29 Pa02/FI02 of <200. The underlying pathophysiology is damage to the endothelial cells lining the pulmonary
30 capillaries resulting in leakage of fluid into the alveoli, sometimes referred to as the exudative stage. The
31 process evolves to include a proliferative stage and sometimes a final fibrotic stage.
32
33
34 1. Acute respiratory distress syndrome.
35
36
37 Time Spent: 73 seconds 39% answ ered correctly. Last updated: [219/2012].
38 Copyright © USMLEWORLD,LLC. SimExam ver. 2011.11.397425
Item: 2 of 46 • \' Mark -<:J C>
Q. Id: 6425 [3831411 Previous NeHt
3
4
5
6 An 80-year-old male suffers from multiple medical problems. While in the hospital, he dies from chronic
7 kidney disease. The histological findings of his renal biopsy are shown on the slide below.
8
9
10
11
12
13 I"
14
15
16
17
18
19 '°
20
21
22
23
24
25
26
27
28
29 Which of the following is the most likely cause of this patient's kidney disease?
30
31
32 x @ A. Renal artery stenosis [4%]
33 .; B. Long-standing diabetes mellitus [88%]
34 C. Cystic kidney disease [2%]
35
36 D. Analgesic abuse [2%]
37 E. Systemic vasculitis [4%]
38
Item: 2 of 46 II P Mark -<:I t>-
Q. Id : 6425 (388147) Previous Next
3
~ • .._,, • • · • • -'• ,._., ._.., ~--· J ._. ~_., , _ ._.,_. L ' , ,..J
4
5 ., B. l ong-standing diabetes mellitus [88%]
6 C. Cystic kidney disease [2%]
7 D. Analgesic abuse [2%]
8
9 E. Systemic vasculitis [4%]
10
11
12 Explanation: User Id: 388147
13 ~
Diabetic nephropathy is a know n complication of both type 1 and type 2 diabetes mellitus, typically appearing
14
at least ten to fifteen years after disease onset. Histopathologically, diabetic nephropathy is characterized by
15
mesangial expansion, glomerular basement membrane thickening, and glomerular sclerosis. When the
16
glomerular sclerosis is nodular in nature (as in the slide above), it is described as a Kimmelstiel-Wilson
17
lesion. The Kimmelstiel-Wilson lesion consists of dense hyaline deposits, and more commonly arises in the
18
glomerular arterioles.
19 -
20 (Choice A) Renal artery stenosis is typically characterized by medial fibroplasia and absent internal elastic
21 lamina in areas of aneurysmal dilatation.
22
23 (Choice C) Simple cysts are fluid-filled sacs lined by a single epithelial layer. No renal structures or tissue
24 are present w ithin these cysts.
25
26 (Choice D) Analgesic abuse can result in a nephropathy characterized by marked thickening of the vasa
27 recta capillaries and intermittent tubular necrosis. Eventually, papillary necrosis, focal and segmental
28 glomerulosclerosis, and interstitial infiltration and fibrosis can occur.
29
30 (Choice E) Histologic findings associated w ith systemic vasculitis range from acute inflammation to
31 multinucleated giant cells to evidence of healing and repair.
32
33 Educational objective:
34 Diabetic nephropathy is characterized by mesangial expansion, glomerular basement membrane thickening,
35 and glomerular sclerosis. If the glomerular sclerosis is nodular, it is described as a Kimmelstiel-Wilson lesion .
36
37 Time Spent: 53 seconds 88% answ ered correctly. l ast updated: [1 /22/2012].
38 Copyright © USMLEWORLD,LLC. SimExam ver. 2011 .11.397425
Item: 3 of 46 II P Mark -<:I t>-
Q. Id : 6426 (388147) Previous Next

4
5
6 Action potential propagation velocity is measured at four different points w ithin the cardiac muscle. The
7 follow ing results are obtained:
8
9 Point 1 - 1.10 m/sec
10 Point 2 - 0.05 m/sec
11 Point 3 - 2.20 m/sec
12 Point 4 - 0.30 m/sec
13 ~
14
W hich of the follow ing locations w ithin the cardiac muscle most likely corresponds to point 2?
15
16
17 X @ A. Left atrium near the openings of the pulmonary veins [9%]
18 ~ B. lnteratrial septun1 near the right AV orifice [53%]
19 - C. Upper part of the interventricular septum [22%]
20
21 D. Subendocardium of the left ventricle [ 11 %]
22 E . Endocardium of the right ventricle [6%]
23
24
25 Explanation: User Id: 388147
26
27 Normally, cardiac impulses originate in the sinoatrial (SA) node. SA node depolarization delivers an electrical
28 impulse to the surrounding atrial m yocardium, w hich carries the action potential to the AV node at a rate of 0.3
29 m/sec. The action potential is delayed at the AV node in order to allow the ventricles to completely fill w ith
30 blood during diastole. The action potential takes nearly 0.13 seconds to travel through the AV node; w ithin the
31 AV node the speed of conduction is only 0.05 m/sec. From the AV node, the action potential enters the
32 His-Purkinje system. Impulses travel rapidly through the Purkinje fibers. Rapid transmission of electrical
33 impulses during this phase of the cardiac cycle is essential to ensure contraction of the ventricles in a
34 bottom-up fashion. Contraction of the ventricles from the apex of the heart tow ard the base is necessary to
35 propel blood upw ard toward the pulmonary artery and aorta . Impulses travel through the His-Purkinje system
36 at a rate of 2.2 m /sec. From the Purkinje fibers, the action potential is transmitted to the ventricular
37 m yocardium w here it travels at a rate of 0.3 m/sec. In summary, the slow est conduction velocity is in the AV
38 node and the fastest conduction occurs in the Purkinje system. The conductivity of the atrial muscle is higher
Item: 3 of 46 II P Mark -<:I t>-
Q. Id : 6426 (388147) Previous Next

4 Point 4 - 0.30 m/sec


5
6 W hich of the follow ing locations w ithin the cardiac muscle most likely corresponds to point 2?
7
8
9 x @ A. Left atrium near the openings of the pulmonary veins [9%]
10 ., B. lnteratrial septum near the right AV orifice [53%]
11 C. Upper part of the interventricular septum [22%]
12
13 ~
D. Subendocardium of the left ventricle [11 %]
14 E. Endocardium of the right ventricle [6%]
15
16
17 Explanation: User Id: 388147
18
19 - Normally, cardiac impulses originate in the sinoatrial (SA) node. SA node depolarization delivers an electrical
20 impulse to the surrounding atrial m yocardium, w hich carries the action potential to the AV node at a rate of 0.3
21 m/sec. The action potential is delayed at the AV node in order to allow the ventricles to completely fill w ith
22 blood during diastole. The action potential takes nearly 0.13 seconds to travel through the AV node; w ithin the
23 AV node the speed of conduction is only 0.05 m/sec. From the AV node, the action potential enters the
24 His-Purkinje system. Impulses travel rapidly through the Purkinje fibers. Rapid transmission of electrical
25 impulses during this phase of the cardiac cycle is essential to ensure contraction of the ventricles in a
26 bottom-up fashion. Contraction of the ventricles from the apex of the heart tow ard the base is necessary to
27 propel blood upw ard tow ard the pulmonary artery and aorta . Impulses travel through the His-Purkinje system
28 at a rate of 2.2 m/sec. From the Purkinje fibers, the action potential is transmitted to the ventricular
29 m yocardium w here it travels at a rate of 0.3 m/sec. In summary, the slow est conduction velocity is in the AV
30 node and the fastest conduction occurs in the Purkinje system. The conductivity of the atrial muscle is higher
31 than that of the ventricular muscle.
32
33 Educational objective:
34 The cardiac action potential conduction velocity is slow est in the AV node and fastest in the Purkinje system.
35 The conductivity of the atrial muscle is higher than that of the ventricular muscle.
36
37 Time Spent: 76 seconds 53% answ ered correctly. Last updated: [1 /8/2012].
38 Copyright © USMLEWORLD,LLC. SimExam ver. 2011.11.397425
1 A

2 Item: 4 of 46 II P Mark -<:I t>-


Q. Id : 6428 (388147) Previous Next
3

5
6 A 62-year-old Caucasian male w ith Paget's disease of the bone has been treated w ith risedronate. He suffers
7 from mild bilateral hearing loss. He has been taking low -dose prednisone for asthma for the last several
8 years. His current complaints include persistent right hip pain that is unresponsive to over-the-counter
9 analgesics. A radiograph of the right hip demonstrates an area of mixed lysis and sclerosis w ith focal
10 periosteal elevation and reactive new bone formation . W hich of the follow ing is the most likely cause of this
11 patient's current complaints?
12
13 ~
14 A. Avascular necrosis [17%]
15 B. Osteoarthritis [10%]
16 C. Renal osteodystrophy [7%]
17
18 ., D. Osteosarcoma [50%]
19 - x @ E. Osteopetrosis [17%]
20
21
22 Explanation: User Id: 388147
23
24 Osteosarcoma is the bone tumor most often associated w ith Paget's disease, arising in the femur, tibia,
25 humerus, or other bones. Radiographic findings of osteosarcoma include destruction of the normal trabecular
26 bone pattern, mixed radiodense and radiolucent areas, periosteal new bone formation, lifting of the cortex, and
27 Codman's triangle. Adjacent soft tissue usually demonstrates ossification in a "sunburst" pattern.
28
(Choice A) Avascular necrosis can be caused by glucocorticoid use (typically high dose}. Radiographic
29
findings include the pathognomonic crescent sign, w hich indicates subchondral collapse.
30
31
(Choice B) Osteoarthritis is common in the elderly, but typically improves w ith over-the-counter analgesics.
32
Radiographic findings include osteophytes and joint space narrowing.
33
34
(Choice C) Renal osteodystrophy encompasses four bony diseases associated w ith advanced chronic renal
35
36 failure: hyperparathyroid bone disease, osteomalacia, mixed uremic osteodystrophy, and aplastic bone.
Radiographic findings are not particularly sensitive for this condition (bone biopsy is preferred}.
37
38
1 A

2 Item: 4 of 46 II P Mark -<:I t>-


Q. Id : 6428 (388147) Previous Next
3
fJi:::IU~r ll :s CUI I ~r ll cur r lfJICIU ll:S (

5
6 A. Avascular necrosis [1 7%]
7 B. Osteoarthritis [10%]
8
9 C. Renal osteodystrophy [7%]
10 ., D. Osteosarcon1a [50%]
11 x @ E. Osteopetrosis [17%]
12
13 ~
14 Explanation: User Id: 388147
15
16 Osteosarcoma is the bone tumor most often associated w ith Paget's disease, arising in the femur, tibia,
17 humerus, or other bones. Radiographic findings of osteosarcoma include destruction of the normal trabecular
18 bone pattern, mixed radiodense and radiolucent areas, periosteal new bone formation, lifting of the cortex, and
19 - Codman's triangle. Adjacent soft tissue usually demonstrates ossification in a "sunburst" pattern.
20
21 (Choice A) Avascular necrosis can be caused by glucocorticoid use (typically high dose}. Radiographic
22 findings include the pathognomonic crescent sign, w hich indicates subchondral collapse.
23
24 (Choice B) Osteoarthritis is common in the elderly, but typically improves w ith over-the-counter analgesics.
25 Radiographic findings include osteophytes and joint space narrow ing.
26
27 (Choice C) Renal osteodystrophy encompasses four bony diseases associated w ith advanced chronic renal
28 failure: hyperparathyroid bone disease, osteomalacia, mixed uremic osteodystrophy, and aplastic bone.
29 Radiographic findings are not particularly sensitive for this condition (bone biopsy is preferred}.
30
31 (Choice E) Osteopetrosis occurs w hen osteoclasts fail to resorb bone, thereby impairing bone modeling and
32 remodeling. Radiographic findings include generalized osteosclerosis and evidence of fractures.
33
34 Educational objective:
35 Paget's disease is associated w ith the development of osteosarcoma .
36
37 Time Spent: 63 seconds 50% answ ered correctly. Last updated: [1 /8/2012].
38 Copyright © USMLEWORLD,LLC. SimExam ver. 2011.11.397425
1 A

2 Item: 5 of 46 II P Mark -<:I t>-


Q. Id : 6433 (388147) Previous Next
3
4

6 A 32-year-old female complains of a lump in her throat. Her past medical history is significant for migraine
7 headaches. Neck examination reveals no abnormalities and a barium S\"lall w is unrevealing . W hich of the
8 following is the most likely diagnosis?
9
10
11 A. Esophageal w eb [12%]
12 B. Zenker diverticulum [8%]
13 ~ X @ C. Retrosternal goiter [25%]
14
15 D. Pseudobulbar paresis [20%]
16 E. Globus hystericus [35%]
17
18
19 - Explanation: User Id: 388147
20
21 Globus hysteric us (also called globus sensation, or "lump in the throat"} is a common occurrence in up to
22 45% of the general population . Most individuals experience the globus sensation intermittently, w ith the
23 remainder describing it as persistent. Emotions commonly trigger globus hystericus. Fear, tension, anger,
24 and mental anguish are frequently reported as exacerbating factors. Multiple organic and functional disorders
25 are also associated w ith globus hysteric us, including gastroesophageal reflux disease, upper esophageal
26 sphincter abnormalities, achalasia, psychiatric disorders, and stress.
27
(Choices A, 8, C, and D) Esophageal w ebs are fragile mucosal folds that cause solid food dysphagia .
28
Zenker's diverticula are mucosal outpouchings caused by motor abnormalities of the esophagus.
29
Retrosternal goiter occurs w hen one or both thyroid lobes grow into the thoracic cavity inlet. Pseudobulbar
30
paresis is caused by conditions such as multiple sclerosis and is characterized by dysarthria, dysphagia,
31
dysphonia, and impaired movement of the tongue and facial muscles.
32
33 All of these conditions should be identifiable on physical examination, endoscopy, or barium sw allow . As a
34
result, they are not likely causes of this patient's complaints.
35
36 Educational objective:
37 Globus hystericus is the sensation of a "lump in the throat" w ithout accompanying physical, endoscopic, or
38 radioloaic findinas of esoohaaeal obstruction .
39
40
1 A

2 Item: 5 of 46 II P Mark -<:I t>-


Q. Id : 6433 (388147) Previous Next
3
4 r l~i:::IUCICr l~:S . l \l~CK. ~Xi:::IJ I IU ICIUUI I r ~V~i:::ll:S f IU i:::IUI IUI r I IC::lllll~:S i:::ll IU i:::I o ar 1ur r I :S\'Vi:::lllQW. l:S Ulll~V ~i:::llU 1y. vvr llCI I UI ll I~
follow ing is the most likely diagnosis?
6
7 A. Esophageal w eb [12%]
8 B. Zenker diverticulum [8%]
9
10 X @ C. Retrosternal gaiter [25%]
11 D. Pseudobulbar paresis [20%]
12
E. Globus hystericus [35%]
13 ~
14
15 Explanation: User Id: 388147
16
17 Globus hysteric us (also called globus sensation, or "lump in the throat"} is a common occurrence in up to
18 45% of the general population . Most individuals experience the globus sensation intermittently, w ith the
19 - remainder describing it as persistent. Emotions commonly trigger globus hystericus. Fear, tension, anger,
20 and mental anguish are frequently reported as exacerbating factors. Multiple organic and functional disorders
21 are also associated w ith globus hysteric us, including gastroesophageal reflux disease, upper esophageal
22 sphincter abnormalities, achalasia, psychiatric disorders, and stress.
23
24 (Choices A, 8, C, and D) Esophageal w ebs are fragile mucosal folds that cause solid food dysphagia .
25 Zenker's diverticula are mucosal outpouchings caused by motor abnormalities of the esophagus.
26 Retrosternal gaiter occurs w hen one or both thyroid lobes grow into the thoracic cavity inlet. Pseudobulbar
27 paresis is caused by conditions such as multiple sclerosis and is characterized by dysarthria, dysphagia,
28 dysphonia, and impaired movement of the tongue and facial muscles.
29
30 All of these conditions should be identifiable on physical examination, endoscopy, or barium sw allow. As a
31 result, they are not likely causes of this patient's complaints.
32
33 Educational objective:
34 Globus hystericus is the sensation of a "lump in the throat" w ithout accompanying physical, endoscopic, or
35 radiologic findings of esophageal obstruction .
36
37 Time Spent: 34 seconds 35% answ ered correctly. Last updated: [1 /8/2012].
38 Copyright © USMLEWORLD,LLC. SimExam ver. 2011.11.397425
1 •
2 Item: 6 of 46 • \ ' Mark -<:J C>
Q. Id: 6433 (383141) Previous NeHt
3
4
5
A 54-year-old male complains of right knee swelling and pain. Microscopic findings of synovial fluid aspiration
7 are shown on the image below.
8
9
10
11
12
13 I"
14
15
16
17
18
19 '°
20
21
22
23
24
25
26
27
28 Which of the following is the most likely cause of this patient's symptoms?
29
30 X @ A. Cartilage wear-and-tear inju1y [1%]
31
B. Immune complex deposition [4%]
32
33 C. Crystal arthropathy [93%]
34 D. Joint infection [2%]
35
E. Traumatic joint injury [0%]
36
37
38
1 A

2 Item: 6 of 46 II P Mark -<:I t>-


Q. Id : 6438 (388147) Previous Next
3
4
5
Explanation: User Id: 388147
7
8 Gout is a common disorder of uric acid metabolism characterized by recurrent inflammatory arthritis, tissue
9 deposition of monosodium urate crystals, and uric acid nephrolithiasis. Joint destruction can occur if the
10 disease is left untreated.
11
On polarizing light microscopy, intracellular monosodium urate crystals are present in the synovial fluid.
12
These crystals are slender, needle-shaped, and negatively birefringent. Normally, the urate crystals have a
13 ~
protective coating of apolipoprotein E or apolipoprotein B. W hen uric acid levels fluctuate or microtrauma
14
occurs, bare urate crystals are shed and exposed to lgG antibodies. The subsequent antibody binding
15
stimulates neutrophil phagocytosis of crystals, resulting in an inflammatory cascade that is responsible for the
16
signs and symptoms of gout.
17
18 (Choice A) Characteristic features of an osteoarthritic joint include cartilage w ear-and-tear, bony eburnation,
19 - dislodging of cartilage and bone pieces ("joint mice"), fibrous-w alled cysts w ithin the subchondral bone, and
20 bony outgrowths at the articular margins. Crystals are not typically present in the joint fluid.
21
22 (Choice B) Characteristic features of a joint affected by rheumatoid arthritis include immune complex
23 deposition, neutrophil accumulation in the synovial fluid, perivascular inflammation of the synovial stroma,
24 osteoclastic activity, and pannus formation. Crystals are not typically present in the joint fluid.
25
26 (Choice D) Characteristic features of a joint infection include the presence of bacteria and neutrophils (as in
27 acute suppurative arthritis), m ycobacteria and caseating granulomas (as in tuberculous arthritis), or
28 spirochetes (as in Lyme disease). Crystals are not typically present in the joint fluid.
29
30 (Choice E) The characteristic feature of traumatic joint injury is a joint effusion that may be serosanguineous
31 or frankly bloody. Crystals are not typically present in the joint fluid.
32
33 Educational objective:
34 Gout is characterized by intracellular monosodium urate crystals that are slender, needle-shaped, and
35 negatively birefringent.
36
37 Time Spent: 64 seconds 93% answ ered correctly. Last updated: [1 /8/2012].
38 Copyright © USMLEWORLD,LLC. SimExam ver. 2011 .11.397425
1 •
2 Item: 7 of 46 • \ ' Mark -<:J C>
Q. Id: 6441 (3831411 Previous NeHt
3
4
5
6 A 23-year-old Caucasian female complains of abdominal pain that precedes her menses. She describes the
pain as moderate to severe. She takes over-the-counter ibuprofen, which provides some pain relief.
8 Laparoscopy demonstrates nodules on the fallopian tubes. Nodule biopsy reveals the histologic findings
9 shown below.
10
11
12
13 I"
14
15
16
17
18
19 '°
20
21
22
23
24
25
26
27
28
29
30 Which of the following is the most likely cause of this patient's symptoms?
31
32 A. Adenomyosis [9o/o]
33
34
B. Cervical cancer [0%]
35 II @ C. Endometnos1s [76%]
36 D. Leiomyoma (8%]
37
E. Pyosalpinx (7%]
1 A

2 Item: 7 of 46 II P Mark -<:I t>-


Q. Id : 6441 [388147) Previous Next
3
••. • .... _.,,_.,,,} .......... .... l"' . ... J
4
5 B. Cervical cancer [0%]
6 ., @ C. Endometriosis [76%]
D. Leiomyoma [8%]
8
9 E. Pyosalpinx [7%]
10
11
12 Explanation: User Id: 388147
13 ~
Endometriosis is a common, progressive condition in reproductive-aged women that is characterized by the
14
presence of endometrial glands and stroma outside the uterus. The three proposed theories of endometriosis
15
pathogenesis include retrograde menstruation, lymphatic and vascular spread, and coelomic metaplasia. The
16
most common sites of endometriosis (in decreasing order of frequency) are the ovaries, cul-de-sac, posterior
17
broad ligaments, uterosacral ligaments, uterus, fallopian tubes, sigmoid colon, appendix, and round
18
ligaments. Significant pelvic pain, dysmenorrhea, infertility, and deep dyspareunia are frequent consequences
19 - of the disease.
20
21 (Choice A) Adenomyosis is characterized by the presence of endometrial tissue and stroma w ithin the
22 uterine myometrium. The image provided above is of the fallopian tubes, not the uterine myometrium.
23
24 (Choice B) The most common form of cervical cancer is squamous cell carcinoma, in w hich malignant
25 keratinocytes invade through the basement membrane. The image provided above has no evidence of
26 malignancy.
27
28 (Choice D) Leiomyoma is characterized by a w horled pattern of smooth muscle bundles w ith intervening
29 connective tissue. The image provided above demonstrates no such pattern.
30
31 (Choice E) Common in pelvic inflammatory disease, pyosalpinx is the distention of the fallopian tube w ith
32 purulent discharge. The image provided above has no evidence of inflammation or bacterial infection.
33
34 Educational Objective:
35 Endometriosis is characterized by the presence of endometrial glands and stroma outside the uterus.
36
37 Time Spent: 43 seconds 76% answ ered correctly. Last updated: [3/27/201 O].
38 Copyright© USMLEWORLD,LLC. SimExam ver. 2011.11.397425
1 A

2 Item: 8 of 46 II P Mark -<:I t>-


Q. Id : 6443 (388147) Previous Next
3
4
5
6 A 23-year-old female presents to clinic w ith mild abdominal discomfort and nausea. Her diet consists mainly
7 of tea and toast, but she often takes vitamin supplements. Her urine is positive for 13-hCG. W hich of the
following supplements should be avoided in this patient?
9
10
11 A. Vitamin D [3%]
12 X @ B. Vitamin C [3%]
13 ~ ~ C. Vitamin A [77%]
14
15 D. Beta-carotene [13%]
16 E. Folic acid [4%]
17
18
19 - Explanation: User Id: 388147
20
21 Preformed vitamin A (retinol) is found in dietary sources such as liver, kidney, egg yolk, and butter. Because
22 this vitamin is lipid-soluble, it is not readily excreted in urine; instead, it accumulates w ithin the stellate cells of
23 the liver. Individuals at greatest risk of experiencing vitamin A toxicity are those w ho ingest large quantities of
24 the vitamin in its synthetic form. Vitamin A supplements typically contain 10,000-50,000 IU per capsule.
25
Excessive vitamin A is highly teratogenic (FDA pregnancy category X), particularly in the first trimester of
26
27 pregnancy. Microcephaly, cardiac anomalies, early epiphyseal closure, grow1h retardation, and spontaneous
abortion can result from maternal ingestion of high doses of retinoids.
28
29 (Choices A, B, and D) Vitamin C, vitamin D, and beta-carotene are classified as FDA pregnancy risk
30 category C. Prenatal supplementation w ith standard doses of these vitamins appears to be safe.
31
32 (Choice E) Folic acid is classified as FDA pregnancy risk category A. Adequate maternal folic acid intake is
33 essential to the fetus and has been show n to prevent neural tube defects.
34
35 Educational Objective:
36 Maternal ingestion of high doses of vitamin A can be highly teratogenic, resulting in fetal microcephaly, cardiac
37 anomalies, early epiphyseal closure, grow1h retardation, or spontaneous abortion.
38
1 A

2 Item: 8 of 46 II P Mark -<:I t>-


Q. Id : 6443 (388147) Previous Next
3
4 A 23-year-old temale presents to clinic w ith mild abdominal d1scomtort and nausea. Her diet consists mainly
5 of tea and toast, but she often takes vitamin supplements. Her urine is positive for 13-hCG. W hich of the
6 following supplements should be avoided in this patient?
7
A. Vitamin D [3%]
9
10
x @ B. Vitamin C [3%]
11 .; C. Vitamin A [77%]
12 D. Beta-carotene [13%]
13 ~
14 E. Folic acid [4%]
15
16 Explanation: User Id: 388147
17
18 Preformed vitamin A (retinal) is found in dietary sources such as liver, kidney, egg yolk, and butter. Because
19 - this vitamin is lipid-soluble, it is not readily excreted in urine; instead, it accumulates w ithin the stellate cells of
20 the liver. Individuals at greatest risk of experiencing vitamin A toxicity are those w ho ingest large quantities of
21 the vitamin in its synthetic form. Vitamin A supplements typically contain 10,000-50,000 IU per capsule.
22
23 Excessive vitamin A is highly teratogenic (FDA pregnancy category X), particularly in the first trimester of
24 pregnancy. Microcephaly, cardiac anomalies, early epiphyseal closure, growth retardation, and spontaneous
25 abortion can result from maternal ingestion of high doses of retinoids.
26
27 (Choices A, B, and D) Vitamin C, vitamin D, and beta-carotene are classified as FDA pregnancy risk
28 category C. Prenatal supplementation w ith standard doses of these vitamins appears to be safe.
29
30 (Choice E) Folic acid is classified as FDA pregnancy risk category A. Adequate maternal folic acid intake is
31 essential to the fetus and has been show n to prevent neural tube defects.
32
33 Educational Objective:
34 Maternal ingestion of high doses of vitamin A can be highly teratogenic, resulting in fetal microcephaly, cardiac
35 anomalies, early epiphyseal closure, growth retardation, or spontaneous abortion.
36
37 Time Spent: 40 seconds 77% answ ered correctly. Last updated: [3/27/201 O].
38 Copyright © USMLEWORLD,LLC. SimExam ver. 2011.11.397425
1 A

2 Item: 9 of 46 II P Mark -<:I t>-


Q. Id : 6456 (388147) Previous Next
3
4
5
6 A young woman w ith a two day history of cough and malaise receives a prescription for oseltamivir. This
7 medication is an appropriate treatment for w hich of the following conditions?
8

10 A. Common cold [2%]


11 B. RSV infection [7%]
12 ., C. Influenza [84%]
13 ~
14 x @ D. Mycop/asma infection [1 %]
15 E. HIV infection [6%]
16
17
18 Explanation: User Id: 388147
19 -
20 Oseltamivir (brand name Tamiflu) is a sialic acid analogue that inhibits the neuraminidases of the influenza A
21 and influenza B viruses, w hich attach to cells and respiratory tract mucins through hemagglutinin binding of
22 glycoconjugate receptors. Neuraminidase cleaves the terminal sialic acid residues on these receptors,
23 allow ing for the release of attached virions from infected cells and subsequent viral spread w ithin the
24 respiratory tract. Neuraminidase inhibitors force the new ly synthesized virions to remain adherent to the host
25 cell surface, ultimately forming viral aggregates that are unable to infect new host cells. Through this
26 mechanism of action, oseltamivir can prevent or shorten the course of influenza A and B infections w hen
27 taken w ithin 48 hours of symptom onset.
28
29 (Choice A) The common cold is typically caused by rhinoviruses, w hich are non-enveloped RNA viruses. No
30 specific antiviral drug is commonly used against rhinoviruses.
31
32 (Choice B) Respiratory syncytial virus (RSV) is the most common cause of bronchiolitis in children under the
33 age of two years. Treatment of RSV bronchiolitis includes administration of humidified supplemental oxygen
34 and, in some cases, aerosolized ribavirin.
35
36 (Choice D) Mycoplasma pneumoniae causes atypical (interstitial} community acquired pneumonia, w hich
37 manifests w ith persistent fever, malaise, cough, and sore throat. Treatment is w ith antibiotics such as
38 tetracycline or erythromycin instead of penicillins because m ycoplasmas lack cell w alls.
1 A

2 Item: 9 of 46 II P Mark -<:I t>-


Q. Id : 6456 (388147) Previous Next
3
" ~ LI. 'vry L.VfJI0.;, 111 0 1111c1,,.u u 11 l1 FUJ
4
5 E. HIV infection [6%]
6
7
8 Explanation: User Id: 388147

10 Oseltamivir (brand name Tamiflu) is a sialic acid analogue that inhibits the neuraminidases of the influenza A
11 and influenza B viruses, w hich attach to cells and respiratory tract mucins through hemagglutinin binding of
12 glycoconjugate receptors. Neuraminidase cleaves the terminal sialic acid residues on these receptors,
13 ~
allow ing for the release of attached virions from infected cells and subsequent viral spread w ithin the
14 respiratory tract. Neuraminidase inhibitors force the new ly synthesized virions to remain adherent to the host
15 cell surface, ultimately forming viral aggregates that are unable to infect new host cells. Through this
16 mechanism of action, oseltamivir can prevent or shorten the course of influenza A and B infections w hen
17 taken w ithin 48 hours of symptom onset.
18
19 - (Choice A) The common cold is typically caused by rhinoviruses, w hich are non-enveloped RNA viruses. No
specific antiviral drug is commonly used against rhinoviruses.
20
21
22 (Choice B) Respiratory syncytial virus (RSV) is the most common cause of bronchiolitis in children under the
23 age of two years. Treatment of RSV bronchiolitis includes administration of humidified supplemental oxygen
24 and, in some cases, aerosolized ribavirin.
25
26 (Choice D) Mycoplasma pneumoniae causes atypical (interstitial) community acquired pneumonia, w hich
27 manifests w ith persistent fever, malaise, cough, and sore throat. Treatment is w ith antibiotics such as
28 tetracycline or erythromycin instead of penicillins because mycoplasmas lack cell w alls.
29
30 (Choice E) The drugs of choice for HIV infection would be nucleoside reverse transcriptase inhibitors,
31 non-nucleoside reverse transcriptase inhibitors, and retroviral protease inhibitors.
32
33 Educational objective:
34 Oseltamivir is an inhibitor of the neuraminidases of influenza A and influenza B viruses. Oseltamivir is used in
35 the prevention of and early treatment of influenza infection.
36
37 Time Spent: 37 seconds 84% answ ered correctly. Last updated: [3/27/201 O].
38 Copyright © USMLEWORLD,LLC. SimExam ver. 2011.11.397425
1 A

2 Item: 10 of 46 II P M ark -<:I t>-


Q. Id : 6460 (388147] Previous Next
3
4
5
6 A radiolabeled amino acid is added to a cell line that is actively producing protein. Three samples of cell
7 protein extracts are obtained at 10, 15 and 25 minutes after the radiolabeled amino acid is added, and
8 subsequently subjected to gel electrophoresis on an agarose gel (see slide below).
9
85 KO
11
12
13 ~
14 45 KO
15
16
17 35 KO
18
19 -
20 10 min 15 min 25 min
21
22
23 W hich of the follow ing statements best represents the experiment results?
24
25 A. The cell line is mainly producing 45 KO protein, w ith 35 KO protein as a byproduct [4%]
26 B. The cell line is mainly producing 85 KO protein that is rapidly degraded by proteases [18%]
27
28 x @ C. The cell line is making all three proteins but the 85 KD protein has the shortest half-life [18%]
29 .; 0. The cell line is making a precursor protein which is subsequently modified via cleavage [59%]
30 E. the cell line is making two proteins w hich are subsequently joined by disulifide bonds [2%]
31
32
33 Explanation: User Id: 388147
34
35 This question describes a protein that undergoes posttranslational cleavage. This scenario most commonly
36 occurs in proteins destined for secretion from the cell. Such proteins are synthesized as a large polypeptide
37 containing the active protein sequence in addition to signaling sequences and protective sequences.
38 Signaling polypeptide sequences facilitate sorting of the protein to its proper destination, such as into a
39
40
1 A

2 Item: 10 of 46 II P M ark -<:I t>-


Q. Id : 6460 (388147] Previous Next
3
4 txp1anat1on: user 1a: 3tsts1 4f
5
6 This question describes a protein that undergoes posttranslational cleavage. This scenario most commonly
7 occurs in proteins destined for secretion from the cell. Such proteins are synthesized as a large polypeptide
8 containing the active protein sequence in addition to signaling sequences and protective sequences.
9 Signaling polypeptide sequences facilitate sorting of the protein to its proper destination, such as into a
secretary vesicle or into the extracellular space. Protective polypeptide sequences prevent premature
11 degradation of the protein before it has reached its target site. Removal of nonfunctional polypeptide
12 sequences once the protein has reached its target site is accomplished by specific endoproteases.
13 ~
Examples of proteins that undergo posttranslational cleavage include:
14 1. Collagen: N-terminal and C-terminal propeptides are cleaved by procollagen peptidases in the
15 extracellular space.
16 2. Trypsinogen: Enterokinase in the duodenal brush border cleaves this pancreatic protease, activating it
17 to trypsin. Trypsin then cleaves and activates other pancreatic proenzymes such as proelastase and
18 procarboxypeptidase.
19 -
20
21 (Choice A) The cell line is producing equal amounts of the 45 KO and 35 KO proteins.
22
23 (Choice B) This is not the best answ er choice because the 85 KO propeptide is not degraded, but rather
24 cleaved into two distinct proteins . In contrast to the endopeptidase at work here, degradative proteases
25 typically act on a protein after it has been signaled for destruction by ubiquitin.
26
27 (Choice C) This choice is not accurate because the 45 KO and 35 KO proteins are not produced at the 10
28 minute time point w hile the 85 KO protein is.
29
30 (Choice E) This choice is not consistent w ith the temporal relationship captured on the electrophoresis gel.
31 Production of the 85 KO protein precedes formation of the 45 KO and 35 KO proteins.
32
33 Educational objective:
34 Proteins destined for export from the cell frequently undergo posttranslational cleavage of protective and
35 signaling sequences either w ithin or outside the cell of origin.
36
37 Time Spent: 84 seconds 59% answ ered correctly. Last updated: [1 /22/2012].
38 Copyrighl © USMLEWORLD,LLC. SimExam ver. 2011 .11.397425
1 A

2 Item: 11 of 46 II P Mark -<:I t>-


Q. Id : 6464 (388147] Previous Next
3
4
5
6 An investigator grow s a cancer cell line after adding a specific growth factor. Processing and agarose gel
7 electrophoresis of the cellular DNA reveals a distinct pattern of discrete bands. The investigator then grow s
8 the same cell line in the absence of the added growth factor. Subsequent DNA electrophoresis reveals
9 fragmentation in multiples of 180 bases. W hich of the follow ing best explains the latter finding?
10

12 A. Transformation [9%]
13 ~
B. Clonal expansion [26%]
14
15 C. Contact inhibition [11 %]
16
17 x @ D. Anaplasia [3%]
18
19 - ~ E. Apoptosis [52%]
20
21
22
23 Explanation: User Id: 388147
24
25 The phenomenon described - the appearance of DNA fragments in multiples of 180 base pairs on gel
26 electrophoresis - is know n as DNA laddering. This finding is a sensitive indicator of apoptosis (programmed
27 cell death}. Laddering occurs due to the action of specific endonucleases during karyorrhexis. These
28 endonucleases cleave DNA at internucleosomal linker regions, w hich occur at 180 base pair intervals in the
29 genome.
30
31 (Choice A) Transformation refers to any process by w hich a cell loses its ability to regulate its growth, thereby
32 becoming malignant. Transformation primarily occurs due to overexpression of oncogenes or loss of function
33 of tumor suppressor genes.
34
35 (Choice B) Clonal expansion can be a malignant or nonmalignant process. Classically this term is used to
36 refer to the clonal expansion of antigen-specific lymphocytes in response to infection. Clonal cells have
37 identical DNA.
38
1 A

2 Item: 11 of 46 II P Mark -<:I t>-


Q. Id : 6464 (388147] Previous Next
3
4 x @ D. Anaplasia [3%]
5
6 .; E. Apoptosis [52%]
7
8
9
10 Explanation: User Id: 388147

12 The phenomenon described - the appearance of DNA fragments in multiples of 180 base pairs on gel
13 ~
electrophoresis - is know n as DNA laddering. This finding is a sensitive indicator of apoptosis (programmed
14 cell death). Laddering occurs due to the action of specific endonucleases during karyorrhexis. These
15 endonucleases cleave DNA at internucleosomal linker regions, w hich occur at 180 base pair intervals in the
16 genome.
17
18 (Choice A) Transformation refers to any process by w hich a cell loses its ability to regulate its growth, thereby
19 - becoming malignant. Transformation primarily occurs due to overexpression of oncogenes or loss of function
of turner suppressor genes.
20
21
22 (Choice B) Clonal expansion can be a malignant or nonmalignant process. Classically this term is used to
23 refer to the clonal expansion of antigen-specific lymphocytes in response to infection. Clonal cells have
24 identical DNA.
25
26 (Choice C) Contact inhibition refers to the phenomenon w hereby cell division halts in G1 in response to
27 contact on all sides by other cells. Malignant cells do not exhibit contact inhibition.
28
29 (Choice D) Anaplasia is a histologic finding associated w ith malignant cells. Anaplastic cells exhibit poor
30 differentiation of specialized cellular structures.
31
32 Educational objective:
33 DNA laddering is a laboratory phenomenon used to identify apoptotic cells and distinguish them from necrotic
34 cells. During karyorrhexis, endonucleases degrade DNA into fragments that are multiples of 180 base pairs;
35 these fragments are equally spaced on gel electrophoresis giving the appearance of a ladder.
36
37 Time Spent: 92 seconds 52% answ ered correctly. Last updated: [3/27/201O].
38 Copyright © USMLEWORLD,LLC. SimEx am ver. 2011 .11.397425
1 A

2 Item: 12 of 46 II P Mark -<:I t>-


Q. Id : 6465 (388147] Previous Next
3
4
5
6 An investigator studies the effect of various drugs on isolated smooth muscle cells. The experimental findings
7 are show n in the graph below.
8
9
10
11
c:
13
14
~ -.c
.Q
( .)
<O

15 c:
0
16 ()
Drug X + Drug X +
17 Drug X
Physostigmine Drug Y
18
19 - Drug Y is most likely to be w hich of the follow ing?
20
21
22 A. Epinephrine [2%]
23 B. Acetylcholine [8%]
24
25 C. Pralidoxime [9%]
26 D. Phentolamine [10%]
27 .; @ E. Atropine [70%]
28
29
30 Explanation: User Id: 388147
31
32 The contractile effect of Drug X on smooth muscle cells is potentiated by physostigmine, w hich is a
33 cholinesterase inhibitor. Therefore, Drug X must be acetylcholine or a similar cholinergic compound that can
34 be inactivated by acetylcholinesterase. The graph also indicates that Drug Y inhibits the contractile action of
35 Drug X. Of all the drugs listed, only atropine is a competitive inhibitor of acetylcholine-like agents, acting at the
36 muscarinic receptors of smooth muscle cells.
37
38
39
40
1 A

2 Item: 12 of 46 II P Mark -<:I t>-


Q. Id : 6465 (388147] Previous Next
3
4
5
6 Explanation: User Id: 388147
7
8 The contractile effect of Drug X on smooth muscle cells is potentiated by physostigmine, w hich is a
9 cholinesterase inhibitor. Therefore, Drug X must be acetylcholine or a similar cholinergic compound that can
10 be inactivated by acetylcholinesterase. The graph also indicates that Drug Y inhibits the contractile action of
11 Drug X. Of all the drugs listed, only atropine is a competitive inhibitor of acetylcholine-like agents, acting at the
muscarinic receptors of smooth muscle cells.
13 ~
14 (Choice A) Epinephrine can oppose, potentiate, or have little effect on smooth muscle contractile response to
15 muscarinic stimulation (depending on the smooth muscle tissue of origin, the smooth muscle expression of
16 alpha- and beta-adrenergic receptors, and the concentration of epinephrine). For example, if this experiment
17 w ere conducted on bronchial smooth muscle, epinephrine w ould inhibit an acetylcholine-like agent. How ever,
18 if the experiment w ere conducted w ith concentrated epinephrine on vascular smooth muscle in the skin, then
19 - the acetylcholine-like agent w ould be potentiated rather than inhibited.
20
21 (Choice B) Acetylcholine is a smooth muscle muscarinic receptor agonist, and w ould thereby increase
22 muscle contraction.
23
24 (Choice C) Pralidoxime is an antidote to organophosphorus anticholinesterases that inhibit cholinesterases
25 through phosphorylation. Pralidoxime reactivates phosphorylated cholinesterases by removing the phosphate
26 group. Consequently, pralidoxime w ould decrease the action of Drug X only if the target cells had been
27 exposed to an organophosphorus anticholinesterase. There is no evidence of such an exposure in this
28 experiment.
29
30 (Choice D) Phentolamine competitively blocks alpha-adrenergic receptors, but does not directly interact w ith
31 the muscarinic receptors of smooth muscle.
32
33 Educational objective:
34 Acetylcholine-like muscarinic agonists cause smooth muscle contraction, are potentiated by physostigmine,
35 and are inhibited by atropine.
36
37 Time Spent: 38 seconds 70% answ ered correctly. Last updated: [11 / 10/2011].
38 Copyright © USMLEWORLD,LLC. SimExam ver. 2011 .11.397425
1 •
2 Item: 13 of 46 ~ \ ' Mark -<:J C>
Q. Id: 6479 (3831471 Previous NeHt
3
4
5
6 A 70-year-old female and a 25-year-old male are each given single doses of diazepam for anxiety attacks.
7 Diazepam plasma levels in each patient over time are plotted on the graph below.
8
9
10
11
12

14
15
16
17 Time
18
19 The 70-year-old female is more likely to experience which of the following?
20
21
A Insomnia (10%]
22
23 v B. Confusion (72%]
24 C. Generalized tonic-clonic seizures (5%]
25 X @ D. Cardiac arrhythmia (1 0%]
26
27 E. Muscle rigidity (4%]
28
29
Explanation: User Id: 388147
30
31
The graph above demonstrates the slowness w ith w hich diazepam is eliminated in the elderly. In the younger
32
adult, diazepam has a half-life of 20-50 hours; in the older adult, however, diazepam has a half-life
33
approaching 90 hours. The metabolism of benzodiazepines is accomplished through two different
34
mechanisms. Some benzodiazepines (diazepam and midazolam) undergo hepatic oxidative reduction, while
35
others (lorazepam) undergo hepatic glucuronide conjugation. The benzodiazepine metabolites are then
36
excreted in the urine.
37
1 A

2 Item: 13 of 46 f.1 P Mark -<:I t>-


Q. Id : 6479 (388147] Previous Next
3
._. ... ._.,~) ... ,._..._ .... t-'._.' I I I,._.,_. ._. I,._.,, 111 '-' V• <-V '-'V I'"'""'' '-' J II I U 1'-' VIU'-'1 ._. ... ._.,~) I •V • • '-' v '-'' ) ._.,._..._...,f"._.' I I I,,_.,_. ._. I,._.,, 111 '-'
4
approaching 90 hours. The metabolism of benzodiazepines is accomplished through tw o different
5
mechanisms. Some benzodiazepines (diazepam and midazolam) undergo hepatic oxidative reduction, w hile
6
others (lorazepam) undergo hepatic glucuronide conjugation. The benzodiazepine metabolites are then
7
excreted in the urine.
8
9
The elderly individual has an increased volume of distribution (secondary to decreased total body w ater,
10
decreased total body mass, and increased body fat) and a reduced hepatic size and blood flow . These
11
factors contribute to a slow ing in benzodiazepine elimination w ith age. Marked increases in the half-life of
12
diazepam make it more likely for the older patient to experience adverse effects such as confusion (ie,
impairment of consciousness, orientation, thought, and attention), paradoxical agitation or aggression,
14
anterograde amnesia, psychomotor retardation, ataxia, w eakness, vertigo, and syncope. Falls and traumatic
15
injuries are serious risks. If diazepam reaches toxic levels, respiratory depression and coma may result.
16
17
(Choice A) Although paradoxical agitation may occur w ith benzodiazepine treatment, insomnia is not a know n
18
adverse effect.
19
20
(Choice C) Benzodiazepines raise the threshold for seizure provocation and are often used for their
21
anticonvulsant properties. Abrupt diazepam w ithdraw al in an epileptic patient can provoke a seizure. As a
22
consequence, the longer t 112 of diazepam in an elderly patient may protect against a diazepam w ithdrawal
23
24 seizure follow ing a single dose.
25
26 (Choice D) Intravenously administered diazepam has reportedly caused ventricular premature complexes
27 and other arrhythmias w hen given prior to cardioversion . Typically, how ever, an arrhythmia w ould be quite
28 unexpected.
29
30 (Choice E) Benzodiazepines can be used as muscle relaxants. Generalized muscle rigidity w ould not be an
31 expected adverse effect.
32
33 Educational objective:
34 Slow er benzodiazepine metabolism in the elderly increases the likelihood of adverse effects. Confusion,
35 anterograde amnesia, and psychomotor retardation are commonly reported in this patient population .
36
37 Time Spent: 77 seconds 72% answ ered correctly. Last updated: [11 / 10/2011].
38 Copyright © USMLEWORLD,LLC. SimExam ver. 2011 .11.397425
1 A

2 Item: 14 of 46 ll P Mark -<:I t>-


Q. Id : 6483 (388147] Previous Next
3
4
5
6 A 17-year-old female is brought to the emergency department by her boyfriend. She is confused and
7 complains of not seeing clearly. She keeps asking for w ater because of extreme thirst. An overdose of w hich
8 of the following drugs is most likely responsible for this patient's condition?
9
10
11 A. Heroin [8%]
12 B. Methylphenidate [ 15%]
13 ~
.; @ C. Atropine [67%]
15
16 D. Diazepam [3%]
17
E. Cocaine [8%]
18
19 -
20
21 Explanation: User Id: 388147
22
23 This patient presents w ith some of the signs and symptoms of anticholinergic syndrome, w hich is caused by
24 an overdose of a substance that inhibits muscarinic cholinergic neurotransmission. The classic clinical
25 manifestations of anticholinergic syndrome are summarized by the mnemonic "hot as a hare, dry as a bone,
26 red as a beet, blind as a bat, and mad as a hatter" (fever, dry skin and mucous membranes, flushing,
27 m ydriasis and cycloplegia, and altered mental status). The altered mental status observed in these patients
28 encompasses confusion, excitation, disorientation, delirium, and psychosis. Coma and respiratory failure can
29 develop in severe cases.
30
31 Numerous drugs have anticholinergic properties, w ith atropine the classic example. Other common
32 anticholinergics include antihistamines (eg, hydroxyzine, diphenhydramine), antipsychotics (eg,
33 chlorpromazine, olanzapine), antispasmodics (eg, dicyclomine, hyoscyamine) and cyclic antidepressants (eg,
34 amitriptyline, desipramine).
35
36 (Choices A, D, and E) Heroin overdose is associated w ith central nervous system depression, seizures, and
37 pinpoint pupils. Benzodiazepine overdose is characterized by decreased neurologic function . Cocaine
38 toxicity features central nervous system stimulation, cardiovascular stimulation, and m ydriasis. Blurred vision
1 A

2 Item: 14 of 46 ll P Mark -<:I t>-


Q. Id : 6483 (388147] Previous Next
3
4 u. u 1azepam [sroJ
5
E. Cocaine [8%]
6
7
8
9 Explanation: User Id: 388147
10
11 This patient presents w ith some of the signs and symptoms of anticholinergic syndrome, w hich is caused by
12 an overdose of a substance that inhibits muscarinic cholinergic neurotransmission. The classic clinical
13 ~
manifestations of anticholinergic syndrome are summarized by the mnemonic "hot as a hare, dry as a bone,
red as a beet, blind as a bat, and mad as a hatter" (fever, dry skin and mucous membranes, flushing,
15 mydriasis and cycloplegia, and altered mental status). The altered mental status observed in these patients
16 encompasses confusion, excitation, disorientation, delirium, and psychosis. Coma and respiratory failure can
17 develop in severe cases.
18
19 - Numerous drugs have anticholinergic properties, w ith atropine the classic example. Other common
anticholinergics include antihistamines (eg, hydroxyzine, diphenhydramine), antipsychotics (eg,
20
21 chlorpromazine, olanzapine), antispasmodics (eg, dicyclomine, hyoscyamine) and cyclic antidepressants (eg,
22 amitriptyline, desipramine).
23
24 (Choices A, D, and E) Heroin overdose is associated w ith central nervous system depression, seizures, and
25 pinpoint pupils. Benzodiazepine overdose is characterized by decreased neurologic function. Cocaine
26 toxicity features central nervous system stimulation, cardiovascular stimulation, and mydriasis. Blurred vision
27 and intense thirst are not common complaints w ith ingestion of these substances.
28
29 (Choice B) Methylphenidate is a piperidine-derived medication that stimulates the cerebral cortex in a manner
30 similar to amphetamines. Acute methylphenidate intoxication can cause confusion, dry mucous membranes,
31 and mydriasis. Blurred vision is not a common complaint.
32
33 Educational objective:
34 Confusion, blurred vision, dry mucous membranes, and intense thirst are associated w ith anticholinergic
35 toxicity.
36
37 Time Spent: 28 seconds 67% answ ered correctly. Last updated: [3/27/201 OJ.
38 Copyright © USMLEWORLD,LLC. SimExam ver. 2011 . 11.397425
1 A

2 Item: 15 of 46 II P Mark -<:I t>-


Q. Id : 6487 (388147] Previous Next
3
4
5
6 A 29-year-old Caucasian female presents for genetic counseling. Both of her children have been diagnosed
7 w ith osteogenesis imperfecta. She and her husband are free of symptoms and have unremarkable family
8 histories. W hich of the follow ing is most likely responsible for their children's disorder?
9
10
11 A. Somatic mutation [27%]
12 B. Incomplete penetrance [28%]
13 ~ ~ @ C. Gern1line mosaicisn1 [30%]
14
D. Trinucleotide repeat expansion [3%]
16 E . Pleiotropy [12%]
17
18
19 - Explanation: User Id: 388147
20
21 Mosaic ism is defined as the presence of two or more genetically different cell lines w ithin the body. Typically,
22 mosaic ism results from a mutation in the first stages of embryonic development. The earlier the mutation
23 happens, the more daughter cells are affected. Mosaicism can be classified as germline, somatic, or
24 both. Germline mosaicism involves only oocytes or spermatocytes, and should be considered w hen a
25 genetic mutation is identified in the offspring but not the parents. The likelihood that this couple w ill have
26 another child affected by the same mutation depends on the proportion of mutant to w ild-type germ cells in the
27 mosaic parent. The presence of two affected children in the family above suggests that the number of mutant
28 germ cells in the mosaic parent is high.
29
30 (Choice A) A somatic mutation is an acquired alteration of somatic cell DNA. Unlike germline mutations,
31 somatic mutations cannot be passed on to offspring.
32
33 (Choice B) Penetrance describes the extent to w hich a gene is phenotypically expressed. Highly penetrant
34 genes are readily apparent in the individual, w hile genes w ith incomplete penetrance only sometimes produce
35 the associated trait or condition to a demonstrable extent. The gene for osteogenesis imperfecta is highly
36 penetrant.
37
38 (Choice D) Various medical disorders are caused by genes that contain repeated trinucleotide sequences,
... ,. . ' ' '
39
40
1 A

2 Item: 15 of 46 II P Mark -<:I t>-


Q. Id : 6487 (388147] Previous Next
3
4
5 Explanation: User Id: 388147
6
7 Mosaic ism is defined as the presence of two or more genetically different cell lines w ithin the body. Typically,
8 mosaic ism results from a mutation in the first stages of embryonic development. The earlier the mutation
9 happens, the more daughter cells are affected. Mosaicism can be classified as germline, somatic, or
10 both. Germline mosaicism involves only oocytes or spermatocytes, and should be considered w hen a
11 genetic mutation is identified in the offspring but not the parents. The likelihood that this couple w ill have
12 another child affected by the same mutation depends on the proportion of mutant to w ild-type germ cells in the
13 ~
mosaic parent. The presence of two affected children in the family above suggests that the number of mutant
14 germ cells in the mosaic parent is high.

16 (Choice A) A somatic mutation is an acquired alteration of somatic cell DNA. Unlike germline mutations,
17 somatic mutations cannot be passed on to offspring.
18
19 - (Choice B) Penetrance describes the extent to w hich a gene is phenotypically expressed. Highly penetrant
genes are readily apparent in the individual, w hile genes w ith incomplete penetrance only sometimes produce
20
21 the associated trait or condition to a demonstrable extent. The gene for osteogenesis imperfecta is highly
22 penetrant.
23
24 (Choice D) Various medical disorders are caused by genes that contain repeated trinucleotide sequences,
25 resulting in the creation of faulty protein products. Such disorders include fragile X syndrome, Huntington's
26 disease, Friedreich's ataxia, and spinocerebellar ataxia.
27
28 (Choice E) Pleiotropy describes the impact a single gene has on multiple phenotypic traits. The genes
29 responsible for most inherited diseases are pleiotropic in that they have numerous and varied clinical
30 manifestations. W hile the osteogenesis imperfecta gene may be pleiotropic, pleiotropy does not explain how
31 the condition arose in this particular family.
32
33 Educational objective:
34 Germline mosaic ism (the presence of two or more genetically different gamete cell lines) should be
35 considered w hen a genetic mutation is identified in the offspring but not the parents.
36
37 Time Spent: 45 seconds 30% answ ered correctly. Last updated: [3/27/201 O].
38 Copyright © USMLEWORLD,LLC. SimExam ver. 2011 .11.397425
1 A

2 Item: 16 of 46 II P Mark -<:I t>-


Q. Id : 6493 (388147] Previous Next
3
4
5
6 A 54-year-old moderately overweight Caucasian female w as diagnosed w ith type 2 diabetes mellitus two
7 years ago. Her glycemia is reasonably w ell controlled w ith oral metformin. She exercises regularly. Her
8 mother died of a stroke at 65 years old. Her father suffers from hypertension, osteoarthritis, and kidney
9 stones. Her blood pressure is 130/80 mm Hg and her heart rate is 76/min. W hich of the follow ing regular
10 activities is most recommended in this patient?
11
12
A. Fish oil consumption [1 8%]
13 ~
14 B. Vitamin E supplementation [8%]
15 C. Daily foot inspection [44%]
D. Monthly pinprick sensation testing [12%]
17
18 X @ E. Complex carbohydrate restriction [1 8%]
19 -
20
21 Explanation: User Id: 388147
22
Diabetes mellitus is associated w ith a variety of complications, including cardiovascular disease, stroke,
23
end-stage renal disease, retinopathy, and ulcerative foot infections. Typically, diabetic foot infections arise
24
secondary to trauma in the setting of preexistent low er extremity vascular and neurologic disease. Careful
25
and regular inspection of the foot is therefore an extremely important component of preventive care in the
26
diabetic. Clinicians are advised to complete comprehensive foot examinations annually on all diabetic
27
patients, w ho should in turn be instructed on prophylactic foot care: feet should be w ashed and closely
28
examined daily; appropriately fitting socks should be changed daily; shoes should be snug and fit the contours
29
of the foot w ell; toe nails should be trimmed to the shape of the toe, w ith cuticles left intact; and exposure to
30
potential trauma should be limited (eg, by w earing shoes indoors and testing w ater temperature before
31
bathing). Therefore, it is most important to recommend daily foot inspection for this patient.
32
33
34 (Choice A) Fish oil supplementation is often recommended for those patients w ith refractory
hypertriglyceridemia but is not as w idely recommended for diabetics, in part because some studies suggest it
35
36 can affect glucose regulation .
37
(Choice B) Routine vitamin E supplementation is not recommended for diabetic patients.
38
1 A

2 Item: 16 of 46 II P Mark -<:I t>-


Q. Id : 6493 (388147] Previous Next
3
4 IX @ E. Complex carbohydrate restriction [18%]
5
6
7 Explanation: User Id: 388147
8
9 Diabetes mellitus is associated w ith a variety of complications, including cardiovascular disease, stroke,
10 end-stage renal disease, retinopathy, and ulcerative foot infections. Typically, diabetic foot infections arise
11 secondary to trauma in the setting of preexistent low er extremity vascular and neurologic disease. Careful
12 and regular inspection of the foot is therefore an extremely important component of preventive care in the
13 ~
diabetic. Clinicians are advised to complete comprehensive foot examinations annually on all diabetic
14 patients, w ho should in turn be instructed on prophylactic foot care: feet should be w ashed and closely
15 examined daily; appropriately fitting socks should be changed daily; shoes should be snug and fit the contours
of the foot w ell; toe nails should be trimmed to the shape of the toe, w ith cuticles left intact; and exposure to
17 potential trauma should be limited (eg, by w earing shoes indoors and testing w ater temperature before
18 bathing). Therefore, it is most important to recommend daily foot inspection for this patient.
19 - (Choice A) Fish oil supplementation is often recommended for those patients w ith refractory
20
21 hypertriglyceridemia but is not as w idely recommended for diabetics, in part because some studies suggest it
22 can affect glucose regulation .
23
24 (Choice B) Routine vitamin E supplementation is not recommended for diabetic patients.
25
26 (Choice D) Neurologic evaluation of Achilles tendon reflex, vibration sense, and pain and temperature
27 sensation should be performed periodically (yearly) on diabetic patients. Such testing does not need to be
28 performed monthly, how ever.
29
30 (Choice E) Carbohydrate consumption should be carefully controlled in diabetics to limit postprandial blood
31 sugars. Significant complex carbohydrate restriction (as employed in low -carbohydrate diets) is not as
32 commonly suggested because of concerns about long-term efficacy and safety.
33
34 Educational objective:
35 Individuals w ith diabetes mellitus should inspect their feet daily for signs of trauma or infection.
36
37 Time Spent: 93 seconds 44% answ ered correctly. Last updated: [3/27/201 O].
38 Copyright © USMLEWORLD,LLC. SimExam ver. 2011 . 11.397425
1 •
2 Item: 17 of 46 • \' Mark -<:J C>
Q. Id : 6497 (383147) Previous NeHt
3
4
5
6 A 29-year-old male is hospitalized with persistent fever and fatigability. He does not respond to antibiotic
7 treatment for his sore throat. Peripheral blood examination reveals the findings shown in the image below.
8
9
10
11
12
13 I"
14
15
16

18
19 '°
20
21
22
23
24
25
26
27
28
29 Displayod "'ithpeimissiw fian Spring•.. Healthcare Ltd.© Cop)<ligbt 2005 by Current Medicine
30
31
32 Which of the following chromosomal abnormalities is most likely present in this patient?
33
34 X @ A. t(9;22) [14%]
35
B. t(8;14) [12%]
36
37 >1 C. t(15.17) (64%]
38 D. t(14;18) (8%]
39
40
1 A

2 Item: 17 of 46 II P Mark -<:I t>-


Q. Id : 6497 (388147] Previous Next
3
4
X @ A. t(9;22) [14%]
5
6 B. t(B; 14) [ 12%]
7 C. t(15;17) [64%]
8
9 D. t(14;18) [8%]
10 E. t(12;21 ) [2%]
11
12
13 ~ Explanation: User Id: 388147
14
15 The azurophilic, needle-shaped cytoplasmic inclusion visible in one of the cells show n above is an Auer rod.
16 The Auer rod consists of fused primary granules and can be seen in the cytoplasm of m yeloblasts,
promyelocytes and monoblasts. Classically, Auer rods are observed only in the peripheral blood smears of
18 patients w ith acute m yeloid leukemia, and acute prom yelocytic leukemia (AML-3} in particular. Acute
19 - promyelocytic leukemia (APL} is a disease characterized by the chromosomal rearrangement t(15; 17}. This
translocation is highly specific for APL, and is not seen in association w ith other leukemias or solid tumors.
20
21 Clinical features of APL include complications of pancytopenia (eg, w eakness, fatigue, infections,
22 hemorrhagic findings} and disseminated intravascular coagulation.
23
24 (Choices A, B, D, and E) The chromosomal rearrangement of t(9;22} is characteristic of chronic
25 m yelogenous leukemia (CML}. In CML, the peripheral blood smear is leukoerythroblastic w ith many immature
26 m yeloid cells in circulation. The chromosomal rearrangements oft(8;14} and t(12;21 } are common in acute
27 lymphocytic leukemia (ALL}. In ALL, the peripheral blood smear typically demonstrates anemia,
28 thrombocytopenia, and lymphoblasts. The chromosomal rearrangement oft(14;18} is common in follicular
29 lymphoma (FL}, although it is not specific for the disease. In FL, the peripheral blood smear often
30 demonstrates malignant cells w ith notches or clefts ("centrocytes"}. Auer rods are not generally seen in
31 association w ith any of these chromosomal abnormalities.
32
33 Educational objective:
34 Auer rods are common findings in acute promyelocytic leukemia, w hich is linked to the chromosomal
35 rearrangement oft( 15; 17}.
36
37 Time Spent: 29 seconds 64% answ ered correctly. Last updated: [10/3/2011].
38 Copyright © USMLEWORLD,LLC. SimExam ver. 2011 .11.397425
1 A

2 Item: 18 of 46 II P Mark -<:I t>-


Q. Id : 6499 (388147] Previous Next
3
4
5
6 A researcher studies different components of the immune response. Which of the following cytokines is
7 produced exclusively by lymphocytes?
8
9
10 x @ A. IL-1 [5%]
11
12 B. IL-2 [66%]
13 ~
14 C. TNF-a [9%]
15
16 D. IFN-a [1 3%]
17
E. GM-CSF [5%]
19 -
20
21
22 Explanation: User Id: 388147
23
24 lnterleukin-2 (IL-2) is produced exclusively by antigen-stimulated T lymphocytes. This potent cytokine
25 functions to stimulate the growth and differentiation of T cells, B cells, NK cells, and macrophages. When IL-2
26 binds to T-cell IL-2 receptors (IL-2Rs) in an autocrine fashion, the stimulated T cells proliferate clonally. Both
27 IL-2 production and IL-2R expression are required for this T cell proliferation, thereby ensuring that only the T
28 cells specific for the provoking antigen will be activated. Should IL-2 be present for a lengthy period, it can
29 promote Fas-mediated apoptosis.
30
(Choice A) lnterleukin-1 (IL-1 ) is chiefly produced by mononuclear phagocytes. It functions to activate
31
32 lymphocytes and triggers numerous systemic changes associated with illness (eg, fever, lethargy, anorexia).
33 (Choice C) TNF-a is a cytokine primarily produced by activated macrophages. It induces the systemic
34
inflammatory response, and in high concentrations, causes symptoms of septic shock and cachexia.
35
36 (Choice D) Interferon-alpha (IFN-a) is chiefly generated by monocytes, macrophages, B cells, and NK
37 cells. It has numerous antiviral functions (eg, viral replication inhibition within cells, protection of uninfected
38 cells from viral infection. stimulation of cvtotoxic lvmphocvte and NK cell antiviral activitv}.
39
40
1 A

2 Item: 18 of 46 II P Mark -<:I t>-


Q. Id : 6499 (388147] Previous Next
3
4
5 D. IFN-a (13%]
6
7 E. GM-CSF (5%]
8
9
10
Explanation: User Id: 388147
11
12
lnterleukin-2 (IL-2) is produced exclusively by antigen-stimulated T lymphocytes. This potent cytokine
13 ~
functions to stimulate the growth and differentiation of T cells, B cells, NK cells, and macrophages. W hen IL-2
14
binds to T-cell IL-2 receptors (IL-2Rs) in an autocrine fashion, the stimulated T cells proliferate clonally. Both
15
IL-2 production and IL-2R expression are required for this T cell proliferation, thereby ensuring that only the T
16
cells specific for the provoking antigen w ill be activated. Should IL-2 be present for a lengthy period, it can
17
promote Fas-mediated apoptosis.
19 - (Choice A) lnterleukin-1 (IL-1) is chiefly produced by mononuclear phagocytes. It functions to activate
20 lymphocytes and triggers numerous systemic changes associated w ith illness (eg, fever, lethargy, anorexia).
21
22 (Choice C) TNF-a is a cytokine primarily produced by activated macrophages. It induces the systemic
23 inflammatory response, and in high concentrations, causes symptoms of septic shock and cachexia.
24
25 (Choice D) Interferon-alpha (IFN-a) is chiefly generated by monocytes, macrophages, B cells, and NK
26 cells. It has numerous antiviral functions (eg, viral replication inhibition w ithin cells, protection of uninfected
27 cells from viral infection, stimulation of cytotoxic lymphocyte and NK cell antiviral activity).
28
29 (Choice E) Granulocyte-macrophage colony-stimulating factor (GM-CSF) is secreted by macrophages, T
30 cells, NK cells, mast cells, endothelial cells, and fibroblasts. GM-CSF stimulates stem cell production of
31 granulocytes and monocytes.
32
33 Educational objective:
34 lnterleukin-2 is exclusively produced by T lymphocytes and functions to stimulate the growth and differentiation
35 of T cells, B cells, NK cells, and macrophages.
36
37 Time Spent: 49 seconds 66% answ ered correctly. Last updated: (212212012].
38 Copyright © USMLEWORLD,LLC. SimExam ver. 2011 .11.397425
1 •
2 Item: 19 of 46 • \' Mark -<:J C>
Q. Id: 6[,{)2 (383147) Previous NeHt
3
4
5
6 A researcher treats an epithelial cell line w ith substance X, w hich lyses intercellular adhesions but leaves cell
7 adhesions to the basement membrane intact. Removing Ca 2 • from the cells' environment causes the same
8 effect. Substance X most likely affects which of the following?
9
10
11 x ·i!l A. lntegrins (11 %)
12 ~ B. Cadhenns (62%)
13 I" C . Hemidesmosomes (16%)
14
15 D. Fibronectin (7%)
16 E. Laminin (3%)
17
18
'" Explanation: Use r Id: 388147
20
21
22
23
24
25
26 Tight junctions
27
28
29
30
31
32 Desmosomes
33
34
35
36
37
1 •
2 Item: 19 of 46 • \' Mark -<:J C>
Q. Id: 6[,{)2 (383147) Previous NeHt
3
4 ' J

5 D. Fibronectin [7%]
6 E. Laminin [3%]
7
8
9 Explanation: User Id: 388147
10
11
12
13 I"
14
15
16 Tight junctions
17
18
'"
20
21
22 Desmosomes
23
24
25
26 Gap junctions ~;g::
27 Hemidesmosomes
28
29
lntegrin-.
30
31 Fibronectln -._:::::;:z:~:<::::::::2:::2:::><:::-==~':'.5~~
32
33
34
35
36
37
1 A

2 Item: 19 of 46 II P Mark -<:I t>-


Q. Id : 6502 (388147] Previous Next
3
4
5 Gap junctions
JL
2:s
6 Hemidesmosomes
7
8
9 lntegrin - .
10
11
Fibronectin -._::::;:z:::::<::::::::::2::2::::><:~::;::>_:5~~:;;::;0:............_
12
13 ~ (~~~~--==~=-~~~)
I ype 4 collagen
14
15
16 Cadherins are calcium-dependent adhesion molecules that help muscle and epithelial cells bind together.
17 Examples of cadherins include desmocollins, desmogleins, and E-cadherin. Cadherins are transmembrane
18 proteins that bind w ithin the cytoplasm to intermediate filaments (keratin) or microfilaments (actin) via
- intermediate proteins (catenin, plakoglobin and desmoplakin), and bind extracellularly to cadherin molecules
on adjacent cells, forming adherens junctions and desmosomes. Importantly, interactions between the
20
21 extracellular domains of adjacent cadherins is calcium-dependent, hence, removing calcium from the
22 extracellular fluid w ill cause dissociation of cadherin-mediated junctions, ultimately leading to loss of cell-cell
23 adhesion. In contrast, cell adhesion to the basement membrane is mediated primarily by hemidesmosomes,
24 w hich do not rely on cadherins.
25
26 (Choice A, C & E) Hemidesmosomes mediate cell binding to the basement membrane. Unlike
27 desmosomes, w hich rely on cadherins, hemidesmosome binding is mediated primarily by integrins.
28
29 (Choice D) Fibronectin is an extracellular matrix glycoprotein that binds cell surface integrins, facilitating cell
30 binding to the extracellular matrix.
31
32 Educational Objective:
33 Cadherins are calcium-dependent transmembrane proteins that participate in desmosome formation via
34 homologous interaction w ith cadherins on adjoining cells. Cadherins do not participate in hemidesmosome
35 function.
36
37 Time Spent: 42 seconds 62% answ ered correctly. Last updated: [11/ 10/2011].
38 Copyright © USMLEWORLD,LLC. SimExam ver. 2011.11.397425
1 •
2 Item: 20 of 46 • \' Mark -<:J C>
Q. Id: 6507 (383147) Previous NeHt
3
4
5
6 A 40-year-old male with advanced AIDS presents to clinic w ith lesions on his face, as seen in the image below.
7
8
9
10
11
12
13 I"
14
15
16
17
18
19 '°
21
22
23
24
25
26
27
28
29
30
31
32
33
34
35 W hich of the following is the most likely diagnosis in this patient?
36
37
38 A. Rosacea (1%]
1 A

2 Item: 20 of 46 II P Mark -<:I t>-


Q. Id : 6507 (388147] Previous Next
3
4
5 W hich of the follow ing is the most likely diagnosis in this patient?
6
7
8 A. Rosacea [1 %]
9 B. Shingles [5%]
10 .; @ C. Molluscum contagiosum [66%]
11
12 D. W arts [8%]
13 ~ E. Kaposi's sarcoma [21 %]
14
15
16 Explanation: User Id: 388147
17
18 Caused by a poxvirus (the MC virus), molluscum contagiosum is a common dermatologic condition
19 - characterized by multiple firm, flesh-colored, dome-shaped papules on the skin and mucous
membranes. The papules are often pruritic and have umbilicated centers containing w hite, curdlike
21 material. In children, the MC virus is typically transmitted through direct skin-to-skin contact or fomites, w ith
22 lesions appearing on the eyelids, face, trunk, or axilla. In adults, the MC virus is sexually transmitted, w ith
23 lesions appearing on the trunk and anogenital region. HIV-positive or otherwise immunocompromised
24 individuals tend to experience a prolonged course characterized by w idely distributed papules numbering in
25 the hundreds and facial involvement.
26
27 (Choice A) Rosacea first manifests w ith easy flushing and is later characterized by persistent erythema,
28 telangiectasia, and sensitivity to sunlight or insults of a chemical or physical nature.
29
30 (Choice B) Shingles, or herpes zoster, is characterized by a painful, vesicular skin eruption along the sensory
31 dermatomes.
32
33 (Choice D) Non-genital w arts are benign tumors most often appearing on the hands and knees. The lesions
34 are typically skin-colored or pink, and vary in appearance from smooth flattened papules to verrucous,
35 papilliform growths.
36
37 (Choice E) Kaposi's sarcoma (KS) presents as multiple blue-violet dermal plaques that appear on the feet
38 and legs before spreading proximally. KS lesions can also develop on the mucosal membranes of the face
. .. .
39
40
1 A

2 Item: 20 of 46 II P Mark -<:I t>-


Q. Id : 6507 (388147] Previous Next
3
LI . VV OI l~ LU / Uj
4
5 E. Kaposi's sarcoma [21 %]
6
7
8 Explanation: User Id: 388147
9
10 Caused by a poxvirus (the MC virus), molluscum contagiosum is a common dermatologic condition
11 characterized by multiple firm, flesh-colored, dome-shaped papules on the skin and mucous
12 membranes. The papules are often pruritic and have umbilicated centers containing w hite, curdlike
13 ~
material. In children, the MC virus is typically transmitted through direct skin-to-skin contact or fomites, w ith
14 lesions appearing on the eyelids, face, trunk, or axilla. In adults, the MC virus is sexually transmitted, w ith
15 lesions appearing on the trunk and anogenital region. HIV-positive or otherwise immunocompromised
16 individuals tend to experience a prolonged course characterized by w idely distributed papules numbering in
17 the hundreds and facial involvement.
18
19 - (Choice A) Rosacea first manifests w ith easy flushing and is later characterized by persistent ery1hema,
telangiectasia, and sensitivity to sunlight or insults of a chemical or physical nature.
21
22 (Choice B) Shingles, or herpes zoster, is characterized by a painful, vesicular skin eruption along the sensory
23 dermatomes.
24
25 (Choice D) Non-genital w arts are benign tumors most often appearing on the hands and knees. The lesions
26 are typically skin-colored or pink, and vary in appearance from smooth flattened papules to verrucous,
27 papilliform grow1hs.
28
29 (Choice E) Kaposi's sarcoma (KS) presents as multiple blue-violet dermal plaques that appear on the feet
30 and legs before spreading proximally. KS lesions can also develop on the mucosal membranes of the face
31 and genitals.
32
33 Educational objective:
34 Caused by a poxvirus (the MC virus), molluscum contagiosum is a common dermatologic condition
35 characterized by multiple firm, flesh-colored, dome-shaped papules on the skin and mucous membranes.
36
37 Time Spent: 27 seconds 66% answ ered correctly. Last updated: [1 0/3/2011].
38 Copyright © USMLEWORLD,LLC. SimExam ver. 2011 .11.397425
1 A

2 Item: 21 of 46 II P Mark -<:I t>-


Q. Id: 6511 (388147] Previous Next
3
4
5
6 The duodenum is sampled in experimental animals after applying various stimuli. W hich of the follow ing
7 triggers copious bicarbonate-rich fluid accumulation in the duodenum?
8
9
10 A. Low osmolarity of duodenal content [2%]
11
12
13 ~ B. Low pH of duodenal content [89%]
14
15
16 C. Insulin release by pancreatic 13-cells [2%]
17 X @
18
19 - D. Sympathetic stimulation of pancreatic acini [6%]
20

22 E. Vagal denervation of the liver [0%]


23
24
25
26
27 Explanation: User Id: 388147
28
29 The delivery of acidic (pH< 4.5) chyme from the stomach to the duodenum triggers gallbladder bile release
30 and stimulates the S cells in the crypts of LieberkOhn to release secretin into systemic circulation. The
31 pancreas is then prompted by secretin to generate bicarbonate-rich fluid, w hich neutralizes the hydrogen ions
32 in chyme. The establishment of a neutral pH in the duodenal lumen prevents injury to the duodenal mucosa
33 and allow s digestive enzymes to function properly.
34
35 (Choice A) Duodenal content w ith low osmolarity (eg, after drinking a significant quantity of w ater) does not
36 trigger copious bicarbonate release.
37
38 (Choice C) Insulin release by pancreatic 13-cells stimulates cellular glucose uptake and glycogen formation in
,, ,. ' ' ,... ,. . '. ,. '
39
40
1 A

2 Item: 21 of 46 II P Mark -<:I t>-


Q. Id: 6511 (388147] Previous Next
3
4 D. Sympathetic stimulation of pancreatic acini [6%]
5
6
7 E. Vagal denervation of the liver [0%]
8
9
10
11
12 Explanation: User Id: 388147
13 ~
14 The delivery of acidic (pH< 4.5) chyme from the stomach to the duodenum triggers gallbladder bile release
15 and stimulates the S cells in the crypts of LieberkOhn to release secretin into systemic circulation. The
16 pancreas is then prompted by secretin to generate bicarbonate-rich fluid, w hich neutralizes the hydrogen ions
17 in chyme. The establishment of a neutral pH in the duodenal lumen prevents injury to the duodenal mucosa
18 and allow s digestive enzymes to function properly.
19 - (Choice A) Duodenal content w ith low osmolarity (eg, after drinking a significant quantity of w ater) does not
20
trigger copious bicarbonate release.
22
23 (Choice C) Insulin release by pancreatic 13-cells stimulates cellular glucose uptake and glycogen formation in
24 the liver and muscles. Secretin promotes increased insulin release.
25
26 (Choice D) In general, the pancreatic acini, islets, and ducts are innervated by postganglionic fibers of the
27 parasympathetic nervous system, w hile the pancreatic blood vessels are innervated by the postganglionic
28 fibers of the sympathetic nervous system. Sympathetic stimulation of the pancreas would not favor
29 bicarbonate release.
30
31 (Choice E) W hile vagal denervation of the liver does occur during hepatic transplant, it does not appear to
32 have a marked impact on its function .
33
34 Educational objective:
35 Duodenal acidity triggers the release of bile and bicarbonate-rich pancreatic fluid.
36
37 Time Spent: 53 seconds 89% answ ered correctly. Last updated: [3/27/201 O].
38 Copyright © USMLEWORLD,LLC. SimExam ver. 2011 . 11.397425
1
2 Item: 22 of 46 II P Mark -<:I t>-
Q. Id : 6516 (388147] Previous Next
3
4
5
6 A 17-year-old hockey player sustains a blov1 to the right knee and is brought to the emergency department
7 w ith severe knee pain . Physical examination reveals gross posterior displacement of the tibia relative to the
8 femur. W hich of the following structures is most likely to have been injured in this patient?
9
10
11 A. Saphenous nerve [4% ]
12 B. Common peroneal nerve [25%]
13 ~
14 C. Tibial nerve [30%]
15 .; D. Popliteal artery [37%]
16
17 x @ E. Popliteal vein [4% ]
18
19
20 Explanation: User Id: 388147
21
In all patients w ith knee dislocation, a neurovascular examination distal to the point of dislocation is necessary,
23 as the popliteal artery, tibial nerve, and common peroneal nerve are at risk for injury. In posterior and anterior
24 dislocations, the popliteal artery is at especially high risk because it is located deep w ithin the popliteal fossa,
25 close to the articular surfaces of the joint, and is tightly fixed proximal and distal to the popliteal fossa by the
26 adductor magnus and soleus muscles, respectively, making it highly susceptible to tearing by traction forces.
27
28 (Choice A) The saphenous nerve provides sensory innervation to the medial leg and foot. It extends from the
29 femoral nerve and courses in close association w ith the saphenous vein.
30
31 (Choice B) The common peroneal nerve is the most commonly injured nerve in the distal leg. It courses
32 superficially and laterally around the head of the fibula and is often injured by blunt trauma to the lateral knee.
33
34 (Choice C) Like the popliteal artery, the tibial nerve courses through the popliteal fossa . However, it is not
35 subject to the same sort of traction forces as the popliteal artery. Penetrating trauma to the popliteal fossa is
36 a common cause of tibial nerve injury.
37
38 (Choice E) The popliteal vein courses superficial to the popliteal artery through the popliteal fossa. It is less
1
2 Item: 22 of 46 II P Mark -<:I t>-
Q. Id : 6516 (388147] Previous Next
3
4
C. Tibial nerve [30%]
5
6 ~ D. Popliteal artery [37%]
7
X @ E. Popliteal vein [4%]
8
9
10
11 Explanation: User Id: 388147
12
13 ~
In all patients w ith knee dislocation, a neurovascular examination distal to the point of dislocation is necessary,
14 as the popliteal artery, tibial nerve, and common peroneal nerve are at risk for injury. In posterior and anterior
15 dislocations, the popliteal artery is at especially high risk because it is located deep w ithin the popliteal fossa,
16 close to the articular surfaces of the joint, and is tightly fixed proximal and distal to the popliteal fossa by the
17 adductor magnus and soleus muscles, respectively, making it highly susceptible to tearing by traction forces.
18
19 (Choice A) The saphenous nerve provides sensory innervation to the medial leg and foot. It extends from the
20 femoral nerve and courses in close association w ith the saphenous vein.
21
(Choice B) The common peroneal nerve is the most commonly injured nerve in the distal leg. It courses
23 superficially and laterally around the head of the fibula and is often injured by blunt trauma to the lateral knee.
24
25 (Choice C) Like the popliteal artery, the tibial nerve courses through the popliteal fossa . How ever, it is not
26 subject to the same sort of traction forces as the popliteal artery. Penetrating trauma to the popliteal fossa is
27 a common cause of tibial nerve injury.
28
29 (Choice E) The popliteal vein courses superficial to the popliteal artery through the popliteal fossa. It is less
30 commonly injured.
31
32 Educational objective:
33 Injury to the popliteal artery is the primary concern w ith both anterior and posterior dislocations of the knee
34 joint. This vessel is rigidly fixed proximal and distal to the knee joint by the adductor magnus and soleus
35 muscles, respectively.
36
37 Time Spent: 42 seconds 37% answ ered correctly. Last updated: [3/27/201 O].
38 Copyright © USMLEWORLD,LLC. SimExam ver. 2011 . 11.397425
1
2 Item: 23 of 46 II P Mark -<:I t>-
Q. Id : 6521 (388147] Previous Next
3
4
5
6 A 34-year-old female complains of lower abdominal pain two days after giving birth to her second child via
7 spontaneous vaginal delivery. Her temperature is 37.8°C (100°F), blood pressure is 11 OlZ.Omm Hg, pulse is
8 11 O/min, and respirations are 18/min. Bimanual examination reveals a tender uterus and foul-smelling vaginal
9 discharge. W hich of the follow ing organisms is most likely responsible for her symptoms?
10
11
12 A. Cytomegalovirus [0%]
13 ~ B. Chlamydia trachomatis [ 18%]
14 x @ C. Pseudomonas aeruginosa [5%]
15
16 D. Herpes simplex virus [0%]
17 E. Bacteroides species [26%]
18 F. Gardnerefla vagina/is [50%]
19
20
21 Explanation: User Id: 388147
22
This patient most likely has endometritis, an infection of the endometrium that can complicate the postpartum
24 period in the mother follow ing both vaginal and caesarian births. Risk factors include premature and
25 prolonged rupture of the membranes and use of devices like scalp electrodes and intrauterine pressure
26 catheters. Signs and symptoms include fever, uterine tenderness, foul smelling discharge and leukocytosis.
27 Cultures from infected tissue most commonly reveal mixed aerobic and anaerobic flora, but Bacteroides
28 organisms are the most commonly isolated.
29
30 (Choice A) CMV is not a known cause of endometritis, but it is the most common infectious disease acquired
31 by fetuses in utero. CMV infection in the new born can cause hepatosplenomegaly, petechiae, low birth
32 w eight, microcephaly, direct hyperbilirubinemia, thrombocytopenia and intracranial calcifications.
33
34 (Choice B) Chlamydia may cause some cases of endometritis, but Bacteroides is more common.
35
36 (Choice C) Pseudomonas is not a common cause of endometritis. Diabetics and burn patients are
37 predisposed to Pseudomonas infection.
38
1 A

2 Item: 23 of 46 II P Mark -<:I t>-


Q. Id : 6521 (388147] Previous Next
3
4
5 F. Gardnerella vagina/is (50%]
6
7
8 Explanation: User Id: 388147
9
10 This patient most likely has endometritis, an infection of the endometrium that can complicate the postpartum
11 period in the mother follow ing both vaginal and caesarian births. Risk factors include premature and
12 prolonged rupture of the membranes and use of devices like scalp electrodes and intrauterine pressure
13 ~
catheters. Signs and symptoms include fever, uterine tenderness, foul smelling discharge and leukocytosis.
14 Cultures from infected tissue most commonly reveal mixed aerobic and anaerobic flora, but Bacteroides
15 organisms are the most commonly isolated.
16
17 (Choice A) CMV is not a know n cause of endometritis, but it is the most common infectious disease acquired
18 by fetuses in utero. CMV infection in the new born can cause hepatosplenomegaly, petechiae, low birth
19 - w eight, microcephaly, direct hyperbilirubinemia, thrombocytopenia and intracranial calcifications.
20
21 (Choice B) Chlamydia may cause some cases of endometritis, but Bacteroides is more common .
22
(Choice C) Pseudomonas is not a common cause of endometritis. Diabetics and burn patients are
24 predisposed to Pseudomonas infection.
25
26 (Choice D) HSV does not cause endometritis. Infection causes tender lesions, most commonly on the
27 mouth or genitals. It can be passed to the infant during birth if active lesions are present on the vagina .
28
29 (Choice F) Gardnerelfa vagina/is causes bacterial vaginosis (BV), not endometritis. BV causes clear to gray
30 discharge that has a fishy odor w hen potassium hydroxide is added.
31
32 Educational objective:
33 Endometritis is an infection of the uterus. Most cases follow delivery. Signs and symptoms include fever,
34 leukocytosis, uterine tenderness and foul-smelling discharge. It is most commonly caused by mixed flora,
35 w ith Bacteroides the most commonly isolated organism.
36
37 Time Spent: 37 seconds 26% answ ered correctly. Last updated: (7/2/201 O].
38 Copyright © USMLEWORLD,LLC. SimExam ver. 2011 .11.397425
1 •
2 Item: 24 of 46 • \' Mark -<:J C>
Q. Id: 6529 (383147) Previous NeHt
3
4
5
6 A father brings his two-year-old daughter to your office for evaluation of a painful rash on her right upper
7 extremity (pictured below).
8
9
10
11
12
13 I"
14
15
16
17
18
19 '°
20
21
22
23

25
26
27
28
29
30
31
32
33 What is the most likely diagnosis?
34
35 A. Nummular eczema [22%]
36
B. Atopic dermatitis (34%]
37
38 C. Shingles (24%]
1 A

2 Item: 24 of 46 II P Mark -<:I t>-


Q. Id : 6529 (388147] Previous Next
3
4
5 W hat is the most likely diagnosis?
6
7 A. Nummular eczema [22%]
8 B. Atopic dermatitis [34 %]
9
10 C. Shingles [24%]
11 D. Measles [9%]
12 X @ E . Psoriasis [1 0%]
13 ~
14
15 Explanation: User Id: 388147
16
17 Of the options listed, herpes zoster (shingles) is the most likely diagnosis. Herpes zoster results from
18 reactivation of latent varicella zoster virus (VN), the virus that causes chicken pox. Initial infection of a naive
19 - host by VN causes chicken pox. Upon resolution of this primary infection, immunity develops and VN
20 retreats to the dorsal root ganglion of cutaneous sensory nerves w here it remains in a latent state enjoying
21 shelter from the immune system . The factors that cause reactivation of latent VN are unknow n. Classically,
22 herpes zoster manifests clinically w ith a prodrome of neuropathic pain isolated unilaterally to one dermatome.
23 Subsequently, crops of grouped vesicles w ith underlying erythema erupt in that same dermatome
accompanied by severe pain .
25
26 (Choice A) Nummular eczema most commonly occurs in the setting of xerosis. Use of harsh soaps and
27 exposure to environmental irritants can exacerbate the condition. Nummular eczema classically presents
28 w ith pruritic, coin-shaped, erythematous patches.
29
30 (Choice B) Atopic dermatitis classically affects infants and young children and is frequently associated w ith a
31 family history of atopy. Infants typically experience eruptions on their cheeks and diaper area, w hile older
32 children are classically affected on flexural surfaces.
33
34 (Choice D) Measles is rarely seen in the United States. The viral exanthem associated w ith measles
35 classically begins on the head and face and spreads dow nward to involve the trunk and low er extremities.
36 The pathognomonic lesions are red/blue Koplik spots on the buccal mucosa.
37
38
39
40
1 A

2 Item: 24 of 46 II P Mark -<:I t>-


Q. Id : 6529 (388147] Previous Next
3
4
5
6 Explanation: User Id: 388147
7
8 Of the options listed, herpes zoster (shingles) is the most likely diagnosis. Herpes zoster results from
9 reactivation of latent varicella zoster virus 0fl.V), the virus that causes chicken pox. Initial infection of a naive
10 host by VZ.V causes chicken pox. Upon resolution of this primary infection, immunity develops and VZ.V
11 retreats to the dorsal root ganglion of cutaneous sensory nerves w here it remains in a latent state enjoying
12 shelter from the immune system. The factors that cause reactivation of latent VZ.V are unknow n. Classically,
13 ~
herpes zoster manifests clinically w ith a prodrome of neuropathic pain isolated unilaterally to one dermatome.
14 Subsequently, crops of grouped vesicles w ith underlying erythema erupt in that same dermatome
15 accompanied by severe pain .
16
17 (Choice A) Nummular eczema most commonly occurs in the setting of xerosis. Use of harsh soaps and
18 exposure to environmental irritants can exacerbate the condition. Nummular eczema classically presents
19 - w ith pruritic, coin-shaped, erythematous patches.
20
21 (Choice B) Atopic dermatitis classically affects infants and young children and is frequently associated w ith a
22 family history of atopy. Infants typically experience eruptions on their cheeks and diaper area, w hile older
23 children are classically affected on flexural surfaces.

25 (Choice D) Measles is rarely seen in the United States. The viral exanthem associated w ith measles
26 classically begins on the head and face and spreads dow nw ard to involve the trunk and low er extremities.
27 The pathognomonic lesions are red/blue Koplik spots on the buccal mucosa.
28
29 (Choice E) Psoriasis classically causes w ell-demarcated erythematous plaques w ith overlying silvery scale
30 on the extensor surfaces of the extremities.
31
32 Educational objective:
33 Herpes zoster results from reactivation of latent VZ.V in the dorsal root ganglion . The classic clinical
34 presentation is eruption of vesicles on an erythematous base isolated to a single dermatome and
35 accompanied by severe neuropathic pain .
36
37 Time Spent: 38 seconds 24% answ ered correctly. Last updated: [101312011].
38 Copyright© USMLEWORLD,LLC. SimExam ver. 2011 .11.397425
1
2 Item: 25 of 46 II P Mark -<:I t>-
Q. Id: 6615 (388147] Previous Next
3
4
5
6 A 24-year-old man is filled w ith rage each time he recalls childhood episodes of physical abuse by his foster
7 father. He channels his aggression into intense basketball matches. This is an example of w hich of the
8 following defense mechanisms?
9
10
11 x @ A. Displacement [36%]
12 B. Acting out [3%]
13 ~
14 C. Passive-aggressive behavior [2%]
15 D. Sublimation [52%]
16
17 E. Projection [7%]
18
19
20 Explanation: User Id: 388147
21
22 Sublimation is a mature defense mechanism that involves converting unacceptable feelings or drives into
23 more socially acceptable ones. This man is channeling his unpleasant memories and emotions into a more
24 productive outlet.

26 (Choice A) Displacement is shifting feelings or conflicts associated w ith one situation or person to another.
27 An example might be a medical resident w ho yells at his medical student after being embarrassed by his
28 attending.
29
30 (Choice B) Acting out is an immature defense mechanism that involves expressing unconscious w ishes or
31 impulses through actions. Temper tantrums are a classic example.
32
33 (Choice C) Passive-aggressive behavior involves expressing aggression tow ards others by passively
34 refusing to meet their needs. It allow s one to express hostile feelings in a non-confrontational manner. An
35 example would be a teenager who intentionally comes home late and then claims it w as accidental after his
36 parents impose a new curfew.
37
38 (Choice E) Projection is a defense mechanism w here an individual attributes his own unacceptable thoughts
,, " ' ' ,, . .,.
39
40
1 A

2 Item: 25 of 46 II P Mark -<:I t>-


Q. Id: 6615 (388147] Previous Next
3
4
C. Passive-aggressive behavior [2%]
5
6 D. Sublimation [52%]
7
8
E. Projection [7%]
9
10
11 Explanation: User Id: 388147
12
13 ~
Sublimation is a mature defense mechanism that involves converting unacceptable feelings or drives into
14 more socially acceptable ones. This man is channeling his unpleasant memories and emotions into a more
15 productive outlet.
16
17 (Choice A) Displacement is shifting feelings or conflicts associated w ith one situation or person to another.
18 An example might be a medical resident w ho yells at his medical student after being embarrassed by his
19 - attending.
20
21 (Choice B) Acting out is an immature defense mechanism that involves expressing unconscious w ishes or
22 impulses through actions. Temper tantrums are a classic example.
23
24 (Choice C) Passive-aggressive behavior involves expressing aggression tow ards others by passively
refusing to meet their needs. It allow s one to express hostile feelings in a non-confrontational manner. An
26 example would be a teenager w ho intentionally comes home late and then claims it w as accidental after his
27 parents impose a new curfew.
28
29 (Choice E) Projection is a defense mechanism w here an individual attributes his own unacceptable thoughts
30 and feelings to another person. An example might be a man w ho desires another woman accusing his w ife of
31 infidelity.
32
33 Educational objective:
34 Sublimation is a mature defense mechanism in w hich a person channels socially unacceptable thoughts or
35 impulses into acceptable actions.
36
37 Time Spent: 28 seconds 52% answ ered correctly. Last updated: [3/27/201 OJ.
38 Copyright © USMLEWORLD,LLC. SimExam ver. 2011 . 11.397425
1
2 Item: 26 of 46 II P Mark -<:I t>-
Q. Id : 6540 (388147] Previous Next
3
4
5
6 A cell-surface receptor w ith intracellular tyrosine kinase activity is identified in an epithelial cell line. W hich of
7 the follow ing best describes the normal activity of this receptor?
8
9
x @ A. Phospholipase C activation [20%]
10
11 ~ B. Transient dimerization [53%]
12 C. Translocation to the nucleus [12%]
13 ~
D. Receptor-mediated internalization [14%]
14
15 E. Voltage-dependent activation [1 %]
16
17
18 Explanation: User Id: 388147
19
Receptors for numerous peptide growth factors and hormones have tyrosine kinase activity. For example,
20
tyrosine kinase receptors transduce signals from hormones like insulin, epidermal growth factor, platelet-
21
derived growth factor and vascular endothelial growth factor to name a few . Tyrosine kinase receptors are
22
transmembrane proteins that have a ligand-binding extracellular region, a hydrophobic transmembrane region,
23
and an intracellular (cytoplasmic) domain w ith inducible tyrosine kinase activity. Ligand binding to the
24
extracellular portion of a tyrosine kinase receptor causes dimerization of two identical receptor
25
subunits. Each subunit phosphorylates the other causing a conformational change in the intracellular
domains of the tyrosine kinase receptor dimer. This conformational change exposes each subunit's catalytic
27
domain, and phosphorylation of tyrosine residues on target proteins ensues.
28
29
(Choice A) Phospholipase C activation is a step in the inositol triphosphate (IP3) second messenger system.
30
31
(Choice C) Translocation to the nucleus occurs w hen ligand binds to a cytoplasmic steroid, thyroxine or
32
retinoid receptor. In these cases, the ligand-receptor complex moves from the cytoplasm to the nucleus and
33
34 subsequently directly binds DNA to modify transcription.
35
(Choice D) Receptor-mediated endocytosis involves pooling of receptors in clathrin-coated regions, or "pits,"
36
in the cell membrane. W hen ligand binds to these receptors, a signal is transduced that results in
37
internalization of the ligand-receptor complex into a vesicle. The LDL receptor functions in this manner.
38
1 A

2 Item: 26 of 46 II P Mark -<:I t>-


Q. Id : 6540 (388147] Previous Next
3
LI . I 'Cl..CJJlVI -11 ICUIOlCU 11 llCI I IOll~OUVI I l I "T F O j
4
5 E. Voltage-dependent activation [1 %]
6
7
8 Explanation: User Id: 388147
9
10 Receptors for numerous peptide growth factors and hormones have tyrosine kinase activity. For example,
11 tyrosine kinase receptors transduce signals from hormones like insulin, epidermal growth factor, platelet-
12 derived growth factor and vascular endothelial growth factor to name a few . Tyrosine kinase receptors are
13 ~
transmembrane proteins that have a ligand-binding extracellular region, a hydrophobic transmembrane region,
14 and an intracellular (cytoplasmic) domain w ith inducible tyrosine kinase activity. Ligand binding to the
15 extracellular portion of a tyrosine kinase receptor causes dimerization of two identical receptor
16 subunits. Each subunit phosphorylates the other causing a conformational change in the intracellular
17 domains of the tyrosine kinase receptor dimer. This conformational change exposes each subunit's catalytic
18 domain, and phosphorylation of tyrosine residues on target proteins ensues.
19 - (Choice A) Phospholipase C activation is a step in the inositol triphosphate (IP3) second messenger system.
20
21
22 (Choice C) Translocation to the nucleus occurs w hen ligand binds to a cytoplasmic steroid, thyroxine or
23 retinoid receptor. In these cases, the ligand-receptor complex moves from the cytoplasm to the nucleus and
24 subsequently directly binds DNA to modify transcription .
25
(Choice D) Receptor-mediated endocytosis involves pooling of receptors in clathrin-coated regions, or "pits,"
27 in the cell membrane. W hen ligand binds to these receptors, a signal is transduced that results in
28 internalization of the ligand-receptor complex into a vesicle. The LDL receptor functions in this manner.
29
30 (Choice E) Voltage-dependent activation occurs w hen presynaptic neurotransmitter-containing vesicles fuse
31 w ith the plasma membrane in response to depolarization.
32
33 Educational objective:
34 Ligand binding to a tyrosine kinase receptor results in formation of receptor dimers and a conformational
35 change that exposes tyrosine kinase active sites.
36
37 Time Spent: 59 seconds 53% answ ered correctly. Last updated: [3/27/201 O].
38 Copyright © USMLEWORLD,LLC. SimExam ver. 2011 .11.397425
1
2 Item: 27 of 46 II P Mark -<:I t>-
Q. Id : 6624 (388147] Previous Next
3
4
5
6 A 28-year-old man tells his physician that his drinking has increased over the past several months as a result
7 of his recent divorce, job loss, and mother's death. The physician responds, "So you're telling me that your
8 excessive drinking is related to recent stresses in your life?" Which of the following interviewing techniques is
9 the physician using?
10
11
A. Empathy [1 %•]
12
13 ~ ~ @ B. Reflection (65%]
14 C. Sbippert [2%•]
15
D. Confrontation (14%]
16
17 E. FaE:ilitatien [18%•]
18
19
Explanation: User Id: 388147
20
21
Reflection is an interview technique where the physician repeats what he has just been told. This technique
22
assures the patient that the physician has been listening to his concerns and allows the patient to clarify any
23
points that have been misunderstood. A similar but more comprehensive technique is summarizing, wherein
24
the physician encapsulates several ideas into one or two statements.
25
26
(Choice A) Empathy is expression of vicarious understanding of a patient's feelings and emotions. An
empathetic statement would be, "I can understand how difficult it must have been for you to face all those
28
losses in your life."
29
30
(Choice C) Support is expression of concern for a patient's experiences and well-being. "This sounds like it
31
has been very hard for you," \'lould be a supportive statement. Support does not involve vicarious
32
experiencing of a patient's situation.
33
34
(Choice D) Confrontation draws the patient's attention to discrepancies in his statements. For example, the
35
physician might confront the patient if he said that he started drinking in response to life stressors but also
36
mentioned that the last several months were not that stressful.
37
38
1
2 Item: 27 of 46 II P Mark -<:I t>-
Q. Id : 6624 (388147] Previous Next
3
4 C. Sbippert [2%•]
5
D. Confrontation [14%]
6
7 E. f':ae:ilitatien [1 gq~l
8
9
10 Explanation: User Id: 388147
11
12 Reflection is an interview technique where the physician repeats what he has just been told. This technique
13 ~
assures the patient that the physician has been listening to his concerns and allows the patient to clarify any
14 points that have been misunderstood. A similar but more comprehensive technique is summarizing, wherein
15 the physician encapsulates several ideas into one or two statements.
16
17 (Choice A) Empathy is expression of vicarious understanding of a patient's feelings and emotions. An
18 empathetic statement would be, "I can understand how difficult it must have been for you to face all those
19 losses in your life."
20
21 (Choice C) Support is expression of concern for a patient's experiences and well-being. ''This sounds like it
22 has been very hard for you," would be a supportive statement. Support does not involve vicarious
23 experiencing of a patient's situation.
24
25 (Choice D) Confrontation draws the patient's attention to discrepancies in his statements. For example, the
26 physician might confront the patient if he said that he started drinking in response to life stressors but also
mentioned that the last several months were not that stressful.
28
29 (Choice E) Facilitation is a basic interviewing technique where the interviewer encourages the patient to talk
30 more about his or her experience. ''Tell me how your drinking increased" would be an example of facilitation.
31
32 Educational objective:
33 There are numerous interview techniques that are useful in obtaining information and establishing
34 rapport. Reflection involves repeating what the patient has said in order to confirm understanding and let the
35 patient know that he is being heard.
36
37 Time Spent: 50 seconds 65% answered correctly. Last updated: [3/27/201 O].
38 Copyright © USMLEWORLD,LLC. SimExam ver. 2011 . 11.397425
1
2 Item: 28 of 46 II P Mark -<:I t>-
Q. Id : 6550 (388147] Previous Next
3
4
5
6 A 24-year-old male presents to clinic w ith chronic nasal discharge and occasional headaches. Physical
7 examination reveals atroRhic;, nasal mucosa and a thinned nasal septum. W hich of the following is the most
8 likely cause of his condition?
9
10
11 A. Aspirin intolerance [1 %]
12 X @ B. Allergic rhinitis [8%]
13 ~ C. Vasomotor rhinitis [5%]
14
15 D. Substance abuse [85%]
16 E. Hay fever [0%]
17
18
19 Explanation: User Id: 388147
20
21 Adults w ho present w ith headaches, chronic nasal discharge, atrophic nasal mucosa, and thinning of the
22 nasal septum are likely to be long-term intranasal cocaine abusers. Other respiratory tract findings
23 associated w ith the vasoconstrictive effects of intranasal cocaine abuse include nasal septum perforation,
24 oropharyngeal ulcers, and osteolytic sinusitis.
25
26 (Choice A) Aspirin intolerance is characterized by asthma, nasal polyps, and aspirin sensitivity. Nasal septal
27 thinning is not seen.

29 (Choices B and E) Allergic rhinitis presents w ith sneezing, rhinorrhea, nasal congestion, postnasal drip, and
30 pruritus of the eyes, nose, and palate. "Hay fever" is a seasonal form of allergic rhinitis provoked by the
31 pollens of specific plants (eg, trees, grasses, w eeds).
32
33 (Choice C) Vasomotor rhinitis features chronic nasal congestion that worsens w ith abrupt changes in
34 temperature or humidity or w ith exposure to odors or alcohol. Headaches, anosmia, and sinusitis are often
35 also reported.
36
37 Educational objective:
38 Chronic intranasal cocaine abuse causes several classic respiratory tract symptoms, including chronic nasal
,. ' ' ' ' '. ,. '
39
40
1 A

2 Item: 28 of 46 II P Mark -<:I t>-


Q. Id : 6550 (388147] Previous Next
3
4
5
A. Aspirin intolerance [1 %]
6
7 X @ B. Allergic rhinitis [8%]
8 C. Vasomotor rhinitis [5%]
9
D. Substance abuse [85%]
10
11 E. Hay fever [0%]
12
13 ~
14 Explanation: User Id: 388147
15
16 Adults w ho present w ith headaches, chronic nasal discharge, atrophic nasal mucosa, and thinning of the
17 nasal septum are likely to be long-term intranasal cocaine abusers. Other respiratory tract findings
18 associated w ith the vasoconstrictive effects of intranasal cocaine abuse include nasal septum perforation,
19 - oropharyngeal ulcers, and osteolytic sinusitis.
20
21 (Choice A) Aspirin intolerance is characterized by asthma, nasal polyps, and aspirin sensitivity. Nasal septal
22 thinning is not seen.
23
24 (Choices B and E) Allergic rhinitis presents w ith sneezing, rhinorrhea, nasal congestion, postnasal drip, and
25 pruritus of the eyes, nose, and palate. "Hay fever" is a seasonal form of allergic rhinitis provoked by the
26 pollens of specific plants (eg, trees, grasses, w eeds).
27
(Choice C) Vasomotor rhinitis features chronic nasal congestion that worsens w ith abrupt changes in
29 temperature or humidity or w ith exposure to odors or alcohol. Headaches, anosmia, and sinusitis are often
30 also reported.
31
32 Educational objective:
33 Chronic intranasal cocaine abuse causes several classic respiratory tract symptoms, including chronic nasal
34 discharge, atrophic nasal mucosa, thinning of the nasal septum (which can lead to perforation), oropharyngeal
35 ulcers, and osteolytic sinusitis.
36
37 Time Spent: 39 seconds 85% answ ered correctly. Last updated: [3/27/201 O].
38 Copyright © USMLEWORLD,LLC. SimExam ver. 2011.11.397425
1
2 Item: 29 of 46 II P Mark -<:I t>-
Q. Id : 6555 (388147] Previous Next
3
4
5
6 A 32-year-old male sustains a knife injury to his back and undergoes a right nephrectomy. His past medical
7 history is insignificant. W hich of the following best describes his glomerular filtration rate (GFR) compared to
8 baseline?
9 Immediately after surgery Six w eeks after surgery
10
11
12 A. 50% 50% [6%]
13 ~
14
15 " @ B. 50% 80% [42%]

16 C. 50% 100% [36%]


17
18 D. 100% 80% [4%]
19 -
20 E. 100% 100% [12%]
21
22
23
Explanation: User Id: 388147
24
25
Although heminephrectomy causes an acute reduction of the total GFR by 50%, the nephrons in the
26
remaining kidney undergo significant hemodynamic and structural adaptations to allow for the single nephron
27
GFR (SNGFR) to rise. The end result is a long-term reduction of total GFR to approximately 80% of the
28
baseline value. W ithin one week of nephrectomy in a kidney donor, the GFR in the remaining kidney has
already increased to 65-70% of the previous two-kidney value, approaching 80% by four to six w eeks
30
post-nephrectomy.
31
32
Educational objective:
33
Heminephrectomy results in compensatory hyperfiltration and hypertrophy of the nephrons in the remaining
34
kidney. Post-nephrectomy, the total GFR increases from 50% to approximately 80% of its original value w ithin
35
several w eeks.
36
37
Time Spent: 43 seconds 42% answ ered correctly. Last updated: [3/27/201 O] .
38
1
2 Item: 30 of 46 • \' Mark -<:J t:>
Q. Id: 656-0 (383147) Previous NeHt
3
4
5
6 A 56-year-old male complains of sexual dysfunction. He is able to achieve sustained erection after taking a
7 drug that inhibits the isoenzyme phosphodiesterase-5. The drug acts at which of the following areas indicated
8 on the image below?
9
10
, I,/ ..... .
• I ' • •
11 '
12 A B a
13 I"
f •
I .
''
I
14
c
15
16 • /
17
18
\ •'
19 '-:· •
20 •
21 • •
/,
• ,
22 I • • /,' •
I
23
·~

• • •
24 Dttp11ytd wtdl p-.dtsloa ft'om Spriapr Healtbcue Ltd.
25 o c.pynp. 2004 by Cumot Modldoo
26
27
28 A. Area A [14%]
29 ~ @ B. Area B (59%]
31
C. Area C [4o/o]
32 D. Area D (16%]
33 E. Area E (7%]
34
35
36 Explanation: User Id: 388147
37
38 The photomicrograph above depicts a hematoxylin- and eosin-stained transverse section of the penis. This
1 A

2 Item: 30 of 46 II P Mark -<:I t>-


Q. Id : 6560 (388147] Previous Next
3
4
A. Area A [14%]
5
6 .; @ B. Area B [59%]
7 C. Area C [4%]
8
D. Area D [1 6%]
9
10 E. Area E [7%]
11
12
13 ~ Explanation: User Id: 388147
14
15 The photomicrograph above depicts a hematoxylin- and eosin-stained transverse section of the penis. This
16 patient's erectile dysfunction resolved after he w as treated w ith a selective phosphodiesterase-5 inhibitor (eg,
17 sildenafil), w hich increases cGMP levels and mediates relaxation in the smooth muscle cells that surround the
18 cavernous venous sinuses of the corpora cavernosa ("Area B"). Through this mechanism,
19 - phosphodiesterase-5 inhibitors facilitate vascular congestion, causing the corpora to become
engorged. Local release of nitric oxide during sexual stimulation increases the activity of soluble guanylate
20
21 cyclase in the smooth muscle cells; simultaneous phosphodiesterase-5 inhibition results in increased steady
22 state intracellular cGMP, sustaining the erection.
23
24 (Choice A) Area A represents the outer margin of the tunic a albuginea surrounding the corpus cavernosum,
25 and may also represent some of the deep fascia of the penis (Buck's fascia) around the tunic a albuginea .
26
27 (Choices C and D) Area C represents part of the deep fascia of the penis (Buck's fascia) surrounding the
28 corpora cavernosa and the corpus spongiosum . Area D represents the corpus spongiosum .
29
(Choice E) Area E represents the urethral lumen w ithin the corpus spongiosum.
31
32 Educational objectives:
33 Phosphodiesterase-5 inhibitors such as sildenafil promote erection through relaxation of venous sinus smooth
34 muscle w ithin the corpora cavernosa . Be able to identify the corpora cavernosa on a transverse section of
35 the penis.
36
37 Time Spent: 30 seconds 59% answ ered correctly. Last updated: [1 0/3/2011].
38 Copyright © USMLEWORLD,LLC. SimExam ver. 2011 .11.397425
1
2 Item: 31of46 • \ ' Mark -<:J C>
Q. Id: 6564 (383147) Previous NeHt
3
4 A 12-year-old male presents with headaches vomiting and visual disturbances. A head x-ray demonstrates
5 intracranial calcifications, and a preliminary diagnosis of craniopharyngioma is made. Which of the following
6 findings on visual field examination is most consistent with the diagnosis? (black = no vision; white = vision
7 intact)
8
9
10 ~
11
Tempotll Nasal T.......
12
13 I"
14 A. ~ (1%)
15
16 ~
17 Tempotll Nasal Temporal
18
19 '°
20
B. [3o/o}
21
22
23 X @ Te~ral Nasal Temporal
24
25 c. [7%)
26
27
28
Te~ral Nasal
29
30
D. [4%)
32
33
34 TofT4l0181 Te~
35
36
37
E. [85%]
38
1 •
2 Item: 31of46 • \ ' Mark -<:J C>
Q. Id: 6564 (383147) Previous NeHt
3
4
5 Explanation: User Id: 388147
6 Nasal Fields
7 Temporal Fields
8
9
10
11
12 Left eye Right eye
13 I"
14
15
16
17
18
19 '°
20
21
22 Optic tract
23
24
25 Meyer's loop
26
27
28 Lateral geniculate body
29
30

32
33
34
35
36
37
38
39
40
1
2 Item: 31of46 ll P Mark -<:I t>-
Q. Id : 6564 (388147] Previous Next
3
4
5
6
7
8
9
10 In embryonic development, the anterior pituitary (adenohypophysis} is formed from an ectodermal diverticulum
11 arising from the oral cavity know n as Rathke's pouch. Craniopharyngiomas are suprasellar tumors arising
12 from remnants of Rathke's pouch . They classically present in the second or third decade w ith headaches,
13 ~
papilledema and signs of pituitary failure and cranial nerve dysfunction . Because these tumors apply
14 pressure on the optic chiasm, bitemporal hemianopsia may occur (Choice E). Classically, these tumors are
15 calcified and can be identified w ith a plain skull x-ray.
16
17 (Choice A) Monocular blindness results from lesions affecting only one optic nerve or retina . Temporal
18 arteritis or amaurosis fugax may be responsible.
19
20 (Choice B) Left homonymous hemianopsia w ould occur w ith lesions to the right optic tract. The optic tracts
21 are formed distal to the optic chiasm and contain fibers from the ipsilateral temporal hemiretina and
22 contralateral nasal hemiretina. They synapse on the lateral geniculate body, pretectal nucleus and superior
23 colliculus.
24
25 (Choice C) Binasal hemianopsia occurs w hen there is bilateral pressure on the optic chiasm. One instance
26 w here this has been described is w ith atherosclerosis of the bilateral intracranial portions of the internal
27 carotid arteries.
28
29 (Choice D) Right low er quadrantanopsia w ould result from a lesion of the contralateral upper
30 geniculocalcarine tract. This tract projects from the lateral geniculate body to the upper section of the
calcarine sulcus.
32
33 Educational Objective:
34 Tumors arising from the sella turcica, such as craniopharyngiomas and pituitary adenomas, apply pressure to
35 the optic chiasm resulting in bitemporal hemianopsia.
36
37 Time Spent: 73 seconds 85% answ ered correctly. Last updated: [11 / 10/2011].
38 Copyright © USMLEWORLD,LLC. SimExam ver. 2011 . 11.397425
1 A

2 Item: 32 of 46 II P Mark -<:I t>-


Q. Id : 6569 (388147] Previous Next
3
4
A 38-year-old female undergoes right-sided mastectomy for breast cancer. After the surgery, she complains
5
of difficulty raising her right arm above her head. W hen she pushes against a w all w ith outstretched arms,
6
there is prominence of the right scaQular angle. W hich of the following muscles is most likely affected?
7
8
9 A. Levator scapulae [9%]
10 B. Pectoralis minor [3%]
11
.; C. Serratus anterior [80%]
12
13 ~ x @ D. Deltoid [4%]
14 E. Trapezius [4%]
15
16
17 Explanation: User Id: 388147
18
19 - Axillary lymph node dissection, commonly a component of breast cancer surgery, can cause injury to the long
20 thoracic nerve. Penetrating wounds to the axillary region can also injure this nerve. The long thoracic nerve is
21 derived from the C5 through C7 spinal nerves and supplies the serratus anterior muscle. The serratus
22 anterior originates from the lateral portions of the first eight ribs and inserts on the medial border of the
23 scapula . It fixes the medial border of the scapula to the chest w all and also rotates the scapula to elevate the
24 glenoid fossa . Paralysis of this muscle results in an inability to raise the arm over the head and protrusion
25 ("winging") of the medial border of the scapula w hen the outstretched arm is pushed forward against
26 resistance.
27
28 (Choice A) The levator scapulae elevates the scapula and rotates the glenoid fossa inferiorly; it is innervated
29 by the dorsal scapular nerve w ith contributions from the C3 through C5 spinal nerves.
30
31 (Choice B) The pectoralis minor muscle originates from the anterior surfaces of the third through fifth ribs
and attaches to the coracoid process of the scapula . It draw s the scapula anteriorly and inferiorly against the
33 chest w all, stabilizing it. The pectoralis minor muscle is innervated by the medial pectoral nerve.
34
35 (Choice D) The deltoid is capable of flexing, medially rotating, abducting, extending and laterally rotating the
36 arm, depending on the region of the deltoid that is contracting. It is innervated by the axillary nerve.
37
38 (Choice E) The trapezius elevates and retracts the scapula . It can also rotate the scapula slightly for
1 A

2 Item: 32 of 46 II P Mark -<:I t>-


Q. Id : 6569 (388147] Previous Next
3
4 v C. Serratus anterior [80%]
5 x @ D. Deltoid [4%]
6
7 E. Trapezius [4%]
8
9
Explanation: User Id: 388147
10
11
Axillary lymph node dissection, commonly a component of breast cancer surgery, can cause injury to the long
12
thoracic nerve. Penetrating wounds to the axillary region can also injure this nerve. The long thoracic nerve is
13 ~
derived from the C5 through C7 spinal nerves and supplies the serratus anterior muscle. The serratus
14
anterior originates from the lateral portions of the first eight ribs and inserts on the medial border of the
15
scapula. It fixes the medial border of the scapula to the chest w all and also rotates the scapula to elevate the
16
glenoid fossa . Paralysis of this muscle results in an inability to raise the arm over the head and protrusion
17
('winging") of the medial border of the scapula w hen the outstretched arm is pushed forward against
18
resistance.
19 -
20
(Choice A) The levator scapulae elevates the scapula and rotates the glenoid fossa inferiorly; it is innervated
21
by the dorsal scapular nerve w ith contributions from the C3 through C5 spinal nerves.
22
23
(Choice B) The pectoralis minor muscle originates from the anterior surfaces of the third through fifth ribs
24
and attaches to the coracoid process of the scapula . It draws the scapula anteriorly and inferiorly against the
25
chest w all, stabilizing it. The pectoralis minor muscle is innervated by the medial pectoral nerve.
26
27
(Choice D) The deltoid is capable of flexing, medially rotating, abducting, extending and laterally rotating the
28
arm, depending on the region of the deltoid that is contracting. It is innervated by the axillary nerve.
29
30
(Choice E) The trapezius elevates and retracts the scapula . It can also rotate the scapula slightly for
31
movement of the glenoid fossa superiorly. The trapezius is innervated by the spinal accessory nerve (CN XI).
33
Educational objective:
34
Axillary lymph node dissection is a common cause of long thoracic nerve injury. Damage to this nerve
35
causes paralysis of the serratus anterior muscle. Clinically, affected patients are unable to raise the arm
36
laterally above the horizontal position and exhibit scapular "winging" w hen the arm is pushed anteriorly against
37
resistance.
38
1
2 Item: 33 of 46 • \> Mark -<J C>-
Q. Id: 6574 (388147) Previous Next
3
4
5
6 In animal experiments, a secretion stimulus is applied to a particular type of tissue. The constituents of the
7 secreted fluid are measured over time, and the following tracings are obtained (see the graph below).
8
9
10
11 HCo,..l-------------------
12
13 ,. c11 - - - -- -
14
15 K-t------------
16
17
nme
18
19 Secfetion Stimulus
20
21 Which of the following tissues was most likely tested?
22
23 x@ A. Small intestine [31%]
24
B. Salivary gland [15%]
25
26 ., G. i;>anGreas (48%)
27 D. Gall91a99er (6Q~)
28
E. Sweat glans (4 %)
29
30
31 Explanation: User Id: 388147
32
The two primary tissues that produce a bicarbonate-rich fluid in response to stimulation are the exocrine
34 pancreas and the epithelial cells of the biliary tract. Bicarbonate secretion in these tissues is mediated
35 predominantly by secretin, which is released into the bloodstream by S endocrine cells of the duodenal
36 mucosa in response to increased luminal acid. Secretin stimulates pancreatic ductal cells to produce an
37 alkaline, HC03- rich fluid. The fluid secreted by the pancreas is alkalinized due to the action of chloride I
38
1 A

2 Item: 33 of 46 II P Mark -<:I t>-


Q. Id: 6574 (388147] Previous Next
3
4 predominantly by secretin, w hich is released into the bloodstream by S endocrine cells of the duodenal
5 mucosa in response to increased luminal acid. Secretin stimulates pancreatic ductal cells to produce an
6 alkaline, HC0 3 - rich fluid. The fluid secreted by the pancreas is alkalinized due to the action of chloride I
7 bicarbonate ion exchange proteins in the apical surfaces of pancreatic ductal cells. Thus, as more
8 bicarbonate is added to the pancreatic secretion, more chloride w ill be removed, as pictured in the graph in
9 the question. In contrast, the potassium concentration in pancreatic exocrine secretions approximates
10 plasma potassium values in the presence or absence of secretin.
11
12 (Choice A) Small intestinal fluid secretions tend to be slightly alkaline, w ith [HC0 3 - ] exceeding [Ci-] on rare
13 ~
occasion. At high rates of net intestinal fluid secretion, such as diarrhea, [HC0 3 -] can increase from 30
14
15 mEq/L to 45 mEq/L w hile [Ci-] can fall from 100 mEq/L to 40 mEq/L. As the net rate of intestinal fluid
16 secretion increases, [K•] also rises from 5 mEq/L to 35 mEq/L, how ever.
17
18 (Choice B) Salivary gland secretions are initially similar to plasma w ith respect to their solute concentrations.
19 - W ithin the salivary ducts, Na and Cl are reabsorbed w hile K and HC03 are secreted. W hen salivary glands
20 are maximally stimulated, how ever, the concentrations of all of these ions return to values near the plasma
21 concentration due to a lack of time for the ductal cells to modify the ion concentrations. Stated differently, the
22 extent of ductal epithelial modification of acinar secretions is inversely related to the fluid rate of flow from the
23 ducts.
24
25 (Choice D) The gallbladder does not secrete bile or electrolytes. Rather, it stores bile produced in the liver
26 and reabsorbs electrolytes and w ater to concentrate the bile acids and cholesterol.
27
28 (Choice E) Sw eat, once it reaches the surface of the skin from the eccrine glands, primarily consists of
29 hypotonic NaCl w ith very little HC0 3 - . Consequently [Ci-] exceeds [HC0 3 -] in sweat.
30
31 Educational objective:
32 Pancreatic exocrine gland fluid secretion is characterized by [HC0 3 -] that exceeds [Cl"], w ith [HC0 3 - ]

34 increasing and [Ci-] decreasing as the rate of fluid secretion increases. Pancreatic fluid secretion increases in
35 response to the hormone secretin.
36
37 Time Spent: 46 seconds 48% answ ered correctly. Last updated: [11 / 10/2011].
38 Copyright © USMLEWORLD,LLC. SimExam ver. 2011 .11.397425
1
2 Item: 34 of 46 II P Mark -<:I t>-
Q. Id : 66-08 (388147] Previous Next
3
4
5
6 An 18-year-old woman has had terrifying nightmares since w itnessing a murder 3 w eeks ago. She describes
7 w aking multiple times each night in cold sweats followed by trouble falling back to sleep. She says she has
8 also experienced increased irritability, social isolation, and difficulty concentrating since this event. In fact, her
9 concentration has been so poor that she had to postpone her recent final exams. What is the most likely
10 diagnosis?
11
12
A. Adjustment disorder [3%]
13 ~
14 .; B. Acute stress disorder [50%]
15 X @ C. Post-traumatic stress disorder [46%]
16
D. Nightmare disorder [0%]
17
18 E. Major depressive disorder [0%]
19
20
21 Explanation: User Id: 388147
22
Acute stress disorder can develop following a life-threatening traumatic event (either experienced or
23
w itnessed). Symptoms include re-experiencing the event via nightmares or flashbacks, disturbed sleep, poor
24
concentration, irritability, hypervigilance and restlessness. The symptoms must cause significant distress or
25
functional impairment. Acute stress disorder does not last more than 4 w eeks.
26
27
(Choice A) Adjustment disorder is characterized by behavioral symptoms that develop w ithin 3 months of a
28
stressor and are considered excessive relative to the expected response. Some symptoms are similar to
29
those in acute stress disorder (sleep disturbances, irritability, poor concentration, etc.), but patients do not
30
typically re-experience the event via flashbacks and nightmares as in acute stress disorder. Furthermore, the
31
inciting stressors in adjustment disorder tend to be of the emotional type (e.g., divorce, moving) rather than
32
life-threatening as in acute stress disorder. Symptoms in adjustment disorder rarely last beyond 6 months of
33
the stressor.

(Choice C) Acute stress disorder and post-traumatic stress disorder (PTSD) cause identical symptoms and
are distinguished by symptom duration. Acute stress disorder becomes PTSD w hen symptoms persist more
37
than 4 w eeks.
38
1 A

2 Item: 34 of 46 II P Mark -<:I t>-


Q. Id : 66-08 (388147] Previous Next
3
4 cxp1anat1on: user 1a: .i1111141
5
6 Acute stress disorder can develop follow ing a life-threatening traumatic event (either experienced or
7 w itnessed). Symptoms include re-experiencing the event via nightmares or flashbacks, disturbed sleep, poor
8 concentration, irritability, hypervigilance and restlessness. The symptoms must cause significant distress or
9 functional impairment. Acute stress disorder does not last more than 4 w eeks.
10
11 (Choice A) Adjustment disorder is characterized by behavioral symptoms that develop w ithin 3 months of a
12 stressor and are considered excessive relative to the expected response. Some symptoms are similar to
13 ~
those in acute stress disorder (sleep disturbances, irritability, poor concentration, etc.), but patients do not
14 typically re-experience the event via flashbacks and nightmares as in acute stress disorder. Furthermore, the
15 inciting stressors in adjustment disorder tend to be of the emotional type (e.g., divorce, moving) rather than
16 life-threatening as in acute stress disorder. Symptoms in adjustment disorder rarely last beyond 6 months of
17 the stressor.
18
19 - (Choice C) Acute stress disorder and post-traumatic stress disorder (PTSD) cause identical symptoms and
are distinguished by symptom duration. Acute stress disorder becomes PTSD w hen symptoms persist more
20
21 than 4 w eeks.
22
23 (Choice D) Patients w ith nightmare disorder w ake frequently from sleep due to frightening dreams w hich they
24 remember vividly. Although there may be significant functional impairment, other elements of the history
25 described above (e.g., exposure to a traumatic event, hypervigilance) are not part of nightmare disorder.
26
27 (Choice E) Major depressive disorder and acute stress disorder have many overlapping symptoms, including
28 irritability, disturbed sleep, and poor concentration. How ever, nightmares and recent trauma are suggestive of
29 acute stress disorder, and symptoms like anhedonia and suicidal ideation w ould be more characteristic of
30 depression.
31
32 Educational objective:
33 Acute stress disorder is marked by flashbacks, hypervigilance, irritability and poor concentration follow ing a
life-threatening event. It is distinguished from PTSD by symptom duration (symptoms last less than 4 w eeks
in acute stress disorder versus more than 4 w eeks in PTSD ).

37 Time Spent: 31 seconds 50% answ ered correctly. Last updated: [3/27/201 O].
38 Copyright © USMLEWORLD,LLC. SimExam ver. 2011 .11.397425
1
2 Item: 35 of 46 II P Mark -<:I t>-
Q. Id : 6584 (388147] Previous Next
3
4
5
6 A 28-year-old w oman reports that she has been hearing the devil's voice for the past 4 months and that she
7 suspects her internal organs have been replaced by somebody else's. She has cut off all communication
8 w ith friends and family and spends most of each day alone in her room w ith her new ly purchased tarot
9 cards. Her presentation is most suggestive of:
10
11
x @ A. Schizotypal personality disorder [9%]
12
13 ~ B. Schizophrenia, paranoid type [26%]
14 C. Schizophreniform disorder [60%]
15
D. Schizoid personality disorder [6%]
16
17 E. Acute stress disorder [0%]
18
19
Explanation: User Id: 388147
20
21
Like schizophrenia, schizophreniform disorder is characterized by the presence of two or more of: delusions,
22
hallucinations, disorganized speech/behavior, or negative symptoms. These two diagnoses are distinguished
23
by the duration of the symptoms. Symptoms of schizophreniform disorder last at least 1 month but less than
24
6 months. The diagnosis of schizophrenia (Choice B) is made w hen symptoms persist beyond 6 months. A
25
brief psychotic episode is marked by similar symptoms but lasts less than 1 month.
26
27
(Choice A) Schizotypal personality disorder is a pervasive pattern of maladaptive behavior characterized by
28
29 magical thinking and odd beliefs including telepathy, superstition, bizarre fantasies, or
preoccupations. Affected individuals often lack close friends. This diagnosis is unlikely here because the
30
patient's symptoms have only existed for a brief period, and auditory hallucinations are not typically found in
31
personality disorders.
32
33
(Choice D) Individuals w ith schizoid personality disorder have no desire for close relationships w ith family,
34
friends, or others. They choose solitary activities, are emotionally detached, take little pleasure in anything,
and are unmoved by praise or criticism. They lack close friends and have a flattened affect, but psychotic
symptoms like hallucinations and delusions are absent.
37
38
1 A

2 Item: 35 of 46 II P Mark -<:I t>-


Q. Id : 6584 (388147] Previous Next
3
4 E. Acute stress disorder [0%]
5
6
7 Explanation: User Id: 388147
8
9 Like schizophrenia, schizophreniform disorder is characterized by the presence of two or more of: delusions,
10 hallucinations, disorganized speech/behavior, or negative symptoms. These two diagnoses are distinguished
11 by the duration of the symptoms. Symptoms of schizophreniform disorder last at least 1 month but less than
12 6 months. The diagnosis of schizophrenia (Choice B) is made w hen symptoms persist beyond 6 months. A
13 ~
brief psychotic episode is marked by similar symptoms but lasts less than 1 month.
14
15 (Choice A) Schizotypal personality disorder is a pervasive pattern of maladaptive behavior characterized by
16 magical thinking and odd beliefs including telepathy, superstition, bizarre fantasies, or
17 preoccupations. Affected individuals often lack close friends. This diagnosis is unlikely here because the
18 patient's symptoms have only existed for a brief period, and auditory hallucinations are not typically found in
19 - personality disorders.
20
21 (Choice D) Individuals w ith schizoid personality disorder have no desire for close relationships w ith family,
22 friends, or others. They choose solitary activities, are emotionally detached, take little pleasure in anything,
23 and are unmoved by praise or criticism. They lack close friends and have a flattened affect, but psychotic
24 symptoms like hallucinations and delusions are absent.
25
26 (Choice E) Acute stress disorder is an anxiety disorder related to posttraumatic stress disorder
27 (PTSD). Symptoms begin shortly after a traumatic event (e.g. death, threat of death or serious injury) and last
28 at least two days. Patients re-experience the trauma through dreams and flashbacks, avoid stimuli
29 associated w ith the trauma, and are hyperarousable. Symptoms lasting more than 4 w eeks fit the criteria for
30 PTSD.
31
32 Educational Objective:
33 The psychotic thought disorders share signs and symptoms but differ in duration . Brief psychotic episodes
34 last less than 1 month. Schizophreniform disorder lasts more than 1 month but less than 6
months. Psychosis in schizophrenia lasts longer than 6 months.

37 Time Spent: 45 seconds 60% answ ered correctly. Last updated: [3/27/201 O].
38 Copyright © USMLEWORLD,LLC. SimEx am ver. 2011.11.397425
1
2 Item: 36 of 46 II P Mark -<:I t>-
Q. Id : 6586 (388147] Previous Next
3
4
5
6 A new screening test for disease X is positive in 250, out of 1,000 affected patients and in apo_ouLoU ,OOO,
7 healthy controls. The screening test is then applied to a population w here the disease prevalence is
8 10,000/ 100,000. How many false positives do you expect per 100,000 individuals in this population?
9
10
11 A. 90 [3%]
12 B. 900 [12%]
13 ~ C. 1,000 [33%]
14
~ @ D. 9,000 [50%]
15
16 E. 90,000 [3%]
17
18
19 Explanation: User Id: 388147
20
21 =
Sensitivity is a test's ability to detect positive cases, and is therefore 250/ 1,000 0.25 for the screening test
22 above. Specificity is a test's ability to exclude negative cases, i.e. the proportion of identified "true negatives"
23 =
out of all subjects w ithout the disease. Here the specificity is (1,000- 100)/ 1,000 0.9. The false positive
24 ratio is (1 - the specificity), or 0.1 in this case. This means that 10% of unaffected individuals w ill test positive
25 for the disease. In a population w here the disease prevalence is 10,000/ 100,000, you should expect 90,000
26 per 100,000 individuals to be free of the disease. If the false positive ratio is 0.1, there would be 9,000 (or 0.1 x
27 90,000) false positives.
28
29 Educational objective:
30 The false positive ratio is (1 - specificity).
31
32 Time Spent: 82 seconds 50% answered correctly. Last updated: [3/27/201 OJ.
33 Copyright © USMLEWORLD,LLC. SimExam ver. 2011 . 11.397425
34
35

37
38
1
2 Item: 37 of 46 II P Mark -<:I t>-
Q. Id : 6588 (388147] Previous Next
3
4
5
6 A group of women between ages 30 and 40, all of w hom have a point mutation in fibroblast growth factor
7 receptor 2 (t_GFR2), are followed for five years. Thi[)y of the women develop breast cancer during this Reriod
8 and 65 do not. A second group of women w ithout the mutation is also follow ed for five years. [.V{~!l\yY<pmen
9 in this group develop breast cancer and iz,§.dQ.il2l) Which of the follow ing best describes the relative risk of
10 breast cancer in women w ith the FGFR2 mutation?
11
12
13 ~
A. 10 [5%]
14 B. 1.3 [16%]
15 .; @ C. 1.5 [59%]
16
17 D. 1.7 [17%]
18 E. 2.0 [4%]
19
20
21 Explanation: User Id: 388147
22
23 The relative risk (RR) is defined as the risk of developing a certain endpoint in patients w ith a specific
24 "exposure" (here, the mutation) divided by the risk in unexposed subjects. In the above scenario, the risk of
25 =
developing breast cancer in subjects w ith the FGFR2 mutation is 30/(30+65) 30/95, and the risk of
26 =
developing breast cancer in subjects w ithout the FGFR2 mutation is 20/(20+ 75) 20/95. Thus, the RR is:
27
28
RR =(30/95)/(20/95) =30/20 =1.5
29 Educational objective:
30 Relative risk is calculated as the risk in exposed subjects divided by the risk in unexposed subjects.
31
32
Time Spent: 201 seconds 59% answ ered correctly. Last updated: [3/27/201 OJ .
33
Copyright © USMLEWORLD,LLC. SimExam ver. 2011 . 11 .397425
34
35
36

38
1
2 Item: 38 of 46 II P Mark -<:I t>-
Q. Id : 6589 (388147] Previous Next
3
4
5
6 A new chemotherapy regimen is tested in 100 patients w ith lung cancer. Forty of the treated patients are still
7 alive one year after diagnosis. Conventional chemotherapy is given to a separate group of 500 patients w ith
8 lung cancer, and one year after diagnosis 100 are alive. How many patients need to be treated w ith the new
9 regimen to save one life as compared to the conventional regimen?
10
11
A . 1 [4%]
12
13 ~ B. 2 [27%]
14 X @ C. 4 [19%]
15
16
v D. 5 [47%]
17 E. 10 [2%]
18
19
20 Explanation: User Id: 388147
21
22 The number needed to treat (NNT) is a reflection of the absolute risk reduction (actually, it is the reciprocal of
23 the absolute risk reduction). In the above scenario, the mortality in the group treated w ith the novel agent is
24 =
(100-40)/ 100 0.6 and the mortality in the conventional chemotherapy group is (500-100)/500 0.8. =
25 = =
Therefore, the absolute risk reduction is 0.8-0.6 0.2, and the NNT is its reciprocal, or 1/0.2 5. This means
26 that for every five patients treated w ith the new chemotherapy drug instead of the conventional therapy, one
27 additional life is saved.
28
29 Educational objective:
30 The number needed to treat is the inverse of the absolute risk reduction.
31
32 Time Spent: 84 seconds 47% answered correctly. Last updated: [3/27/201 OJ.
33 Copyright © USMLEWORLD,LLC. SimExam ver. 2011 . 11.397425
34
35
36
37
1
2 Item: 39 of 46 II P Mark -<:I t>-
Q. Id : 6591 (388147] Previous Next
3
4
5
6 A ~year;olc!.HIV-negative male is brought to your office w ith confusion and inappropriate behavior for one
7 w eek. Today he! 13ssaulted; his co-worker because he felt he w as ::.Staring" at him. He appears agitated but
8 reports that he feels fine. He has not traveled recently. Magnetic resonance imaging show s an area of
9 il1£!eased_signal in the right temporal lobe. W hich of the follow ing organisms is most likely to be isolated from
10 the affected area?
11
12
A. Bunyavirus [4%]
13 ~
14 .; B. Herpesvirus [59%]
15 x @ C. Paramyxovirus [3%]
16
17 D. Toxoplasma [24%]
18 E. Cryptococcus [7%]
19 F. Acanthamoeba [4%]
20
21
22 Explanation: User Id: 388147
23
24 Herpes simplex encephalitis is the most common form of viral encephalitis. It is typically caused by HSV-1,
25 the same virus responsible for oral herpes simplex lesions. Symptoms of HSV encephalitis are similar to
26 those noted in other forms of encephalitis, including fever, headache, seizures, altered level of consciousness
27 and/or coma . Because HSV tends to affect the temporal lobe, there are some unique clinical findings in HSV
28 encephalitis, such as olfactory hallucinations or anosmia, temporal lobe seizures, personality change, bizarre
29 or psychotic behavior, delirium, aphasia, and/or hemiparesis. Brain imaging may show edema and
30 hemorrhage in the temporal and frontal lobes. Treatment is w ith IV acyclovir.
31
32 (Choice A) Bunyavirus encephalitis is caused by the California encephalitis group of viruses. This virus group
33 is transmitted to people by mosquitoes. Encephalitis caused by the California viruses is rarely fatal and
34 commonly causes a transient infection w ith headache and fever.
35
36 (Choice C) The paramyxoviridae include the mumps and measles viruses as w ell as respiratory syncytial
37 virus and the parainfluenza virus. Both measles and mumps may cause encephalitis; measles causes the
38 chronic encephalitis know n as subacute sclerosing panencephalitis.
1 A

2 Item: 39 of 46 II P Mark -<:I t>-


Q. Id : 6591 (388147] Previous Next
3
4 cxp1anat1on: user 1a: .i1111141
5
6 Herpes simplex encephalitis is the most common form of viral encephalitis. It is typically caused by HSV-1 ,
7 the same virus responsible for oral herpes simplex lesions. Symptoms of HSV encephalitis are similar to
8 those noted in other forms of encephalitis, including fever, headache, seizures, altered level of consciousness
9 and/or coma . Because HSV tends to affect the temporal lobe, there are some unique clinical findings in HSV
10 encephalitis, such as olfactory hallucinations or anosmia, temporal lobe seizures, personality change, bizarre
11 or psychotic behavior, delirium, aphasia, and/or hemiparesis. Brain imaging may show edema and
12 hemorrhage in the temporal and frontal lobes. Treatment is w ith IV acyclovir.
13 ~
14 (Choice A) Bunyavirus encephalitis is caused by the California encephalitis group of viruses. This virus group
15 is transmitted to people by mosquitoes. Encephalitis caused by the California viruses is rarely fatal and
16 commonly causes a transient infection w ith headache and fever.
17
18 (Choice C) The paramyxoviridae include the mumps and measles viruses as w ell as respiratory syncytial
19 - virus and the parainfluenza virus. Both measles and mumps may cause encephalitis; measles causes the
chronic encephalitis know n as subacute sclerosing panencephalitis.
20
21
22 (Choice D) Ring-enhancing brain lesions in HIV patients are commonly the result of toxoplasma infection.
23 Symptoms depend in part on the area of the brain involved.
24
25 (Choice E) Cryptococcus is a fungal organism capable of causing chronic meningitis in HIV-positive patients.
26 Symptoms include fever, headache, confusion and gait abnormalities.
27
28 (Choice F) Acanthamoeba is a protozoa! organism that causes a fatal encephalitis in AIDS patients and other
29 chronically immunosuppressed patients. After invading the CNS, masses of organisms accumulate into
30 space-occupying lesions similar to those seen in toxoplasmosis.
31
32 Educational objective:
33 HSV encephalitis is the most common viral encephalitis. Both immunocompetent and immunosuppressed
34 patients can be affected. The temporal lobe is the main region of the brain affected, so in addition to fever,
35 headache and malaise, patients may experience seizures, personality changes and psychosis.
36
37 Time Spent: 56 seconds 59% answ ered correctly. Last updated: [3/27/201 O].
38 Copyright © USMLEWORLD,LLC. SimExam ver. 2011.11 .397425
1
2 Item: 40 of 46 II P Mark -<:I t>-
Q. Id: 6612 (388147] Previous Next
3
4
5
6 A young female develops confusion, abdominal pain and muscle weakness after starting antibiotics for
7 uncomplicated cystitis. Her urine turns dark on standing. She likes to tan and does not describe any skin
8 rash associated w ith sun exposure. W hich of the follow ing conversions is most likely impaired in this patient?
9 Aminolevulinic Acid Protoporphyrin IX Herne
10 F
11
12
13 ~
l
Porphobilinogen Protoporphyrinogen IX
14
15
16
i D
17 Hydroxymethylbilane Coproporphyrinogen Ill
18
19
20
ic
Uroporphyrinogen I Uroporphyrinogen 111
21
22
23 .; A. A [39%]
24
25 X @ B. B [25%]
26 C. C [1 7%]
27 D. D [5%]
28
29 E. E [6%]
30 F. F [8%]
31
32
33 Explanation: User Id: 388147
34
35 The figure in the question illustrates the synthesis of heme. Enzymatic defects in the heme synthesis
36 pathw ay result in porphyria; defects in different enzymes cause different clinical manifestations. The case
37 illustrates typical manifestations of acute intermittent porphyria (AIP), a disease resulting from an autosomal
38 dominant defect in hydroxymethylbilane (HMB} synthase (AKA porphobilinogen deaminase and
. . . .. . . . , .
1 A

2 Item: 40 of 46 II P Mark -<:I t>-


Q. Id: 6612 (388147] Previous Next
3
lllUSll i:::lt~S l y pn;a1 r r 1ar Ill ~Sli:::IUUI IS UI i:::ICUt~ Ull~I r r llll~l ll p u r pr 1y r ICI \All}, i:::I UIS~i:::IS~ I ~SUllU 1y I r ur r I i:::ll 1 i:::IUlUSUI r ICU
4
5 dominant defect in hydroxymethylbilane (HMB) synthase (AKA porphobilinogen deaminase and
6 uroporphyrinogen I synthetase) that causes excessive levels of porphobilinogen and aminolevulinic acid in the
7 urine. Many patients w ith this condition may never experience symptoms. W hen symptoms do occur, they
8 typically do not begin until after puberty and usually follow consumption of drugs that increase heme
9 biosynthesis. Typical clinical findings during an acute attack of AIP include severe vague abdominal pain,
10 nausea and vomiting and motor/sensory neuropathy. Cutaneous findings and photosensitivity are absent and
11 patients are completely symptom-free between attacks. Urine from an affected patient during an attack w ill
12 classically darken during standing w ith exposure to sunlight.
13 ~
14 The enzymes of heme synthesis are located both in the cytosol and the mitochondria. Enzymes A and B are
15 hydroxymethylbilane (HMB) synthase and uroporphyrinogen Ill (URO Ill) synthase, respectively. As discussed,
16 a deficiency of HMB synthase results in AIP. URO Ill synthase deficiency results in congenital erythropoietic
17 porphyria, a severely mutilating porphyria characterized by photosensitivity beginning in infancy, erythrodontia
18 and hemolytic anemia . Enzyme C is URO decarboxylase, deficiency of w hich results in porphyria cutanea
19 - tarda (PCT, the most common form of porphyria). PCT can be congenital or acquired by exposure to
aromatic hydrocarbons, estrogen supplementation or hepatitis C. Cutaneous photosensitivity is the cardinal
20
21 manifestation.
22
23 Coproporphyrinogen Ill oxidase is enzyme D. Deficiency of this enzymes results in hereditary coproporphyria
24 (HCT). The type of porphyria has systemic manifestations similar to AIP together w ith the photosensitivity
25 seen in PCT. Deficiency of enzyme E, protoporphyrinogen oxidase, results in variegate porphyria (VP). The
26 clinical manifestations of VP are similar to HCT; how ever, cutaneous photosensitivity is more prominent than
27 other symptoms. Enzyme F is ferrochelatase, w hich is the last enzyme in the heme synthesis pathway;
28 deficiency of this enzyme results in erythropoietic protoporphyria characterized by cutaneous photosensitivity
29 beginning in early childhood. The activity of ferrochelatase is also decreased in lead intoxication.
30
31 Educational objective:
32 Acute intermittent porphyria is caused by deficiency of HMB synthase and is characterized by intermittent
33 episodes of abdominal pain w ith neurological manifestations after exposure to an offending medication. Urine
34 from patients during attacks w ill classically darken upon standing in sunlight. Skin photosensitivity is
35 characteristically absent.
36
37 Time Spent: 62 seconds 39% answ ered correctly. Last updated: [11 / 10/2011].
38 Copyright © USMLEWORLD,LLC. SimExam ver. 2011 .11.397425
1
2 Item: 41 of 46 • \' Mark -<:J C>
Q. Id: 6605 (383147) Previous NeHt
3
4
5
6 A 30-year-old male is being evaluated for difficulty swallowing. Physical examination reveals facial and tongue
7 fasciculations. His family pedigree with respect to this dis order is depicted below.
8
9 50
10
11
12
13 I"
14
15
16 37
17
18 30
19
20 Which of the following best explains the pedigree findings?
21
22
A Pleiotropy (2%I
23
24 B. Variable expressivity (5%]
25 C. Incomplete penetrance (11 %]
26
X @ D. Genetic heterogeneity (8%]
27
28 E. Genetic linkage (1 6%]
29 F. Polyploidy (1o/o]
30
G. Ant1c1pat1on (55%]
31
32
33 Explanation: User Id: 388147
34
35 This pedigree depicts an X-linked recessive disease that manifests in successive generations at earlier and
36 earlier ages, a finding known as genetic anticipation. Anticipation is associated with diseases caused by
37 expansions of trinucleotide repeat sequences, because the repeat expands with successive generations
38 causina earlier disease onset. All known trinucleotide reoeat diseases cause neuroloaic svmotoms. The
39
1 A

2 Item: 41 of 46 II P Mark -<:I t>-


Q. Id : 6605 (388147] Previous Next
3
4 Iv G. Anticipation [55%]
5
6
7 Explanation: User Id: 388147
8
9 This pedigree depicts an X-linked recessive disease that manifests in successive generations at earlier and
10 earlier ages, a finding know n as genetic anticipation. Anticipation is associated w ith diseases caused by
11 expansions of trinucleotide repeat sequences, because the repeat expands w ith successive generations
12 causing earlier disease onset. All know n trinucleotide repeat diseases cause neurologic symptoms. The
13 ~
patient described exhibits symptoms and a pedigree consistent w ith X-linked bulbospinal muscular atrophy
14 (Kennedy disease).
15
16 (Choice A) Pleiotropy refers to the ability of a single genetic defect to have multiple simultaneous phenotypic
17 effects.
18
19 - (Choice B) Expressivity is variability in phenotypic severity among individuals w ith the same genetic mutation.
20
21 (Choice C) Penetrance refers to the proportion of individuals w ith a given genotype that express the
22 associated phenotype.
23
24 (Choice D) Genetic heterogeneity exists w hen mutations of different genes cause similar phenotypes.
25
26 (Choice E) Genetic linkage describes alleles that tend to be inherited jointly, usually because they are located
27 near one another on the same strand of DNA.
28
29 (Choice F) Polyploidy occurs w hen more than two complete sets of homologous chromosomes exist w ithin
30 an organism or cell. An example is triploidy (3n).
31
32 Educational objective:
33 Anticipation is the manifestation of an inherited disease at younger and younger ages in successive
34 generations. It is a phenomenon associated w ith trinucleotide repeat sequences. Such diseases commonly
35 exhibit neurologic symptoms.
36
37 Time Spent: 55 seconds 55% answered correctly. Last updated: [11 / 10/2011].
38 Copyright © USMLEWORLD,LLC. SimExam ver. 2011 .11.397425
1
2 Item: 42 of 46 II P Mark -<:I t>-
3 a. Id : 66-06 (388147] Previous Next

4
5 A 21-year-old female presents to your office in her 81" w eek of gestation for prenatal counseling. She has no
6 significant past medical or family history. Her husband suffers from recurrent hemarthroses, and his uncle
7 died due to hemorrhage following a tooth extraction. W hich of the follow ing best describes their child's risk of
8 suffering from this condition?
9
10 A. 75% [2%]
11
12 x @ B. 50% [23%]
13 ~ C. 25% [20%]
14
15 D. 10% [1 %]
16 E. 0% [55%]
17
18
19 Explanation: User Id: 388147
20
21 The father here most likely suffers from hemophilia. Hemophilia A and hemophilia B have similar clinical
22 features, but hemophilia A (classic hemophilia) results from factor VIII deficiency, w hile hemophilia B
23 (Christmas disease) is caused by factor IX deficiency. Both diseases are inherited in an X-linked recessive
24 manner. X-linked recessive disorders affect males almost exclusively, except for rare instances of females
25 w ith only one X-chromosome active, as in Turner syndrome. In X-linked recessive disorders, the children of
26 affected fathers and noncarrier mothers w ill not be affected by the disease, though all female offspring w ill be
27 carriers. Thus, the fetus above has no chance of being affected by the disease regardless of gender, but if
28 the child is female, she w ill be a carrier. Alternatively, if this had been a mating of a carrier female and an
29 unaffected male, half of the male children would be affected and half of the female children would be carriers.
30
31 (Choice A) For an autosomal dominant disorder, if two heterozygotes mate, 75% of the offspring w ill be
32 affected by the disease.
33
34 (Choice B) For an autosomal dominant disorder w here just one parent is a heterozygote for the disease, 50%
35 of the offspring w ill be affected.
36
37 (Choice C) For an autosomal recessive disorder w here both parents are heterozygotes for the disease, 25%
38 of the offsorina w ill be affected.
39
40
1 A

2 Item: 42 of 46 II P Mark -<:I t>-


Q. Id : 66-06 (388147] Previous Next
3
4 x @ B. 50% [23%]
5
6 C. 25% [20%]
7 D. 10% [1 %]
8
9 E. 0% [55%]
10
11
12 Explanation: User Id: 388147
13 ~
14 The father here most likely suffers from hemophilia. Hemophilia A and hemophilia B have similar clinical
15 features, but hemophilia A (classic hemophilia) results from factor VIII deficiency, w hile hemophilia B
16 (Christmas disease) is caused by factor IX deficiency. Both diseases are inherited in an X-linked recessive
17 manner. X-linked recessive disorders affect males almost exclusively, except for rare instances of females
18 w ith only one X-chromosome active, as in Turner syndrome. In X-linked recessive disorders, the children of
19 - affected fathers and noncarrier mothers w ill not be affected by the disease, though all female offspring w ill be
carriers. Thus, the fetus above has no chance of being affected by the disease regardless of gender, but if
20
21 the child is female, she w ill be a carrier. Alternatively, if this had been a mating of a carrier female and an
22 unaffected male, half of the male children w ould be affected and half of the female children w ould be carriers.
23
24 (Choice A) For an autosomal dominant disorder, if two heterozygotes mate, 75% of the offspring w ill be
25 affected by the disease.
26
27 (Choice B) For an autosomal dominant disorder w here just one parent is a heterozygote for the disease, 50%
28 of the offspring w ill be affected.
29
30 (Choice C) For an autosomal recessive disorder w here both parents are heterozygotes for the disease, 25%
31 of the offspring w ill be affected.
32
33 Educational Objective:
34 In X-linked recessive disorders, the children of affected fathers and noncarrier mothers w ill not be affected by
35 the disease, though all of the female children w ill be carriers.
36
37 Time Spent: 54 seconds 55% answ ered correctly. Last updated: [3/27/201 OJ.
38 Copyright © USMLEWORLD,LLC. SimEx am ver. 2011.11.397425
1
2 Item: 43 of 46 II P Mark -<:I t>-
Q. Id : 7238 (388147] Previous Next
3
4
5 The following vignette applies to the next 2 items. The items in the set must be answered in sequential order.
6 Once you click Proceed to Next Item, you will not be able to add or change an answer.
7
8
A 52-year-old male presents to your office with a two week history of sllor:tnes§..of breath. He has noticed
9
decreased exercise tolerance thel.!imits.bi§.daily,.i!ctjl(ities. He also complains of a dry cough. A DopJJ.ler
10
probe placed at the cardiac apex reveals the following blood flow pattern across this patient's mitral valve (see
11
the slide below).
12
13 ~
14
15
16
17
18 Toward the
19 probe
20
21
22 Away from
23 the probe
24
25
26
27 Time
28
29 Item 1 of 2
30 The point marked by the arrow best corresponds to which of the following?
31
32
A. First heart sound [32%]
33
34 X @ B. Second heart sound [22%]
35 .; C. Third heart sound [28%]
36
D. Fourth heart sound [6%]
37
38 E. Mid-systolic click [11 %]
1
2 Item: 43 of 46 II P Mark -<:I t>-
Q. Id : 7238 (388147] Previous Next
3
4
5
A. First heart sound [32%]
6
7 X @ B. Second heart sound [22%]
8 ~ C. Third heart sound [28%]
9
D. Fourth heart sound [6%]
10
11 E. Mid-systolic click [11 %]
12
13 ~
14 Explanation: User Id: 388147
15
The third heart sound is a low -pitched sound heard during rapid filling of the ventricle in mid-diastole. It most
16
likely results from sudden deceleration of blood as the elastic limits of the ventricular chamber are reached
17
during rapid ventricular filling (which is indicated by the arrow in the above diagram). A third heart sound is
18
normal in children, but in adults, typically indicates ventricular failure. The increased end-systolic volumes and
19
pressures present in patients w ith heart failure are the reason that third heart sounds are heard in these
20
patients.
21
22 (Choice A) The first heart sound is the result of audible closure of the atrioventricular valves at the beginning
23 of systole. Blood w ould not be flow ing through the mitral valve tow ard the probe at this point.
24
25 (Choice B) The second heart sound is the result of closure of the semilunar (aortic and pulmonic) valves at
26 the end of systole. Follow ing this sound is the period of isovolumetric relaxation of the ventricle prior to
27 opening of the atrioventricular valves. No blood w ould cross the mitral valve at this point.
28
29 (Choice D) Presence of a fourth heart sound is typically pathologic. It is the result of forceful atrial contraction
30 pushing blood into an overfilled or stiffened ventricle. The period of atrial contraction is indicated by the
31 smaller curve to the right of the curve indicated by the arrow above.
32
33 (Choice E) A mid-systolic click is associated w ith mitral valve prolapse. In this setting there may be some
34 blood flow away from the probe across the mitral valve during systole.
35
36 Educational objective:
37 The third heart sound is the result of blood turbulence w ithin the ventricle during the period of rapid diastolic
38 filling . In adults it occurs most commonly in the setting of cardiac failure (either systolic of diastolic failure).
1
2 Item: 44 of 46 II P Mark -<:I t>-
Q. Id : 7239 (388147] Previous Next
3
4
5
6 Item 2 of 2
7 On physical examination, there are decreased breath sounds at the rigt)Uupg_lias~iwith.dulloess to
8 percussion . Thoracentesis is performed and t§.literp_of pleureUluict;are removed from the right pleural
9 cavity. W hich of the follow ing pleural fluid findings is most consistent w ith congestive heart failure as the
10 cause of the effusion in this patient?
11
12
13 ~
A. High protein content [30%]
X @
14
15
B. High amylase content [4%]
16
17
18 C. Low LOH content [46%]
19
20
21 0 . Low glucose content [17%]
22
23
24 E. High w hite blood cell count [3%]
25
26
27
28
29 Explanation: User Id: 388147
30
31 Congestive heart failure causes transudative pleural effusions. This is in contrast to exudative pleural
32 effusions, w hich result from infection, malignancy, or a variety of other factors. A transudative effusion is the
33 result of increased intravascular pressure in the pulmonary circulation as a result of backup of blood flow
34 through the failing left ventricle. This increased pressure leads to leakage of fluid from the pulmonary
35 capillaries. W hen assessing a pleural effusion, the fluid should be sent for the follow ing tests: protein,
36 glucose, LOH, cell count/cytology, Gram stain and culture. Exudates w ill demonstrate pleural fluid
37 protein/serum protein > 0.5, pleural fluid LOH/serum LOH > 0.6 or a pleural fluid LOH more than two-thirds the
38 upper limit of normal serum LOH. Transudates w ill lack these features.
1
2 Item: 44 of 46 II P Mark -<:I t>-
Q. Id : 7239 (388147] Previous Next
3
4
5 0 . Low glucose content [17%]
6
7
8 E. High w hite blood cell count [3%]
9
10
11
12
Explanation: User Id: 388147
13 ~
14
Congestive heart failure causes transudative pleural effusions. This is in contrast to exudative pleural
15
effusions, w hich result from infection, malignancy, or a variety of other factors. A transudative effusion is the
16
result of increased intravascular pressure in the pulmonary circulation as a result of backup of blood flow
17
through the failing left ventricle. This increased pressure leads to leakage of fluid from the pulmonary
18
capillaries. When assessing a pleural effusion, the fluid should be sent for the follow ing tests: protein,
19
glucose, LOH, cell count/cytology, Gram stain and culture. Exudates w ill demonstrate pleural fluid
20
protein/serum protein > 0.5, pleural fluid LOH/serum LOH > 0.6 or a pleural fluid LOH more than two-thirds the
21
upper limit of normal serum LOH. Transudates w ill lack these features.
22
23 (Choice A) High protein content is consistent w ith an exudative pleural effusion as described above.
24
25 (Choice B) High amylase content is suggestive of pancreatitis, pancreatic pseudocyst, esophageal rupture or
26 lung adenocarcinoma .
27
28 (Choice D) Low glucose content is seen in the setting of malignancy or infection.
29
30 (Choice E) A high w hite blood cell count may be indicative of an empyema .
31
32 Educational objective:
33 Exudates w ill demonstrate pleural fluid protein/serum protein > 0.5, pleural fluid LOH/serum LOH > 0.6 or a
34 pleural fluid LOH more than two-thirds the upper limit of normal serum LOH. Transudates w ill lack these
35 features.
36
37 Time Spent: 52 seconds 46% answ ered correctly. Last updated: [1 /8/2012].
38 Copyright © USMLEWORLD,LLC. SimExam ver. 2011 . 11.397425
1
2 Item: 45 of 46 • \' Mark -<:J C>
Q. Id: 7216 (383147) Previous NeHt
3
4
5
6
7 The following vignette applies to the next 2 items. The items in the set must be answered in sequential order.
8 Once you click Proceed to Next Item, you will not be able to add or change an answer.
9
10
A blood sample is taken from a 10- ear-olct.Patient with jaundice. It is tested for hemolysis in buffered salt
11
solutions at different salt concentrations, and the results are plotted on the graph below:
12
13 I"
14
15
16
17
18
"'
·~
0
19 E
Ql
20 .i::
21 ~
22
23
24 NaCl concentration
25
26
Item 1 of 2
27
Which of the following would you expect most on this patient's blood smear?
28
29
30 ~ A. Dense red blood cells without central pallor [55%]
31 X @ B. Fragmented red blood cells [12%]
32
C. ' Bull's eye" appearing red blood cells [9%]
33
34 D. Red blood cells with multiple surface projections [5%]
35 E. Sickle cells [1 6%]
36
F. Normal appearing red blood cells [3%]
37
38
1
2 Item: 45 of 46 II P Mark -<:I t>-
Q. Id : 7216 (388147] Previous Next
3
4
E . Sickle cells [1 6%]
5
6 F. Normal appearing red blood cells [3%]
7
8
Explanation: User Id: 388147
9
10
The osmotic fragility test (which is the name of the test described in this vignette) is the primary diagnostic
11
test for hereditary spherocytosis. In this test, blood samples are exposed to decreasing strengths of saline
12
solution and the degree of hemolysis is measured at each concentration. The red blood cells in patients w ith
13 ~
hereditary spherocytosis reveal an increased susceptibility (i.e. membrane lysis) to hypotonic saline
14
compared w ith normal controls, as can be seen in the graph above. On a peripheral smear, a characteristic
15
finding is dense, spherical erythrocytes w ith absent or diminished central pallor. The mean corpuscular
16
hemoglobin concentration (MCHC) is often elevated, and this is practically the only disease process that w ill
17
cause this abnormality.
18
19 (Choice B) Fragmented red blood cells occur in conditions that cause microangiopathic hemolytic anemia
20 (MAHA) such as thrombotic thrombocytopenic purpura, hemolytic-uremic syndrome, disseminated
21 intravascular coagulation, and prosthetic valve hemolysis.
22
23 (Choice C) "Bull's eye" appearing red blood cells describe target cells (also know n as codocytes), w hich can
24 be seen in patients w ith liver disease and thalassemia as w ell as those w ho have had a splenectomy.
25
26 (Choice D) Red blood cells w ith multiple surface projections (know n as burr cells or echinocytes) can occur
27 in patients w ith uremia or pyruvate kinase deficiency.
28
29 (Choice E) Sickle cells occur in sickle cell disease. In this process, mechanical fragility is increased due to
30 the tendency of sickle cells to deform and become rigid at low oxygen tensions. How ever, the osmotic fragility
31 of sickle cells is decreased, not increased as in hereditary spherocytosis.
32
33 Educational objective:
34 Osmotic fragility is increased in the red blood cells of patients w ith hereditary spherocytosis, and the
35 peripheral blood smear characteristically show s dense spherocytes w ithout central pallor.
36
37 Time Spent: 59 seconds 55% answ ered correctly. Last updated: [1 /22/2012].
38 Copyright © USMLEWORLD,LLC. SimExam ver. 2011 . 11.397425
1
2 Item: 46 of 46 II P Mark -<:I t>-
3 a. Id : 7217 (388147] Previous Next

4
5
6 Item 2 of 2
7 Physical examination of this patient show s splenomegaly. W hich of the following is most likely to be
8 prevented by performing a splenectomy in this patient?
9
10
11 x @ A. Liver cirrhosis [9%]
12
~ B. Gallstones [44 %]
13 ~
14
C. Recurrent infections [15%]
15
16 D. Arterial thrombosis [29%]
17
18 E. Malignancy [3%]
19
20
21
22 Explanation: User Id: 388147
23
24 Hereditary spherocytosis is a common form of hemolytic anemia. The hemolysis in this process occurs
25 primarily in the spleen, w here the abnormal spherocytes are quickly identified and removed from the
26 circulation. Splenectomy is the cornerstone of treatment for moderate to severe forms of this disease, as it
27 w ill significantly decrease hemolysis, thereby leading to increased hemoglobin, decreased jaundice, and a
28 decreased incidence of bilirubin gallstones. A splenectomy should only be performed after four years of age
29 and only after a patient has received both the pneumococcal and H. influenzae vaccines,as asplenic patients
30 are at risk for severe infections caused by encapsulated organisms.
31
(Choice A) Liver cirrhosis is a common cause of splenomegaly due to elevated splenic venous pressure in
32
the setting of portal hypertension. Cirrhosis, how ever, is not a known complication of hereditary spherocytosis.
33
34
(Choice C) Recurrent infections by encapsulated organisms is a potentially severe complication seen in
35 asplenic patients as described above.
36
37 (Choice D) Thrombotic events can occur in patients w ith hereditary spherocytosis, but the risk is increased
38 followina a solenectomv. not decreased.
39
40
1
2 Item: 46 of 46 II P Mark -<:I t>-
Q. Id : 7217 (388147] Previous Next
3
4 D. Arterial thrombosis [29%]
5
6 E. Malignancy [3%]
7
8
9
Explanation: User Id: 388147
10
11
Hereditary spherocytosis is a common form of hemolytic anemia . The hemolysis in this process occurs
12
primarily in the spleen, w here the abnormal spherocytes are quickly identified and removed from the
13 ~
circulation . Splenectomy is the cornerstone of treatment for moderate to severe forms of this disease, as it
14
w ill significantly decrease hemolysis, thereby leading to increased hemoglobin, decreased jaundice, and a
15
decreased incidence of bilirubin gallstones. A splenectomy should only be performed after four years of age
16
and only after a patient has received both the pneumococcal and H. influenzae vaccines,as asplenic patients
17
are at risk for severe infections caused by encapsulated organisms.
18
19 (Choice A) Liver cirrhosis is a common cause of splenomegaly due to elevated splenic venous pressure in
20 the setting of portal hypertension . Cirrhosis, how ever, is not a know n complication of hereditary spherocytosis.
21
22 (Choice C) Recurrent infections by encapsulated organisms is a potentially severe complication seen in
23 asplenic patients as described above.
24
25 (Choice D) Thrombotic events can occur in patients w ith hereditary spherocytosis, but the risk is increased
26 follow ing a splenectomy, not decreased.
27
28 (Choice E) Hematologic malignancy may occur in patients w ith hereditary spherocytosis due to chronic
29 stimulation of the marrow and reticuloendothelial system . How ever, this is not a w ell-substantiated
30 complication and is not necessarily improved follow ing a splenectomy.
31
32 Educational objective:
33 The spleen is the primary site for hemolysis in patients w ith hereditary spherocytosis, and a splenectomy is
34 often the treatment of choice because it results in an increased hemoglobin, decreased hemolysis, decreased
35 jaundice, and reduced incidence of bilirubin gallstones.
36
37 Time Spent: 54 seconds 44% answ ered correctly. Last updated: [1 /8/2012].
38 Copyright © USMLEWORLD,LLC. SimExam ver. 2011 . 11.397425
2 Item: 1 of 46 II P Mark -<:I t>-
Q. Id : 6427 (388147) Previous Next
3
4
A 67-year-old Caucasian male is brought to the emergency department complaining of confusion, irritability,
5
and a shock-like sensation that radiates to his feet on neck flexion . He has a long history of alsoJ:!ol abuse and
6
has been hospitalized for alcohol intoxication and acute pancreatitis. At presentation, his b lood glucose:.,l~yeU
7
i§..52 mg/dll. Tw o days later, the j;!atient feels better. His only complaint is some numbness of his feet.
8
Physical examination reveals dee eas~.bilatereLYjbr13tory_sepsation over the feet and ankles. Muscle
9
strength is preserved. W hich of the follow ing is the most likely cause of this patient's current complaints?
10
11
12 A. Multiple sclerosis [3%]
13 8 . Guillain-8arre syndrome [4% ]
14
., C. Cobalamin deficiency [76%]
15
16 x @ D. Diabetes mellitus, type 2 [1 2%]
17 E. Sensory stroke [5%]
18
19 -
20 Explanation: User Id: 388147
21
22 Nutritional deficiencies are common in alcoholics and the elderly, w ith cobalamin (vitamin 812) and folate
23 malabsorption of particular concern in this patient population. W hile vitamin 8 12 stores are gradually depleted
24 over a period of years, folate stores and absorption are affected by alcohol abuse w ithin a period of w eeks.
25 W hile the treatment of vitamin 8 12 deficiency w ith folate may improve hematologic abnormalities such as a
26 megaloblastic anemia, it w ill not reverse any neurologic dysfunction caused by the vitamin 8 12 deficiency. In
27 fact, the usage of folate alone can w orsen demyelination and cause abnormal m yelin synthesis. The low er
28 extremity numbness and decreased sensation seen in this patient could therefore have been precipitated by
29 the administration of folate alone.
30
31 (Choice A) Multiple sclerosis is characterized by numerous neurologic abnormalities, including Lhermitte's
32 phenomenon (a shock-like sensation radiating to the feet w ith neck flexion) and some cognitive dysfunction .
33 How ever, cobalamin deficiency can cause both of those signs as w ell, and this patient's history suggests that
34 nutritional deficiency is more likely than multiple sclerosis.
35
36 (Choice B) Guillain-8arre syndrome is an acute postinfectious polyneuropathy characterized by nonspecific
37 prodromal malaise and nausea follow ed by progressive, ascending paralysis.
38
2 Item: 1 of 46 II P Mark -<:I t>-
Q. Id : 6427 (388147) Previous Next
3
4 t:. ;:,ensory stroKe l o 7oJ
5
6
Explanation: User Id: 388147
7
8
Nutritional deficiencies are common in alcoholics and the elderly, w ith cobalamin (vitamin 812) and folate
9
malabsorption of particular concern in this patient population. W hile vitamin 8 12 stores are gradually depleted
10
over a period of years, folate stores and absorption are affected by alcohol abuse w ithin a period of w eeks.
11
W hile the treatment of vitamin 8 12 deficiency w ith folate may improve hematologic abnormalities such as a
12
megaloblastic anemia, it w ill not reverse any neurologic dysfunction caused by the vitamin 8 12 deficiency. In
13
fact, the usage of folate alone can w orsen demyelination and cause abnormal m yelin synthesis. The low er
14
extremity numbness and decreased sensation seen in this patient could therefore have been precipitated by
15
the administration of folate alone.
16
17 (Choice A) Multiple sclerosis is characterized by numerous neurologic abnormalities, including Lhermitte's
18 phenomenon (a shock-like sensation radiating to the feet w ith neck flexion) and some cognitive dysfunction.
19 - How ever, cobalamin deficiency can cause both of those signs as w ell, and this patient's history suggests that
20 nutritional deficiency is more likely than multiple sclerosis.
21
22 (Choice B) Guillain-8arre syndrome is an acute postinfectious polyneuropathy characterized by nonspecific
23 prodromal malaise and nausea follow ed by progressive, ascending paralysis.
24
25 (Choice D) Diabetes mellitus is associated w ith metabolic nerve damage that results in a symmetrical
26 sensory neuropathy that initially affects the low er extremities. How ever, this patient presented w ith
27 hypoglycemia rather than hyperglycemia, w hich reduces the likelihood that he has diabetes mellitus type 2.
28
29 (Choice E) Sensory stroke is characterized by numbness of the face, arm, and leg on one side of the body
30 w ithout an accompanying motor deficit.
31
32 Educational objective:
33 Cobalamin (vitamin 8 12) and folate deficiencies are common in elderly alcoholics. Cobalamin deficiency is
34 associated w ith neurologic and hematologic abnormalities, w hereas folate deficiency is associated w ith
35 hematologic abnormalities alone.
36
37 Time Spent: 70 seconds 76% answ ered correctly. Last updated: [1 /8/2012].
38 Copyright © USMLEWORLD,LLC. SimExam ver. 2011.11.397425
Item: 2 of 46 II P Mark -<:I t>-
Q. Id : 6429 (388147) Previous Next
3
4
A 31-year-old male complains of such diffculty in chew ing that he no longer buys gum. He also describes
5
diJ'.)IOJ'.)ia w hen reading or w atching television for long periods. He has no extremity w eakness. He has no
6
other medical problems and takes no medication. He does not use tobacco, alcohol or illicit drugs. This
7
patient's condition is most likely associated w ith w hich of the follow ing neoplastic processes?
8
9
10 /\. /\Elrenal tblmer [Hlq~]
11 B. Hodgkin lymphoma [9%]
12
.; @ C. Thyn1oma [63%]
13
14 D. Lllng cancer [11 q~l
15 E. ~ypernephrema [2q~]
16
17
18 Explanation: User Id: 388147
19 -
20 Myasthenia gravis is an acquired autoimmune condition characterized by fluctuating w eakness of the facial,
21 periocular, bulbar, and pelvic girdle muscles. Ptosis, diplopia, dysarthria, dysphagia, and fatigable chew ing
22 are common presenting signs and symptoms. Pathophysiologically, an antibody-mediated, T cell-dependent
23 attack on the postsynaptic neuromuscular junction is responsible for the w eakness associated w ith this
24 condition. Approximately 60-70% of patients w ith m yasthenia gravis have hyperplastic thymic tissue, and an
25 additional 10-15% have thymomas. Complete removal of the thymus is typically of significant benefit in this
26 patient population.
27
28 (Choice A) Adrenal tumors include adrenal adenomas, adrenocortical carcinomas, pheochromocytomas, and
29 metastases. Myasthenia gravis is not associated w ith adrenal tumors.
30
31 (Choice B) Hodgkin lymphoma is linked w ith several paraneoplastic syndromes, such as cholestatic liver
32 disease, alcohol-induced pain, skin lesions, neurologic syndromes, and nephrotic syndrome. Myasthenia
33 gravis is not associated w ith Hodgkin lymphoma.
34
(Choice D) Eaten-Lambert syndrome is an autoimmune disorder of the neuromuscular junction seen
35
primarily in patients w ith small cell lung cancer. It is much less common than m yasthenia gravis, and tends to
36
present more w ith extremity w eakness as opposed to w eakness involving the eyes or bulbar musculature.
37
Also, lung cancer w ould be unlikely in a young patient w ith no known smoking history.
38
Item: 2 of 46 II P Mark -<:I t>-
Q. Id : 6429 (388147) Previous Next
3
4 .; @ C. Thymoma (63%]
5
6 D. Lllng cancer [11 %•]
7 E. ~yflerneflhrema [2q~]
8
9
10 Explanation: User Id: 388147
11
12 Myasthenia gravis is an acquired autoimmune condition characterized by fluctuating w eakness of the facial,
13 periocular, bulbar, and pelvic girdle muscles. Ptosis, diplopia, dysarthria, dysphagia, and fatigable chew ing
14 are common presenting signs and symptoms. Pathophysiologically, an antibody-mediated, T cell-dependent
15 attack on the postsynaptic neuromuscular junction is responsible for the w eakness associated w ith this
16 condition. Approximately 60-70% of patients w ith m yasthenia gravis have hyperplastic thymic tissue, and an
17 additional 10-15% have thymomas. Complete removal of the thymus is typically of significant benefit in this
18 patient population .
19 - (Choice A) Adrenal tumors include adrenal adenomas, adrenocortical carcinomas, pheochromocytomas, and
20
metastases. Myasthenia gravis is not associated w ith adrenal tumors.
21
22 (Choice B) Hodgkin lymphoma is linked w ith several paraneoplastic syndromes, such as cholestatic liver
23 disease, alcohol-induced pain, skin lesions, neurologic syndromes, and nephrotic syndrome. Myasthenia
24 gravis is not associated w ith Hodgkin lymphoma.
25
26 (Choice D) Eaten-Lambert syndrome is an autoimmune disorder of the neuromuscular junction seen
27 primarily in patients w ith small cell lung cancer. It is much less common than m yasthenia gravis, and tends to
28 present more w ith extremity w eakness as opposed to w eakness involving the eyes or bulbar musculature.
29 Also, lung cancer w ould be unlikely in a young patient w ith no know n smoking history.
30
31 (Choice E) Hypernephroma (renal cell carcinoma) is linked w ith several paraneoplastic syndromes, such as
32 anemia, hepatic dysfunction, fever, hypercalcemia, cachexia, erythrocytosis, amyloidosis, thrombocytosis,
33 and polym yalgia rheumatica . Myasthenia gravis is not associated w ith renal cell carcinoma .
34
35 Educational objective:
36 Myasthenia gravis is an autoimmune disease that affects the postsynaptic acetylcholine receptors of the
37 neuromuscular junction and presents w ith w eakness of the eye and bulbar musculature that w orsens w ith
38 prolonged activity. There is a common association of m yasthenia gravis w ith thymoma.
Item: 3 of 46 II P Mark -<:I t>-
Q. Id : 6607 (388147) Previous Next

4
5
6 A young patient presents complaining of knee pain and leg sw elling and is diagnosed w ith 6 steosarcoma. His
7 past medical history is significant for enucleation of his right eye years ago due to retinoblastoma. W hich of
8 the following best explains his condition?
9
10
x @ A. Defective DNA repair [40%]
11
12 .; B. Loss of heterogeneity [48%]
13 C. Polyploidy [11 %]
14
D. Parental imprinting [0%]
15
16 E. Trinucleotide expansion [1 %]
17
18
19 - Explanation: User Id: 388147
20
This patient most likely has a mutation in the retinoblastoma gene (RB 1), a tumor suppressor gene found on
21
chromosome 13 that, w hen defective, predisposes to retinoblastoma . Two copies of the RB 1 gene exist in
22
the human genome, one on each chromosome 13. Both copies of the RB 1 tumor suppressor gene must be
23
lost for oncogenesis to occur. This concept reflects the Knudsen "two hit" hypothesis of tumorigenesis.
24
Some patients, such as the patient described above, have a germline defect in one copy of their RB 1 gene
25
(may be either spontaneous or inherited), and hence are born w ith the first "hit" required for tumor formation.
26
Somatic mutations typically provide the second hit for malignant transformation. Patients w ho develop
27
retinoblastoma due to germline mutations are also at increased risk for other primary malignancies such as
28
osteosarcoma .
29
30
(Choice A) Defective DNA repair predisposes to tumor formation in diseases such as xeroderma
31
pigmentosum, a heritable disease resulting from defective nucleotide excision repair that predisposes to early
32
skin cancer of all forms.
33
34
(Choice C) Polyploidy occurs w hen more than two complete sets of homologous chromosomes exist w ithin
35
an organism or cell. A human disease associated w ith cells of nonstandard ploidy is a partial hydatidiform
36
mole, w hich is typically 69XXX, 69XXY or 69 'XYY. The chromosomes here include one maternal and two
37
paternal sets.
38
Item: 3 of 46 II P Mark -<:I t>-
Q. Id : 6607 (388147) Previous Next

4 E. Trinucleotide expansion [1 %]
5
6
7 Explanation: User Id: 388147
8
9 This patient most likely has a mutation in the retinoblastoma gene (RB 1), a tum or suppressor gene found on
10 chromosome 13 that, w hen defective, predisposes to retinoblastoma . Tw o copies of the RB1 gene exist in
11 the human genome, one on each chromosome 13. Both copies of the RB 1 tumor suppressor gene must be
12 lost for oncogenesis to occur. This concept reflects the Knudsen "tw o hit" hypothesis of tumorigenesis.
13 Some patients, such as the patient described above, have a germline defect in one copy of their RB 1 gene
14 (may be either spontaneous or inherited), and hence are born w ith the first "hit" required for tumor formation.
15 Somatic mutations typically provide the second hit for malignant transformation. Patients w ho develop
16 retinoblastoma due to germline mutations are also at increased risk for other primary malignancies such as
17 osteosarcoma .
18
19 - (Choice A) Defective DNA repair predisposes to tumor formation in diseases such as xeroderma
20 pigmentosum, a heritable disease resulting from defective nucleotide excision repair that predisposes to early
21 skin cancer of all forms.
22
23 (Choice C) Polyploidy occurs w hen more than two complete sets of homologous chromosomes exist w ithin
24 an organism or cell. A human disease associated w ith cells of nonstandard ploidy is a partial hydatidiform
25 mole, w hich is typically 69XXX, 69XXY or 69 XYY. The chromosomes here include one maternal and tw o
26 paternal sets.
27
28 (Choice D) The concept of parental imprinting refers strictly to the preferential transcription of genes from one
29 or the other of a homologous pair of chromosomes depending on the parental origin of the chromosome.
30
31 (Choice E) Trinucleotide expansion occurs to a greater magnitude in successive generations, as in
32 Huntington disease, causing genetic anticipation.
33
34 Educational objective:
35 Retinoblastoma is a childhood tumor that results from separate mutagenic events that inactivate both copies
36 of the RB 1 tumor suppressor gene (one copy of this gene is present on each chromosome 13). Germline
37 mutations in the RB 1 gene cause "hereditary" retinoblastoma, w hich is also associated w ith other primary
38 tumors such as osteosarcoma .
1 •
2 Item: 4 of 46 • \' Mark -<:J C>
Q. Id: 6439 (383141) Previous NeHt
3

5
6
7
8
9
.. ~-
' ...
-
10 •
11
12
13
14

••
• •

" I
- •

"' •

~

~
,
;.,

15 ... ... ' r •


16
• • J.J,,
17
•• • .....? ' •' •• ~(.
18
19 '° .. , ' :>


20
21
22
23

"
r, •

,.;r""-.'.I
•• •

"'
#
'\ I
~
- ..
1-

24 • JI.
I
25
26
27
28
29

30 Oispla)'ed wilhpennissioo fuxnSp:inger Heallhcatel.!d. e ~ 1995, ~ by C\Jrrenl..Medicine
31
32 W hich of the following is the most likely cause of this patient's symptoms?
33
34
A. Cartilage wear-and-tear injury [2%]
35
36 B. Immune complex deposition [4o/o]
37 C. Crystal arthropathy [3%]
1 •
2 Item: 4 of 46 • \' Mark -<:J C>
Q. Id: 6439 (383141) Previous NeHt
3
are shown on the image below.
5
6
7
..
-

" ,...
-,
~
8
9
\. •
~

• ,..
10 ~

.., .. •
11 •
12 ... \ r .a
13
14
•• • • • •../
~
: •• .Q.3(.
15
16
~\.

(~ I • .. , ' ,. •

17
18 •
,~~
.,r I
~

I -( ~
•• .. ..
--
19 • JI " I
20 r-
21 •
~
'J.
22
23
24
25
26
27
• -
Displayed wilhpennissiw fimiSp:inger Heallhcarelfd. © Copyrigbt 1995, 2004 by CU.rent Medicine
28
29 Which of the following is the most likely cause of this patient's symptoms?
30
31
32
A. Cartilage wear-and-tear injury [2%]
33 B. Immune complex deposition [4%]
34 C. Crystal arthropathy [3o/o]
35
~ @ D. Joint 1nfect1on [90%]
36
37 E. Traumatic joint injury (1 %]
38
1 A

2 Item: 4 of 46 II P Mark -<:I t>-


Q. Id : 6439 (388147) Previous Next
3
E. Traumatic joint injury [1 %]
5
6
7 Explanation: User Id: 388147
8
9 The image above demonstrates numerous polymorphonuclear leukocytes and Gram-positive cocci, a
10 combination that is clearly indicative of joint infection.
11
The most common causative organisms of infectious arthritis are Neisseria gonorrhoeae, Staphylococcus,
12
Streptococcus, Haemophilus influenzae, and gram-negative bacilli (eg, E. coli, Salmonella, Pseudomonas).
13
Children younger than two years of age are especially vulnerable to H. influenzae arthritis, w hile older children
14
and older adults tend to develop S. aureus arthritis. During late adolescence and early adulthood, N.
15
gonorrhoeae arthritis is most common (especially in w omen). Lastly, sickle cell disease patients of all ages
16
are prone to infection w ith Salmonella.
17
18 (Choice A) Characteristic features of an osteoarthritic joint include cartilage w ear-and-tear, bony eburnation,
19 dislodging of cartilage and bone pieces ("joint mice"), fibrous-w alled cysts w ithin the subchondral bone, and
20 bony outgrowths at the articular margins.
21
22 (Choice B) Characteristic features of a joint affected by rheumatoid arthritis include immune complex
23 deposition, neutrophil accumulation in the synovial fluid, perivascular inflammation of the synovial stroma,
24 osteoclastic activity, and pannus formation.
25
26 (Choice C) Characteristic features of a joint affected by gout include intracellular monosodium urate crystals
27 that are slender, needle-shaped, and negatively birefringent.
28
29 (Choice E) The characteristic feature of traumatic joint injury is a joint effusion that may be serosanguineous
30 or frankly bloody.
31
32 Educational objective:
33 The presence of numerous polymorphonuclear leukocytes and Gram-positive cocci in the joint space is
34 indicative of infection. Some of the most common causative organisms of infectious arthritis are Neisseria
35 gonorrhoeae and Staphylococcus aureus.
36
37 Time Spent: 28 seconds 90% answ ered correctly. Last updated: [1 0/26/2011].
38 Copyright © USMLEWORLD,LLC. SimEx am ver. 2011 . 11.397425
1 A

2 Item: 5 of 46 II P Mark -<:I t>-


Q. Id : 6444 (388147) Previous Next
3
4

6 An infant born to an api:iarently: healthy: 43-year-old female has ~acial abnormalities and muscle_s[1asms that
7 require treatment w ith calciurpjpfusio[J. He also experiences recurrent Candida infectiorfs. The infant's
8 condition is best explained by failed development of w hich of the following?
9
10
11
A. Second branchial cleft [3%]
12 B. Third branchial arch [10%]
13
14 C. Third pharyngeal pouch [79%]
15
16 x@ D. Fourth branchial arch [6%]
17
E. Fourth branchial cleft [1 %]
18
19
20
21 Explanation: User Id: 388147
22
23 DiGeorge syndrome is a T-cell deficiency characterized by the deletion of a gene involved in the formation of
24 the branchial arch and great vessels (chromosome 22q11 ). This syndrome results from the failed
25 development of the third and fourth pharyngeal pouches (the predecessors of the thymus, parathyroid glands,
26 and the ultimobranchial body}. Consequently, individuals w ith DiGeorge syndrome have defective T
27 cell-mediated immunity (secondary to an absent or hypoplastic thymus), tetany (secondary to absent
28 parathyroid glands), and cardiac and great vessel abnormalities. The reduction in circulating T lymphocytes
29 typically causes an increased susceptibility to certain fungal and viral infections.
30
(Choices A and E) W hen the second branchial cleft is not obliterated as expected during the seventh w eek of
31
embryonic development, a branchial cleft cyst can arise on the lateral neck. This branchial cleft cyst has an
32
epithelial lining that may connect to the overlying skin via a sinus tract. The fourth branchial cleft involutes
33
34 during embryonic development and is not responsible for any definitive structures.
35
(Choices B and D) Derivatives of the third pharyngeal arch include the stylopharyngeus muscle, hyoid bone,
36 and glossopharyngeal nerve. Derivatives of the fourth pharyngeal arch include the intrinsic muscles of the
37
larynx and pharynx, the thyroid cartilage, cricoid cartilage, epiglottic cartilage, arytenoid cartilage, and vagus
38
1 A

2 Item: 5 of 46 II P Mark -<:I t>-


Q. Id : 6444 (388147) Previous Next
3
4
B. Third branchial arch [10%]
6
.; C. Thirlil pharyngeal pouch [79%]
7
8 x@ D. Fourth branchial arch [6%]
9
10 E. Fourth branchial cleft [1 %]
11
12
13 Explanation: User Id: 388147
14
15 DiGeorge syndrome is a T-cell deficiency characterized by the deletion of a gene involved in the formation of
16 the branchial arch and great vessels (chromosome 22q11 ). This syndrome results from the failed
17 development of the third and fourth pharyngeal pouches (the predecessors of the thymus, parathyroid glands,
18 and the ultimobranchial body}. Consequently, individuals w ith DiGeorge syndrome have defective T
19 cell-mediated immunity (secondary to an absent or hypoplastic thymus), tetany (secondary to absent
20 parathyroid glands), and cardiac and great vessel abnormalities. The reduction in circulating T lymphocytes
21 typically causes an increased susceptibility to certain fungal and viral infections.
22
23 (Choices A and E) W hen the second branchial cleft is not obliterated as expected during the seventh w eek of
24 embryonic development, a branchial cleft cyst can arise on the lateral neck. This branchial cleft cyst has an
25 epithelial lining that may connect to the overlying skin via a sinus tract. The fourth branchial cleft involutes
26 during embryonic development and is not responsible for any definitive structures.
27
28 (Choices B and D) Derivatives of the third pharyngeal arch include the stylopharyngeus muscle, hyoid bone,
29 and glossopharyngeal nerve. Derivatives of the fourth pharyngeal arch include the intrinsic muscles of the
30 larynx and pharynx, the thyroid cartilage, cricoid cartilage, epiglottic cartilage, arytenoid cartilage, and vagus
31 nerve. These tissues are not involved in DiGeorge syndrome.
32
33 Educational Objective:
34 DiGeorge syndrome is a T-cell deficiency resulting from failed development of the third and fourth pharyngeal
35 pouches.
36
37 Time Spent: 33 seconds 79% answ ered correctly. Last updated: [3/27/201 O].
38 Copyright © USMLEWORLD,LLC. SimExam ver. 2011 .11.397425
1 A

2 Item: 6 of 46 II P Mark -<:I t>-


Q. Id : 6601 [388147) Previous Next
3
4
5
A previously healthy 6-year-old boy presents to the emergency department complaining of abdominal r:iain.
7 His mother reports that he has had n,a usea, and vomiting for the past two days but no.f.e.yers . She also notes
8 that he has been urinatipg .!];!Of.e; than usual and seems to be br,eatbipg heq)(ily. Urine analysis is positive for
9 ketones. W hich of the follow ing patterns of serum abnormalities is most likely in this patient
10
Potassium Sodium Glucose
11
12
13 "' @ A. [47%1
t t
14
B. t t t [14%1
15
16 C. I t t [19%1
17 0. I I t [14%1
18
19 -
20
Explanation: User Id: 388147
21
22
The most likely diagnosis here is diabetic ketoacidosis (OKA) from uncontrolled (undiagnosed) type I diabetes
23
mellitus. Patients in OKA classically present w ith polyuria, polydipsia, abdominal pain/tenderness, nausea,
24
vomiting, fatigue, stupor and tachypnea . Hyperketonemia occurs secondary to insulin deficiency, w hich
25
promotes lipolysis and ketone body production. (Ketone bodies function as an alternate source of energy for
26
tissues unable to utilize glucose.) The presence of acetone, one such ketone, in the blood lends a "fruity" or
27
"sw eet" odor to the breath of patients in OKA. Ketones are acids, and thus cause acidosis w hen present in
28
large amounts. Patients often become tachypneic to compensate for the metabolic acidosis. Polyuria occurs
29
in uncontrolled diabetes as the result of osmotic diuresis secondary to glycosuria . Acidemia and
30
carbohydrate depletion in smooth muscle cells have been know n to cause painful dysmotility, distension, and
31
ileus in the stomach and intestines.
32
33
In short, metabolic derangements commonly observed in OKA include metabolic acidosis, ketonemia and
34
ketonuria, hyperglycemia and glycosuria, hyponatremia, hypovolemia and hyperkalemia . Hyperkalemia
35
occurs both because acidemia drives cells' K•1H• exchange and because insulin's tendency to drive
36
potassium into cells is missing. Paradoxically, total body potassium stores are actually depleted due to
37
urinary losses (osmotic diuresis) and GI losses (vomiting). The serum sodium is decreased in OKA because
38
1 A

2 Item: 6 of 46 II P Mark -<:I t>-


Q. Id : 6601 [388147) Previous Next
3
4 u. i T [1'1'1o]
5
t t
7
8
Explanation: User Id: 388147
9
10 The most likely diagnosis here is diabetic ketoacidosis (OKA) from uncontrolled (undiagnosed) type I diabetes
11 mellitus. Patients in OKA classically present w ith polyuria, polydipsia, abdominal pain/tenderness, nausea,
12
vomiting, fatigue, stupor and tachypnea . Hyperketonemia occurs secondary to insulin deficiency, w hich
13 promotes lipolysis and ketone body production. (Ketone bodies function as an alternate source of energy for
14
tissues unable to utilize glucose.) The presence of acetone, one such ketone, in the blood lends a "fruity" or
15 "sw eet" odor to the breath of patients in OKA. Ketones are acids, and thus cause acidosis w hen present in
16 large amounts. Patients often become tachypneic to compensate for the metabolic acidosis. Polyuria occurs
17 in uncontrolled diabetes as the result of osmotic diuresis secondary to glycosuria . Acidemia and
18 carbohydrate depletion in smooth muscle cells have been know n to cause painful dysmotility, distension, and
19 ileus in the stomach and intestines.
20
21 In short, metabolic derangements commonly observed in OKA include metabolic acidosis, ketonemia and
22 ketonuria, hyperglycemia and glycosuria, hyponatremia, hypovolemia and hyperkalemia . Hyperkalemia
23
occurs both because acidemia drives cells' K•1H• exchange and because insulin's tendency to drive
24
potassium into cells is missing. Paradoxically, total body potassium stores are actually depleted due to
25
urinary losses (osmotic diuresis) and GI losses (vomiting). The serum sodium is decreased in OKA because
26
of the osmotic activity of glucose (serum sodium drops 1.6 mEq/L for every 100 mg/dl rise in blood glucose)
27
and because of the hyperglycemia-induced osmotic diuresis, w hich results in sodium and free w ater
28
losses. Blood levels of NH 3 are increased in OKA and other early starvation states due to increased
29
30 proteolysis under the influence of insulin counterregulatory hormones such as glucagon, epinephrine and
31 cortisol.
32
33 Educational objective:
34 Metabolic derangements associated w ith OKA include metabolic acidosis, ketonemia, hyperglycemia,
35 hyperkalemia and hyponatremia. Hyperammonemia can also be observed due to muscle degradation.
36
37 Time Spent: 71 seconds 47% answ ered correctly. Last updated: [2/16/2012].
38 Copyright © USMLEWORLD,LLC. SimExam ver. 2011 .11.397425
1 •
2 Item: 7 of 46 • \ ' Mark -<:J C>
Q. Id: 6453 (383141) Previous NeHt
3
4 A young male complains of severe headaches and periodic vertigo. An MRI image of the head is shown
5 below.
6

8
9
10
11
12
13
14
15
16
17
18
19 '°
20
21
22
23
24
25
26
27
28
29
30
31
32
33
34
35
36
37
38
39
40
1 •
2 Item: 7 of 46 • \ ' Mark -<:J C>
Q. Id: 6453 (383141) Previous NeHt
3
4
5
6

8
9
10
11
12
13
14
15
16
17
18
19 '°
20
21
22
23
24
25
26
27
28
29
30
31
32
33
34
35
36
37
38
39
40
1 A

2 Item: 7 of 46 II P Mark -<:I t>-


Q. Id : 6453 (388147) Previous Next
3
4 Which of the following is the most likely diagnosis?
5
6
A. DanEly 'A'alker synElreme [11 %•]
8 B. i\rnelEl Chiari malfermatien [6Qq~]

9 C. Spina 9ifi8a cystic a [1 q~l


10
D. Syringobulbia [3%]
11
12 x @ E. Aqueductal stenosis [25%]
13
14
Explanation: User Id: 388147
15
16
Arnold-Chiari malformations (ACMs) are common congenital structural anomalies of the brain. The ACM is
17
characterized by the downward displacement of elongated cerebellar tonsils through the foramen magnum
18
and into the upper cervical canal (as shown in the midsagittal MRI above). Cerebrospinal fluid flow is blocked
19
as a result of this malformation. Common additional findings include a small posterior fossa, caudal
20
displacement of the medulla, hydrocephalus, and lumbar myelomeningocele.
21
22
(Choice A) The Dandy-Walker malformation is characterized by an enlarged posterior fossa. The absent or
23
shrunken cerebellar vermis is replaced by a large, midline cyst that represents an expanded fourth ventricle.
24
25
(Choice C) Spina bifida cystica (also called myelomeningocele) describes the herniation of a sac containing
26
CSF, spinal cord, and nerve roots through a defect in the vertebral arches and dura.
27
28
(Choice D) Syringobulbia features fluid-filled cavities within the medulla oblongata.
29
30
(Choice E) Aqueductal stenosis is a condition in which narrowing or occlusion of the aqueduct of Sylvius
31
obstructs CSF flow. Hydrocephalus commonly results.
32
33
Educational objective:
34
Arnold-Chiari malformations are characterized by the downward displacement of elongated cerebellar tonsils
35
through the foramen magnum and into the upper cervical canal.
36
37
38 Time Spent: 44 seconds 60% answered correctly. Last updated: [1 0/3/2011].
1 A

2 Item: 8 of 46 II P Mark -<:I t>-


Q. Id : 6455 (388147) Previous Next
3
4
5
6 A young patient is evaluated for ambiguous genitalia. The vagina ends in a blind pouch and a unilateral
7 inguinal hernia is present. Serum testosterone and LH levels are high and FSH level is normal. W hich of the
follow ing is the most likely cause of this patient's condition?
9
10
A. 45,XO karyotype [4%]
11
12 X @ B. MOllerian agenesis [13%]
13 C. Leydig-cell agenesis [4%]
14
D. Androgen insensitivity [73%]
15
16 E. Adrenal hyperplasia [6%]
17
18
19 Explanation: User Id: 388147
20
Complete androgen insensitivity, formerly called testicular feminization syndrome, is a rare X-linked disorder.
21
Affected individuals have a 46,XY karyotype but demonstrate a female phenotype. This condition arises
22
because of an absence of testosterone receptors in target tissues throughout the body. Serum testosterone
23
levels are high, as are luteinizing hormone (LH) levels (because of hypothalamic-pituitary gland resistance to
24
androgen feedback). Follicle-stimulating hormone levels are normal. Eventually, the circulating androgens
25
are peripherally converted into estradiol. Breast development, external genitalia, overall body habitus, and
26
psychological outlook are all consistent w ith female gender. How ever, the vagina ends abruptly in a blind
27
pouch and the wolffian and mOllerian structures are absent. The testes are located in the abdomen, inguinal
28
canal, or labia majora. They produce testosterone but do not engage in spermatogenesis.
29
30
31
(Choice A) Turner syndrome ( 45,XO) typically presents w ith primary amenorrhea, mild mental retardation,
32 short stature, webbed neck, shielded chest and ovarian dysgenesis. LH and FSH levels are elevated w hile
estrogen levels are low secondary to ovarian failure.
33
34
(Choice B) MOllerian agenesis is a cause of primary amenorrhea in w hich the uterus, fallopian tubes, and
35
proximal vagina fail to develop. LH, FSH, and estrogen levels are normal.
36
37
(Choice C) Leydig-cell agenesis is a rare form of male pseudohermaphroditism that can present w ith
38
1 A

2 Item: 8 of 46 II P Mark -<:I t>-


Q. Id : 6455 (388147) Previous Next
3
4
5
6 Explanation: User Id: 388147
7
Complete androgen insensitivity, formerly called testicular feminization syndrome, is a rare X-linked disorder.
9 Affected individuals have a 46,XY karyotype but demonstrate a female phenotype. This condition arises
10 because of an absence of testosterone receptors in target tissues throughout the body. Serum testosterone
11 levels are high, as are luteinizing hormone (LH) levels (because of hypothalamic-pituitary gland resistance to
12 androgen feedback). Follicle-stimulating hormone levels are normal. Eventually, the circulating androgens
13 are peripherally converted into estradiol. Breast development, external genitalia, overall body habitus, and
14 psychological outlook are all consistent w ith female gender. How ever, the vagina ends abruptly in a blind
15 pouch and the w olffian and mOllerian structures are absent. The testes are located in the abdomen, inguinal
16 canal, or labia majora. They produce testosterone but do not engage in spermatogenesis.
17
18 (Choice A) Turner syndrome ( 45,XO) typically presents w ith primary amenorrhea, mild mental retardation,
19 short stature, w ebbed neck, shielded chest and ovarian dysgenesis. LH and FSH levels are elevated w hile
20 estrogen levels are low secondary to ovarian failure.
21
22 (Choice B) MOllerian agenesis is a cause of primary amenorrhea in w hich the uterus, fallopian tubes, and
23 proximal vagina fail to develop. LH, FSH, and estrogen levels are normal.
24
25 (Choice C) Leydig-cell agenesis is a rare form of male pseudohermaphroditism that can present w ith
26 complete external female genitalia. LH and FSH levels are increased, w hile testosterone levels are decreased.
27
28 (Choice E) Congenital adrenal hyperplasia can present w ith various sex characteristic abnormalities, salt
29 w asting, and failure to thrive in infants. LH and FSH levels are typically decreased w hile testosterone levels
30 are increased.
31
32 Educational objective:
33 Complete androgen insensitivity is characterized by a male genotype and a female phenotype w ith normal
34 external female genitalia and a vagina ending in a blind pouch . Serum testosterone levels and LH levels are
35 high, w hile FSH levels are normal.
36
37 Time Spent: 92 seconds 73% answ ered correctly. Last updated: [3/27/201 O].
38 Copyright © USMLEWORLD,LLC. SimExam ver. 2011 .11.397425
1 A

2 Item: 9 of 46 II P Mark -<:I t>-


Q. Id : 6466 (388147) Previous Next
3
4
5
6 A 42-year-old Caucasian female suffering from episodic headache and Ralpitations is diagnosed w ith a
7 right-sided_suRrarenal· mass on CT scan . Her blood pressure is,200illQ mm Hg and her heart rate is
8 92.aJJ!d. A single dose of a drug brings her blood pressure to normal for a period of three days. W hich of the
follow ing drugs w as most likely used in this patient?
10
11
12
A. Prazosin [12%]
13 B. Phentolamine [15%]
14 .; @ C. Phenoxybenzan1ine [62%]
15
16
D. Phenylephrine [3%]
17 E. Esmolol [8%]
18
19
20 Explanation: User Id: 388147
21
22 This patient's history and clinical presentation suggest she has excess catecholamines in circulation
23 secondary to pheochromocytoma. Catecholamines establish a hyperadrenergic state by stimulating
24 peripheral vascular alpha-adrenergic receptors (predominantly the a 1 receptors) and cardiac 131
25 receptors. Of all the drugs listed, only phenoxybenzamine is a long-acting alpha receptor
26 blocker. Phenoxybenzamine irreversibly alkylates alpha-adrenergic receptors, resulting in a half life of
27 approximately 24 hours. Consequently, the effects of a single dose of phenoxybenzamine can last three to
28 four days.
29
(Choice A) Prazosin competitively and reversibly inhibits the action of endogenous catecholamines at the a 1
30
receptors on vascular smooth muscle. It has a half life of three hours and a duration of action of 10-24 hours.
31
32 (Choice B) Phentolamine is a non-selective alpha blocker that competitively and reversibly inhibits
33
endogenous catecholamines at the alpha-adrenergic receptors. Phentolamine has a half-life of twenty
34
minutes and a duration of action of less than an hour.
35
36 (Choice D) Phenylephrine is an alpha-adrenergic agonist (a1 > a2) w ith w eak beta-adrenergic activity. Its
37 duration of action is a few hours at most.
38
1 A

2 Item: 9 of 46 II P Mark -<:I t>-


Q. Id : 6466 (388147) Previous Next
3
4 v @ C. Phenoxybenzan1ine [62%]
5
D. Phenylephrine [3%]
6
7 E. Esmolol [8%]
8

Explanation: User Id: 388147


10
11
This patient's history and clinical presentation suggest she has excess catecholamines in circulation
12
secondary to pheochromocytoma. Catecholamines establish a hyperadrenergic state by stimulating
13
peripheral vascular alpha-adrenergic receptors (predominantly the a 1 receptors) and cardiac 131
14
receptors. Of all the drugs listed, only phenoxybenzamine is a long-acting alpha receptor
15
blocker. Phenoxybenzamine irreversibly alkylates alpha-adrenergic receptors, resulting in a half life of
16
approximately 24 hours. Consequently, the effects of a single dose of phenoxybenzamine can last three to
17
four days.
18
19 (Choice A) Prazosin competitively and reversibly inhibits the action of endogenous catecholamines at the a 1
20 receptors on vascular smooth muscle. It has a half life of three hours and a duration of action of 10-24 hours.
21
22 (Choice B) Phentolamine is a non-selective alpha blocker that competitively and reversibly inhibits
23 endogenous catecholamines at the alpha-adrenergic receptors. Phentolamine has a half-life of twenty
24 minutes and a duration of action of less than an hour.
25
26 (Choice D) Phenylephrine is an alpha-adrenergic agonist (a1 > a2) w ith weak beta-adrenergic activity. Its
27 duration of action is a few hours at most.
28
29 (Choice E) Esmolol competitively and reversibly inhibits endogenous catecholamines at the 131 receptors,
30 w ith little effect on the 132 receptors (except at high doses). Esmolol has a half life of approximately nine
31 minutes and a duration of action of 30 minutes.
32
33 Educational objective:
34 Phenoxybenzamine is a non-competitive, irreversible alpha receptor blocker. It has a half life of approximately
35 24 hours, w ith the effects of a single dose lasting three to four days.
36
37 Time Spent: 40 seconds 62% answ ered correctly. Last updated: [3/27/201 O].
38 Copyright © USMLEWORLD,LLC. SimExam ver. 2011 .11.397425
1 A

2 Item: 10 of 46 II P M ark -<:I t>-


Q. Id : 6469 (388147] Previous Next
3
4
5
6 A two-day-old infant dies in the intensive care unit. He w as born at 25 w eeks of gestation to an apparently
7 healthy, 38-year-old female. He suffered from moderate respiratory distress secondary to hyaline membrane
8 disease. Postmortem brain findings include a large left-sided intraventricular clot. W hich of the follow ing is
9 the most likely cause of this finding?

11
A. Postmortem artifact [ 19%]
12
13 B. Cephalhematoma [1 9%]
14 X @ C. Berry aneurysm [11 %]
15
~ D. Germinal matrix bleeding [33%]
16
17 E. Subarachnoid hemorrhage [18%]
18
19
Explanation: User Id: 388147
20
21
lntraventricular hemorrhage (IVH) is a prominent cause of brain injury in the premature infant, typically
22
developing w ithin the first five days of life. Risk factors for IVH include birth before 32 weeks of gestation, birth
23
24 w eight of less than 1500 grams, intrapartum asphyxia, respiratory distress syndrome, and prolonged
resuscitation. IVH most often begins in the germinal matrix, w hich is a highly cellular and vascular in the
25
subventricular region that generates neurons and glia during fetal development. No network fibers are present
26
to support the vasculature in the germinal matrix, leaving it vulnerable to hemorrhage. Periods of
27
hemodynamic instability that modify cerebral blood flov1 appear to increase the risk of bleeding. After 32
28
w eeks of gestation, the germinal matrix has subsided in prominence, w ith an associated decline in cellularity
29
and vascularity.
30
31
(Choice A) Postmortem clots are bright purple to red, gelatinous clots w ithin the vasculature. Unlike thrombi,
32
these clots are not attached to the vessel w all and lack lines of Zahn .
33
34
(Choice B) The cephalhematoma is a hemorrhage w ithin the circular region of the scalp that enters the lower
35
uterus during childbirth. It is associated w ith sudden delivery, disproportion between diameter of fetal head and
36
birth canal, and inappropriate forceps use.
37
38
1 A

2 Item: 10 of 46 II P M ark -<:I t>-


Q. Id : 6469 (388147] Previous Next
3
LI . V C I 1 1111 10 1 I I I OU II\ UIC"CUll 1!::;1 L.J.J F U J
4
5 E. Subarachnoid hemorrhage [1 8%]
6
7
8 Explanation: User Id: 388147
9
lntraventricular hemorrhage (IVH) is a prominent cause of brain injury in the premature infant, typically
11 developing w ithin the first five days of life. Risk factors for IVH include birth before 32 w eeks of gestation, birth
12 w eight of less than 1500 grams, intrapartum asphyxia, respiratory distress syndrome, and prolonged
13 resuscitation. IVH most often begins in the germinal matrix, w hich is a highly cellular and vascular in the
14 subventricular region that generates neurons and glia during fetal development. No network fibers are present
15 to support the vasculature in the germinal matrix, leaving it vulnerable to hemorrhage. Periods of
16 hemodynamic instability that modify cerebral blood flow appear to increase the risk of bleeding . After 32
17 w eeks of gestation, the germinal matrix has subsided in prominence, w ith an associated decline in cellularity
18 and vascularity.
19
20 (Choice A) Postmortem clots are bright purple to red, gelatinous clots w ithin the vasculature. Unlike thrombi,
21 these clots are not attached to the vessel w all and lack lines of Zahn.
22
23 (Choice B) The cephalhematoma is a hemorrhage w ithin the circular region of the scalp that enters the lower
24 uterus during childbirth. It is associated w ith sudden delivery, disproportion between diameter of fetal head and
25 birth canal, and inappropriate forceps use.
26
27 (Choices C and E) Hypertension and cigarette smoking are two of the more significant risk factors for the
28 development of saccular (berry) aneurysms. These aneurysms do not exist at birth, but rather develop over
29 time. Rupture of a saccular aneurysm is the most common cause of subarachnoid hemorrhage, typically
30 occurring in the fifth decade of life. Saccular aneurysms and subarachnoid hemorrhages are not expected in
31 neonates.
32
33 Educational objective:
34 lntraventricular hemorrhage is a significant cause of brain injury in the premature infant, most often arising in
35 the germinal matrix .
36
37 Time Spent: 60 seconds 33% answ ered correctly. Last updated: [3/27/201 O].
38 Copyright © USMLEWORLD,LLC. SimExam ver. 2011 .11.397425
1 A

2 Item: 11 of 46 II P Mark -<:I t>-


Q. Id : 6475 (388147] Previous Next
3
4
5
6 A 64-year-old male is treated w ith combined chemotherapy for a m ediastipal mass. Two months later, he
7 develops high-fre~ue cy hearing loss. W hich of the follow ing drugs is most likely responsible for his change
8 in condition?
9
10
A. Prednisone [3%]
12 B. Etoposide [ 15%]
13 ~ C. Cisplatin [64%]
14
15
X @ D. Doxorubicin [1 0%]
16 E . Methotrexate [6%]
17 F. Ondansetron [2%]
18
19
20 Explanation: User Id: 388147
21
22 Certain medications have the adverse effect of causing tinnitus and hearing loss by damaging hair cells w ithin
23 the organ of Corti of the cochlear membranous labyrinth. Of all the drugs listed, cisplatin is the most
24 ototoxic. Electrophysiologic studies have show n that cisplatin primarily damages the apical stereocilia on hair
25 cells. On audiogram, a subclinical loss of response to high frequencies occurs in 74-1 00% of patients
26 receiving cumulative cisplatin doses of 200 mg/m 2 Approximately 6% of patients receiving cisplatin develop
27 clinical hearing loss, w ith children particularly at risk.
28
29 (Choices A, B, D, and E) Adverse neurologic effects caused by glucocorticoids include vertigo and
30 psychosis. Etoposide can cause alopecia, gastrointestinal irritation, and m yelosuppression. Doxorubicin
31 (adriamycin) is associated w ith alopecia, m yocardial damage, and m yelosuppression . Methotrexate can
32 cause m yelosuppression and fatty change in the liver. None of these drugs is considered particularly ototoxic .
33
34 (Choice F) Ondansetron is the only drug listed that is not a chemotherapeutic agent. Instead, it is typically
35 used as an antiemetic in patients undergoing cancer chemotherapy. Major adverse effects of ondansetron
36 include headache and constipation.
37
38 Educational objective:
39
40
1 A

2 Item: 11 of 46 II P Mark -<:I t>-


Q. Id : 6475 (388147] Previous Next
3
4
5 A. Prednisone [3%]
6
B. Etoposide [1 5%]
7
8 C. Cisplatin [64%]
9 X @ D. Doxorubicin [10%]
10
E. Methotrexate [6%]
12 F. Ondansetron [2%]
13
14
15
Explanation: User Id: 388147
16
Certain medications have the adverse effect of causing tinnitus and hearing loss by damaging hair cells w ithin
17
the organ of Corti of the cochlear membranous labyrinth. Of all the drugs listed, cisplatin is the most
18
ototoxic. Electrophysiologic studies have show n that cisplatin primarily damages the apical stereocilia on hair
19
cells. On audiogram, a subclinical loss of response to high frequencies occurs in 74- 100% of patients
20
21 receiving cumulative cisplatin doses of 200 mg/m 2 · Approximately 6% of patients receiving cisplatin develop
22 clinical hearing loss, w ith children particularly at risk.
23
24 (Choices A, B, D, and E) Adverse neurologic effects caused by glucocorticoids include vertigo and
25 psychosis. Etoposide can cause alopecia, gastrointestinal irritation, and myelosuppression. Doxorubicin
26 (adriamycin) is associated w ith alopecia, myocardial damage, and myelosuppression. Methotrexate can
27 cause myelosuppression and fatty change in the liver. None of these drugs is considered particularly ototoxic.
28
29 (Choice F) Ondansetron is the only drug listed that is not a chemotherapeutic agent. Instead, it is typically
30 used as an antiemetic in patients undergoing cancer chemotherapy. Major adverse effects of ondansetron
31 include headache and constipation.
32
33 Educational objective:
34 Cisplatin is an ototoxic chemotherapeutic agent thought to damage hair cells in the cochlear membranous
35 labyrinth .
36
37 Time Spent: 35 seconds 64% answ ered correctly. Last updated: [3/27/201 O].
38 Copyright © USMLEWORLD,LLC. SimExam ver. 2011 .11.397425
1 A

2 Item: 12 of 46 II P Mark -<:I t>-


Q. Id : 6476 (388147] Previous Next
3
4
5
6 A 62-year-old male is treated for ventricular arrhythmia. A few w eeks after his therapy is initiated, he returns
7 to clinic w ith dyspnea, cough, chest pain, and malaise. Physical examination reveals bilateral inspiratory
8 crackles. Chest x-ray demonstrates patchy areas of interstitial infiltration . W hich of the follow ing drugs is
9 most likely responsible for this patient's symptoms?
10
11
A. Sotalol [6%]
13 B. Procainamide [7%]
14 ~ C. Amiodarone [80%]
15
X @ D. Lidocaine [3%]
16
17 E. Diltiazem [4% ]
18
19
20 Explanation: User Id: 388147
21
Dyspnea, cough, dry inspiratory crackles, and patchy interstitial infiltration on radiograph are characteristic
22
clinical features of inflammation and fibrosis in the pulmonary interstitium. Of the drugs listed, only
23
amiodarone is known to cause these toxic changes in the lung. Most commonly, amiodarone is responsible
24
for a chronic condition termed amiodarone interstitial pneumonitis that has an incidence of 2-7% and presents
25
w ith slow ly progressive dyspnea and nonproductive cough . The condition is usually reversible w ith a
26
reduction in dose or discontinuance of the drug, though corticosteroids may be necessary in severe cases.
27
Other notable reactions to oral amiodarone include arrhythmias, hepatic injury, thyroid abnormalities, ocular
28
29 changes, and bluish-gray skin discoloration.
30
(Choices A, D, and E) Adverse effects of sotalol include torsades de pointes and excessive beta blockade.
31
Lidocaine can cause dose-dependent central nervous system stimulation or depression and cardiovascular
32
33
depression. Diltiazem can cause bradycardia, decreased m yocardial contractility, flushing, hypotension, and
34 gastrointestinal disturbances (eg, nausea, constipation).
35
(Choice B) Procainamide can cause a reversible syndrome similar to systemic lupus erythematosus.
36
37
Educational objective:
38
1 A

2 Item: 12 of 46 II P Mark -<:I t>-


Q. Id : 6476 (388147] Previous Next
3
• • - - J - -· · - •- ••• -·· - •- •• - -· . - - • - • • - •• • •• - - ·· -· · -·· ••• J • •• • • • • -· · • • • - • • •• - - •· - -· · • - • • •• - • •• - • -·r J •- • • •• • • -· . - - , •• - • - • - ·· •• -
4 to clinic w ith dyspnea, cough, chest pain, and malaise. Physical examination reveals bilateral inspiratory
5 crackles. Chest x-ray demonstrates patchy areas of interstitial infiltration. W hich of the following drugs is
6 most likely responsible for this patient's symptoms?
7
8
9 A. Sotalol [6%]
10 B. Procainamide [7%]
11
C. An1iodarone [80%]
13 x @ D. Lidocaine [3%]
14 E. Diltiazem [4%]
15
16
17 Explanation: User Id: 388147
18
19 Dyspnea, cough, dry inspiratory crackles, and patchy interstitial infiltration on radiograph are characteristic
20 clinical features of inflammation and fibrosis in the pulmonary interstitium. Of the drugs listed, only
21 amiodarone is known to cause these toxic changes in the lung. Most commonly, amiodarone is responsible
22 for a chronic condition termed amiodarone interstitial pneumonitis that has an incidence of 2-7% and presents
23 w ith slow ly progressive dyspnea and nonproductive cough . The condition is usually reversible w ith a
24 reduction in dose or discontinuance of the drug, though corticosteroids may be necessary in severe cases.
25 Other notable reactions to oral amiodarone include arrhythmias, hepatic injury, thyroid abnormalities, ocular
26 changes, and bluish-gray skin discoloration.
27
28 (Choices A, D, and E) Adverse effects of sotalol include torsades de pointes and excessive beta blockade.
29 Lidocaine can cause dose-dependent central nervous system stimulation or depression and cardiovascular
30 depression. Diltiazem can cause bradycardia, decreased m yocardial contractility, flushing, hypotension, and
31 gastrointestinal disturbances (eg, nausea, constipation}.
32
33 (Choice B) Procainamide can cause a reversible syndrome similar to systemic lupus erythematosus.
34
35 Educational objective:
36 Amiodarone is associated w ith a delayed onset interstitial pneumonitis that occurs in isolation . Procainamide
37 can cause a delayed onset pleuritis generally associated w ith a drug-induced systemic lupus erythematosus
38 syndrome.
1 •
2 Item: 13 of 46 • \' Mark -<:J C>
Q. Id : 6478 (388147) Previous NeHt
3
4
5
6 A 44-year-old male with long-standing an~losing ~ondylitis complains of e(lisodic abdominal pain.
7 Endoscopy of his upper gastrointestinal tract demonstrates a small gastric ulcer in the antrum. Misoprostol is
8 prescribed. Which of the following pathways in the parietal cell is affected by misoprostol?
9
10
11
12

14
15
16
17
18
19 '°
20
21
22 E
23
24
25 A. Pathway A (3%]
26
27 B. Pathway B (3%]
28 X @ C. Pathway C [5%1]
29 ~ D. Pathway 0 [79%]
30
31 E. Pathway E (1Oo/o]
32
33
Explanation: User Id: 388147
34
35
Patients with severe back pain secondary to ankylosing spondylitis are likely to have lengthy histories of
36
NSAIO use. Many NSAJOs inhibit cyclooxygenase-1 (COX-1). interfering with prostaglandin formation.
37
Prostaglandins such as PGE 1 stimulate gastric epithelial cell mucus production and appear to decrease
38
1 A

2 Item: 13 of 46 II P Mark -<:I t>-


Q. Id : 6478 (388147] Previous Next
3
4 A. Pathw ay A [3%]
5
B. Pathw ay B [3%]
6
7 X @ C. Pathw ay C [5%]
8 .; D. Pathw ay D [79%]
9
E. Pathw ay E [10%]
10
11
12
Explanation: User Id: 388147
14
Patients w ith severe back pain secondary to ankylosing spondylitis are likely to have lengthy histories of
15 NSAID use. Many NSAIDs inhibit cyclooxygenase-1 (COX-1}, interfering w ith prostaglandin formation.
16
Prostaglandins such as PGE, stimulate gastric epithelial cell mucus production and appear to decrease
17
parietal cell acid secretion . Consequently, long-term NSAID use can leave gastric mucosa w ith little
18
protection from acidic juices. The small gastric ulcer observed on endoscopy in this patient is likely a benign
19
manifestation of peptic ulcer disease. Misoprostol, the medication he w as prescribed, is a PGE1 analog
20
(receptor agonist) that can be used to help prevent and treat NSAID-induced peptic ulcers.
21
22
(Choices A, B, and C) Acetylcholine, histamine, and gastrin can stimulate parietal cells to produce gastric
23
acid through interaction w ith the M3 receptor, H2 receptor, and CCK2 receptor, respectively. Drugs such as
24
cimetidine or ranitidine are often used to selectively block the H2 receptors in parietal cells. Misoprostol does
25
not directly interact w ith the M3 receptor, H2 receptor, or CCK2 receptor, how ever.
26
27
(Choice E) Misoprostol may decrease gastric acid secretion by binding to parietal cell receptors. In the above
28
29 figure, "E" represents the luminal membrane H• pump, w hich is affected by proton pump inhibitors (eg,
30 omeprazole). Misoprostol does not directly affect this pump, how ever.
31
32 Educational objective:
33 Misoprostol is a PGE1 analog that binds to plasma membrane prostaglandin receptors. In the upper
34 gastrointestinal tract, misoprostol stimulates gastric epithelial cell mucus production and appears to decrease
35 parietal cell acid secretion.
36
37 Time Spent: 31 seconds 79% answ ered correctly. Last updated: [11 / 10/2011].
38 Copyright © USMLEWORLD,LLC. SimExam ver. 2011 .11.397425
1 A

2 Item: 14 of 46 ll P Mark -<:I t>-


Q. Id : 648-0 (388147] Previous Next
3
4
5
6 A 21-year-old male suffers from chronic nasal blockage. He uses over-the-counter nasal decongestants
7 occasionally: w ith minimal symptom relief. He also describes difficultyj?r.eathing and nocturnal w heezing
8 w heri_takipg asi:iir.ip..The latter symptoms are most likely related to an increase in w hich of the following?
9
10
11
X @ A. Prostaglandin E [6%]
12 B. Thromboxane A 2 [ 10%]
13
C. Phospholipase ~ [5%]
15 D. Leukotrienes [76%]
16
E. Prostacyclin [3%]
17
18
19 Explanation: User Id: 388147
20
21 This patient appears to be suffering from chronic rhinitis, intranasal polyposis, and aspirin-sensitive asthma .
22 Approximately 10% of asthmatic patients (especially those w ith frequent nasal polyps and sinusitis) w ill
23 develop nasal congestion or bronchospasm after ingesting aspirin or NSAIDs. Although the mechanism of
24 aspirin-induced asthma has not been entirely elucidated, it appears that selective inhibition of the
25 cyclooxygenase pathway in arachidonic acid metabolism promotes the formation of bronchoconstrictive
26 leukotrienes. Leukotrienes C 4 , D 4 , and E 4 are considered some of the most important mediators of asthmatic
27
bronchoconstriction .
28
29 (Choice A) Prostaglandins appear to be less important than leukotrienes in mediating asthmatic
30 bronchoconstriction.
31
32
(Choices B and E) Thromboxane ~ and prostacyclin (PGl 2 } are products of the cyclooxygenase pathway in
33
34 arachidonic acid metabolism. Because aspirin inhibits the cyclooxygenase pathw ay, thromboxane ~and
35 prostacyclin levels would be decreased rather than increased by aspirin.
36
37 (Choice C) Since phospholipase A 2 releases arachidonic acid from membrane lipids, it is conceivable that an
38 increase in phospholipase A, levels could promote leukotriene production . However, aspirin is not known to
39
40
1 A

2 Item: 14 of 46 ll P Mark -<:I t>-


Q. Id : 648-0 (388147] Previous Next
3
4
5 C. Phospholipase ~ [5%]
6 D. Leukotrienes [76%]
7
E. Prostacyclin [3%]
8
9
10
Explanation: User Id: 388147
11
12 This patient appears to be suffering from chronic rhinitis, intranasal polyposis, and aspirin-sensitive asthma .
13 Approximately 10% of asthmatic patients (especially those w ith frequent nasal polyps and sinusitis} w ill
develop nasal congestion or bronchospasm after ingesting aspirin or NSAIDs. Although the mechanism of
15 aspirin-induced asthma has not been entirely elucidated, it appears that selective inhibition of the
16 cyclooxygenase pathway in arachidonic acid metabolism promotes the formation of bronchoconstrictive
17 leukotrienes. Leukotrienes C 4 , D 4 , and E 4 are considered some of the most important mediators of asthmatic
18
bronchoconstriction.
19
20
(Choice A) Prostaglandins appear to be less important than leukotrienes in mediating asthmatic
21
bronchoconstriction .
22
23
24 (Choices B and E) Thromboxane ~and prostacyclin (PGl 2 } are products of the cyclooxygenase pathway in
25 arachidonic acid metabolism. Because aspirin inhibits the cyclooxygenase pathway, thromboxane ~ and
26 prostacyclin levels would be decreased rather than increased by aspirin.
27
28 (Choice C) Since phospholipase A 2 releases arachidonic acid from membrane lipids, it is conceivable that an
29 increase in phospholipase ~ levels could promote leukotriene production. How ever, aspirin is not know n to
30
significantly increase phospholipase ~ levels.
31
32
33 Educational objective:
34 The pathogenesis of aspirin-sensitive asthma appears to involve increased production of the lipoxygenase
35 products of arachidonic acid metabolism (eg, bronchoconstrictive leukotrienes}.
36
37 Time Spent: 33 seconds 76% answ ered correctly. Last updated: [3/27/201 O].
38 Copyright © USMLEWORLD,LLC. SimExam ver. 2011 .11.397425
1 A

2 Item: 15 of 46 II P Mark -<:I t>-


Q. Id : 6485 (388147] Previous Next
3
4
5
6 A 65-year-old Caucasian male presents to clinic w ith a painless neck mass. Physical examination reveals
7 right-sided submandibular lyml) adenopathy. Biopsy is consistent w ith squam ous cell carcinoma . Which of
8 the follow ing is the most likely primary site for the cancer?
9
10
11
X @ A. Parotid gland [ 19%]
12 B. Thyroid gland [10%]
13 C. Tongue [61%]
14
D. Lymph nodes [6%]
16 E. Stomach [4%]
17
18
19 Explanation: User Id: 388147
20
21 The vast majority of cancers in the head and neck are squamous cell carcinomas (HNSCCs}, w hich are
22 strongly associated w ith the use of tobacco and alcohol. HNSCCs tend to develop in the oral cavity, w ith the
23 ventral tongue, floor of the mouth, low er lip, soft palate, and gingiva the most common sites of origin. This
24 type of cancer often infiltrates adjacent tissue, and may eventually metastasize locally to cervical lymph nodes
25 or distally to mediastinal lymph nodes, lungs, liver, or bones.
26
27 (Choices A, B, D, and E) Squamous cell carcinoma does not typically develop in the parotid gland, thyroid
28 gland, lymph nodes, or stomach.
29
30 Educational objective:
31 The vast majority of cancers in the head and neck are squamous cell carcinomas.
32
33 Time Spent: 41 seconds 61 % answ ered correctly. Last updated: [3/27/201 OJ.
34 Copyright © USMLEWORLD,LLC. SimEx am ver. 2011 .11.397425
35
36
37
38
1 A

2 Item: 16 of 46 II P Mark -<:I t>-


Q. Id : 6488 (388147] Previous Next
3
4
5
6 The V 2 vasopressin receptor is a polypeptide w ith seven transmembrane domains. It also has an additional
7
anchoring to the inner aspect of the plasma membrane at the carboxyl tail. The anchoring is most likely
8
accomplished by w hich of the following?
9
10
11 x @ A. Phosphorylation [35%]
12 B. Sulfation [1 0%]
13
C. Glucuronidation [11 %]
14
15 D. Palmitoylation [23%]
E. Hydroxylation [21 %]
17
18
19 Explanation: User Id: 388147
20
21 Palmitoylation is a process in w hich fatty acids are covalently anchored to plasma membrane cysteine
22 residues, thereby increasing the hydrophobicity of a protein . Many G protein-coupled receptors have carboxyl
23 tails palmitoylated on cysteine residues. The V 2 vasopressin receptor is a prime example of such a receptor,
24 w ith a carboxyl tail palmitoylated on cysteines 341 and 342 .
25
26 (Choices A, 8, C, and E) Phosphorylation, sulfation, glucuronidation, and hydroxylation are not associated
27 w ith the anchoring of receptor carboxyl tails to the plasma membrane. Phosphorylation is an extremely
28 important regulatory process, causing a number of enzymes and receptors to be sw itched "on" or "off'
29 through the use of protein kinases or phosphatases; in mitochondria, oxidative phosphorylation allow s for the
30 production of ATP from ADP. The liver employs sulfation and glucuronidation to detoxify acetaminophen, w ith
31 glucuronidation also important in the metabolism and processing of bilirubin, steroid hormones, carcinogens,
32 and other drugs. Praline can be hydroxylated to form hydroxyproline, an important component of collagen .
33 Scurvy is characterized by inadequate praline hydroxylation secondary to ascorbic acid deficiency.
34
35 Educational objective:
36 Palmitoylation increases protein hydrophobicity by anchoring receptor carboxyl tails to the plasma membrane.
37
38
1 A

2 Item: 17 of 46 II P Mark -<:I t>-


Q. Id : 6489 (388147] Previous Next
3
4
5 A 56-year-old male with a history of hyperteosiop, piabetes mellitus, and hypercholesterolemia experiences an
6 episode of squeezing r etrpsternaLcbes\_pain while walking uphill. He takes one tablet of nit oglyceri.ll
7 sublingually and obtains prompt pain relief. Which of the following are the most likely hemodynamic changes
8 associated with the intake of this drug?
9
Heart rate End-systolic left ventricular volume
10
11
12
13 A. L [8%]
14
i
15 X @ B. [17%]
L L
16
C. L No change [10%]
18
19 - D. r No change [10%]
20
21
E. t L [30%]
22
23 F. No change [23%]
L
24
25 G. i [3%]
i
26
27
28
Explanation: User Id: 388147
29
30 Nitroglycerin relieves anginal pain by reducing myocardial oxygen consumption relative to coronary perfusion.
31
This reduction is accomplished by relaxing vascular smooth muscle in the veins more than in the arteries.
32 Hemodynamically, nitroglycerin rapidly decreases ventricular preload (end diastolic volume) through systemic
33
venodilation, which reduces venous return to the heart and decreases effective circulating blood volume
34
through venous blood pooling. Cardiovascular reflexes compensate for the drop in stroke volume, cardiac
35
output, and mean arterial pressure with sympathetic increases in heart rate and myocardial contractility. In
36
the new steady state, preload, afterload, end systolic ventricular volume, and cardiac output are somewhat
37
reduced, while total peripheral resistance remains relatively unchanged. Due to the reduction in stroke work
38
1 A

2 Item: 17 of 46 II P Mark -<:I t>-


Q. Id : 6489 (388147] Previous Next
3
4
X @ B. ! ! [17%]
5
6
7
C. ! No change [1 0%]
8
9
D. t No change [1 0%]
10
11 E. t ! [30%]
12
F. No change ! [23%]
13
14
15 G. t t [3%]
16

18 Explanation: User Id: 388147


19
20 Nitroglycerin relieves anginal pain by reducing myocardial oxygen consumption relative to coronary perfusion.
21 This reduction is accomplished by relaxing vascular smooth muscle in the veins more than in the arteries.
22 Hemodynamically, nitroglycerin rapidly decreases ventricular preload (end diastolic volume) through systemic
23 venodilation, which reduces venous return to the heart and decreases effective circulating blood volume
24 through venous blood pooling. Cardiovascular reflexes compensate for the drop in stroke volume, cardiac
25 output, and mean arterial pressure with sympathetic increases in heart rate and myocardial contractility. In
26 the new steady state, preload, afterload, end systolic ventricular volume, and cardiac output are somewhat
27 reduced, while total peripheral resistance remains relatively unchanged. Due to the reduction in stroke work
28 associated with the smaller ventricular volumes, decreased ejection time, and decreased afterload during the
29 cardiac cycle, the ratio of myocardial oxygen consumption to coronary blood flow is decreased in the new
30 steady state. Consequently, myocardial ischemia and anginal pain are relieved.
31
32 Educational objective:
33 Nitroglycerin relieves angina pectoris by producing net systemic venodilation. Myocardial oxygen consumption
34 is decreased relative to coronary perfusion through a reduction in steady state ventricular volumes that
35 overcome the associated reflexive increase in heart rate.
36
37 Time Spent: 66 seconds 30% answered correctly. Last updated: [3/27/201 O].
38 Copyright © USMLEWORLD,LLC. SimExam ver. 2011 .11.397425
1 A

2 Item: 18 of 46 II P Mark -<:I t>-


Q. Id : 6492 (388147] Previous Next
3
4
5
6 A patient w ith an inherited defect of the epithelial j'.1rotein CFTR has body fluids w ith the characteristics listed in
7 the table below .
8 Sodium Chloride
9
Sw eat t t
10 Bronchial secretion ! !
11
12
W hich of the following best explains the different findings in these body fluids?
13
14
15 A. Low expression of CFTR in sw eat glands [5%]
16 B. Abnormal folding of CFTR in bronchial epithelial cells [26%]
17
X @ C. Rapid destruction of abnormal CFTR in bronchial epithelial cells [3%]
19 .; D. Difference in tissue-specific functioning of CFTR [64 %]
20 E. Compensatory overexpression of sodium channels in sw eat glands [2%]
21
22
23 Explanation: User Id: 388147
24
25 Cystic fibrosis is an autosomal recessive disorder that affects exocrine glands and the epithelia lining
26 respiratory, gastrointestinal, and reproductive organs. Mutation of the cystic fibrosis transmembrane
27 conductance regulator (CFTR) gene causes the epithelial chloride channel protein to operate incorrectly.
28
29 CFTR function varies from tissue to tissue. W ithin the sweat glands, normal CFTR reabsorbs luminal
30 chloride ions and enhances sodium reabsorption; mutated CFTR, in contrast, diminishes sodium chloride
31 reabsorption and increases sw eat tonicity. W ithin the respiratory and intestinal epithelium, normal CFTR
32 allow s for active luminal chloride secretion; mutated CFTR, in contrast, reduces luminal chloride secretion
33 and increases luminal sodium absorption. In this latter case, passive reabsorption of w ater from the lumen
34 depletes the w ater content of the surface fluid layer overlying the mucosal cells. This isotonic, low-volume
35 surface fluid layer in the lungs and intestines contributes to the production of thick, viscid secretions low in
36 both sodium and chloride.
37
38 (Choices A, B, C, and E) Class I mutations of the CFTR gene cause defective protein synthesis, leading to a
1 A

2 Item: 18 of 46 II P Mark -<:I t>-


Q. Id : 6492 (388147] Previous Next
3
4 B. Abnormal folding of CFTR in bronchial epithelial cells [26%]
5 X @ C. Rapid destruction of abnormal CFTR in bronchial epithelial cells [3%]
6
~ D. Difference in tissue-specific functioning of CFTR [64%]
7
8 E. Compensatory overexpression of sodium channels in sw eat glands [2%]
9
10
11 Explanation: User Id: 388147
12
13 Cystic fibrosis is an autosomal recessive disorder that affects exocrine glands and the epithelia lining
14 respiratory, gastrointestinal, and reproductive organs. Mutation of the cystic fibrosis transmembrane
15 conductance regulator (CFTR) gene causes the epithelial chloride channel protein to operate incorrectly.
16
17 CFTR function varies from tissue to tissue. Within the sweat glands, normal CFTR reabsorbs luminal
chloride ions and enhances sodium reabsorption; mutated CFTR, in contrast, diminishes sodium chloride
19 reabsorption and increases sw eat tonicity. Within the respiratory and intestinal epithelium, normal CFTR
20 allow s for active luminal chloride secretion; mutated CFTR, in contrast, reduces luminal chloride secretion
21 and increases luminal sodium absorption . In this latter case, passive reabsorption of w ater from the lumen
22 depletes the w ater content of the surface fluid layer overlying the mucosal cells. This isotonic, low -volume
23 surface fluid layer in the lungs and intestines contributes to the production of thick, viscid secretions low in
24 both sodium and chloride.
25
26 (Choices A, B, C, and E) Class I mutations of the CFTR gene cause defective protein synthesis, leading to a
27 total absence of the CFTR protein at the apical surface of epithelial cells. Class II mutations of the CFTR
28 gene are associated w ith abnormalities in protein folding and processing, w hich result in degradation of the
29 aberrant protein before it reaches the surface. Lastly, epithelial sodium channel overexpression occurs in
30 Liddle's syndrome. None of these states is responsible for the electrolyte differences described in the table
31 above.
32
33 Educational objective:
34 Electrolyte differences in the body fluids of cystic fibrosis patients develop because of tissue-specific variation
35 in the functioning of the cystic fibrosis transmembrane conductance regulator.
36
37 Time Spent: 41 seconds 64% answ ered correctly. Last updated: [3/27/201 O].
38 Copyright © USMLEWORLD,LLC. SimExam ver. 2011 .11.397425
1 A

2 Item: 19 of 46 II P Mark -<:I t>-


Q. Id : 6494 (388147] Previous Next
3
4
5
6 A young Caucasian male has prominent skeletal abnormalities including short stature, short fingers, and
7 round face.:His serum calcium level is low and his parath~roid hormone level is high. W hich of the follow ing
8 is the most likely cause of this patient's condition?
9
10
A. ThirEl pharyngeal pebie:h hypeplasia [Hlq~l
11
12 B. Parathyroid adenoma (10%]
13 C. MeElbillary thyrei8 e:are:inema [2q~l
14
D. 1,25-dihydroxycholecalciferol overproduction (1 %]
15
16 .; @ E. End-organ resistance to PTH (71 % ]
17
18
Explanation: User Id: 388147
20
Albright hereditary osteodystrophy (AHO) is the classic phenotype of pseudohypoparathyroidism type
21
1a. AHO is characterized by skeletal and developmental defects (short stature, short metacarpal and
22
metatarsal bones) and end-organ resistance to parathyroid hormone (PTH), thyroid stimulating hormone
23
(TSH), and luteinizing hormone/follicle stimulating hormones (LH/FSH). These hormones stimulate G,a-
24
25 mediated pathways in select tissues (G, is a G protein that facilitates PTH actions on bone and kidney). Most
26 noticeable clinically is PTH resistance, w hich manifests w ith hypocalcemia, hyperphosphatemia, and elevated
27 PTH.
28
29 (Choice A) A hypoplastic third pharyngeal pouch is associated w ith DiGeorge syndrome, w hich is
30 characterized by variable hypoparathyroidism (low PTH), hypocalcemia, defective T cell-mediated immunity,
31 and cardiac and great vessel abnormalities.
32
33 (Choice B) Parathyroid adenoma is a common cause of hyperparathyroidism, manifesting w ith
34 hypercalcemia, bone pain, nephrolithiasis, gastrointestinal disturbances, depression, fatigue, and cardiac
35 valve calcifications ("painful bones, renal stones, abdominal groans, and psychic moans").
36
37 (Choice C) Medullary thyroid carcinoma is associated w ith multiple endocrine neoplasia (MEN) syndrome 2A
38 and 28. Features of MEN 2A include medullary carcinoma, pheochromocytoma, and parathyroid hyperplasia .
1 A

2 Item: 19 of 46 II P Mark -<:I t>-


Q. Id : 6494 (388147] Previous Next
3
4 G . Mem.mary 1nyre1s carc1nema [£~"]

5 D. 1,25-dihydroxycholecalciferol overproduction [1 %]
6
.; @ E. End-organ resistance to PTH [71 %]
7
8
9 Explanation: User Id: 388147
10
11 Albright hereditary osteodystrophy (AHO} is the classic phenotype of pseudohypoparathyroidism type
12 1a. AHO is characterized by skeletal and developmental defects (short stature, short metacarpal and
13 metatarsal bones} and end-organ resistance to parathyroid hormone (PTH}, thyroid stimulating hormone
14 (TSH}, and luteinizing hormone/follicle stimulating hormones (LH/FSH}. These hormones stimulate G,a-
15
mediated pathways in select tissues (G, is a G protein that facilitates PTH actions on bone and kidney}. Most
16
17 noticeable clinically is PTH resistance, w hich manifests w ith hypocalcemia, hyperphosphatemia, and elevated
18 PTH.

20 (Choice A) A hypoplastic third pharyngeal pouch is associated w ith DiGeorge syndrome, w hich is
21 characterized by variable hypoparathyroidism (low PTH}, hypocalcemia, defective T cell-mediated immunity,
22 and cardiac and great vessel abnormalities.
23
24 (Choice B) Parathyroid adenoma is a common cause of hyperparathyroidism, manifesting w ith
25 hypercalcemia, bone pain, nephrolithiasis, gastrointestinal disturbances, depression, fatigue, and cardiac
26 valve calcifications ("painful bones, renal stones, abdominal groans, and psychic moans"}.
27
28 (Choice C) Medullary thyroid carcinoma is associated w ith multiple endocrine neoplasia (MEN} syndrome 2A
29 and 28. Features of MEN 2A include medullary carcinoma, pheochromocytoma, and parathyroid hyperplasia.
30
31 (Choice D) Overproduction of the active form of vitamin D (1, 25-dihydroxycholecalciferol} is associated w ith
32 increased serum calcium and decreased PTH levels.
33
34 Educational objective:
35 End-organ resistance to PTH is associated w ith elevated serum PTH and lov1 serum calcium levels.
36
37 Time Spent: 115 seconds 71 % answ ered correctly. Last updated: [3/27/201 O].
38 Copyright © USMLEWORLD,LLC. SimEx am ver. 2011 .11.397425
1 A

2 Item: 20 of 46 II P Mark -<:I t>-


Q. Id : 6503 (388147] Previous Next
3
4
5
6
7
8
W hich of the following best explains the observed findings?
9
10
11 A. DNA rearrangement [7%]
12 X @ B. Transcription factor effect [1 0%]
13
C. DNA splicing [27%]
14
15 .; D. RNA splicing [56%]
16 E. DNA mutation [ 1% ]
17
18
19 Explanation: User Id: 388147

21 The diagram show s a mature mRNA molecule binding w ith its complementary DNA sequence. "Mature"
22 mRNA refers to mRNA that has been processed and is ready for export from the nucleus and translation into
23 protein. mRNA processing (post-transcriptional modification) involves three main steps:
24
1. Removal of introns: lntrons are non-coding segments of a gene that are transcribed together w ith
25
exons into the primary mRNA transcript. lntrons must be removed for protein translation to occur, an
26
energy-dependent process facilitated by snRNPs and snRNAs. The DNA loops seen in the above
27
image are the regions of the gene that code for introns, w hich have since been removed in the post-
28
transcriptional modification of the corresponding mRNA.
29
2. 5' capping: 7-methyl-guanosine is added to the 5' end of the primary mRNA transcript via a 5'-5'
30
triphosphate linkage. This modification helps stabilize mRNA in the cytosol.
31
3. Poly-A tail addition: Follow ing transcription, a tail rich in adenine residues is added to the mRNA
32
transcript by poly-A polymerase. The poly-A tail stabilizes the 3' end of mRNA in the cytosol.
33
34
35 (Choice A) DNA (gene) rearrangement occurs during antibody and T-cell receptor production and
36 maturation. In these instances, the mRNA produced fully complements the DNA sequence from w hich it is
37 transcribed, w ithout the loops pictured above.
38
1 A

2 Item: 20 of 46 II P Mark -<:I t>-


Q. Id : 6503 (388147] Previous Next
3
4 Explanation: User Id: 388147
5
6 The diagram show s a mature mRNA molecule binding w ith its complementary DNA sequence. "Mature"
7 mRNA refers to mRNA that has been processed and is ready for export from the nucleus and translation into
8 protein. mRNA processing (post-transcriptional modification) involves three main steps:
9
10 1. Removal of introns: lntrons are non-coding segments of a gene that are transcribed together w ith
11 exons into the primary mRNA transcript. lntrons must be removed for protein translation to occur, an
12 energy-dependent process facilitated by snRNPs and snRNAs. The DNA loops seen in the above
13 image are the regions of the gene that code for introns, w hich have since been removed in the post-
14 transcriptional modification of the corresponding mRNA.
15 2. 5' capping: 7-methyl-guanosine is added to the 5' end of the primary mRNA transcript via a 5'-5'
16 triphosphate linkage. This modification helps stabilize mRNA in the cytosol.
17 3. Poly-A tail addition: Follow ing transcription, a tail rich in adenine residues is added to the mRNA
18 transcript by poly-A polymerase. The poly-A tail stabilizes the 3' end of mRNA in the cytosol.
19 -
(Choice A) DNA (gene) rearrangement occurs during antibody and T-cell receptor production and
21 maturation. In these instances, the mRNA produced fully complements the DNA sequence from w hich it is
22 transcribed, w ithout the loops pictured above.
23
24 (Choice B) Transcription factors affect RNA polymerase's affinity for specific genes by binding to DNA
25 promoter sequences, to one another, and to RNA polymerase itself. Some transcription factors stimulate
26 gene transcription and others have an inhibitory function.
27
28 (Choice C) DNA splicing does not occur under normal circumstances.
29
30 (Choice E) A DNA mutation would be reflected in the mRNA transcript and would therefore not affect
31 complementary binding .
32
33 Educational objective:
34 Newly transcribed mRNA undergoes splicing, a process w hereby introns are removed, before it can leave the
35 nucleus. Thus, the exons in DNA w ill find complementary binding in mature mRNA, but DNA introns w ill
36 not. If mature mRNA is bound to the DNA from w hich it w as transcribed, there w ill be loops of unbound DNA
37 flanked by stretches of bound DNA.
38
1 A

2 Item: 21 of 46 II P Mark -<:I t>-


Q. Id : 6506 (388147] Previous Next
3
4 Duc.herme musc,ular dystrophy is a disease of progressive muscle w eakness caused by a mutation in the
5 dystrophin gene. T ypically, symptoms start before ag.M, w ith patients w h,eelchair-bOUJld by age 12. In
6 contrast, Becker muscular dystrophy, a disease caused by a mutation in the same gene, is marked by
7 symptoms beginning in,Jate:_cpildhood, w ith patients remaining able to. "fellS.ioto their late teenage years. The
8 difference in clinical presentation between these two conditions is best explained by:
9
10
A. Pelygenie: Elisease (6%•]
11
12 ., @ B. Allelic heterogeneity [46%]
13 C. Variable expressivity [41 %]
14
D. i\ntitiflatien (2%•]
15
16 E. lne:emfilete flenetrane:e (5%•]
17
18
19 Explanation: User Id: 388147
20
Allelic heterogeneity describes instances w hen different mutations in the same genetic locus cause similar
22 phenotypes. The underlying concept is that more than one type of mutation is possible in a given gene, and
23 different mutations can cause alterations in or loss of protein function resulting in disease. Both Duchenne
24 and Becker muscular dystrophy result from mutations in the dystrophin gene, w hich resides on the short arm
25 of the X chromosome. They present differently, how ever, because the mutation causing Duchenne dystrophy
26
causes complete loss of dystrophin function, w hile the mutation causing Becker dystrophy results in a
27
structurally abnormal but partially functional dystrophin gene product. Allelic heterogeneity is not to be
28 confused w ith genetic heterogeneity, w here mutations of different genes cause similar phenotypes, or w ith
29 phenotypic heterogeneity, w here mutations in the same gene result in different phenotypes.
30
31
(Choice A) Polygenic diseases result from defects in numerous, often unrelated genes. A classic example is
32 type II diabetes, w hich clearly has a heritable component but cannot be traced to any specific gene in most
33 cases.
34
35
(Choice C) Expressivity refers to variations in phenotypic severity between individuals w ith the same gene
36 mutation.
37
38 (Choice D) Anticipation occurs in trinucleotide repeat diseases such as Huntington disease, and refers to an
1 A

2 Item: 21 of 46 II P Mark -<:I t>-


Q. Id : 6506 (388147] Previous Next
3
4
5
6 Explanation: User Id: 388147
7
8 Allelic heterogeneity describes instances w hen different mutations in the same genetic locus cause similar
9 phenotypes. The underlying concept is that more than one type of mutation is possible in a given gene, and
10 different mutations can cause alterations in or loss of protein function resulting in disease. Both Duchenne
11 and Becker muscular dystrophy result from mutations in the dystrophin gene, w hich resides on the short arm
12 of the X chromosome. They present differently, how ever, because the mutation causing Duchenne dystrophy
13 causes complete loss of dystrophin function, w hile the mutation causing Becker dystrophy results in a
14 structurally abnormal but partially functional dystrophin gene product. Allelic heterogeneity is not to be
15 confused w ith genetic heterogeneity, w here mutations of different genes cause similar phenotypes, or w ith
16 phenotypic heterogeneity, w here mutations in the same gene result in different phenotypes.
17
18 (Choice A) Polygenic diseases result from defects in numerous, often unrelated genes. A classic example is
19 type II diabetes, w hich clearly has a heritable component but cannot be traced to any specific gene in most
20 cases.

22 (Choice C) Expressivity refers to variations in phenotypic severity between individuals w ith the same gene
23 mutation.
24
25 (Choice D) Anticipation occurs in trinucleotide repeat diseases such as Huntington disease, and refers to an
26 earlier onset of disease w ith successive generations due to progressive expansion of the repeat.
27
28 (Choice E) Penetrance refers to the proportion of individuals w ith a given genotype that express the
29 associated phenotype. Incomplete penetrance occurs w hen less than 100% of individuals w ith a genotype
30 express its phenotype.
31
32 Educational objective:
33 Allelic heterogeneity exists w hen different mutations in the same genetic locus cause similar phenotypes. The
34 heterogeneity of disease may be explained by the fact that some mutations cause complete loss of protein
35 function w hile others cause only partial loss of protein function.
36
37 Time Spent: 80 seconds 46% answ ered correctly. Last updated: [3/27/201 O].
38 Copyright © USMLEWORLD,LLC. SimExam ver. 2011 .11.397425
1 A

2 Item: 22 of 46 II P Mark -<:I t>-


Q. Id : 6508 (388147] Previous Next
3
4
5
6 A 10-year-old male comi:ilains of exercise intolerance. He describes ei:iisodes of br~thlessness, chest
7 tightness, and w heezing after moderate exercise, especially in cold w eatherc W hich one of the follow ing pairs
8 of chemical mediators and antagopist§. js most involved in the pathogenesis of this boy's condition?
9
10
11
A. Histamine - clemastine [14%]
12 x @ B. Prostaglandin E 2 - indomethacin [6%]
13
14
C. Leukotriene 0 4 - montelukast [76%]
15 0 . Thromboxane A 2 - aspirin [4%]
16
17 E. Serotonin - sumatriptan [ 1% ]
18
19
Explanation: User Id: 388147
20
21
The presentation of exercise intolerance, breathlessness, chest tightness, w heezing, and sensitivity to cold
w eather in this child is strongly suggestive of exercise-induced asthma . The airway obstruction associated
23 w ith asthma develops secondary to two major mechanisms: 1) pulmonary inflammation caused by
24 interactions between cells and chemical mediators; and 2) bronchial hyperresponsiveness to stimuli such as
25 exercise or cold climate. Eosinophils and mast cells synthesize cysteinyl-containing leukotrienes (eg,
26
leukotriene C4, leukotriene 0 4, leukotriene E4) that play important roles in the pathogenesis of bronchial
27
asthma by inducing bronchospasm and increasing bronchial mucus secretion. An anti-leukotriene medication
28 like montelukast antagonizes leukotriene 0 4 activity at the cysteinyl leukotriene receptor.
29
30 (Choices A, B, D, and E) Asthma is not associated w ith the activity of histamine, prostaglandin E 2
31
32 thromboxane Ai, or serotonin and therefore cannot be treated w ith antagonists of these chemical mediators.
33
34 Educational objective:
35 Synthesized by eosinophils and mast cells, leukotriene 0 4 plays an important role in the pathogenesis of
36 bronchial asthma by inducing bronchospasm and increasing bronchial mucus secretion. An anti-leukotriene
37 medication like montelukast antagonizes leukotriene 0 4 activity at the cysteinyl leukotriene receptor.
38
1 A

2 Item: 23 of 46 II P Mark -<:I t>-


Q. Id: 6512 (388147] Previous Next
3
4
5
6 A ~year-old Caucasian male coml'.)lains of iqcreased fatigability, dry mouth, and impote ce. Phys ical
7 examination reveals mild hei:>atomegaly and atroJ)Qi<;,,,t estes. His fasting blood glucose level is ,252..mg/dl.
8 His urine is pos[tive:.foi:. glucose but negative for ketones and protein. He also says that he developed a
9 '\strerigeJ ap" despite spending almost all his time indoors. W hich of the follow ing is most likely responsible
10 for the skin changes in this patient?
11
12
X @ A. High serum ACTH concentration (46%]
13
14 .; B. Dermal hemosiderin deposition (44%]
15 C. Abnormal melanocyte migration (2%]
16
17
0 . Impaired tyrosine metabolism [4% ]
18 E. Reduced serum copper content (4%]
19
20
21 Explanation: User Id: 388147
22
Hemochromatosis is an autosomal recessive disease characterized by the excessive gastrointestinal
24 absorption of iron. Affected individuals store 0.5-1 .0 grams of iron each year in the form of hemosiderin w ithin
25 the dermis and various parenchymal organs. The disease is typically silent in early adulthood, presenting only
26 once at least 20 grams of iron have been accumulated. After the age of 40, common clinical manifestations
27 include liver disease w ith hepatomegaly, skin hyperpigmentation (particularly in sun-exposed areas), diabetes
28 mellitus secondary to pancreatic islet destruction, impotence, arthropathy, and cardiac dysfunction and
29 enlargement. Laboratory findings include mildly elevated liver function tests, elevated plasma iron w ith more
30 than 50% saturation of transferrin, and elevated serum ferritin.
31
32 (Choice A) Increased levels of serum ACTH (as seen in primary adrenal insufficiency) can cause skin
33 hyperpigmentation w hen ACTH binds to melanocyte-stimulating hormone receptors.
34
35 (Choice C) Piebaldism (partial albinism) and W aardenburg syndrome are thought to be caused by defects in
36 melanocyte proliferation and migration. Additionally, the blue-gray macules referred to as "Mongolian spots"
37 arise w hen melanocytes are halted in the dermis as they migrate from the neural crest to the epidermis.
38
1 A

2 Item: 23 of 46 II P Mark -<:I t>-


Q. Id: 6512 (388147] Previous Next
3
LI . 111 l tJOll CU l Y I V~ll IC I I IClOUVll::.1 11 l "t FU J
4
5 E. Reduced serum copper content [4%]
6
7
8 Explanation: User Id: 388147
9
10 Hemochromatosis is an autosomal recessive disease characterized by the excessive gastrointestinal
11 absorption of iron. Affected individuals store 0.5-1 .0 grams of iron each year in the form of hemosiderin w ithin
12 the dermis and various parenchymal organs. The disease is typically silent in early adulthood, presenting only
13 once at least 20 grams of iron have been accumulated . After the age of 40, common clinical manifestations
14 include liver disease w ith hepatomegaly, skin hyperpigmentation (particularly in sun-exposed areas), diabetes
15 mellitus secondary to pancreatic islet destruction, impotence, arthropathy, and cardiac dysfunction and
16 enlargement. Laboratory findings include mildly elevated liver function tests, elevated plasma iron w ith more
17 than 50% saturation of transferrin, and elevated serum ferritin.
18
19 (Choice A) Increased levels of serum ACTH (as seen in primary adrenal insufficiency ) can cause skin
20 hyperpigmentation w hen ACTH binds to melanocyte-stimulating hormone receptors.
21
22 (Choice C) Piebaldism (partial albinism) and W aardenburg syndrome are thought to be caused by defects in
melanocyte proliferation and migration . Additionally, the blue-gray macules referred to as "Mongolian spots"
24 arise w hen melanocytes are halted in the dermis as they migrate from the neural crest to the epidermis.
25
26 (Choice D) Melanin is a brow n-black pigment formed in melanocytes w hen tyrosine is oxidized to form
27 dihydroxyphenylalanine (a reaction catalyzed by the tyrosinase enzyme). Individuals w ith albinism have
28 melanocytes that do not produce melanin because of absent or defective tyrosinase.
29
30 (Choice E) Serum copper content is decreased in individuals w ith Menkes' disease, w hich is associated w ith
31 abnormally pigmented, kinky hair and hypopigmented irises. Hyperpigmentation of the skin is not classically
32 seen.
33
34 Educational objective:
35 The skin hyperpigmentation seen in hematochromatosis results from hemosiderin deposition w ithin the dermis
36
37 Time Spent: 77 seconds 44% answ ered correctly. Last updated: [11/ 10/2011].
38 Copyright © USMLEWORLD,LLC. SimEx am ver. 2011 .11.397425
1 A

2 Item: 24 of 46 II P Mark -<:I t>-


Q. Id : 6517 (388147] Previous Next
3
4
5
6 A 45-year-old homeless man complains of vague ab,dornioal discomfort. He has a long history of hepatiti§..C,
7 infection and alcohol abuse. Based on this history, w hich of the follow ing structures below is most likely to
8 have increased pressures?
9
10
11
12
13
14
15
16
17
18
19 -
20
21
22
23

25
26
27
28
29
30
31
32 X @ A. A [51%]
33
34 B. 8 [1 2%]
35 C. C [1 %]
36 D. D [36%]
37
38
1 A

2 Item: 24 of 46 II P Mark -<:I t>-


Q. Id : 6517 (388147] Previous Next
3
\J. v L' FOJ
4
5 D. D [36%]
6
7
8 Explanation: User Id: 388147
9
10 Both chronic hepatitis C infection and chronic alcohol abuse are associated w ith hepatic cirrhosis. Cirrhosis
11 is an irreversible process caused by chronic injury to the hepatic parenchyma. In cirrhosis, hepatocyte
12 necrosis and extensive fibrosis cause distortion of the hepatic vascular bed and formation of disorganized
13 regenerative nodules. Disruption of the hepatic vascular bed causes complications like portal hypertension,
14 portosystemic anastomoses, splenomegaly and ascites.
15
16 Splenomegaly occurs in cirrhosis w hen portal hypertension prevents drainage of blood from the spleen via the
17 splenic vein. Recall that the portal venous system is valveless, so retrograde flow of portal blood away from
18 the liver occurs readily w hen there is portal hypertension.
19
20 (Choice A) Normally, the inferior vena cava receives blood from the portal system via the hepatic vein after it
21 has been filtered by the liver. In portal hypertension, the systemic venous circulation serves as a low pressure
22 reservoir into w hich portal blood can drain via portosystemic anastomoses. These anastomoses occur at the
23 rectum, esophagus, umbilicus and retroperitoneum.

25 (Choice B) The celiac trunk is an arterial vessel that arises directly from the aorta as pictured. The splenic
26 artery is a branch of the celiac trunk.
27
28 (Choice C) The aorta does not communicate directly w ith the portal venous system. Aortic pressures are
29 elevated in systemic hypertension. Prolonged aortic hypertension may lead to formation of an abdominal
30 aortic aneurysm.
31
32 Educational Objective:
33 Cirrhosis causes portal hypertension due to distortion of the hepatic vascular bed. The increased pressures
34 in the portal venous system cause splenic vein hypertension and splenomegaly secondary to venous
35 accumulation.
36
37 Time Spent: 79 seconds 36% answ ered correctly. Last updated: [11 / 10/2011].
38 Copyright © USMLEWORLD,LLC. SimExam ver. 2011 .11.397425
1 A

2 Item: 25 of 46 II P Mark -<:I t>-


Q. Id : 6522 (388147] Previous Next
3
4
5
6 A new test is designed for diagnosing parvovirus 819 infection . The test uses parvovirus 819 antigen
7 attached to w ells along w ith a chromogen for a peroxidase enzyme. W hat else should be added to make the
8 test kit complete?
9
10
X @ A. Antibodies against parvovirus 819 [33%]
11
12 B. i\nti9e8ies against perexiElase (2Q%,]
13 C. Anti-human immunoglobulin [35%]
14
D. Latex particles [9%]
15
16 E. Cemplement e:empenents (<l%•]
17
18
19 Explanation: User Id: 388147
20
Indirect ELISA (enzyme-linked immunosorbent assay) is a serologic test that identifies the presence of
21
antibody directed against a know n target antigen in a patient's serum. The steps are as follow s:
22
23 1. A know n antigen (from parvovirus 819 in this case) is fixed to the bottom of a w ell.
24 2. Patient serum is added. If present, antigen-specific antibodies bind and remain fixed to the w ell. The
plate is then w ashed to remove unbound patient antibodies.
26 3. Anti-human immunoglobulin antibody (Choice C) coupled to a substrate-modifying enzyme ("enzyme-
27 linked"} is added. This antibody binds to antigen-bound antibodies in the w ells. The plate is again
28 w ashed.
29 4. A substrate, or chromogen, is added, w hich is modified by the enzyme to elicit a detectable signal
30 such as a color change.
31
32
(Choice A) This kit tests for the presence of antibodies against parvovirus 8 19 in patient serum .
33
34
(Choice B) Peroxidase is a substrate-modifying enzyme that, w hen bound to the anti-human immunoglobulin
35
added in Step #3 above serves as a marker for the antibodies being sought.
36
37
(Choice D) The latex agglutination assay can be used to detect a specific antigen in a sample of
38
1 A

2 Item: 25 of 46 II P Mark -<:I t>-


Q. Id : 6522 (388147] Previous Next
3
4 Explanation: User Id: 388147
5
6 Indirect ELISA (enzyme-linked immunosorbent assay) is a serologic test that identifies the presence of
7 antibody directed against a know n target antigen in a patient's serum . The steps are as follow s:
8
1. A know n antigen (from parvovirus 819 in this case) is fixed to the bottom of a w ell.
9
2. Patient serum is added. If present, antigen-specific antibodies bind and remain fixed to the w ell. The
10
plate is then w ashed to remove unbound patient antibodies.
11
3. Anti-human immunoglobulin antibody (Choice C) coupled to a substrate-modifying enzyme ("enzyme-
12
linked"} is added. This antibody binds to antigen-bound antibodies in the w ells. The plate is again
13
w ashed.
14
4. A substrate, or chromogen, is added, w hich is modified by the enzyme to elicit a detectable signal
15
such as a color change.
16
17
18 (Choice A) This kit tests for the presence of antibodies against parvovirus B 19 in patient serum .
19
20 (Choice B) Peroxidase is a substrate-modifying enzyme that, w hen bound to the anti-human immunoglobulin
21 added in Step #3 above serves as a marker for the antibodies being sought.
22
23 (Choice D) The latex agglutination assay can be used to detect a specific antigen in a sample of
24 interest. This assay is carried out by adding the sample to a collection of specific antibodies fixed to latex
beads.
26
27 (Choice E) Complement fixation tests involve combining complement w ith know n antigen, unkno\"ln patient
28 serum and antibody-coated sheep erythrocytes.
29
30 Educational objective:
31 The ELISA is a serologic test that identifies antibodies in patient serum specific for an antigen of
32 interest. Know n antigen is fixed to a plate and patient serum is added. The next step is the addition of anti-
33 human immunoglobulin antibodies, each fixed to an enzyme such as peroxidase. These bind the antigen-
34 bound immunoglobulin from the patient's serum, and w hen a chromogen substrate is subsequently added, it
35 is modified by the linked enzymes to a detectable form (color change).
36
37 Time Spent: 69 seconds 35% answ ered correctly. Last updated: [3/27/201 O].
38 Copyright © USMLEWORLD,LLC. SimExam ver. 2011 .11.397425
1 A

2 Item: 26 of 46 II P Mark -<:I t>-


Q. Id : 6524 (388147] Previous Next
3
4
5
6 A 24-year-old male accidentally nicks his chin w hile shaving. Over the next tw o days, his cut grow s
7 increasingly inflamed and develops a yellow ish C[!JS,t. W hich of the follow ing is most likely responsible for this
8 change?
9
10
11
A. Pseudomonas aeruginosa [1 %]
12
~ B. Staphylococcus aureus [71 %]
13
14
15
x@ C. Staphylococcus epidermidis [22%]
16 D. Group B Streptococcus [5%]
17
18 E. Bacteroides fragilis [1 %]
19
20
21
22 Explanation: User Id: 388147
23
24 This patient has prolonged erythema and a yellow ("honey") crust on a cutaneous w ound, classic findings in
25 impetigo. The most common causes of impetigo are Staphylococcus aureus and Group A Streptococcus.
Impetigo can also complicate chronic inflammatory lesions like eczema (secondary impetigo).
27
28 (Choice A) P. aeruginosa does not cause impetigo, but often underlies skin diseases like ecthyma
29 gangrenosum, hot tub folliculitis, pyoderma, green nail syndrome and botryomycosis.
30
31 (Choice C) S. epidermidis is a component of normal skin flora and is not commonly associated w ith skin
32 disease. It should be regarded as a contaminant if found in a w ound culture.
33
34 (Choice D) Group B streptococcus is most classically associated w ith neonatal sepsis, chorioamnionitis and
35 endometritis.
36
37 (Choice E) B. fragilis is most commonly associated w ith intraabdominal abscesses follow ing abdominal
38 trauma or surgery. It is also a cause of endometritis and intraabdominal abscesses follow ing gynecologic
39 '
40
1 A

2 Item: 26 of 46 II P Mark -<:I t>-


Q. Id : 6524 (388147] Previous Next
3
V ...... '-'~""f" ' ~ T'""'"' ... '"''"'"'"'-' "'"'"' """"'""" l ' ' ' "'J
4
5
x @ C. Staphylococcus epidern1idis [22%]
6
7 D. Group B Streptococcus [5%]
8
9 E. Bacteroides fragilis [ 1%]
10
11
12
13 Explanation: User Id: 388147
14
15 This patient has prolonged erythema and a yellow ("honey") crust on a cutaneous wound, classic findings in
16 impetigo. The most common causes of impetigo are Staphylococcus aureus and Group A Streptococcus.
17 Impetigo can also complicate chronic inflammatory lesions like eczema (secondary impetigo).
18
19 (Choice A) P. aeruginosa does not cause impetigo, but often underlies skin diseases like ecthyma
20 gangrenosum, hot tub folliculitis, pyoderma, green nail syndrome and botryomycosis.
21
22 (Choice C) S. epidermidis is a component of normal skin flora and is not commonly associated w ith skin
23 disease. It should be regarded as a contaminant if found in a wound culture.
24
25 (Choice D) Group B streptococcus is most classically associated w ith neonatal sepsis, chorioamnionitis and
endometritis.
27
28 (Choice E) B. fragilis is most commonly associated w ith intraabdominal abscesses following abdominal
29 trauma or surgery. It is also a cause of endometritis and intraabdominal abscesses following gynecologic
30 procedures.
31
32 Educational objective:
33 Impetigo is a superficial infection of the skin associated w ith erythema and a yellow "honey" crust. It is most
34 commonly caused by S. aureus and Group A Streptococcus. In cases caused by S. aureus, bullae may form
35 due to toxin production {bullous impetigo).
36
37 Time Spent: 34 seconds 71 % answ ered correctly. Last updated: [3/27/201 O].
38 Copyright © USMLEWORLD,LLC. SimExam ver. 2011 .11.397425
1 A

2 Item: 27 of 46 II P Mark -<:I t>-


Q. Id : 6526 (388147] Previous Next
3
4
5
6 A scientist is studying acquired antibiotic resistance in a Rarticular strain of bacteria . Resistant bacteria are
7 noted to have decr~asedjotrecellulaLconceptr.ations_oUQe,J3n,tibiotiG...,of ipter.e st. W hen the scientist adds
8 protons to the growth media, intrabacterial concentrations of the drug sharply increase. The most likely
9 mechanism of acquired antibiotic resistance in these bacteria is:
10
11
12
A. Poor drug penetrance into the cell [21 %]
13 B. Enzymatic drug destruction [1 6%]
14 .; @ C. Drug pumped out of the cell [56%]
15
16
D. Impaired drug-target binding [5%]
17 E. Alternative route of metabolism bypassing the drug effect [1 %]
18
19
20 Explanation: User Id: 388147
21
22 This question describes bacteria that develop resistance by decreasing their intracellular accumulation of
23 drug. Potentially, this could occur by bacteria decreasing their permeability to the drug or acquiring an ability
24 to pump the drug out. In general, most antibiotics freely cross bacterial membranes, making efflux pumping
25 the more likely mechanism of resistance here. Efflux pumping is an active, energy-requiring process. Most
26 antibiotic efflux pumps are powered by moving protons (hydrogen ions) along their concentration gradient out
of the cell coupled w ith simultaneously expelling the antibiotic against its concentration gradient. W hen the
28 scientist here saturates the medium w ith protons, she eliminates the hydrogen ion concentration gradient
29 energy source, thereby inhibiting the efflux pumps.
30
31 Educational objective:
32 Bacterial antibiotic efflux pumps require energy to move antibiotics against their concentration gradients out of
33 the cell. This energy is most commonly derived from a proton gradient, though sodium gradients or direct
34 ATP expenditure may also be used in some cases.
35
36 Time Spent 181 seconds 56% answered correctly. Last updated: [11 / 10/2011].
37 Copyright © USMLEWORLD,LLC. SimExam ver. 2011 .11.397425
38
1 A

2 Item: 28 of 46 II P Mark -<:I t>-


Q. Id : 6531 (388147] Previous Next
3
4
5
6 An infant w ith a complicated family history is diagnosed w ith a genetic abnormali!Y mapRed to the short arm of
7 chnomosome 4. /ll. ROint mutation substituting Arg foLGly in,Jf1e 375 position of fibroblast growth facto
8 receptor:&_(E.GER-3) is identified. The patient is most likely to have w hich of the follow ing?
9
10
11
A. Fused and extra digits [3%]
12 B. Short extremities [70%]
13 C. Lens ectopy [9%]
14
15
X @ D. Bone fragility [ 13%]
16 E . Ew ing sarcoma [5%]
17
18
19 Explanation: User Id: 388147
20
21 A common cause of dwarfism, achondroplasia is an autosomal dominant condition characterized by point
22 mutation of the gene for fibroblast growth factor receptor 3 (FGFR-3) on chromosome band 4p16.3. This
23 mutation causes a defect in paracrine cell signaling that leads to increased function of FGFR-3, w hich is
24 responsible for inhibition of cartilage proliferation. Consequently, individuals w ith achondroplasia have
25 severely restricted chondrocyte proliferation in the growth plate cartilage, decreased endochondral
26 ossification, and reduced cellular hypertrophy and cartilage matrix production . These changes cause specific
27 clinical manifestations, including shortened proximal extremities (with a normal trunk length) and an enlarged
head w ith frontal bossing.
29
30 (Choice A) Fused digits (syndactyly) and extra digits (polydactyly) can occur as isolated events or as features
31 of systemic conditions. For instance, syndactyly is commonly seen in Apert syndrome (which is caused by
32 missense substitution mutations of FGFR-2 on chromosome band 1Oq25-q26), and polydactyly is commonly
33 seen in Patau syndrome (which is caused by trisomy 13).
34
35 (Choice C) Lens ectopy is a hallmark of Marfan syndrome, w hich is typically caused by mutation of the FBN1
36 gene on chromosome 15q21. 1.
37
38 (Choice D) Bone fragility is characteristic of conditions such as osteogenesis imperfecta, w hich is caused by
,. ,. ' ,. '
39
40
1 A

2 Item: 28 of 46 II P Mark -<:I t>-


Q. Id : 6531 (388147] Previous Next
3
4 E. Ew ing sarcoma [5%]
5
6
7 Explanation: User Id: 388147
8
9 A common cause of dwarfism, achondroplasia is an autosomal dominant condition characterized by point
10 mutation of the gene for fibroblast growth factor receptor 3 (FGFR-3) on chromosome band 4p16.3. This
11 mutation causes a defect in paracrine cell signaling that leads to increased function of FGFR-3, w hich is
12 responsible for inhibition of cartilage proliferation . Consequently, individuals w ith achondroplasia have
13 severely restricted chondrocyte proliferation in the growth plate cartilage, decreased endochondral
14 ossification, and reduced cellular hypertrophy and cartilage matrix production . These changes cause specific
15 clinical manifestations, including shortened proximal extremities (with a normal trunk length) and an enlarged
16 head w ith frontal bossing.
17
18 (Choice A) Fused digits (syndactyly) and extra digits (polydactyly) can occur as isolated events or as features
19 of systemic conditions. For instance, syndactyly is commonly seen in Apert syndrome {which is caused by
20 missense substitution mutations of FGFR-2 on chromosome band 1Oq25-q26), and polydactyly is commonly
21 seen in Patau syndrome (which is caused by trisomy 13).
22
23 (Choice C) Lens ectopy is a hallmark of Marfan syndrome, w hich is typically caused by mutation of the FBN1
24 gene on chromosome 15q21.1.
25
26 (Choice D) Bone fragility is characteristic of conditions such as osteogenesis imperfecta, w hich is caused by
27 mutation of gene COL 1A1 on band 17q21 or mutation of gene COL1A2 on band 7q22 .1.

29 (Choice E) Ew ing sarcoma is a bone tumor that most commonly manifests in the pediatric population. It is
30 associated w ith t(11 ;22), w hich is a joining of the EWS gene from chromosome 22 to the FL/1 gene on
31 chromosome 11 .
32
33 Educational objective:
34 Achondroplasia is characterized by a point mutation substituting Arg for Gly in the 375 position of the gene for
35 fibroblast growth factor receptor 3 (FGFR-3) on chromosome band 4p16.3
36
37 Time Spent: 55 seconds 70% answ ered correctly. Last updated: [3/27/201 O].
38 Copyright © USMLEWORLD,LLC. SimExam ver. 2011 .11.397425
1 A

2 Item: 29 of 46 II P Mark -<:I t>-


Q. Id : 6533 (388147] Previous Next
3
4 An 1nrant surrenng rrom recurrent 1nrecuons 1s aiagnosea w nn an 1ntrace11u1ar messaging aonormauty. 1 ne
5 abnormal gene codes for a cytoplasmic tyrosine kinase. Normally, this tyrosine kinase is activated in CD19-
6 positive cells by antigen exposure, resulting in cell maturation. The abnormal gene responsible for this patient'
7 s condition can be mapped to w hich of the follow ing?
8
9 A. Chromosome 7 [15%]
10
X @ B. Chromosome 16 [8%]
11
12 C. Chromosome 20 [6%]
13 D. Chromosome 22 [30%]
14
E. X chromoson1e [42%]
15
16
17
Explanation: User Id: 388147
18
19 Bruton agammaglobulinemia (X-linked agammaglobulinemia} is a rare X-linked genetic condition that results
20 from a defect in the signal transduction molecule known as Bruton's tyrosine kinase (BTK}. This protein is
21 important in the maturation of B cells; w ithout it, B cells cannot leave the bone marrow to enter circulation, and
22 antibodies are not produced. As a consequence, recurrent bacterial infections (especially of the respiratory
23 tract} are observed in affected infants. Laboratory evaluation demonstrates normal T cell numbers, but low or
24 absent B cell numbers (<1 % of CD19 or CD20 positive lymphocytes are present in circulation}.
25
26 (Choices A, 8, C, and D) Some of the more common diseases associated w ith genetic mutations on
27 chromosome 7 include cystic fibrosis, Ehlers-Danlos syndrome, and osteogenesis imperfecta, w hile some of
28 the more common diseases associated w ith genetic mutations on chromosome 16 are polycystic kidney
disease and tuberous sclerosis. One of the more common diseases associated w ith chromosome 20 is
30 maturity onset diabetes of the young type 1, and one of the more common diseases associated w ith
31 chromosome 22 is DiGeorge syndrome. Bruton agammaglobulinemia is not associated w ith defects on any
32 of these chromosomes.
33
34 Educational objective:
35 Bruton agammaglobulinemia is characterized by an intracellular messaging abnormality that results in
36 recurrent infections. The defective gene involved is on the X chromosome.
37
38
1 A

2 Item: 29 of 46 II P Mark -<:I t>-


Q. Id : 6533 (388147] Previous Next
3
- . .. .
4 positive cells by antigen exposure, resulting in cell maturation. The abnormal gene responsible for this patient'
5 s condition can be mapped to w hich of the follow ing?
6
7
A. Chromosome 7 [ 15%]
8
9 X @ B. Chromosome 16 [8%]
10 C. Chromosome 20 [6%]
11
12
D. Chromosome 22 [30%]
13 E. X chromoson1e [42%]
14
15
16 Explanation: User Id: 388147
17
18 Bruton agammaglobulinemia (X-linked agammaglobulinemia) is a rare X-linked genetic condition that results
19 from a defect in the signal transduction molecule know n as Bruton's tyrosine kinase (BTK). This protein is
20 important in the maturation of B cells; w ithout it, B cells cannot leave the bone marrow to enter circulation, and
21 antibodies are not produced. As a consequence, recurrent bacterial infections (especially of the respiratory
22 tract) are observed in affected infants. Laboratory evaluation demonstrates normal T cell numbers, but low or
23 absent B cell numbers (<1% of CD 19 or CD20 positive lymphocytes are present in circulation).
24
25 (Choices A, B, C, and D) Some of the more common diseases associated w ith genetic mutations on
26 chromosome 7 include cystic fibrosis, Ehlers-Danlos syndrome, and osteogenesis imperfecta, w hile some of
27 the more common diseases associated w ith genetic mutations on chromosome 16 are polycystic kidney
28 disease and tuberous sclerosis. One of the more common diseases associated w ith chromosome 20 is
maturity onset diabetes of the young type 1, and one of the more common diseases associated w ith
30 chromosome 22 is DiGeorge syndrome. Bruton agammaglobulinemia is not associated w ith defects on any
31 of these chromosomes.
32
33 Educational objective:
34 Bruton agammaglobulinemia is characterized by an intracellular messaging abnormality that results in
35 recurrent infections. The defective gene involved is on the X chromosome.
36
37 Time Spent: 34 seconds 42% answ ered correctly. Last updated: [3/27/201 O].
38 Copyright © USMLEWORLD,LLC. SimExam ver. 2011 .11.397425
1 A

2 Item: 30 of 46 II P Mark -<:I t>-


Q. Id : 6541 (388147] Previous Next
3
4
5
6 A 6-year-old female is diagnosed w ith acute lym~hocytic leukemia and undergoes allogeneic bone marrow
7 transplantation . Three months after the procedure, her WBC count is 4 500/mm 3 and karyotyping from a
8 serum sample reveals 46 XY composition. W hich of the follow ing best explains this finding?
9
10
/\. Male psebiElehermaphreElitism [1 7q~l
11
12 B. RaEliatien in8blE:e8 Gell injblry [1 q~l
13 ~ @ C. Daner karyetype [7€iq~l
14
D. Graft-versus-host disease [5%]
15
16 E. Cell-mediated transplant rejection [ 1% ]
17 F. Blast E:risis [Qq~l
18
19
20 Explanation: User Id: 388147
21
22 The process of bone marrow transplantation begins by conditioning the recipient w ith high-dose
23 chemotherapy or radiation to destroy all host bone marrow . Subsequently, HLA-matched donor hematopoietic
24 stem cells are infused intravenously. These stem cells preferentially engraft in the bone marrov1 due to
25 homing signals and are able to proliferate and regenerate the bone marrow . The engrafted stem cells are
26 thereafter the source of all hematopoietic cell lines (i.e. all erythrocytes, platelets and leukocytes) in the
27 recipient. The recipient patient described above is female, but her peripheral leukocytes have a male
28 karyotype. A male marrow donor would explain this finding.
29
(Choice A) Male pseudohermaphroditism occurs in genetic males (XY) w ho are feminized due to a defect in
31 testosterone production or action. An example is androgen insensitivity syndrome, w hich results from
32 mutations in the androgen receptor.
33
34 (Choice B) Radiation causes double-stranded fractures in DNA. These are difficult to repair properly
35 because there is no template strand from w hich to work.
36
37 (Choice D) Graft-versus-host disease may occur in hematopoietic stem cell transplant patients, but would
38 not cause a male karyotype. Graft-versus host disease results from destruction of recipient tissues by donor
1 A

2 Item: 30 of 46 II P Mark -<:I t>-


Q. Id : 6541 (388147] Previous Next
3
4
5 Explanation: User Id: 388147
6
7 The process of bone marrov1 transplantation begins by conditioning the recipient w ith high-dose
8 chemotherapy or radiation to destroy all host bone marrow . Subsequently, HLA-matched donor hematopoietic
9 stem cells are infused intravenously. These stem cells preferentially engraft in the bone marrow due to
10 homing signals and are able to proliferate and regenerate the bone marrow . The engrafted stem cells are
11 thereafter the source of all hematopoietic cell lines (i.e. all erythrocytes, platelets and leukocytes) in the
12 recipient. The recipient patient described above is female, but her peripheral leukocytes have a male
13 karyotype. A male marrow donor w ould explain this finding.
14
15 (Choice A) Male pseudohermaphroditism occurs in genetic males (XY) w ho are feminized due to a defect in
16 testosterone production or action. An example is androgen insensitivity syndrome, w hich results from
17 mutations in the androgen receptor.
18
19 (Choice B) Radiation causes double-stranded fractures in DNA. These are difficult to repair properly
20 because there is no template strand from w hich to w ork.
21
22 (Choice D) Graft-versus-host disease may occur in hematopoietic stem cell transplant patients, but w ould
23 not cause a male karyotype. Graft-versus host disease results from destruction of recipient tissues by donor
24 T-lymphocytes.
25
26 (Choice E) Acute rejection of solid organ transplants is cell-mediated and involves destruction of the
27 transplanted organ by the recipient patient's T-cells. This is usually due to HLA-antigen mismatch.
28
29 (Choice F) Blast crisis occurs in patients w ith chronic m yelogenous leukemia and refers to an accelerated
phase of the disease that is poorly responsive to therapy and is often fatal.
31
32 Educational objective:
33 W hen a patient receives a bone marrow transplant, the donor stem cells engraft in the bone marrow and
34 produce all of the hematopoietic cell lines for the recipient. Thus, the recipient's peripheral blood cells w ill be
35 genetically different from the rest of the body's cells.
36
37 Time Spent: 77 seconds 76% answ ered correctly. Last updated: [3/27/201 O].
38 Copyright © USMLEWORLD,LLC. SimExam ver. 2011 .11.397425
1 A

2 Item: 31of46 ll P Mark -<:I t>-


Q. Id : 6544 (388147] Previous Next
3
4
5
6 A grohormone peptide molecule has a signal sequence w ith a hydrophobic core. A large deletion in the
7 hyd OJlllobiG..,cor~..would most likely result in the protein's accumulation in w hich of the follow ing
8 compartments?
9
10
11
A. Endoplasmic reticulum [ 19%]
12 ~ B. Cytosol [44 %]
13 X @ C. Golgi complex [18%]
14
15 D. Endosomes [8%]
16 E . Secretion granules [11 %]
17
18
19 Explanation: User Id: 388147
20
21 Ribosomes facilitate the translation of mRNA into protein. This process begins in the cytosol, w hen mRNA
22 binds to a ribosome and initiates translation. Proteins destined for the endoplasmic reticulum possess
23 N-terminal peptide signaling sequences that identify them as such . These N-terminal sequences are
24 translated in the cytosol and are rapidly recognized by signal recognition particles (SRPs), w hich halt
25 translation and target the ribosome to protein pores in the rough endoplasmic reticulum (RER). Once the
26 ribosome is bound to the RER, the SRP dissociates and translation proceeds w ith the polypeptide being fed
27 into the RER through the membrane pore. Signal sequences that call for protein targeting to the RER are
28 composed of 15 to 20 amino acid residues, most of w hich are hydrophobic. Deletion of this hydrophobic
29 signaling sequence would cause failure of SRP binding and failure to translocate appropriate proteins into the
30 RER. These proteins would therefore inappropriately accumulate in the cytosol.

32 (Choices A, C, D & E) After a protein is targeted to the endoplasmic reticulum, signal peptidase cleaves the
33 signal peptide region from the elongating polypeptide w ithin the RER. W ithin the ER, proteins undergo folding,
34 glycosylation and coupling w ith other subunits required for function. From the ER, proteins are exported to the
35 Golgi complex via vesicles. In the Golgi, proteins undergo further posttranslational modification and are sorted
36 towards their ultimate destinations, be it into the extracellular space, into secretary granules for storage and
37 release upon stimulus, into endosomes and lysosomes for the degradation of phagocytosed material, or into
38 the plasma membrane.
1 A

2 Item: 31of46 ll P Mark -<:I t>-


Q. Id : 6544 (388147] Previous Next
3
4 x @ C. Golgi complex [18%]
5
6 D. Endosomes [8%]
7
E. Secretion granules [11 %]
8
9
10
11 Explanation: User Id: 388147
12
13 Ribosomes facilitate the translation of mRNA into protein. This process begins in the cytosol, w hen mRNA
14 binds to a ribosome and initiates translation. Proteins destined for the endoplasmic reticulum possess
15 N-terminal peptide signaling sequences that identify them as such. These N-terminal sequences are
16 translated in the cytosol and are rapidly recognized by signal recognition particles (SRPs), w hich halt
17 translation and target the ribosome to protein pores in the rough endoplasmic reticulum (RER). Once the
18 ribosome is bound to the RER, the SRP dissociates and translation proceeds w ith the polypeptide being fed
19 into the RER through the membrane pore. Signal sequences that call for protein targeting to the RER are
20 composed of 15 to 20 amino acid residues, most of w hich are hydrophobic. Deletion of this hydrophobic
21 signaling sequence would cause failure of SRP binding and failure to translocate appropriate proteins into the
22 RER. These proteins would therefore inappropriately accumulate in the cytosol.
23
24 (Choices A, C, D & E) After a protein is targeted to the endoplasmic reticulum, signal peptidase cleaves the
25 signal peptide region from the elongating polypeptide w ithin the RER. Within the ER, proteins undergo folding,
26 glycosylation and coupling w ith other subunits required for function. From the ER, proteins are exported to the
27 Golgi complex via vesicles. In the Golgi, proteins undergo further posttranslational modification and are sorted
28 towards their ultimate destinations, be it into the extracellular space, into secretary granules for storage and
29 release upon stimulus, into endosomes and lysosomes for the degradation of phagocytosed material, or into
30 the plasma membrane.

32 Educational objective:
33 The signal peptides that designate proteins for extrusion into the rough endoplasmic reticulum are N-terminal
34 hydrophobic sequences 15 to 20 amino acid residues in length. Removal of these signal sequences would
35 cause accumulation of ER proteins in the cytosol.
36
37 Time Spent: 51 seconds 44% answ ered correctly. Last updated: [3/27/201 O].
38 Copyright © USMLEWORLD,LLC. SimExam ver. 2011 .11.397425
1 •
2 Item: 32 of 46 • \' Mark -<:J C>
Q. Id: 6613 (383147) Previous NeHt
3
4
5
6 An infant born to an apparently health}' 39-year-old female experiences feeding problems and occ asional
7 vomiting The patient has severe metabolic acidosis and an increased amount of branched-c hain a-ke!o
8 acids in the blood and urine. The enzyme deficient in this patient is dependent on which of the following?
9
10
A. Biotin (18%)
11
12 ., B. Th1amuie (50%)
13 x ~' C. Folic acid (3%)
14
D. Pyridoxine (26%)
15
16 E. Retinol (2%)
17
18
19 Explanation: Use r Id: 388147
20
21
22
23
Phenylalanine

l Acetyl CoA +- - - - - - - - +----


Oxidative decarboxylation
Leuclne

Tyrosine
24 (Branched chain a-keto
25 I acid dehydrogenase)
I
26 I
27 L - + Fumarate TCA cycle
28
29
30
31

Succinyl CoA
33
34
35 T
Methy1malony1 CoA Oxidative decart>oxylatlon
36 (Branched chain a-keto
37 acid dehydrogenase)
38
39
T I
40
1 A

2 Item: 32 of 46 II P Mark -<:I t>-


Q. Id : 6613 (388147] Previous Next
3
4
5
6 Explanation: User Id: 388147
7
8
9
10
Phenylalanine

l Acetyl CoA +- - - - - - - - t ----


Oxidative decarboxylation
Leucine

Tyrosine
11 (Branched chain a-keto
12 I acid dehydrogenase)
I
13 I
14 ' - + Fumarate TCA cycle
15
16
17
18
19
Succinyl CoA
20
21
22 T
Methylmalonyl CoA
Oxidative decarboxytation
23 (Branched chain a-keto
24 acid dehydrogenase)
25 T
Propionyl CoA
+- ______ _[ _____ Valine
26
27
T lsoleucine
28
29 The infant described is most likely suffering from maple syrup urine disease (branched-chain ketoaciduria).
30 This disorder is characterized by a defect in the branched-chain alpha-keto acid dehydrogenase complex
31 resulting in an inability to degrade branched chain alpha-amino acids (leucine, isoleucine and valine) and their
alpha-ketoacid metabolites. This defect causes branched chain amino acids and alpha-ketoacids to
33 accumulate in the plasma and urine. Excessive levels of these amino acids and alpha-ketoacids causes
34 CNS toxicity, w hile the presence of large quantities of these compounds in the urine causes a characteristic
35 sw eet odor that smells like burnt sugar or maple syrup. Maple syrup urine disease can be life threatening if
36 left untreated; typical early symptoms include poor feeding and vomiting. Early diagnosis (prenatally or w ithin
37 the first w eek of life) w ith subsequent early restriction of branched chain amino acids in the diet is advocated.
38 Alpha-ketoacid decarboxylase requires thiamine pyrophosphate (vitamin B 1) as a coenzyme. Another
1 A

2 Item: 32 of 46 II P Mark -<:I t>-


Q. Id : 6613 (388147] Previous Next
3
4 The infant described is most likely suffering from maple syrup urine disease (branched-chain ketoaciduria).
5 This disorder is characterized by a defect in the branched-chain alpha-keto acid dehydrogenase complex
6 resulting in an inability to degrade branched chain alpha-amino acids (leucine, isoleucine and valine) and their
7 alpha-ketoacid metabolites. This defect causes branched chain amino acids and alpha-ketoacids to
8 accumulate in the plasma and urine. Excessive levels of these amino acids and alpha-ketoacids causes
9 CNS toxicity, w hile the presence of large quantities of these compounds in the urine causes a characteristic
10 sw eet odor that smells like burnt sugar or maple syrup. Maple syrup urine disease can be life threatening if
11 left untreated; typical early symptoms include poor feeding and vomiting. Early diagnosis (prenatally or w ithin
12 the first w eek of life) w ith subsequent early restriction of branched chain amino acids in the diet is advocated.
13 Alpha-ketoacid decarboxylase requires thiamine pyrophosphate (vitamin B 1) as a coenzyme. Another
14 enzyme that requires thiamine as a cofactor is transketolase.
15
16 (Choice A) Biotin is a cofactor required by enzymes that are involved in carboxylation such as acetyl-CoA
17 carboxylase and pyruvate carboxylase. Biotin deficiency is very uncommon.
18
19 (Choice C) Folic acid plays a crucial role in single-carbon transfer reactions in the form of tetrahydrofolate. It
20 is required for the synthesis of purines, thymine and certain amino acids. Folic acid deficiency can cause
21 neural tube defects in the fetus as w ell as megaloblastic anemia in adults.
22
23 (Choice D) Pyridoxine (pyridoxal, pyridoxamine, vitamin B6) functions as a cofactor in numerous reactions
24 including transamination, decarboxylation, condensation and deamination. The antituberculosis agent
25 isoniazid can induce a pyridoxine deficiency, so vitamin B6 supplementation is required w hen this drug is
26 prescribed.
27
28 (Choice E) Vitamin A (retinal) is converted to retinaldehyde and retinoic acid. W hile vitamin A does not
29 function as a coenzyme, it is required for normal vision, certain reproductive functions, maintenance of
30 epithelial differentiation and normal growth. Vitamin A deficiency causes night blindness, follicular
31 hyperkeratosis and xerophthalmia.

33 Educational objective:
34 Maple syrup urine disease is caused by a defect in the branched-chain alpha-keto acid dehydrogenase
35 complex, w hich is responsible for the catabolism of the branched chain alpha-amino acids leucine, valine and
36 isoleucine. This enzyme complex requires thiamine (vitamin B 1) as a coenzyme.
37
38
1 A

2 Item: 33 of 46 II P Mark -<:I t>-


Q. Id : 6551 (388147] Previous Next
3
4
5
6 An infant w ith hydrops fetalis is born to a couple w ho recently emigrated from Thailand. The placenta is
7 edematous The infant is 12ale and edematous, and has a massively enlargecj, Jjy~r. Both parents have strong
8 family histories of hemoglobinopathy. A maternal blood smear is most likely to demonstrate w hich of the
9 following?
10
11
12
x @ A. Sickle cells [12%]
13 B. Schistocytes [23%]
14 C. Megaloblasts [5%]
15
16 D. Microcytosis [49%]
17 E. Rouleaux formation [11 %]
18
19
20 Explanation: User Id: 388147
21
22 The major hemoglobin in adults is hemoglobin A, w hich contains one pair of alpha chains and one pair of beta
23 chains. The reduction or absence of one or more of these globin chains is a common hematologic condition
24 know n as thalassemia. The degree of globin chain imbalance determines the severity of clinical
25 manifestations. Some individuals are asymptomatic, w hile others demonstrate pallor, irritability, growth
26 retardation, edema, hepatosplenomegaly, jaundice, and signs of ·extramedullary hematopoiesis. Thalassemic
27 individuals have microcytic and hypochromic red blood cells, w ith occasional target cells seen . The infant
28 described above appears to have Hb Barts (hydrops fetalis ), a severe form of alpha thalassemia
29 characterized by four nonfunctional alpha globin loci.
30
31 (Choices A and B) Sickle cells are associated w ith various hemoglobinopathies, such as sickle cell anemia.
32 Sickle cell anemia manifestations in infants and children include dactylitis, acute episodic pain, and splenic
sequestration . Schistocytes are associated w ith microangiopathic hemolytic anemias, such as disseminated
34 intravascular coagulation (DIC}. DIC manifests in the neonate w ith bleeding in the gastrointestinal tract and at
35 venipuncture sites.
36
37 (Choices C and E) Megaloblastic red blood cells tend to be associated w ith deficiencies of vitamin B 12 or
38 folate. Rouleaux red blood cell formation is characteristic of multiple m yeloma .
1 A

2 Item: 33 of 46 II P Mark -<:I t>-


Q. Id : 6551 (388147] Previous Next
3
4 x @ A. Sickle cells [12%]
5
6 B. Schistocytes [23%]
7 C. Megaloblasts [5%]
8 D. Microcytosis [49%]
9
10 E. Rouleaux formation [ 11 %]
11
12
Explanation: User Id: 388147
13
14
The major hemoglobin in adults is hemoglobin A, w hich contains one pair of alpha chains and one pair of beta
15
chains. The reduction or absence of one or more of these globin chains is a common hematologic condition
16
know n as thalassemia . The degree of globin chain imbalance determines the severity of clinical
17
manifestations. Some individuals are asymptomatic, w hile others demonstrate pallor, irritability, growth
18
retardation, edema, hepatosplenomegaly, jaundice, and signs of extramedullary hematopoiesis. Thalassemic
19
individuals have microcytic and hypochromic red blood cells, w ith occasional target cells seen. The infant
20
described above appears to have Hb Barts (hydrops fetalis}, a severe form of alpha thalassemia
21
characterized by four nonfunctional alpha globin loci.
22
23
(Choices A and B) Sickle cells are associated w ith various hemoglobinopathies, such as sickle cell anemia .
24
Sickle cell anemia manifestations in infants and children include dactylitis, acute episodic pain, and splenic
25
sequestration. Schistocytes are associated w ith microangiopathic hemolytic anemias, such as disseminated
26
intravascular coagulation (DIC). DIC manifests in the neonate w ith bleeding in the gastrointestinal tract and at
27
venipuncture sites.
28
29
(Choices C and E) Megaloblastic red blood cells tend to be associated w ith deficiencies of vitamin 8 12 or
30
31 folate. Rouleaux red blood cell formation is characteristic of multiple myeloma .
32
Educational objective:
34 Thalassemia is associated w ith microcytic, hypochromic red blood cells. Occasional target cells may be
35 seen.
36
37 Time Spent: 45 seconds 49% answ ered correctly. Last updated: [3/27/201 O].
38 Copyright © USMLEWORLD,LLC. SimExam ver. 2011 .11.397425
1 A

2 Item: 34 of 46 II P Mark -<:I t>-


Q. Id : 6556 (388147] Previous Next
3
4
5
6 A 23-year-old male has normal se,c ooda[Y sexuaLchaf acteristics and norrnal.)?loocj_testosterpoeJ evels, but
7 the testosterone concentration in his seminiferous tubules and epididymis is abnormally low . W hich of the
8 following cells are most likely dysfunctional in this patient?
9
10
~ A. Sertoli cells [41 %]
11
12 X @ B. Leydig cells [39%]
13 C. Spermatogenic cells [1 0%]
14
15
D. Ciliated columnar cells [9%]
16 E . Smooth muscle cells [1 %]
17
18
19 Explanation: User Id: 388147
20
21 The seminiferous tubules and epididymis must contain high concentrations of testosterone relative to the
22 testosterone in circulation for spermatogenesis and sperm maturation to occur. The high concentration of
23 localized testosterone is normally maintained by androgen-binding protein (ABP) w ithin the seminiferous
24 tubule. W hen ABP binds to testosterone and dihydrotestosterone, it makes these hormones less lipophilic,
25 thereby concentrating them w ithin the luminal fluid. ABP is synthesized and secreted into the tubule lumen by
26 Sertoli cells of the seminiferous epithelium. Therefore, Sertoli cell dysfunction is most likely in this patient.
27
28 (Choice B) Leydig cells are mainly responsible for synthesizing and secreting testosterone. They are located
29 in the testicular interstitium, outside of the seminiferous tubules.
30
31 (Choice C) Spermatogonia are intermediates in the production of spermatozoa . They do not secrete ABP,
32 and are located w ithin the epithelium of seminiferous tubules.
33
(Choice D) Although Sertoli cells secrete ABP and are columnar in shape, they lack true cilia.

(Choice E) Smooth muscle cells do not secrete ABP. They are present external to the basal lamina of the
37 seminiferous epithelium (particularly in the ductuli efferentes) and surround the lumen of the epididymis.
38
1 A

2 Item: 34 of 46 II P Mark -<:I t>-


Q. Id : 6556 (388147] Previous Next
3
..- M . vc1 lVll .... cu~ L-t 1 FU J
4
5 x@ B. Leydig cells [39%]
6 C. Spermatogenic cells [10%]
7
D. Ciliated columnar cells [9%]
8
9 E. Smooth muscle cells [1 %]
10
11
12 Explanation: User Id: 388147
13
14 The seminiferous tubules and epididymis must contain high concentrations of testosterone relative to the
15 testosterone in circulation for spermatogenesis and sperm maturation to occur. The high concentration of
16 localized testosterone is normally maintained by androgen-binding protein (ABP) w ithin the seminiferous
17 tubule. When ABP binds to testosterone and dihydrotestosterone, it makes these hormones less lipophilic,
18 thereby concentrating them w ithin the luminal fluid. ABP is synthesized and secreted into the tubule lumen by
19 Sertoli cells of the seminiferous epithelium. Therefore, Sertoli cell dysfunction is most likely in this patient.
20
21 (Choice B) Leydig cells are mainly responsible for synthesizing and secreting testosterone. They are located
22 in the testicular interstitium, outside of the seminiferous tubules.
23
24 (Choice C) Spermatogonia are intermediates in the production of spermatozoa . They do not secrete ABP,
25 and are located w ithin the epithelium of seminiferous tubules.
26
27 (Choice D) Although Sertoli cells secrete ABP and are columnar in shape, they lack true cilia.
28
29 (Choice E) Smooth muscle cells do not secrete ABP. They are present external to the basal lamina of the
30 seminiferous epithelium (particularly in the ductuli efferentes) and surround the lumen of the epididymis.
31
32 Educational objective:
33 Androgen-binding protein (ABP) is synthesized by the Sertoli cells of the seminiferous epithelium. ABP is
secreted by these cells into the seminiferous tubule lumen to maintain the high local concentration of
testosterone necessary for normal sperm production and maturation.

37 Time Spent: 48 seconds 41 % answ ered correctly. Last updated: [3/27/201 O].
38 Copyright © USMLEWORLD,LLC. SimExam ver. 2011.11 .397425
1 A

2 Item: 35 of 46 II P Mark -<:I t>-


Q. Id : 6561 (388147] Previous Next
3
4
5
6 A hormonal agent stimulates gluconeogenesis and glycogenolysis in liver cells. The agent is most likely to act
7 through w hich of the follow ing intracellular proteins?
8
9 X @ A. Receptor tyrosine kinase [20%]
10
11
B. Phospholipase ~ [1 0%]
12 C. Small G protein (Ras) [3%]
13
D. G, protein [60%]
14
15 E. Gq protein [6%]
16
17
18 Explanation: User Id: 388147
19
20 Glucagon is a hormonal agent that promotes glycogenolysis and gluconeogenesis in hepatocytes. Glucagon
21 enhances the activity of adenylate cyclase in hepatocytes (and in m yocardiocytes and adipocytes as w ell),
22 thereby increasing intracellular cyclic AMP (cAMP). In hepatocytes, cAMP initiates a series of intracellular
23 reactions that include the activation of glycogen phosphorylase. In other cells, glucagon has been show n to
24 bind to a seven transmembrane G-protein-coupled receptor. Adenylate cyclase is thereby stimulated, most
25 likely through an activated G, intermediate. Note that epinephrine, w hich can also be considered a hormone,
26 has a similar effect on hepatocytes through the binding of 132 receptors. 132 receptor occupancy is generally
27 associated w ith the sequential activation of G, and then adenylate cyclase.
28
29
(Choice A) Growth factors and insulin act on receptors that have intracytoplasmic tails w ith intrinsic tyrosine
30
kinase activity.
31
32
(Choice B) Phospholipase ~is a non-specific pro-inflammatory enzyme that acts on membrane
33
34 phospholipids to generate arachidonic acid and its prostaglandin and leukotriene metabolites.

(Choice C) W hen catalyzed to bind GTP, Ras (Ras-GTP) becomes a dow nstream intracellular messenger
37 that triggers the mitogen-activated protein (MAP) kinase cascade. The MAP kinase cascade is involved in
38 control of cell entry into the cell cycle, but does not directly stimulate hepatocyte glucose production.
1 A

2 Item: 35 of 46 II P Mark -<:I t>-


Q. Id : 6561 (388147] Previous Next
3
4
5
Explanation: User Id: 388147
6
7 Glucagon is a hormonal agent that promotes glycogenolysis and gluconeogenesis in hepatocytes. Glucagon
8 enhances the activity of adenylate cyclase in hepatocytes (and in m yocardiocytes and adipocytes as w ell),
9 thereby increasing intracellular cyclic AMP (cAMP). In hepatocytes, cAMP initiates a series of intracellular
10 reactions that include the activation of glycogen phosphorylase. In other cells, glucagon has been show n to
11 bind to a seven transmembrane G-protein-coupled receptor. Adenylate cyclase is thereby stimulated, most
12
likely through an activated G, intermediate. Note that epinephrine, w hich can also be considered a hormone,
13
14
has a similar effect on hepatocytes through the binding of 132 receptors. 132 receptor occupancy is generally
associated w ith the sequential activation of G, and then adenylate cyclase.
15
16
17 (Choice A) Growth factors and insulin act on receptors that have intracytoplasmic tails w ith intrinsic tyrosine
18 kinase activity.
19
20 (Choice B) Phospholipase ~ is a non-specific pro-inflammatory enzyme that acts on membrane
21 phospholipids to generate arachidonic acid and its prostaglandin and leukotriene metabolites.
22
23 (Choice C) W hen catalyzed to bind GTP, Ras (Ras-GTP) becomes a dow nstream intracellular messenger
24 that triggers the mitogen-activated protein (MAP) kinase cascade. The MAP kinase cascade is involved in
25 control of cell entry into the cell cycle, but does not directly stimulate hepatocyte glucose production.
26
27 (Choice E) Gq is activated by hormone binding to a, and V1 (vasopressin) receptors. It then stimulates the
28 PLC-IP 3 -DAG second messenger pathw ay, w hich is not significantly involved in hormonal augmentation of
29
hepatocyte glycogenolysis and gluconeogenesis.
30
31
Educational objective:
32
Hormonal stimulation of hepatic glycogenolysis and gluconeogenesis is generally achieved through the
33
activation of G, by glucagon. This intracellular G protein stimulates adenylate cyclase, w hich raises
34
intracellular cAMP.

37 Time Spent: 19 seconds 60% answ ered correctly. Last updated: [3/27/201 O].
38 Copyright © USMLEWORLD,LLC. SimExam ver. 2011.11.397425
1 A

2 Item: 36 of 46 II P Mark -<:I t>-


Q. Id : 6565 (388147] Previous Next
3
4
5
6 A young girl complains of heat intolerance and sweating. Physical examination reveals excessive sweating in
7 the axillae. Destruction of which of the following structures would prevent this?
8
9
10
A. Cervical sympathetic ganglion [36%]
11 B. Thoracic splanchnic nerves [8%]
12 ., @ C. Thoracic sympathetic trunk [46%]
13
14 D. Celiac ganglion [3%]
15 E. \lagl!s neF¥e [7%•]
16
17
18 Explanation: User Id: 388147
19
20 Sweating is a function of the sympathetic nervous system. Sympathetic postganglionic nerves innervate
21 eccrine and apocrine (but not sebaceous} glands. The eccrine gland is responsible for hyperhidrosis, a
22 syndrome of sympathetic hyperactivity that causes increased sweating, particularly affecting the palms, soles
23 and axillae. The postganglionic sympathetic neurons that synapse on eccrine glands are cholinergic.
24 Systemic anticholinergic medications, local anticholinergic injectable medications such as botulinum toxin, or
25 surgical sympathectomy may be used to treat hyperhidrosis. Surgical sympathectomy for axillary
26 hyperhidrosis targets the T2 sympathetic ganglion (Choice C).
27
28 (Choice A) The superior cervical sympathetic ganglion provides sympathetic innervation to the tarsal muscle,
29 lacrimal gland, radial muscle of the iris and all salivary glands.
30
31 (Choices B & D) The greater, lesser and least thoracic splanchnic nerves transmit sympathetic
32 preganglionic neurons into the abdomen where they synapse on postganglionic cell bodies within the celiac
33 and aorticorenal ganglia to provide sympathetic innervation to the abdominal viscera. They may also pass
34 directly to the adrenal medulla without synapsing.
35
(Choice E) The vagus nerve carries all parasympathetic preganglionic fibers innervating the viscera of the
37 thorax, foregut and midgut. Prior to the emergence of safer and more effective therapies, vagotomy was used
38 to treat peptic ulcer disease.
1 A

2 Item: 36 of 46 II P Mark -<:I t>-


Q. Id : 6565 (388147] Previous Next
3
M . \...rCI V l "-01:;:, y 11 ltJOU ICU"- ~01 l~llVI I L'-'V / Uj
4
5 B. Thoracic splanchnic nerves [8%]
6 .; @ C. Thoracic sympathetic trunk [46%]
7
D. Celiac ganglion [3%]
8
9 E. \lagbls neF¥e [7%•]
10
11
12 Explanation: User Id: 388147
13
14 Sw eating is a function of the sympathetic nervous system. Sympathetic postganglionic nerves innervate
15 eccrine and apocrine (but not sebaceous) glands. The eccrine gland is responsible for hyperhidrosis, a
16 syndrome of sympathetic hyperactivity that causes increased sw eating, particularly affecting the palms, soles
17 and axillae. The postganglionic sympathetic neurons that synapse on eccrine glands are cholinergic.
18 Systemic anticholinergic medications, local anticholinergic injectable medications such as botulinum toxin, or
19 surgical sympathectomy may be used to treat hyperhidrosis. Surgical sympathectomy for axillary
20 hyperhidrosis targets the T2 sympathetic ganglion (Choice C).
21
22 (Choice A) The superior cervical sympathetic ganglion provides sympathetic innervation to the tarsal muscle,
23 lacrimal gland, radial muscle of the iris and all salivary glands.
24
25 (Choices B & D) The greater, lesser and least thoracic splanchnic nerves transmit sympathetic
26 preganglionic neurons into the abdomen w here they synapse on postganglionic cell bodies w ithin the celiac
27 and aorticorenal ganglia to provide sympathetic innervation to the abdominal viscera . They may also pass
28 directly to the adrenal medulla w ithout synapsing.
29
30 (Choice E) The vagus nerve carries all parasympathetic preganglionic fibers innervating the viscera of the
31 thorax, foregut and midgut. Prior to the emergence of safer and more effective therapies, vagotomy w as used
32 to treat peptic ulcer disease.
33
34 Educational objective:
35 Sw eating is mediated by cholinergic postganglionic fibers of the sympathetic nervous system.

37 Time Spent: 42 seconds 46% answ ered correctly. Last updated: [3/27/201 O].
38 Copyright © USMLEWORLD,LLC. SimExam ver. 2011 .11.397425
1 •
2 Item: 37 of 46 • \ ' Mark -<:J C>
Q. Id: 6508 (383147) Previous NeHt
3
4
5
6 The tissue pictured below was obtained from a healthy male and then stained with red safranin O.
7
8
9
10
11
12
13
14
15
16
17
18
19 '°
20
21
22
23
24
25
26
27
28 Di6J>)ayed v.ith pemiission fran Springer Healthcare Ltd.© C'op)'ngbt 2005 by Current Medicine
29
30 The red-stained area has a large content of which of the following?
31
32
A. Laminin [8%]
33
34 B. Dystrophin [4%)
35 v C. Collagen type II [60%]
36
X '@ D. Sphingomyelin [14%)
38 E. Keratin (14%)
39
~c: CartJlage
40 I Ofv1s1on: ._4usculoskeletal, Skin, & Connectwe Tissue
41 •
1 A

2 Item: 37 of 46 II P Mark -<:I t>-


Q. Id : 6568 (388147] Previous Next
3
M . L OI 11111111 LU F UJ
4
5 B. Dystrophin [4%]
6 .; C. Collagen type II [60%]
7
X @ D. Sphingomyelin [14% ]
8
9 E. Keratin [14% ]
10
11
12 Explanation: User Id: 388147
13
14 This image show s a sample of articular cartilage stained by red safranin 0 . Articular cartilage is the form of
15 cartilage found on bone w ithin joint spaces. The specific cartilage present in articular cartilage is hyaline
16 (glass-like) cartilage. Even w ithout staining, hyaline cartilage can be identified by its microscopic appearance,
17 w ith chondrocytes embedded in a matrix of proteoglycans and collagen, as above. The predominant collagen
18 found in hyaline cartilage is type II collagen. Red safranin O is a histologic stain that colors cartilage, mast cell
19 granules and mucin red. In this image, the hyaline cartilage can be visualized fusing w ith subchondral bone
20 (blue w ith clear spaces). The articular surface of the sample is illustrated by the solid blue band at the top.
21
22 (Choice A) Laminins are a class of glycoproteins that help epithelial cells adhere to the basement membrane.
23
24 (Choice B) Dystrophin is a cytoplasmic protein found in muscle. It is defective in Duchenne muscular
25 dystrophy.
26
27 (Choice D) Sphingomyelin is classically found in the myelin sheath around nerves. Niemann-Pick disease
28 results from defective degradation of sphingomyelin due to a defect in sphingomyelinase.
29
30 (Choice E) Keratin is a cytoplasmic intermediate filament protein produced by squamous cells of the skin.
31
32 Educational Objective:
33 Red safranin 0 stains cartilage, mast cell granules and mucin red. Based on histologic appearance alone,
34 cartilage can be identified by the presence of cells surrounded by a glassy matrix of ground substance and
35 collagen .
36
Time Spent: 25 seconds 60% answ ered correctly. Last updated: [10/3/2011].
38 Copyright © USMLEWORLD,LLC. SimExam ver. 2011 . 11.397425
1 A

2 Item: 38 of 46 II P Mark -<:I t>-


Q. Id : 6570 (388147] Previous Next
3
4
5
6 A 32-year-old female is scheduled for surgery. A diagram illustrating her pelvic viscera is shov1n below.
7 Which of these structures is likely to be found in the inguinal canal?
8
A
9
10
11
12
13
14
15
16
17 D
18
19 E
20
21 A. A [14%]
22
23 B. B [8%]
24 C. C[15%]
25 ~ @ D. 0 [62%]
26
27
E. E [0%]
28
29 Explanation: User Id: 388147
30
31 During embryonic development, the gonads in both males and females are bound superiorly and inferiorly by a
32 fibrous band of peritoneum known as the gubernaculum. Shortening of this structure as the fetus develops
33 pulls the gonads from their original position high in the retroperitoneum down into the pelvis in females and into
34 the scrotum in males. In males, the gubernaculum attaches inferiorly to the scrotum. In females, it passes
35 through the inguinal canal and attaches to the labia majora. In the adult female, the portion of the
36 gubernaculum inferior to the ovary becomes the ovarian ligament and the round ligament of the uterus
37 (ligamentum teres uteri), while the superior portion persists as part of the suspensory ligament of the ovary.
1 A

2 Item: 38 of 46 II P Mark -<:I t>-


Q. Id : 6570 (388147] Previous Next
3
...... .... l'"" ' "J
4
5 " @ D. D [62%]
6 E. E [0%]
7
8
9 Explanation: User Id: 388147
10
11 During embryonic development, the gonads in both males and females are bound superiorly and inferiorly by a
12 fibrous band of peritoneum know n as the gubernaculum. Shortening of this structure as the fetus develops
13 pulls the gonads from their original position high in the retroperitoneum dow n into the pelvis in females and into
14 the scrotum in males. In males, the gubernaculum attaches inferiorly to the scrotum . In females, it passes
15 through the inguinal canal and attaches to the labia majora. In the adult female, the portion of the
16 gubernaculum inferior to the ovary becomes the ovarian ligament and the round ligament of the uterus
17 (ligamentum teres uteri), w hile the superior portion persists as part of the suspensory ligament of the ovary.
18
19 In the image above, the round ligament of the uterus (Choice D) is the structure that contacts the uterus in
20 approximately the same location as the ovarian ligament and courses inferolaterally tow ard the deep inguinal
21 ring (not pictured}.
22
23 (Choice A) The ureter is the structure entering the urinary bladder. The bladder lies anterior to the uterus.
24
25 (Choice B) The fallopian tubes enter the lateral aspects of the uterus, transmitting ova from the ovaries to the
26 uterus.
27
28 (Choice C) The suspensory ligament lies proximal to the ovary. The fibrous band that projects from the ovary
29 to the uterus is the ovarian ligament. Neither of these structures passes through the inguinal canal.
30
31 (Choice E) The cervix is the distal portion of the uterus that lies w ithin the vagina .
32
33 Educational objective:
34 The round ligament of the uterus is a vestige of the gubernaculum. It projects from the uterus through the
35 inguinal canal into the labia majora.
36
37 Time Spent: 28 seconds 62% answ ered correctly. Last updated: [11 / 10/2011].
Copyright © USMLEWORLD,LLC. SimExam ver. 2011 .11.397425
1 A

2 Item: 39 of 46 II P Mark -<:I t>-


Q. Id : 6575 (388147] Previous Next
3
4
5
6 A 32-year-old Caucasian female suffers from episodic pal!)itations, sw eating and abdominal pain . During
7 these episodes, her systolic blood pressure ranges from 20g;220..L!lm Hg, her diastolic blood !)ressure ranges
8 from 11 o-,µ_g.;mm Hg and her heart rate measures 00-120/min. Her urine contains a IJ!gh_copcentr.ation of
9 m,etaoephrines. W hich of the follow ing reactions in the pathway show n below requires methylation?
10
1 2
11 DOPA - - -+ Dopamine
Tyrosine - - --
12
13
14
15 Vanillylmandelic
5 4
l 3
Acid - - - Epinephrine - - Norepinephrine
16
17
18
X @ A. 1,4 [1 6%]
19
20 B. 3,4 [21 %]
21 C. 2,3 [10%]
22
D. 3,5 [13%]
23
24 E. 4,5 [40%]
25
26
27 Explanation: User Id: 388147
28
29 Norepinephrine is converted to epinephrine through methylation in step 4 of the diagram above. Mediating this
30 step is the enzyme phenylethanolamine-N-methyl transferase, a transmethylase that utilizes S-adenosyl
31
methionine as the methyl group donor. In step 5, epinephrine is methylated by catechol-0-methyltransferases
32 to produce metanephrine (3-methoxyepinephrine). Monoamine oxidase can then oxidize metanephrine to
33
form biologically inactive vanillylmandelic acid (VMA). VMA is the primary end-product of both norepinephrine
34 and epinephrine metabolism.
35
36 This patient described most likely has a pheochromocytoma, w hich is generally a benign tumor of the adrenal
37
medulla that hypersecretes epinephrine and norepinephrine, and rarely dopamine as w ell. As a result, urinary
38 concentrations of metanephrine and vanillylmandelic acid are typically elevated.
1 A

2 Item: 39 of 46 II P Mark -<:I t>-


Q. Id : 6575 (388147] Previous Next
3
4
5
X @ A. 1,4 [1 6%]
6
7 B. 3,4 [2 1%]
8 C. 2,3 [10%]
9
10
D. 3,5 [1 3%]
11 E. 4,5 [40%]
12
13
14 Explanation: User Id: 388147
15
16 Norepinephrine is converted to epinephrine through methylation in step 4 of the diagram above. Mediating this
17 step is the enzyme phenylethanolamine-N-methyl transferase, a transmethylase that utilizes S-adenosyl
18 methionine as the methyl group donor. In step 5, epinephrine is methylated by catechol-0-methyltransferases
19 to produce metanephrine (3-methoxyepinephrine). Monoamine oxidase can then oxidize metanephrine to
20 form biologically inactive vanillylmandelic acid (VMA). VMA is the primary end-product of both norepinephrine
21 and epinephrine metabolism.
22
23 This patient described most likely has a pheochromocytoma, w hich is generally a benign tumor of the adrenal
24 medulla that hypersecretes epinephrine and norepinephrine, and rarely dopamine as w ell. As a result, urinary
25 concentrations of metanephrine and vanillylmandelic acid are typically elevated.
26
27 The conversion of tyrosine to DOPA in step 1 of the above diagram is the rate-limiting step in catecholamine
28 synthesis. In this step, tyrosine hydroxylase adds a hydroxyl group to the aromatic ring on tyrosine. DOPA is
29 then decarboxylated to dopamine by DOPA decarboxylase, a pyridoxal phosphate enzyme, in step
30 2. Dopamine is subsequently hydroxylated to norepinephrine by dopamine-13-hydroxylase, a copper-
31 containing enzyme, in step 3.
32
33 Educational objective:
34 The increased synthesis and secretion of catecholamines by a pheochromocytoma elevates plasma
35 catecholamines, urinary metanephrine and urinary vanillylmandelic acid.
36
37 Time Spent: 64 seconds 40% answ ered correctly. Last updated: [11/ 10/2011].
38 Copyright © USMLEWORLD,LLC. SimExam ver. 2011 .11.397425
1 A

2 Item: 40 of 46 II P Mark -<:I t>-


Q. Id : 6578 (388147] Previous Next
3
4
5
6 A 46-year-old truck driver presents to the emergency department w ith sudden-onset shortness of breath. His
7 past medical history is not significant. Physical examination reveals bilateral breath sounds w ith no w heezing
8 or crackles. His right calf is sw ollen and tender to QalQatio . W hich of the follow ing sets of arterial blood gas
9 findings is most expected in this patient?
10 pH PaC02 (mm Hg) Pa02 (mm Hg)
11
12
13 A. 7.22 22 98 [6%]
14
15 X @ B. 7.32 51 64 [50%]
16
G. ~ ~ 00 l&%l-
17
18 G-. ~ ~ 96 f&%l-
19 -
20 E. 7.51 31 71 [32%]
21
22
23 Explanation: User Id: 388147
24
25 This middle-aged patient presents w ith sudden onset dyspnea w ithout accompanying evidence of anxiety,
26 asthma/COPD, pneumonia or m yocardial infarction. He does, how ever, have two risk factors for deep
27 venous thrombosis: prolonged immobility (> 4 hours) as a truck driver and age (> 40 years).
28
29 In the setting of two risk factors for DVT, the presence of acute onset dyspnea and a tender, swollen calf are
30 strongly suggestive of pulmonary embolism (PE). Significant pulmonary embolism is characterized by
31 hypoxemia secondary to a pulmonary ventilation-perfusion (V/Q) mismatch. The hypoxemia leads to
32 hyperventilation and respiratory alkalosis. An increase in pH and reductions in Pa0 2 and PaC0 2 would
33 therefore be expected. Approximate normal arterial blood gas values include pH of 7.35-7.45, Pa0 2 of 80-95
34 mm Hg, PaC0 2 of 35-45 mm Hg, and HC0 3 - of 22-26 mEq/L. Therefore, in the chart above, only the pH of
35
7.51 , PaC0 2 of 31 mm Hg and Pa0 2 of 71 mm Hg correspond to the hypoxemia and respiratory alkalosis
36
37 expected in acute pulmonary embolism.
38
1 A

2 Item: 40 of 46 II P Mark -<:I t>-


Q. Id : 6578 (388147] Previous Next
3
4 This middle-aged patient presents w ith sudden onset dyspnea w ithout accompanying evidence of anxiety,
5 asthma/COPD, pneumonia or myocardial infarction. He does, how ever, have two risk factors for deep
6 venous thrombosis: prolonged immobility(> 4 hours) as a truck driver and age (> 40 years).
7
8 In the setting of two risk factors for DVT, the presence of acute onset dyspnea and a tender, swollen calf are
9 strongly suggestive of pulmonary embolism (PE). Significant pulmonary embolism is characterized by
10 hypoxemia secondary to a pulmonary ventilation-perfusion (V/Q) mismatch. The hypoxemia leads to
11 hyperventilation and respiratory alkalosis. An increase in pH and reductions in Pa0 2 and PaC0 2 would
12 therefore be expected. Approximate normal arterial blood gas values include pH of 7.35-7.45, Pa0 2 of 80-95
13
mm Hg, PaC0 2 of 35-45 mm Hg, and HC0 3 - of 22-26 mEq/L. Therefore, in the chart above, only the pH of
14
7.51 , PaC0 2 of 31 mm Hg and Pa0 2 of 71 mm Hg correspond to the hypoxemia and respiratory alkalosis
15
16 expected in acute pulmonary embolism.
17
18 (Choice A) A decreased pH, a decreased PaC02 and a normal Pa0 2 most likely indicates metabolic acidosis
19 w ith attempted respiratory compensation as may occur in a variety of conditions including all forms of anion
20 gap acidosis.
21
22 (Choice B) A slightly decreased pH, increased PaC0 2 and a decreased Pa0 2 would be most expected in a
23 patient w ith chronic obstructive pulmonary disease (hypoxia and carbon dioxide retention due to
24 hypoventilation ).
25
26 (Choice C) This is a normal ABG reading.
27
28 (Choice D) A slightly increased pH in the presence of a high-normal PaC0 2 and a normal Pa0 2 indicates a
29
possible early metabolic alkalosis.
30
31
Educational objective:
32
In the setting of two risk factors for DVT, the presence of acute onset dyspnea and a tender, swollen calf are
33
strongly suggestive of pulmonary embolism (PE). A significant PE causes hypoxemia and respiratory
34
alkalosis (increased pH, decreased PaC0 2 ) .
35
36
37 Time Spent: 209 seconds 32% answ ered correctly. Last updated: [3/27/201 O].
38 Copyright © USMLEWORLD,LLC. SimExam ver. 2011.11.397425
3
4
Item: 41 of 46 II P Mark -<:I t>-
Q. Id : 6580 (388147] Previous Next
5
6
7
8 A phospholipid infused into experimental animals causes severe bi;_oochocoostrjctiQn, vasocoostrjction and
9 platelet aggregation w ith microthrombus formation . These findings can be attributed to w hich of the follow ing
10 intracellular signaling mechanisms?
11
12 A. Protein kinase A - adenylate cyclase [1 7%]
13
14 B. Guanylate cyclase - NO synthase [2%]
15 ~ @ C. lnositol triphosphate - calcium [39%]
16 D. Tyrosine kinase - MAP kinase [3%]
17
18 E. Cyclooxygenase - prostacyclin [39%]
19
20
Explanation: User Id: 388147
21
22
Platelet activating factor (PAF) is a phospholipid inflammatory mediator that causes severe
23
bronchoconstriction, vasoconstriction and platelet aggregation w ith microthrombus formation . At low ,
24 physiologic concentrations, PAF is a vasodilator that also promotes leukocyte adhesion to the endothelium,
25 diapedesis (transmigration), degranulation and oxidative bursts. PAF is produced by platelets, basophils, mast
26
cells, neutrophils, monocytes, macrophages and endothelial cells. It affects platelets through a G
27
protein-coupled receptor (Gq) that activates phospholipase C (PLC). PLC in turn generates inositol
28
29 al
triphosphate (IP and diacylglycerol from membrane phospholipids. IP 3 -mediated release of calcium from the
30 endoplasmic reticulum is thought to be a major intracellular signaling pathw ay that promotes platelet activation .
31
32 (Choice A) Protein kinase A is activated by cAMP, w hich is produced by adenylate cyclase activated by ligand
33 binding to a G protein-coupled receptor (G,).
34
35 (Choice B) The vasodilation associated w ith low concentrations of PAF is accomplished through increased
36 production of nitric oxide by endothelial nitric oxide synthase. Nitric oxide activates soluble guanylate cyclase
37 in vascular smooth muscle cells to produce cGMP ultimately leading to relaxation of vascular smooth muscle.
38
39 (Choice D) PAF binds to a platelet surface G protein-coupled receptor, not a receptor w ith intrinsic tyrosine
1 A

2 Item: 41 of 46 II P Mark -<:I t>-


Q. Id : 6580 (388147] Previous Next
3
4 -··-...·-··-··-···
5 Platelet activating factor (PAF) is a phospholipid inflammatory mediator that causes severe
6 bronchoconstriction, vasoconstriction and platelet aggregation w ith microthrombus formation. At low,
7 physiologic concentrations, PAF is a vasodilator that also promotes leukocyte adhesion to the endothelium,
8 diapedesis (transmigration), degranulation and oxidative bursts. PAF is produced by platelets, basophils, mast
9
cells, neutrophils, monocytes, macrophages and endothelial cells. It affects platelets through a G
10 protein-coupled receptor (G0 ) that activates phospholipase C (PLC). PLC in turn generates inositol
11
triphosphate (IP 3 ) and diacylglycerol from membrane phospholipids. IP 3 -mediated release of calcium from the
12
13 endoplasmic reticulum is thought to be a major intracellular signaling pathw ay that promotes platelet activation.
14
15 (Choice A) Protein kinase A is activated by cAMP, w hich is produced by adenylate cyclase activated by ligand
16 binding to a G protein-coupled receptor (G,).
17
18 (Choice B) The vasodilation associated w ith low concentrations of PAF is accomplished through increased
19 production of nitric oxide by endothelial nitric oxide synthase. Nitric oxide activates soluble guanylate cyclase
20 in vascular smooth muscle cells to produce cGMP ultimately leading to relaxation of vascular smooth muscle.
21
22 (Choice D) PAF binds to a platelet surface G protein-coupled receptor, not a receptor w ith intrinsic tyrosine
23 kinase activity. Occupancy of the latter receptor type triggers the mitogen-activated protein kinase (MAP
24 kinase) cascade in some cells and is also responsible for insulin signaling.
25
26 (Choice E) Prostacyclin strongly inhibits platelet functions and increases platelet cAMP synthesis by
27 stimulating adenylate cyclase. Cyclooxygenase enzymes convert arachidonic acid to PGH 2 , w hich is the
28 precursor to prostacyclin.
29
30 Educational objective:
31 The IP 3 -mediated release of calcium from the endoplasmic reticulum is a major intracellular signaling pathway
32
that contributes to platelet activation . Platelet activating factor promotes platelet aggregation, primarily through
33
the G/ IP 3 signaling pathw ay. Excessive PAF levels can cause platelet aggregation w ith microthrombus
34
35 formation and constriction of smooth muscle in the bronchi, blood vessels and upper digestive tract.
36
37 Time Spent: 28 seconds 39% answ ered correctly. Last updated: [3/27/201 O].
38 Copyright © USMLEWORLD,LLC. SimExam ver. 2011 .11.397425
1 A

2 Item: 42 of 46 II P Mark -<:I t>-


Q. Id : 6595 (388147] Previous Next
3
4
5
6 A 21-year-old female complains of midabdominal pain that occasionally wakes her from sleep and tends to be
7 relieved by food. Endoscopy shows a small mucosal ulcer in the duodenal bulb. Lansoprazole therapy is
8 initiated. Which of the following is the best justification for adding multi-antibiotic therapy to this patient's
9 treatment regimen?
10
11
A. Ule:er healing [15%•l
12
13 B. Symptem relief [2%•l
14 C. Disease transmissien [1 g~l
15
~ @ D. Recurrence risk [54%]
16
17 E. Malignant transfermatien [28g~l

18
19
Explanation: User Id: 388147
20
21
This patient is suffering from peptic ulcer disease (PUD). Over 70% of PUD cases are the result of H. pylori
22
23 infection, with the remainder attributable to NSAID overuse, Zollinger-Ellison syndrome, and idiopathic
causes. H. pylori colonizes the gastric mucosa, so the exact mechanism by which it causes duodenal
24
ulceration is unclear. Diagnosis of H. pylori infection can be made via the urease breath test or endoscopic
25
duodenal biopsy. Treatment requires complete eradication of H. pylori to prevent recurrence. Triple therapy
26
(proton pump inhibitor, clarithromycin, and metronidazole or amoxicillin) or quadruple therapy (proton pump
27
28 inhibitor, bismuth subsalicylate, metronidazole, and tetracycline) are appropriate.
29
(Choice A) While eradication of H. pylori certainly promotes ulcer healing, antibiotics do not specifically work
30
toward this purpose. Agents like bismuth subsalicylate and sucralfate, which coat ulcer beds, have been
31
shown to directly promote healing.
32
33
(Choice B) Antibiotics contribute little to symptom relief. Antacids like calcium carbonate, as well as proton
34
pump inhibitors and H2 blockers provide better symptomatic relief.
35
36
37 (Choice C) H. pylori is passed from person to person by oral-oral or by fecal-oral transmission. The
38 incidence is highest in developing countries.
1 A

2 Item: 42 of 46 II P Mark -<:I t>-


Q. Id : 6595 (388147] Previous Next
3
E . l'l li::lllYF li::lf ll ~r i::IF ISI UI rrli::IUUF I t:CS)'8J
4
5
6
Explanation: User Id: 388147
7
8
This patient is suffering from peptic ulcer disease (PUD}. Over 70% of PUD cases are the result of H. pylori
9
infection, w ith the remainder attributable to NSAID overuse, Zollinger-Ellison syndrome, and idiopathic
10
causes. H. pylori colonizes the gastric mucosa, so the exact mechanism by w hich it causes duodenal
11
ulceration is unclear. Diagnosis of H. pylori infection can be made via the urease breath test or endoscopic
12
duodenal biopsy. Treatment requires complete eradication of H. pylori to prevent recurrence. Triple therapy
13
(proton pump inhibitor, clarithromycin, and metronidazole or amoxicillin} or quadruple therapy (proton pump
14
inhibitor, bismuth subsalicylate, metronidazole, and tetracycline} are appropriate.
15
16
(Choice A) While eradication of H. pylori certainly promotes ulcer healing, antibiotics do not specifically work
17
tow ard this purpose. Agents like bismuth subsalicylate and sucralfate, w hich coat ulcer beds, have been
18
show n to directly promote healing.
19
20
(Choice B) Antibiotics contribute little to symptom relief. Antacids like calcium carbonate, as w ell as proton
21
pump inhibitors and H2 blockers provide better symptomatic relief.
22
23
24 (Choice C) H. pylori is passed from person to person by oral-oral or by fecal-oral transmission . The
25 incidence is highest in developing countries.
26
27 (Choice E) H. pylori infection has been associated w ith mucosa-associated lymphoid tissue (MALT}
28 lymphoma and gastric adenocarcinoma. Eradication of H. pylori can cause complete tumor resolution in
29 some cases of gastric malignancy, especially MALT lymphoma. How ever, duodenal ulcers do not carry the
30 risk of malignancy.
31
32 Educational objective:
33 While proton pump inhibitors may provide symptomatic relief in peptic ulcer disease (PUD}, antibiotic therapy
34 is necessary to eradicate H. pylori, the organism responsible for the majority of PUD. Using multiple
35 antibiotics reduces the risk of disease recurrence.
36
37 Time Spent: 39 seconds 54% answ ered correctly. Last updated: [3/27/201 O].
38 Copyright © USMLEWORLD,LLC. SimExam ver. 2011 .11.397425
1 A

2 Item: 43 of 46 II P Mark -<:I t>-


Q. Id: 7212 (388147] Previous Next
3
4
5
6
7 The follow ing vignette applies to the next 2 items. The items in the set must be answered in sequential order.
8
Once you click Proceed to Next Item, you w ill not be able to add or change an answ er.
9
10
A 12-year-old Caucasian female is found to have multiple_bf.QWnjsh spots on her body .during a routine
11
physical examination . She had a cougle of them at birth, but they have increased in number recently. She
12
has also noticed two soft flesh-colored, nop,;lepde~ i:iai:iules on her upper chest that she describes as "acne".
13
14
Item 1of2
15 The latter finding most likely represents w hich of the follow ing?
16
17
18 A. Epidermoid cyst [11 %]
19 B. Inflammatory lesion [2%]
20 C. Vascular malformation [16%]
21
22 D. Dermal hyperplasia [18%]
23 ~ @ E. Benign tumor [54%]
24
25
26 Explanation: User Id: 388147
27
28 This patient most likely has neurofibromatosis type 1 (NF1, von Recklinghausen disease), w hich is an
29 autosomal dominant condition resulting from a mutation in the neurofibromin-1 (NF1) gene on chromosome
30 17, a tumor suppressor gene. The clinical findings needed for the diagnosis of NF1 include the presence of
31 two or more of the follow ing: six or more cafe-au-lait spots (which represent the pigmented lesions described
32 in this patient), intertriginous freckling, two or more cutaneous neurofibromas or one plexiform neurofibroma,
33 an optic nerve glioma, characteristic bony lesions, iris Lisch nodules, or a first-degree relative w ith NF1 . The
34 soft, flesh-colored nodules on this patient's chest are characteristic for neurofibromas, w hich are benign
35 growths of Schwann cells and other neural elements. These lesions rarely undergo malignant transformation .
36
37
(Choice A) Epidermoid cysts (sebaceous cyst) are keratin-filled, cystic papules w ith a central punctum . The
38 label "sebaceous" is a misnomer and should not be used because these cysts do not arise from sebaceous
1 A

2 Item: 43 of 46 II P Mark -<:I t>-


Q. Id: 7212 (388147] Previous Next
3
4
5
Explanation: User Id: 388147
6
7
This patient most likely has neurofibromatosis type 1 (NF1, von Recklinghausen disease}, w hich is an
8
autosomal dominant condition resulting from a mutation in the neurofibromin-1 (NF1) gene on chromosome
9
17, a tumor suppressor gene. The clinical findings needed for the diagnosis of NF1 include the presence of
10
tw o or more of the follow ing: six or more cafe-au-lait spots (which represent the pigmented lesions described
11
in this patient}, intertriginous freckling, two or more cutaneous neurofibromas or one plexiform neurofibroma,
12
an optic nerve glioma, characteristic bony lesions, iris Lisch nodules, or a first-degree relative w ith NF1. The
13
soft, flesh-colored nodules on this patient's chest are characteristic for neurofibromas, w hich are benign
14
growths of Schwann cells and other neural elements. These lesions rarely undergo malignant transformation .
15
16 (Choice A) Epidermoid cysts (sebaceous cyst) are keratin-filled, cystic papules w ith a central punctum. The
17 label "sebaceous" is a misnomer and should not be used because these cysts do not arise from sebaceous
18
glands.
19
20 (Choice B) An "inflammatory lesion" is a term that can describe numerous lesions, but there are no
21 characteristic cutaneous inflammatory lesions associated w ith neurofibromatosis type 1.
22
23 (Choice C) Vascular malformations can occur in a variety of syndromes. Commonly observed vascular
24 lesions include nevus flammeus, hemangiomas of infancy, cherry angiomas, spider angiomas, and port w ine
25 stains. No vascular lesions are characteristically seen in patients w ith NF1.
26
27 (Choice D) A dermatofibroma can be easily confused w ith a neurofibroma. How ever, a dermatofibroma is
28 characteristically a firm, hyperpigmented papule that is tethered to the epidermis, w hereas a neurofibroma is
29 typically a soft, flesh-colored papule.
30
31 Educational objective:
32 Neurofibromas are benign neural neoplasms that can occur sporadically or in the setting of neurofibromatosis
33 type 1. The diagnosis of NF1 can be made in the presence of two or more of the follow ing: six or more
34 cafe-au-lait spots, intertriginous freckling, an optic nerve glioma, bony lesions, iris Lisch nodules, or a
35 first-degree relative w ith NF1.
36
37 Time Spent: 60 seconds 54% answ ered correctly. Last updated: [1/8/2012].
38 Copyright © USMLEWORLD,LLC. SimExam ver. 2011 .11.397425
1 A

2 Item: 44 of 46 II P Mark -<:I t>-


Q. Id: 7213 (388147] Previous Next
3
4
5
6 Item 2 of 2
7 A careful examination of her two brothers reveals similar brow nish spots along w ith flesh-colored nodules and
8 axillary freckling. Examination of their parents and all grandparents does not show any skin abnormalities.
9 W hich of the follow ing best explains the observed findings?
10
11
12
A. Variable expression [23%]
13 B. Incomplete penetrance [33%]
14 C. Anticipation [6%]
15
16
v D. Gem1line mosaicism [30%]
17 x @ E. Pleiotropy [7%]
18
19
20 Explanation: User Id: 388147
21
22 Neurofibromatosis type 1 (NF 1) is an autosomal dominant condition w ith variable expressivity but 100%
23 penetrance. The only three explanations for observed generational "skipping" in NF 1 are the follow ing: the
24 presence of a new germline mutation, an incorrect family tree (i.e. the father of the child is not truly the father),
25 or the failure to diagnose the syndrome in predecessors w ith subtle findings. In the case described above, all
26 three siblings are affected and a thorough examination has revealed no signs of disease in any
27 predecessors. The most likely explanation for this finding is a novel germline mutation in one of the parents.
28 Germline (or gonadal} mosaic ism is a term used to describe the presence of two sets of precursor cells for
29 the production of sperm and eggs. In this case, one set likely carries the mutated NF1 gene w hile the other
30 does not.
31
(Choice A) Variable expression refers to the ability of people w ith the same genetic mutation to have a varying
32
phenotypic expression of that mutation.
33
34
(Choice B) NF1 exhibits 100% penetrance. Penetrance refers to the proportion of patients w ith a mutated
35
gene w ho have the associated phenotype. Penetrance differs from expression in that penetrance simply
36 refers to the presence or absence of phenotypic expression of a mutated gene, w hile expression refers to the
37
degree of phenotypic expression of the genetic mutation.
38
1 A

2 Item: 44 of 46 II P Mark -<:I t>-


Q. Id: 7213 (388147] Previous Next
3
4 l;. Ant1c1pauon [b'roJ
5 .; D. Germline mosaicism [30%]
6
x @ E. Pleiotropy [7%]
7
8
9 Explanation: User Id: 388147
10
11 Neurofibromatosis type 1 (NF1) is an autosomal dominant condition w ith variable expressivity but 100%
12 penetrance. The only three explanations for observed generational "skipping" in NF 1 are the following: the
13 presence of a new germline mutation, an incorrect family tree (i.e. the father of the child is not truly the father),
14 or the failure to diagnose the syndrome in predecessors w ith subtle findings. In the case described above, all
15 three siblings are affected and a thorough examination has revealed no signs of disease in any
16 predecessors. The most likely explanation for this finding is a novel germline mutation in one of the parents.
17 Germline (or gonadal) mosaic ism is a term used to describe the presence of two sets of precursor cells for
18 the production of sperm and eggs. In this case, one set likely carries the mutated NF1 gene w hile the other
19 does not.
20
21 (Choice A) Variable expression refers to the ability of people w ith the same genetic mutation to have a varying
22 phenotypic expression of that mutation.
23
24 (Choice B) NF1 exhibits 100% penetrance. Penetrance refers to the proportion of patients w ith a mutated
25 gene w ho have the associated phenotype. Penetrance differs from expression in that penetrance simply
26 refers to the presence or absence of phenotypic expression of a mutated gene, w hile expression refers to the
27 degree of phenotypic expression of the genetic mutation.
28
29 (Choice C) Anticipation occurs w hen the severity of a genetic mutation increases w ith each successive
30 generation, and is classically seen in Huntington's disease.
31
(Choice E) Pleiotropy refers to a single genetic mutation that causes multiple different phenotypic traits.
32
W hile the neurofibromin gene mutation in NF 1 does exhibit pleiotropy, this does not explain the presence of
33
34 disease in one generation and the absence of disease in preceding generations.
35
Educational objective:
36 Neurofibromatosis type 1 is an autosomal dominant disorder w ith 100% penetrance and variable expressivity.
37
Novel germline mutations can cause NF1 in families w ith no prior family history of this disease.
38
1 A

2 Item: 45 of 46 II P Mark -<:I t>-


Q. Id : 7218 (388147] Previous Next
3
4
5
6
7 The follow ing vignette applies to the next 2 items. The items in the set must be answered in sequential order.
8
Once you click Proceed to Next Item, you w ill not be able to add or change an answ er.
9
10
A 41-year-old male comes to the emergency department w ith a three day history of fever, chills, and
11
non:.RrpductjY,~,cough. He also complains of severe w eakness. He is a know n alcoJ:lolic and intravenous drug
12
abuser, and w as Rreviously hospitalized for al£.9~1-related pancreatitis. His temperature is 38.3°C 101°F),
13
blood l;)ressure is 125/7~Hg, and heart rate is.J.02/min, regula~. On physical examination there is mild
14
yellow ish discoloration of the iSclerae. There is a soft, sysl olic murm ur at the ~eftjgw~Lste[,nal border that
15
increases in inte sity UROn inspiration. Scattered bilateral rhonchi are heard on auscultation of the lungs. The
16
abdomen is soft and non-tender. Laboratory studies show:
17
18 Hemoglobin 11 .5 g/dL
19 Leukocyte count 17,000/mm 3
20 Platelet count 92,000/mm 3
21 Total bilirubin 3.5 mg/dL
22 AST 11 2 U/L
23 ALT 89 U/L
24 Alkaline phosphatase 11 0 U/L
25
26 Item 1 of 2
27 W hich of the follow ing is most likely to be present in this patient?
28
29
/\. Mitral regYrgitatien [1 2q~]
30
31 Q. lnterventricYlar septal Elefect [2q~]
32 C. i\ertic stenesis [4q~l
33
D. Tricuspid regurgitation (79%]
34
35 E. /\ertic regYrgitatien [2q~l

36 X @ F. Pulmonic regurgitation [2%]


37
38
1 A

2 Item: 45 of 46 II P Mark -<:I t>-


Q. Id : 7218 (388147] Previous Next
3
4
5 Users of intravenous drugs are more likely to develop right-sided infective endocarditis and subsequent
6 pathology on the right side of the heart, including damage to the tricuspid and pulmonary valves. Tricuspid
7 regurgitation is a potential cause of right-sided heart failure, w hich can result in several characteristic signs
8 and symptoms including jugular venous distention, hepatomegaly w ith systolic palpable pulsations in the liver,
9 hepatojugular reflux, ascites, and peripheral edema . A blow ing, holosystolic murmur heard best along the left
10 low er sternal border that is intensified during inspiration and reduced w ith standing is characteristic of
11 tricuspid regurgitation. W hile this patient does not exhibit all of these signs and symptoms, his fever and
12 elevated leukocyte count in the presence of the characteristic murmur of tricuspid regurgitation likely
13 represent tricuspid valve infective endocarditis. In addition, the elevated liver enzymes and jaundice indicate a
14 combination of alcoholic hepatitis and hepatic congestion from right-sided heart failure.
15
(Choice A) Mitral regurgitation is associated w ith an intense, holosystolic murmur best heard over the apex
16
that radiates to the axilla.
17
18 (Choice B) lnterventricular septal defects (VSD) produce a loud, holosystolic murmur heard best along the
19 left low er sternal border occasionally associated w ith a palpable systolic thrill. The history of intravenous drug
20 abuse as w ell as the fact that this patient has no prior history of a heart murmur makes tricuspid regurgitation
21
much more likely than a VSD.
22
23 (Choice C) Aortic stenosis causes a crescendo-decrescendo systolic murmur best heard at the base in the
24 second right intercostal space w ith radiation to the carotids.
25
26 (Choice E) Aortic regurgitation is associated w ith a high-pitched decrescendo diastolic murmur best heard in
27 the third intercostal space along the left sternal border.
28
29 (Choice F) Pulmonic regurgitation is rarely clinically significant and produces a murmur that is nearly
30 indistinguishable from aortic regurgitation.
31
32 Educational objective:
33 A blow ing holosystolic murmur heard best along the low er left sternal border that is intensified during
34 inspiration and reduced during standing is characteristic of tricuspid regurgitation. Infective endocarditis
35 associated w ith right-sided heart failure is common in abusers of intravenous drugs.
36
37 Time Spent: 121 seconds 79% answ ered correctly. Last updated: [1 /8/2012].
38 Copyright © USMLEWORLD,LLC. SimExam ver. 2011.11 .397425
1 •
2 Item: 46 of 46 • \' Mark -<:J C>
Q. Id: 7219 (383147) Previous NeHt
3
4
5
6 Item 2 of 2
7 The patient is started on an intravenous antibiotic for his condition but develops severe pruritus and a rash
8 immediately upon infusion. The antibiotic used in this patient demonstrates the following dynamics when
9 tested in several healthy volunteers.
10
11
12
13 c
14
15
-
.Q
~
16
17
-~
(1)
c
c
18
19
8
Ql
20 c
21
E
~
22 ·-
"'
.s;;;
23 'O
24 8
25 co
26
27 Rate of drug infusion
28
29 Which of the following antibiotics was most likely given to this patient?
30
31
A. Penicillin G [35%]
32
33 B. Vancomyc1n [49o/o]
34 C. Gentamicin [3%]
35
X IOI D. Trimethoprim/sulfamethoxazole [9%)
36
37 E. lmipenem/cilastatin [3%]
38
1 A

2 Item: 46 of 46 II P Mark -<:I t>-


Q. Id : 7219 (388147] Previous Next
3
4 A. Penicillin G [35%]
5
B. Vancon1ycin [49%]
6
7 C. Gentamicin [3%]
8 X @ D. Trimethoprim/sulfamethoxazole [9%]
9
E. lmipenem/cilastatin [3%]
10
11
12
Explanation: User Id: 388147
13
14
Vancomycin inhibits cell wall synthesis of gram-positive bacteria by binding to the D-Ala-0 -Ala terminus of
15 nascent peptidoglycan pentapeptides, thus inhibiting further peptidoglycan elongation and cross-linking. One
16 of the most common adverse reactions associated with the use of vancomycin is the "red man" syndrome.
17 This adverse reaction is the result of widespread histamine release via nonspecific mast cell degranulation
18 that occurs when vancomycin is infused too rapidly. It is important to understand that this is not an lgE
19 mediated allergic reaction. It can be lessened or prevented by decreasing the rate of infusion as
20 demonstrated in the graph shown above.
21
22 (Choices A & E) The most common adverse reactions associated with Penicillin G and other beta-lactam
23 agents (such as imipenem/cilastatin) are lgE mediated hypersensitivity reactions. Thus the rate of drug
24 infusion has little effect on histamine release.
25
26 (Choice C) Gentamicin is an aminoglycoside antibiotic. Classic side effects associated with this class of
27 drugs include ototoxicity and nephrotoxicity.
28
29 (Choice D) Trimethoprim/sulfamethoxazole can cause megaloblastic anemia, thrombocytopenia, leukopenia,
30 and toxic epidermal necrolysis, although these reactions are uncommon.
31
32 Educational objective:
33 Rapid infusion of vancomycin can lead to widespread release of histamine from mast cells resulting in a
34 condition known as the "red man" syndrome. It is important to understand that this is not an lgE mediated
35 allergic reaction . It can be prevented by using a slower rate of infusion.
36
37 Time Spent: 61 seconds 49% answered correctly. Last updated: [1 /22/2012].
38 Copyright © USMLEWORLD,LLC. SimExam ver. 2011 .11.397425
2 Item: 1 of 46 II P Mark -<:I t>-
Q. Id : 6424 (388147) Previous Next
3
4
5
6 A 31-year-old Caucasian female has been experiencing exertional shortness of breath and cw~ghtness
7 over the last several months. Physical examination reveals a right ventricular heave, on chest palpation and
8 J?ilater.al itting edenia. ol the:...anktes. ECG demonstrates right ventricular hyRertr,0~y. W hich of the follow ing
9 is the most likely cause of this patient's condition?
10
11
12 .; A. Pulmonary endothelial dysfunction (54 % ]
13 X @ B. Chronic obstructive pulmonary disease (23%]
14 C. Right E:erenary artery narrewing [4%•]
15
16 D. ThinneEl t1o1niE:a meElia ef the p1o1lmenary arteries [1Qq~]
17 E. Al~'eelar hyaline memerane fermatien [Qq~l
18
19 -
20 Explanation: User Id: 388147
21
22 Idiopathic pulmonary arterial hypertension (IPAH, formerly called primary pulmonary hypertension) is
23 characterized by increased pulmonary artery pressure and right ventricular failure. The pathogenesis of
24 pulmonary hypertension follow s a predictable course: 1) injury to the pulmonary endothelium secondary to an
25 idiopathic mechanism or a know n medical condition (eg, cardiac defects, collagen vascular diseases,
26 pulmonary embolism); 2) narrow ing of the pulmonary vascular bed w ith resultant right ventricular hypertrophy;
27 3) accelerated vascular injury secondary to increased pulmonary artery pressure; 4) further rise in right
28 ventricular afterload; 5) dilation of the right ventricle. Once right ventricular hypertrophy has developed, clinical
29 manifestations include exertional dyspnea, angina, peripheral edema, increased intensity of the second heart
30 sound (pulmonic component), a systolic ejection murmur, and right ventricular heave.
31
(Choice B) Chronic obstructive pulmonary disease is a know n cause of pulmonary hypertension, but would
32
be unlikely in a young female.
33
34
(Choice C) Right coronary artery narrow ing can be associated w ith angina or m yocardial infarction if severe.
35 It is not typically associated w ith pulmonary hypertension.
36
37 (Choice D) Thickening (not thinning) of the endothelium, smooth muscle, and intimal cells in conjunction w ith
38 concentric laminar intimal fibrosis is consistent w ith oulmonarv hvoertension.
39
40
2 Item: 1 of 46 II P Mark -<:I t>-
Q. Id : 6424 (388147) Previous Next
3
4 x ® l:j, vnrornc oosirucuve pu1monary msease [L,j"loJ
5 C. Right cerenary artery narre Ning [4%•]
1

6 D. ThinneEl t1omica meElia ef the fHJlmenary arteries [1 gq~]


7
8 E. Ali,ceelar hyaline memerane fermatien [Qq~l

9
10
Explanation: User Id: 388147
11
12
Idiopathic pulmonary arterial hypertension (IPAH, formerly called primary pulmonary hypertension) is
13
characterized by increased pulmonary artery pressure and right ventricular failure. The pathogenesis of
14
pulmonary hypertension follows a predictable course: 1) injury to the pulmonary endothelium secondary to an
15
idiopathic mechanism or a known medical condition (eg, cardiac defects, collagen vascular diseases,
16
pulmonary embolism); 2) narrowing of the pulmonary vascular bed with resultant right ventricular hypertrophy;
17
3) accelerated vascular injury secondary to increased pulmonary artery pressure; 4) further rise in right
18
ventricular afterload; 5) dilation of the right ventricle. Once right ventricular hypertrophy has developed, clinical
19 - manifestations include exertional dyspnea, angina, peripheral edema, increased intensity of the second heart
20
sound (pulmonic component), a systolic ejection murmur, and right ventricular heave.
21
22 (Choice B) Chronic obstructive pulmonary disease is a known cause of pulmonary hypertension, but would
23 be unlikely in a young female.
24
25 (Choice C) Right coronary artery narrowing can be associated with angina or myocardial infarction if severe.
26 It is not typically associated with pulmonary hypertension.
27
28 (Choice D) Thickening (not thinning) of the endothelium, smooth muscle, and intimal cells in conjunction with
29 concentric laminar intimal fibrosis is consistent with pulmonary hypertension.
30
31 (Choice E) Alveolar hyaline membrane formation occurs in acute respiratory distress syndrome (ARDS) and
32 hyaline membrane disease of neonates. It is not typically associated with pulmonary hypertension.
33
34 Educational objective:
35 Pulmonary endothelial dysfunction is characteristic of pulmonary arterial hypertension.
36
37 Time Spent: 264 seconds 54% answered correctly. Last updated: [1 /8/2012].
38 Copyright © USMLEWORLD,LLC. SimExam ver. 2011.11.397425
Item: 2 of 46 II P Mark -<:I t>-
Q. Id : 6432 (388147) Previous Next
3
4
5
6 A 32-year-old African American female complains of having heavy menses and passing clots occasionally.
7 She also complains of heaviness and a pal~able mass in the low er abdomen. Her laboratory findings include
8 the follow ing:
9 Hemoglobin 10.2 g/L
10
WBC count 4,600/mm 3
11
12
Platelets 170,000/mm 3
13
14 W hich of the follow ing is the most likely cause of this patient's complaints?
15
16 A. Endometriosis [21 % ]
17 v B. Leion1yon1a [66%]
18
19 - X @ C. Ovarian cyst [8%]
20 D. Cervical cancer [2%]
21 E. Endometritis [3%]
22
23
24 Explanation: User Id: 388147
25
26 Uterine leiomyomas ('libroids") are benign tumors comprised of uterine smooth muscle cells. W hile some
27 leiomyomas are asymptomatic, others are associated w ith pelvic pressure and pain, menorrhagia, or
28 reproductive difficulties. Leiomyomas are most common in w omen in their thirties and forties, w ith black
29 w omen experiencing a tw o- to threefold increased risk of developing these benign tumors.
30
31 (Choice A) Endometriosis consists of endometrial glands and stroma outside of the uterine cavity or
32 musculature. Most w omen w ith the condition are asymptomatic, but some experience pain, dysmenorrhea,
33 or infertility.
34
35 (Choice C) Ovarian cysts are relatively common findings in females of all ages, and may represent a w ide
36 range of conditions including physiologic processes, neoplastic processes (whether benign or malignant),
37 polycystic ovary syndrome, or pelvic inflammatory disease. Rarely does an ovarian cyst cause menorrhagia.
38
Item: 2 of 46 II P Mark -<:I t>-
Q. Id : 6432 (388147) Previous Next
3
4 A. Endometriosis [21 %]
5
6 ~ B. l eion1yoma [66%]
7 X @ C. Ovarian cyst [8%]
8 D. Cervical cancer [2%]
9
10 E . Endometritis [3%]
11
12
Explanation: User Id: 388147
13
14
Uterine leiomyomas ("fibroids"} are benign tumors comprised of uterine smooth muscle cells. While some
15
leiomyomas are asymptomatic, others are associated w ith pelvic pressure and pain, menorrhagia, or
16
reproductive difficulties. Leiomyomas are most common in women in their thirties and forties, w ith black
17
women experiencing a two- to threefold increased risk of developing these benign tumors.
18
19 - (Choice A) Endometriosis consists of endometrial glands and stroma outside of the uterine cavity or
20 musculature. Most women w ith the condition are asymptomatic, but some experience pain, dysmenorrhea,
21 or infertility.
22
23 (Choice C) Ovarian cysts are relatively common findings in females of all ages, and may represent a w ide
24 range of conditions including physiologic processes, neoplastic processes (whether benign or malignant},
25 polycystic ovary syndrome, or pelvic inflammatory disease. Rarely does an ovarian cyst cause menorrhagia.
26
27 (Choice D) Cervical cancer is often asymptomatic, but can present w ith abnormal vaginal bleeding, postc.oital
28 bleeding, or a change in vaginal discharge. While it is a possible concern in this patient, leiomyoma is more
29 likely.
30
31 (Choice E) Endometritis is an inflammation of the endometrium commonly associated w ith pelvic
32 inflammatory disease, the postpartum state, intrauterine foreign bodies or growths, and radiation therapy.
33
34 Educational objective:
35 Leiomyomas are benign uterine smooth muscle tumors that frequently present w ith menorrhagia.
36
37 Time Spent: 68 seconds 66% answered correctly. Last updated: [1 /8/2012].
38 Copyright © USMLEWORLD,LLC. SimExam ver. 2011.11.397425
Item: 3 of 46 II P Mark -<:I t>-
Q. Id : 6435 (388147) Previous Next

4
5 A 64-year-old Caucasian male complains of crampy abdominal pain w hile passing stool over the last two
6 days. He has also noticed some blood in his stool. He denies vomiting, fever or urinary symptoms. He
7 experienced myocardial infarction six years ago. Physical examination reveals decreased bowel sounds.
8 The right femoral and dorsalis ~edis ~ulses are w eak. Colonoscopy visualizing the rectum, sigmoid colon and
9 a portion of the descending colon is performed. Which of the follow ing findings are most likely in this patient?
10
11 A. Cobblestone mucosa and transmural inflammation [6%]
12
B. Crypt abscesses and multiple pseudopolyps [11 %]
13
14 .; @ C. Mucosal hemorrhage and patchy areas of necrosis [63%]
15 D. Neoplastic cells infiltrating beyond the lamina propria [1 6%]
16
E. Macrophages w ith accumulated PAS-positive granules [4%]
17
18
19 - Explanation: User Id: 388147
20
21 lschemic colitis is a common cause of low er gastrointestinal hemorrhage in the elderly. Typically, patients
22 present w ith crampy abdominal pain, tenderness to palpation, and bloody stool. The tw o primary
23 mechanisms that produce ischemic colitis are hypoperfusion secondary to diminished cardiac output (as
24 seen in cardiac disease or prolonged shock) and occlusion of the bow el vascular supply (as seen w ith
25 atheroma, thrombosis, or embolism). The w eak low er extremity pulses in this patient indicate he likely has
26 extensive atherosclerosis.
27
28 Histologic findings depend upon the cause and severity of the ischemia. Initially, mucosal hemorrhage,
29 ecchymoses, and patchy necrosis are seen . If the ischemia persists and the injury extends dow n into the
30 muscularis, the bow el w all thickens and becomes edematous. Frank blood may enter the bow el
31 lumen. Finally, transmural infarction is observed, w hich may result in bow el perforation.
32
33 (Choices A, 8, D, and E) Cobblestone mucosa and transmural inflammation are characteristic of Crohn's
34 disease. Crypt abscesses and multiple pseudopolyps are consistent w ith ulcerative colitis. Neoplastic cells
35 infiltrate beyond the lamina propria in colonic adenocarcinoma. Macrophages w ith accumulated PAS-positive
36 granules suggest Whipple's disease. All of these conditions can present w ith abdominal pain, and all but
37 Whipple's disease commonly present w ith blood in the stool. Hov1ever, ischemic colitis is the most strongly
38 associated w ith cardiovascular disease (as seen in this patient). Also, inflammatory bow el disease is
Item: 3 of 46 II P Mark -<:I t>-
Q. Id : 6435 (388147) Previous Next

4 B. Crypt abscesses and multiple pseudopolyps [11 %]


5
6 ~ @ C. Mucosal hen1orrhage and patchy areas of necrosis [63%]
7 D. Neoplastic cells infiltrating beyond the lamina propria [16%]
8 E. Macrophages w ith accumulated PAS-positive granules [4%]
9
10
11 Explanation: User Id: 388147
12
13 lschemic colitis is a common cause of low er gastrointestinal hemorrhage in the elderly. Typically, patients
14 present w ith crampy abdominal pain, tenderness to palpation, and bloody stool. The two primary
15 mechanisms that produce ischemic colitis are hypoperfusion secondary to diminished cardiac output (as
16 seen in cardiac disease or prolonged shock) and occlusion of the bow el vascular supply (as seen w ith
17 atheroma, thrombosis, or embolism). The w eak low er extremity pulses in this patient indicate he likely has
18 extensive atherosclerosis.
19 -
20 Histologic findings depend upon the cause and severity of the ischemia. Initially, mucosal hemorrhage,
21 ecchymoses, and patchy necrosis are seen . If the ischemia persists and the injury extends down into the
22 muscularis, the bow el w all thickens and becomes edematous. Frank blood may enter the bow el
23 lumen. Finally, transmural infarction is observed, w hich may result in bow el perforation .
24
25 (Choices A, B, D, and E) Cobblestone mucosa and transmural inflammation are characteristic of Crohn's
26 disease. Crypt abscesses and multiple pseudopolyps are consistent w ith ulcerative colitis. Neoplastic cells
27 infiltrate beyond the lamina propria in colonic adenocarcinoma . Macrophages w ith accumulated PAS-positive
28 granules suggest Whipple's disease. All of these conditions can present w ith abdominal pain, and all but
29 Whipple's disease commonly present w ith blood in the stool. How ever, ischemic colitis is the most strongly
30 associated w ith cardiovascular disease (as seen in this patient). Also, inflammatory bow el disease is
31 predominantly a disease of young adults.
32
33 Educational objective:
34 Early ischemic colitis is characterized by mucosal hemorrhage and patchy areas of necrosis. Eventually the
35 bow el w all thickens and becomes edematous, and transmural infarction develops.
36
37 Time Spent: 138 seconds 63% answ ered correctly. Last updated: [1 /8/2012].
38 Copyright © USMLEWORLD,LLC. SimExam ver. 2011.11.397425
1 •
2 Item: 4 of 46 • \' Mark -<:J C>
Q. Id: 6440 (383141) Previous NeHt
3

5
6 A picture of a boy with a complicated family history is shown below.
7
8
9
10
11
12
13
14
15
16
17
18
19 '°
20
21
22
23
Displayed wilh pennissioo franSpinger Heallhcare Lid. e Copyrig!lt 2000, 2003 by C\llteDI Medicine
24
25
26 Which of the following proteins is not synthesized properly in this patient?
27
28
A. Fibrillin (10%]
29
30 B. Dystrophin (3%]
31 ~ C. Collagen [80o/o]
32
X @ D. Albumin (1%]
33
34 E. LDL receptor (3%]
35 F. Ferritin (3%]
36
37
38 Explanation: User Id: 388147
1 A

2 Item: 4 of 46 II P Mark -<:I t>-


Q. Id : 6440 (388147) Previous Next
3

5
6 Explanation: User Id: 388147
7
Osteogenesis imperfecta (01) is a rare inherited disorder of bone fragility that is associated with mutations of
8
the genes COL 1A1 and COL 1A2. Because these genes encode proteins that combine to form type I
9
collagen, mutations of COL 1A1 and COL 1A2 result in qualitative and quantitative type I collagen defects. Type
10
11
I collagen is an important structural component of bone, fibrous tissue, tendon, ligament, skin, and sclerae.
Approximately 30% of the human body by weight is composed of type I collagen.
12
13
The severity of 0 1varies considerably, with four disease subtypes described. Clinical manifestations of 01
14
include brittle bones, short stature, scoliosis, blue sclerae, hearing loss, skin and ligament laxity, and easy
15
bruisability. Blue sclerae can be seen in other rare disorders, such as progeria, Menkes syndrome, and cutis
16
laxa.
17
18 (Choices A, B, D, E, and F)
19 Marfan syndrome is characterized by defects in fibrillin, a glycoprotein important in elastic fiber formation.
20
21 Deficiency of dystrophin, a protein that connects muscle fibers to the extracellular matrix, is associated with
22 muscular dystrophy.
23
24 Low levels of the protein transporter albumin are typically caused by decreased hepatic production or
25 increased excretion via the bov1el or kidney. LDL receptor mutations are associated with familial
26 hypercholesterolemia.
27
28 Ferritin is the primary intracellular iron storage protein and is measured in the workups for iron-deficiency
29 anemia and iron overload disorders.
30
None of these proteins (whether synthesized properly or improperly) is associated with blue sclerae, however.
31
32 Educational objective:
33
Blue sclerae are a classic finding of osteogenesis imperfecta, a disease characterized by defective type I
34
collagen formation .
35
36
37 1. Osteogenesis imperfecta. A study of 160 family members.
38
1 A

2 Item: 5 of 46 II P Mark -<:I t>-


Q. Id : 6610 (388147) Previous Next
3
4

6 An 8-year-old male fasts for 15 hours w hile camping in the mountains w ith his parents. He develops lethargy,
7 confusion and vomiting. His blood glucose level is 45 mg/di, and his blood ketone level is low . W hich of the
8 follow ing enzymes is most likely deficient in this patient?
9
10
A. Glucose-6-phosphatase [22%]
11
12 .; @ B. Acyl-CoA dehydrogenase [33%]
13 C. Acetyl CoA carboxylase [13%]
14
D. Glycogen phosphorylase [20%]
15
16 E. Pyruvate kinase [6%]
17 F. Glucose-6-phosphate dehydrogenase [6%]
18
19
20 Explanation: User Id: 388147
21
22 The patient described is in a state of starvation. Under normal circumstances, a prolonged fast w ill result in a
23 decrease in insulin and an increase in glucagon resulting in mobilization of stored energy. Glycogen is
24 degraded during the first 24 hours of a fast; gluconeogenesis is used subsequently to provide glucose formed
25 from metabolic intermediates such as glycerol, lactate and alpha-ketoacids derived from the breakdov1n of
26 protein and triglycerides. Triglycerides are degraded during fasting to form ketone bodies, w hich may be used
27 as an alternate source of energy by most tissues in the body. Blood levels of ketone bodies are significantly
28 increased w ithin the first few days of a fast and should be elevated in this patient. This patient's failure to
29 synthesize ketone bodies indicates a defect in either beta-oxidation or ketone body synthesis. Briefly, beta-
30 oxidation proceeds by the sequential removal of two-carbon units (acetyl-CoA} from fatty acids. Beta-
31 oxidation is a four-step process; the first step is catalyzed by acyl CoA dehyd~ogenase. W hen the amount of
32 acetyl-CoA produced during beta-oxidation exceeds the capacity of the TCA cycle; the excess acetyl-CoA is
33 shunted into the production of ketone bodies (acetoacetate, 3-hydroxybutyrate and acetone}.
34
35 There are three forms of acyl CoA dehydrogenase: short-, medium- and long-chain. Medium chain acyl CoA
36 dehydrogenase deficiency is the most common genetic defect in beta-oxidation and is a common inborn error
37 of metabolism in general. The symptoms start in first few years of life w ith prolonged fasting. Symptoms
38 include nausea, vomiting and hypoglycemia.
1 A

2 Item: 5 of 46 II P Mark -<:I t>-


Q. Id : 6610 (388147) Previous Next
3
4 ux1ua u ur 11~ a 1 uu r -~lefJ pruce~~ , lr 1t::- 111~l ~lefJ 1~ cata1yLeu uy acy1 vuA ue11 y uruye1 1a~ e. vvr1er1 tr1t::- arr1uu11t u1
acetyl-CoA produced during beta-oxidation exceeds the capacity of the TCA cycle; the excess acetyl-CoA is
6 shunted into the production of ketone bodies (acetoacetate, 3-hydroxybutyrate and acetone).
7
8 There are three forms of acyl CoA dehydrogenase: short-, medium- and long-chain. Medium chain acyl CoA
9 dehydrogenase deficiency is the most common genetic defect in beta-oxidation and is a common inborn error
10 of metabolism in general. The symptoms start in first few years of life w ith prolonged fasting. Symptoms
11 include nausea, vomiting and hypoglycemia.
12
13 (Choice A) Glucose-6-phosphatase dephosphorylates glucose-6-phosphate to free glucose. Glucose-6-
14 phosphate must be dephosphorylated to free glucose before it can be released into the circulation during
15 glycogenolysis and gluconeogenesis. Deficiency of this enzyme results in fasting hypoglycemia and
16 excessive accumulation of glycogen in the liver and kidneys.
17
18 (Choice C) Acetyl CoA carboxylase (ACC) catalyzes the first step in fatty acid synthesis and is the major site
19 of regulation for fatty acid synthesis. This enzyme is naturally inhibited during fasting to decrease synthesis of
20 fatty acids.
21
22 (Choice D) Glycogen phosphorylase is responsible for cleaving the alpha 1,4 glycosidic linkage between
23 glucose residues in glycogen thereby liberating glucose-1-phosphate. Deficiency of this enzyme results in
24 glycogen storage disease type V (muscle phosphorylase deficiency or McArdle disease) or type VI (liver
25 phosphorylase deficiency or Hers disease). Both of these conditions are characterized by hypoglycemia and
26 hyperketosis.
27
28 (Choices E & F) Pyruvate kinase and glucose-6-phosphate dehydrogenase are enzymes in the glycolytic and
29 pentose phosphate pathw ays, respectively. Deficiency of these enzymes does not result in hypoglycemia;
30 excessive hemolysis is the major clinical manifestation .
31
32 Educational objective:
33 Hypoglycemia after prolonged fasting w ith inappropriately low ketone bodies suggests impaired 13-
34 oxidation. Medium chain acyl CoA dehydrogenase deficiency is the most common genetic defect in beta-
35 oxidation.
36
37 Time Spent: 70 seconds 33% answ ered correctly. Last updated: [2/16/2012].
38 Copyright © USMLEWORLD,LLC. SimEx am ver. 2011.11.397425
1 A

2 Item: 6 of 46 II P Mark -<:I t>-


Q. Id : 6440 (388147) Previous Next
3
4
5
Which of the follow ing is involved in signal transmission from cell membrane to nucleus?
7
8
9 A. Myc [1 3%]
10 B. Ras [60%]
11 C. Rb [3%]
12
13
D. Cyclin D 1 [19%]
14 X @ E. BRCA-1 [4%]
15 F. p53 [2%]
16
17
18 Explanation: User Id: 388147
19
20 Ras is an oncoprotein component of the MAP-kinase signal transduction system . It is present in an inactive
21 (GDP-containing) or an active (GTP-containing) form. The phosphorylated, active Ras protein transmits a
22 stimulus from the receptor on the cell surface into the nucleus, promoting mitogenesis. Up to 20% of all
23 human tumors contain mutated Ras proteins that have reduced GTPase activity. These mutated Ras
24 proteins remain in an excited GTP-containing state, stimulating neoplastic growth through continuous
25 pathologic activation of the mitogenic signaling pathw ay.
26
27 (Choices A, C, D, E, and F) The product of c-myc is a nuclear phosphoprotein that functions as a
28 transcription activator controlling cell proliferation, differentiation, and apoptosis. Rb is a nuclear
29 phosphoprotein that regulates the G1->S checkpoint. Cyclin D 1 is a promoter of G1 to S-phase transition
30 during the cell cycle. BRCA-1 encodes a DNA repair enzyme that is associated w ith breast and ovarian
31 cancer w hen mutated. p53, the "molecular policeman," forces a cell to halt in the G1 phase of the cell cycle
32 w hen DNA damage is identified. None of these is a GTP-binding oncoprotein.
33
34 Educational Objective:
35 Ras is a GTP-binding, signal-transducing oncoprotein.
36
37 Time Spent: 41 seconds 60% answ ered correctly. Last updated: [3/27/201 O].
38 Copyright © USMLEWORLD,LLC. SimExam ver. 2011 . 11.397425
1 •
2 Item: 7 of 46 • \ ' Mark -<:J C>
Q. Id: 6616 (383141) Previous NeHt
3
4
Hormonal stimulation causes an increase in enzyme synthesis w ithin a cell. W hich of the following dashed
=-~
5
lines would most likely correspond to the change in enzyme kinetics caused by the increased concentration of
6
enzyme compared to the original state (solid line)?

8
9 1/v
10
11
A
12
13
14
15
16
17 1/(S] (49%]
A.
18
19 1/v
20
21
B
22
23 X @
24
25
26
27 111s1 [33%.]
28
29 1/v
30
31
32 c
33

- --
34
35
36
37 1/($) (7%]
C.
38
1 •
2 Item: 7 of 46 • \ ' Mark -<:J C>
Q. Id: 6616 (383141) Previous NeHt
3
4
5 1/v
6
B
8
9
10
11
12
13 1/[SJ 1<1<1%1
14
15 1/v
16
17
18 c
19
20
21
22 --- ---
23 C. 1/[S) [7%]
24
25 1/v
26
27
28 D
29
30
31
32
33 D. 1/[S] [12%]
34
35
36 Explanation: User Id: 388147
37
38 Enzymes catalyze most chemical reactions in the body. In an enzyme-catalyzed chemical reaction, an
1 A

2 Item: 7 of 46 II P Mark -<:I t>-


Q. Id : 6616 (388147) Previous Next
3
4 Enzymes catalyze most chemical reactions in the body. In an enzyme-catalyzed chemical reaction, an
5 enzyme (E) binds to the substrate forming an enzyme-substrate complex (ES). The chemical reaction
6 catalyzed by the enzyme then takes place thereby forming the enzyme-product (EP) complex. The enzyme
and product subsequently dissociate freeing the enzyme to catalyze another reaction. The velocity of a
8 chemical reaction is primarily dependent on the concentrations of enzyme, substrate and product. High
9 concentrations of enzyme and substrate promote higher reaction velocities. High concentrations of product or
10 low concentrations of enzyme or substrate w ill result in low reaction velocities (law of mass action). The
11 kinetics of an enzyme catalyzed reaction can be best summarized using the Michaelis-Menten equation .
12 Plotting velocity of a reaction (Y-axis) against the substrate concentration (X-axis) creates a Michaelis-Menten
13 plot. As the concentration of the substrate increases, the velocity of reaction also increases up to a maximal
14 velocity (Vmax) resulting in a hyperbolic plot. To create a linear plot, the Michaelis-Menten equation can be
15 rearranged by plotting 1/velocity on the Y-axis and 1/substrate on the X-axis w ith KmNmax as the slope and
16
1N max as the Y-intercept. This is know n as the Linew eaver-Burke plot; each of the images given in the
17
18
question are Linew eaver-Burke plots. An increase in enzyme concentration would increase the Vmax of a
19 reaction . Because the Y-intercept in a Linew eaver-Burke plot is 1N max>An increase in V max would IO\'ler the
20 Y-intercept of the Linew eaver-Burke plot. Therefore, an increased enzyme concentration w ill lower the
21 Y-intercept in a Lineweaver-Burke plot as pictured in Graph A. The enzyme affinity for substrate (Km) would
22 be unchanged in this scenario, so the X-intercept on the Lineweaver-Burke plot would remain unchanged.
23
24 (Choice B) In this graph, the Vmax in both enzymes systems is identical. The Dashed line represents the
25 enzyme system in the absence of a competitive inhibitor w hile the solid line represents the enzyme system in
26 the presence of a competitive inhibitor. The key observation in this scenario is that in the presence of a
27 competitive inhibitor, the Km of the enzyme system w ill increase causing a rightward shift of the X-intercept of
28 the Linew eaver-Burke plot (-1/Km).
29
30 (Choices C & D) These graphs do not represent scenarios caused by modifications of enzyme levels or
31 enzyme inhibition.
32
33 Educational objective:
34 An increase in the amount of enzyme available in a particular system, such as a cell, w ill cause an increase in
35 V msx but no change in Km in Michaelis-Menten kinetics. This would cause a decrease in the Y-intercept
36
(1N maxl and no change in the X-intercept (-1/Km) in a Linew eaver-Burke plot.
37
38
1 A

2 Item: 8 of 46 II P Mark -<:I t>-


Q. Id : 6450 (388147) Previous Next
3
4
5
6 A 75-year-old Caucasian male complains of knee pain that restricts his daily activities. The pain is w orse in
7 the latter half of the day. His fa,ther.bad a similar problem before his death. The family has alw ays lived in a
cold climate and enjoys an active lifestyle. W hich of the follow ing is the most significant risk factor for this
9 patient's condition?
10
11
12
A. Gender (2%]
13 X @ B. Cold climate (4%]
14
C. Family history (18%]
15
16 D. Age (75%]
17 E. Race (1 %]
18
19
20 Explanation: User Id: 388147
21
22 Osteoarthritis (or degenerative joint disease) is a chronic condition characterized by the progressive erosion
23 of articular cartilage, primarily w ithin the large w eight-bearing joints. Pain is w orse w ith activity (end of the
24 day). In most patients, osteoarthritis (OA) appears insidiously as a phenomenon of aging; therefore, age is
25 considered a highly significant risk factor for developing the disease. The mechanical stresses placed on the
26 joint are of particular importance, as seen by the increased prevalence of OA in individuals w ho are obese or
27 w ho have previous joint deformities. Characteristic physical features of an osteoarthritic joint include cartilage
28 w ear-and-tear, bony eburnation, dislodging of cartilage and bone pieces ("joint mice"), fibrous-w alled cysts
29 w ithin the subchondral bone, and bony outgrowths at the articular margins.
30
31 (Choice A) After 55 years of age, w omen are more likely than men to have osteoarthritis. W omen w ith
32 osteoarthritis are more likely to have affected knees and hands, w hereas men w ith osteoarthritis are more
33 likely to have affected hips.
34
35 (Choice B) Cold climate has not been associated w ith increased incidence of osteoarthritis, although some
36 patients report an improvement in symptoms w hen in a w arm, dry climate.
37
38 (Choice C) Several studies have suggested there are genetic factors involved in the development of
.' . , . , ' ' ,. .
39
40
1 A

2 Item: 8 of 46 II P Mark -<:I t>-


Q. Id : 6450 (388147) Previous Next
3
4 C. Family history [18%]
5
6
D. Age [75%]
7 E. Race [1 %]

9
10 Explanation: User Id: 388147
11
12 Osteoarthritis (or degenerative joint disease) is a chronic condition characterized by the progressive erosion
13 of articular cartilage, primarily w ithin the large w eight-bearing joints. Pain is w orse w ith activity (end of the
14 day). In most patients, osteoarthritis (OA) appears insidiously as a phenomenon of aging; therefore, age is
15 considered a highly significant risk factor for developing the disease. The mechanical stresses placed on the
16 joint are of particular importance, as seen by the increased prevalence of OA in individuals w ho are obese or
17 w ho have previous joint deformities. Characteristic physical features of an osteoarthritic joint include cartilage
18 w ear-and-tear, bony eburnation, dislodging of cartilage and bone pieces ("joint mice"), fibrous-w alled cysts
19 w ithin the subchondral bone, and bony outgrowths at the articular margins.
20
21 (Choice A) After 55 years of age, w omen are more likely than men to have osteoarthritis. W omen w ith
22 osteoarthritis are more likely to have affected knees and hands, w hereas men w ith osteoarthritis are more
23 likely to have affected hips.
24
25 (Choice B) Cold climate has not been associated w ith increased incidence of osteoarthritis, although some
26 patients report an improvement in symptoms w hen in a w arm, dry climate.
27
28 (Choice C) Several studies have suggested there are genetic factors involved in the development of
29 osteoarthritis (and some subsets of OA in particular). How ever, family history is not the most significant risk
30 factor.
31
32 (Choice E) There does not appear to be a significant correlation between osteoarthritis incidence and race.
33
34 Educational objective:
35 Age is the most pov1erful risk factor for osteoarthritis.
36
37 Time Spent: 78 seconds 75% answ ered correctly. Last updated: [3/27/201 O].
38 Copyright © USMLEWORLD,LLC. SimExam ver. 2011 .11.397425
1 A

2 Item: 9 of 46 II P Mark -<:I t>-


Q. Id : 6457 (388147) Previous Next
3
4
5
6 Ligation of the common bile duct in animals causes bacterial overgrowth in the proximal small intestine. It is
7 suggested that conjugated bile salts have strong antimicrobial properties. W hich of the follow ing is the best
8 explanation of these properties?

10
11
A. Bacterial enzyme inhibition [ 15%]
12 ~ @ B. Surface membrane disruption [76%]
13 C. Microbial RNA denaturation [3%]
14
15
D. Microbial DNA denaturation [5%]
16 E . Ribosomal disaggregation [1 %]
17
18
19 Explanation: User Id: 388147
20
21 Common bile duct ligation causes extrahepatic cholestasis, preventing bile salts (conjugated bile acids) from
22 entering the small intestine. Bile salts are amphipathic molecules that have a hydrophobic component at one
23 end and a hydrophilic component at the opposing end. As a consequence, bile salts congregate at lipid-w ater
24 interfaces, functioning as emulsifying agents. In addition to facilitating the digestion of dietary lipids, bile salts
25 destroy gram-negative bacteria by disrupting bacterial outer membranes. W hen bile salts are absent in the
26 small intestine, gram-negative enteric bacilli may proliferate to excess; counts of cecal gram-negative aerobic
27 bacilli are significantly increased in experimental animals w ho have obstructed common bile ducts.
28
29 (Choices A, C, D, and E) Bile salts are emulsifying agents not known to significantly affect bacterial
30 intracellular enzymes, organelles, or macromolecules. Indirectly, how ever, these bacterial components could
31 be secondarily compromised after emulsification of the outer bacterial membrane.
32
33 Educational objective:
34 Bile salts are microbicidal emulsifying agents that disrupt the outer membranes of gram-negative enteric
35 bacilli. Chronic cholestasis permits bacterial overgrowth and translocation across the gut w all in the distal
36 small intestine.
37
38 Time Spent: 160 seconds 76% answ ered correctly. Last updated: [3/27/201 O].
1 A

2 Item: 10 of 46 II P M ark -<:I t>-


Q. Id : 6458 (388147] Previous Next
3
4 An investigator isolates an organism that has a haploid genome. It has inner and outer cell membranes but no
5 nuclear membrane. It is sensitive to some but not all penicillin antibiotics. W hich of the follow ing organisms
6 w as isolated?
7
8
9 X @ A. Virus [5%]
B. Chlamydia [1 8%]
11 C. Mycoplasma [1 7%]
12
13 D. Yeast [4%]
14 E. Gram-negative rod [56%]
15
16
17 Explanation: User Id: 388147
18
19 Prokaryotes are microorganisms w ith haploid genomes consisting of a single chromosome or DNA strand; no
20 cell nuclei are present. A prokaryotic cell that has a cell w all sandwiched between outer and inner plasma
21 membranes is most likely a gram-negative bacterium, a classification further confirmed by sensitivity to
22 penicillins {which inhibit peptidoglycan cross-linking in the bacterial cell w all).
23
24 (Choice A) DNA viruses have haploid DNA and can have outer membrane envelopes after budding through
25 the plasma or nuclear membranes of infected cells. How ever, DNA viruses lack inner membranes and do not
26 have peptidoglycan cell w alls, rendering them insensitive to penicillins.
27
28 (Choice B) Chlamydiae are small, obligate intracellular bacteria that may have multiple copies of circular,
29 plasmid-like genomes. Similar to typical gram-negative bacteria, chlamydiae have double membranes and
30 periplasmic spaces. How ever, they generally lack peptidoglycan cell w alls and therefore are insensitive to
31 penicillins. Cysteine-rich outer membrane proteins w ith S-S bridges replace peptidoglycan in maintaining the
32 rigidity of the chlamydia! membrane.
33
34 (Choice C) Mycoplasmas are very small bacteria that have single, circular DNA genomes. Mycoplasmas are
35 enclosed by plasma membranes but lack cell w alls, and are therefore insensitive to penicillins. Some
36 research suggests m ycoplasmas evolved from gram-positive bacteria.
37
D) Yeasts are unicellular fungal forms that do not form hyphae (multicellular branching filaments).
38
39
-(Choice
. . .. . " .

40
1 A

2 Item: 10 of 46 II P M ark -<:I t>-


Q. Id : 6458 (388147] Previous Next
3
4
5
D. Yeast [4%]
6 E. Gram-negative rod [56%]
7
8
9 Explanation: User Id: 388147

11 Prokaryotes are microorganisms w ith haploid genomes consisting of a single chromosome or DNA strand; no
12 cell nuclei are present. A prokaryotic cell that has a cell w all sandwiched between outer and inner plasma
13 membranes is most likely a gram-negative bacterium, a classification further confirmed by sensitivity to
14 penicillins (which inhibit peptidoglycan cross-linking in the bacterial cell w all).
15
16 (Choice A) DNA viruses have haploid DNA and can have outer membrane envelopes after budding through
17 the plasma or nuclear membranes of infected cells. How ever, DNA viruses lack inner membranes and do not
18 have peptidoglycan cell w alls, rendering them insensitive to penicillins.
19
20 (Choice B) Chlamydiae are small, obligate intracellular bacteria that may have multiple copies of circular,
21 plasmid-like genomes. Similar to typical gram-negative bacteria, chlamydiae have double membranes and
22 periplasmic spaces. How ever, they generally lack peptidoglycan cell w alls and therefore are insensitive to
23 penicillins. Cysteine-rich outer membrane proteins w ith S-S bridges replace peptidoglycan in maintaining the
24 rigidity of the chlamydia! membrane.
25
26 (Choice C) Mycoplasmas are very small bacteria that have single, circular DNA genomes. Mycoplasmas are
27 enclosed by plasma membranes but lack cell w alls, and are therefore insensitive to penicillins. Some
28 research suggests mycoplasmas evolved from gram-positive bacteria .
29
30 (Choice D) Yeasts are unicellular fungal forms that do not form hyphae (multicellular branching filaments).
31 Fungi are eukaryotes that have diploid genomes and chitinous cell w alls impervious to penicillins.
32
33 Educational objective:
34 A gram-negative bacterium has a haploid genome and a peptidoglycan cell w all sandwiched between an outer
35 membrane and an inner plasma membrane.
36
37 Time Spent: 30 seconds 56% answ ered correctly. Last updated: [3/27/201 O].
38 Copyright © USMLEWORLD,LLC. SimExam ver. 2011 .11.397425
1 A

2 Item: 11 of 46 II P Mark -<:I t>-


Q. Id : 6462 (388147] Previous Next
3
4
5
6 A eukaryotic cell contains equimolar amounts of 1BS and 28S rRNA molecules. W hich of the follow ing best
7 explains this finding?
8
9
.; @ A. Both rRNA molecules are transcribed from a single chromosome as a single transcript that
10
subsequently undergoes cleavage [55%]
12 B. The rRNA molecules are paired after transcription by an energy-requiring biochemical event [9%]
13 C. The rRNI\ melecbiles are transcri9e8 frem Elifferent chremesemes ey a single RN/\ pelymerase
14 that is regbilateEl te preElbice similar ameblnts ef each melecbile [1 Q%,]
15
D. A single precursor rRNA is transcribed and undergoes subsequent amplification to produce many
16
copies [7%]
17
18 E. The rRNA molecules are transcribed from different chromosomes but the transcription is
19 controlled by a single inducer [9%]
20
21
22 Explanation: User Id: 388147
23
There are four main forms of ribosomal RNA (rRNA) that occur in eukaryotic cells, classified as the 28S, 1BS,
24
5.BS and 5S forms based on their unique surface area and molecular w eight. The 28S, 1BS and 5.BS rRNA
25
molecules are formed by a series of steps beginning w ith transcription of a single rRNA gene into a pre-rRNA
26
molecule by RNA polymerase I. The 45S transcript is subsequently cleaved w ithin the nucleolus by
27
ribonucleases to form the 28S, 1BS and 5.BS rRNA molecules. The 5S rRNA is formed separately by
28
transcription of the 5S rRNA gene by RNA polymerase Ill.
29
30
Remember that Svedberg units (S) are not additive (Le. do not expect 28S + 1BS + 5.BS to equal 45S), but
31
are instead based on molecules' size and hov1 they move during centrifugation in a viscous medium.
32
33
34 (Choice B) W hen protein synthesis is initiated, ribosomal subunits are paired in an energy (GTP}-requiring
35
process. This does not, how ever, ensure equimolar production of ribosomal subunits.
36
(Choice C) In eukaryotic cells, RNA polymerase I transcribes the 28S, 1BS and 5.BS rRNA molecules from
37
the same chromosome. Cleavage of a single 45S pre-rRNA into equimolar amounts of these three rRNA
38
1 A

2 Item: 11 of 46 II P Mark -<:I t>-


Q. Id : 6462 (388147] Previous Next
3
4 controlled by a single inducer [9%]
5
6
7 Explanation: User Id: 388147
8
9 There are four main forms of ribosomal RNA (rRNA) that occur in eukaryotic cells, classified as the 28S, 1BS,
10 5.BS and 5S forms based on their unique surface area and molecular w eight. The 28S, 1BS and 5.BS rRNA
molecules are formed by a series of steps beginning w ith transcription of a single rRNA gene into a pre-rRNA
12 molecule by RNA polymerase I. The 45S transcript is subsequently cleaved w ithin the nucleolus by
13 ribonucleases to form the 28S, 1BS and 5.BS rRNA molecules. The 5S rRNA is formed separately by
14 transcription of the 5S rRNA gene by RNA polymerase Ill.
15
16 Remember that Svedberg units (S) are not additive (i.e. do not expect 28S + 1BS + 5.BS to equal 45S), but
17 are instead based on molecules' size and how they move during centrifugation in a viscous medium.
18
19 (Choice B) W hen protein synthesis is initiated, ribosomal subunits are paired in an energy (GTP)-requiring
20 process. This does not, however, ensure equimolar production of ribosomal subunits.
21
22 (Choice C) In eukaryotic cells, RNA polymerase I transcribes the 28S, 1BS and 5.BS rRNA molecules from
23 the same chromosome. Cleavage of a single 45S pre-rRNA into equimolar amounts of these three rRNA
24 types ensures that equal amounts of each are produced.
25
26 (Choice D) During production of the 28S, 1BS and 5.BS rRNA molecules, a single precursor rRNA is
27 transcribed. How ever, it subsequently undergoes cleavage, not amplification.
28
29 (Choice E) The 28S, 1BS and 5.BS rRNA molecules are transcribed from the same rRNA gene w hile the 5S
30 rRNA is transcribed separately.
31
32 Educational objective:
33 In eukaryotes, the 28S, 1BS and 5.BS rRNA molecules are formed together w hen a single 45S pre-rRNA is
34 cleaved, hence they occur in equimolar amounts. The 45S pre-rRNA itself is transcribed by RNA polymerase
35 I. The eukaryotic 5S rRNA is formed separately by RNA polymerase Ill.
36
37 Time Spent: 142 seconds 55% answ ered correctly. Last updated: [3/27/201 O].
38 Copyright © USMLEWORLD,LLC. SimExam ver. 2011 .11.397425
1 A

2 Item: 12 of 46 II P Mark -<:I t>-


3 a. Id : 6467 (388147] Previous Next

4
5
6 A patient w ith type 2 diabetes mellitus is treated w ith an oral hypoglycemic agent The therapy induces
7 differentiation of preadipocytes into adipocytes and increases GLUT 4 transporter expression on the adipocyte
8 cell membrane. W hich of the follow ing agents w as most likely used in this patient?
9
10
X @ A. Glyburide [14%]
11
B. Repaglinide [5%]
13 C. Rosiglitazone [57%]
14
D. Metformin [20%]
15
16 E. Acarbose [4%]
17
18
19 Explanation: User Id: 388147
20
21
As a member of the thiazolidinedione class, rosiglitazone is a peroxisome proliferator-activated receptorv
22 agonist that increases transcription of insulin-responsive genes. (GLUT4, an insulin-responsive
23 transmembrane glucose transporter expressed in adipocytes and skeletal m yocytes, is encoded by one such
24 gene.) Consequently, rosiglitazone increases insulin sensitivity in target cells and increases the number of
25 insulin-responsive adipocytes. Glitazones do not promote pancreatic 13 cell insulin release, and therefore do
26 not cause hypoglycemia .
27
28 (Choice A) Glyburide is an oral hypoglycemic agent classified as a second-generation sulfonylurea. It closes
29 a K+ channel in the pancreatic 13 cell membrane, inducing depolarization and L-type calcium channel opening .
30 The increased Ca++ influx stimulates 13 cell insulin release. Insulin sensitivity in target tissues is also
31
increased, and hepatic glucose output is reduced .
32
(Choice B) Repaglinide, a meglitinide derivative, is a short-acting, insulinotropic agent used as an adjunct to
33
diet and exercise in the treatment of type 2 diabetes mellitus. Its mechanism of action is similar to that of the
34
sulfonylureas. Unlike the sulfonylureas, how ever, repaglinide does not stimulate insulin release w hen
35
extracellular glucose levels are low .
36
37 (Choice D) Metformin is a biguanide w hich, unlike glyburide and repaglinide, can be used as an oral
38 hvooalvcemic in the absence of B cell function. It acts bv decreasina aluconeoaenesis. and increasina
39
40
1 A

2 Item: 12 of 46 II P Mark -<:I t>-


Q. Id : 6467 (388147] Previous Next
3
4
5
Explanation: User Id: 388147
6
7 As a member of the thiazolidinedione class, rosiglitazone is a peroxisome proliferator-activated receptorv
8
agonist that increases transcription of insulin-responsive genes. (GLUT4, an insulin-responsive
9
transmembrane glucose transporter expressed in adipocytes and skeletal m yocytes, is encoded by one such
10
gene.) Consequently, rosiglitazone increases insulin sensitivity in target cells and increases the number of
11
insulin-responsive adipocytes. Glitazones do not promote pancreatic 13 cell insulin release, and therefore do
not cause hypoglycemia .
13
14
(Choice A) Glyburide is an oral hypoglycemic agent classified as a second-generation sulfonylurea . It closes
15 a K + channel in the pancreatic 13 cell membrane, inducing depolarization and L-type calcium channel opening.
16 The increased Ca++ influx stimulates 13 cell insulin release. Insulin sensitivity in target tissues is also
17 increased, and hepatic glucose output is reduced.
18
19 (Choice B) Repaglinide, a meglitinide derivative, is a short-acting, insulinotropic agent used as an adjunct to
20 diet and exercise in the treatment of type 2 diabetes mellitus. Its mechanism of action is similar to that of the
21 sulfonylureas. Unlike the sulfonylureas, how ever, repaglinide does not stimulate insulin release w hen
22 extracellular glucose levels are low.
23
24 (Choice D) Metformin is a biguanide w hich, unlike glyburide and repaglinide, can be used as an oral
25 hypoglycemic in the absence of 13 cell function. It acts by decreasing gluconeogenesis, and increasing
26 glycolysis in target cells.
27
28 (Choice E) Acarbose is an intestinal brush border a-glucosidase inhibitor that decreases postprandial
29 hyperglycemia by reducing gastrointestinal absorption of polymeric glucose.
30
31 Educational objective:
32 Rosiglitazone is a thiazolidinedione that heightens target cell sensitivity to insulin through increased
33 transcription and plasma membrane expression of insulin-responsive genes. For example, rosiglitazone
34 increases GLUT 4 expression on adipocyte membranes. Rosiglitazone may also induce differentiation of
35 preadipocytes into insulin-responsive adipocytes.
36
37 Time Spent: 42 seconds 57% answ ered correctly. Last updated: [7/21201 O].
38 Copyright © USMLEWORLD,LLC. SimExam ver. 2011 .11.397425
1 A

2 Item: 13 of 46 II P Mark -<:I t>-


Q. Id : 6471 (388147] Previous Next
3
4
5
6 A 23-year-old Caucasian female suffering from chronic diarrhea presents for drainage of a perjneal
7 abscess. Her physical examination reveals mouthU1cers but no lymphadenopathy, arthritis, or skin
8 rash. W hich of the follow ing is the most common area of involvement for this patient's disease?
9
10
11 A. Stomach [3%]
12 B. Duodenum [5%]
C. Jejunum [7%]
14
15 D. Ileum [60%]
16 X @ E . Sigmoid colon [24%]
17
18
19 Explanation: User Id: 388147
20
21 Crohn's disease is a chronic, idiopathic, inflammatory condition that classically presents w ith abdominal pain,
22 diarrhea, and low -grade fever. Aphthous ulcers of the mouth may be observed, as in this patient. Eventually,
23 intestinal obstruction, fistulae, or perianal disease (eg, fissures, abscesses} develop in most individuals w ith
24 Crohn's disease. Although any area of the gastrointestinal tract may be affected, the most frequently involved
25 site is the terminal ileum.
26
27 (Choice A) The stomach and esophagus are rarely affected in Crohn's disease.
28
29 (Choices B and C) The duodenum and jejunum are occasionally involved in Crohn's disease, but less often
30 than is the terminal ileum.
31
32 (Choice E) The colon is the gastrointestinal tract site second most frequently involved in Crohn's disease.
33
34 Educational objective:
35 The terminal ileum is the most commonly involved site in Crohn's disease.
36
37 Time Spent: 204 seconds 60% answ ered correctly. Last updated: [3/27/201 O].
38 Copyright © USMLEWORLD,LLC. SimExam ver. 2011 .11.397425
1 •
2 Item: 14 of 46 • \ ' Mark -<:J C>
Q. Id: 6431 (3831471 Previous NeHt
3
4
5
6 A 65-year-old male is being treated with w arfarin for chronic atrial fibrillation. His prothrombin time (PT) is
7 closely monitored to allow for warfarin dosing adjustment when necessary. He begins taking cimetidine due
8 to some stomach discomfort. Which of the following changes in PT would be most expected in this patient
9 once he begins taking cimetidine? (Note that the cimetidine start time is indicated by a black arrow on the
10 graph below.)
11
12 PT
13

15
16
17
18
19
20
21
22 Time (days)
23
24
A. A (25%]
25
26 ,; @ B. B (54%]
27 C. C [4%]
28
0 . 0 (12%]
29
30 E. E [4%]
31
32
Explanation: User Id: 388147
33
34
A wide variety of drugs are oxidatively metabolized by cytochrome P-450 (P450) in the liver. The activity of
35
this enzyme family can be modified by the presence of other drugs through a phenomenon described as
36
enzyme induction and inhibition. Cimetidine, for example, slows the metabolism of warfarin through inhibition
37
of P450. Consequently, the patient described above \'Viii have an increase in his warfarin steady state level
38
1 A

2 Item: 14 of 46 ll P Mark -<:I t>-


Q. Id : 6481 (388147] Previous Next
3
4
5
6 't Time (days)
7
8
9
A. A [25%]
10 ~ @ B. B [54%]
11 C. C [4%]
12
13
D. D [12%]
E. E [4%]
15
16
17 Explanation: User Id: 388147
18
19 A w ide variety of drugs are oxidatively metabolized by cytochrome P-450 (P450} in the liver. The activity of
20 this enzyme family can be modified by the presence of other drugs through a phenomenon described as
21 enzyme induction and inhibition . Cimetidine, for example, slow s the metabolism of w arfarin through inhibition
22 of P450. Consequently, the patient described above w ill have an increase in his w arfarin steady state level
23 (and PT} due to his concomitant use of cimetidine. The normal half life (tY2} of w arfarin is approximately 40
24 hours, and a new w arfarin steady state level w ill not be established until four elimination half lives have
25 elapsed (160 hours, or 6.7 days}. Moreover, once a change in the w arfarin steady state level occurs it w ill not
26 become clinically manifest for at least 24 hours, as four half lives of factor VII (approximately 24 hours total}
27 must elapse for the PT to stabilize. Therefore, this patient w ould be expected to have a gradual increase in
28 his PT over the course of several days.
29
30 (Choices A, C, D, and E) The PT w ould increase rather than decrease in this patient once he began taking
31 cimetidine. This increase w ould be gradual and not abrupt.
32
33 Educational objective:
34 Cimetidine slow s the metabolism of w arfarin, resulting in a gradual increase of the prothrombin time over a
35 period of several days.
36
37 Time Spent: 266 seconds 54% answ ered correctly. Last updated: [11 / 10/2011].
38 Copyright © USMLEWORLD,LLC. SimExam ver. 2011 .11.397425
1 A

2 Item: 15 of 46 II P Mark -<:I t>-


3 a. Id : 6486 (388147] Previous Next

4
5
6 A 37-year-old HIV-positive man is hospitalized for confusion and sleep disturbances. He is a know n IV drug
7 abuser. Physical examination reveals_yellov1 sclera,e, abdominal distension w ith fluid w ave, and ankle
8 edema. Jerky, involuntal}' hand movements are also present. Laboratory findings are show n below.
9
Anti-HAV lgM egative
10 Anti-HAV lgG Positive
11 HBsAg Positive
12 Anti-HBsAg Negative
13
Anti-HCV Positive
14

Which of the following is most likely to improve this patient's confusion?


16
17
18 A. High-volume paracentesis [1 3%]
19 B. Crystalloid infusion [1 3%]
20
21
x @ C. Urine alkalinization [15%]

22 v D. Intestinal content acidification [31 %]


23 E. Immediate portacaval shunting [29%]
24
25
26 Explanation: User Id: 388147
27
28 This patient's laboratory results indicate an exposure to hepatitis A, hepatitis B, and hepatitis C, w hich
29 explains his jaundice and ascites. In light of this baseline hepatic impairment, the acute onset confusion,
30 sleep disturbances, and involuntary hand movements are suggestive of hepatic encephalopathy. Hepatic
31 encephalopathy is characterized by neuropsychiatric abnormalities in the setting of severe liver dysfunction
32 (eg, cirrhosis).
33
34 When portal blood is diverted into systemic circulation, neurotoxic substances such as ammonia are allow ed
35 to accumulate and eventually breach the blood-brain barrier. One of the classic treatments for hepatic
36 encephalopathy is lactulose, a nonabsorbable disaccharide that is degraded by colonic bacteria to form lactic
37 acid and acetic acid. The contents of the gastrointestinal tract become acidified, and NH3 is converted into
38 NH .+. More NH_ diffuses from the blood into the out lumen. w here it too is converted into NH .+. An osmotic
39
40
1 A

2 Item: 15 of 46 II P Mark -<:I t>-


Q. Id : 6486 (388147] Previous Next
3
.... . " ........................................ ~ .................................... ' LV' ' "J
4
5 E. Immediate portacaval shunting [29%]
6
7
8 Explanation: User Id: 388147
9
This patient's laboratory results indicate an exposure to hepatitis A, hepatitis B, and hepatitis C, w hich
10
11
explains his jaundice and ascites. In light of this baseline hepatic impairment, the acute onset confusion,
sleep disturbances, and involuntary hand movements are suggestive of hepatic encephalopathy. Hepatic
12
encephalopathy is characterized by neuropsychiatric abnormalities in the setting of severe liver dysfunction
13
14
(eg, cirrhosis}.

W hen portal blood is diverted into systemic circulation, neurotoxic substances such as ammonia are allow ed
16
to accumulate and eventually breach the blood-brain barrier. One of the classic treatments for hepatic
17
encephalopathy is lactulose, a nonabsorbable disaccharide that is degraded by colonic bacteria to form lactic
18
acid and acetic acid. The contents of the gastrointestinal tract become acidified, and NH 3 is converted into
19
20 NH 4 • . More NH 3 diffuses from the blood into the gut lumen, w here it too is converted into NH 4 • . An osmotic
21 effect is established in the colon, and the resulting luminal distension promotes peristalsis. As the level of
22 ammonia in circulation gradually declines, the symptoms of hepatic encephalopathy improve.
23
24 (Choices A, 8, C, and E) High-volume paracentesis w ould relieve some of the abdominal pressure
25 associated w ith ascites, but w ould not address the greater threat of hepatic encephalopathy. Crystalloid
26 infusion is useful in the fluid resuscitation of hypovolemic individuals, but w ould likely only exacerbate the
27 edema and ascites in this patient. Urine alkalinization decreases the solubility of basic compounds, potentially
28 aggravating this patient's condition by decreasing NH 3 excretion. Immediate portocaval shunting w ould likely
29 w orsen the hepatic encephalopathy because it w ould redirect the ammonia-rich portal blood into circulation
30 w ithout any hepatic processing.
31
32 Educational objective:
33 Lactulose is useful in treating hepatic encephalopathy because it acidifies the contents of the gastrointestinal
34
tract, converting NH 3 into NH 4 •.
35
36
37 Time Spent: 190 seconds 31% answ ered correctly. Last updated: [3/27/201 O].
38 Copyright © USMLEWORLD,LLC. SimExam ver. 2011 .11.397425
1 A

2 Item: 16 of 46 II P Mark -<:I t>-


Q. Id : 6490 (388147] Previous Next
3
4
5
6 A 64-year-old Caucasian male w ith atrial fibrillation develops persistent malaise headache sore throat d!)l
7 cough and low-grade fever. He is treated w ith w arfarin and e!}'throm ycin. W hile he uses antibiotics, his
8 international normalized ratio (INR) is 2.5. His symptoms improve rapidly, and he discontinues erythromycin
9 after seven days of treatment. He returns to work on the same dose of w arfarin, and his INR two days later is
10 1.2 W hich of the following best explains the difference in the INR values?
11
12
A. Liver enzyme induction [ 19%]
13
14 .; B. lnhieitien ef hef')atis an.ig metaeelism [68%•]
15 X @ C. Competitive renal tubular drug secretion [4%]
D. Tachyphylaxis [ 1% ]
17
18 E. Pharmacodynamic interaction [7%]
19
20
21 Explanation: User Id: 388147
22
This patient has been prescribed erythromycin for an apparent atypical pneumonia and w arfarin for
23
prophylaxis of thromboembolism associated w ith atrial fibrillation . W hile taking erythromycin, the patient's
24
w arfarin dosage w as adjusted appropriately to achieve a therapeutic INR value (usually between 2.0 to 3.0 for
25
orally anticoagulated patients). Upon resolution of the respiratory infection, discontinuation of erythromycin
26
w ithout a change in w arfarin dosage resulted in a reduced, sub-therapeutic INR. The most likely explanation
27
for this variation in INR is a pharmacokinetic interaction between erythromycin and w arfarin. Erythromycin
28
and other macrolide antibiotics can slow the metabolism of w arfarin by inhibiting hepatic cytochrome P-450
29
microsomal enzymes (eg, CYP3A4 and CYP2C9 isoforms). Patients on w arfarin must have their INRs
30
monitored closely (and, if necessary, their w arfarin dosages adjusted} w hen drugs like erythromycin are
31
32 started or discontinued.
33
34 (Choice A) Induction of the hepatic cytochrome P450 microsomal enzymes would increase w arfarin
metabolism. Discontinuation of a drug that induces these enzymes (eg, phenobarbital) would increase the
35
INR of a patient on a stable w arfarin dose.
36
37
(Choice C) W arfarin metabolism by hepatic cytochrome P450 microsomal enzymes produces inactive
38
1 A

2 Item: 16 of 46 II P Mark -<:I t>-


Q. Id : 6490 (388147] Previous Next
3
I 111~ fJi:::IU~r ll r li:::I~ u~r I fJI e~c.11ueu er Yll n u r r 1y cu I I UI i:::ll I i:::lfJfJi:::ll er It i:::llYfJICi:::ll fJI 1eur r IUI 110 i:::ll IU W i:::ll I i:::ll u I I UI
4
5 prophylaxis of thromboembolism associated w ith atrial fibrillation. W hile taking erythromycin, the patient's
6 w arfarin dosage w as adjusted appropriately to achieve a therapeutic INR value (usually between 2.0 to 3.0 for
7 orally anticoagulated patients). Upon resolution of the respiratory infection, discontinuation of erythromycin
8 w ithout a change in w arfarin dosage resulted in a reduced, sub-therapeutic INR. The most likely explanation
9 for this variation in INR is a pharmacokinetic interaction between erythromycin and w arfarin . Erythromycin
10 and other macrolide antibiotics can slow the metabolism of w arfarin by inhibiting hepatic cytochrome P-450
11 microsomal enzymes (eg, CYP3A4 and CYP2C9 isoforms ). Patients on w arfarin must have their INRs
12 monitored closely (and, if necessary, their w arfarin dosages adjusted) w hen drugs like erythromycin are
13 started or discontinued.
14
15 (Choice A) Induction of the hepatic cytochrome P450 microsomal enzymes w ould increase w arfarin
metabolism. Discontinuation of a drug that induces these enzymes (eg, phenobarbital) w ould increase the
17 INR of a patient on a stable w arfarin dose.
18
19 (Choice C) W arfarin metabolism by hepatic cytochrome P450 microsomal enzymes produces inactive
20 metabolites including diastereomeric alcohols (which are excreted in the urine) and inactive 7-hydroxywarfarin
21 (which is excreted in bile). Even if the renal secretion of some inactive metabolites w ere significant,
22 competitive inhibition of this process w ould not affect the elimination half life of the parent w arfarin compound.
23
24 (Choice D) Tachyphylaxis is a progressively diminishing response to a drug upon repeated
25 administration . Tachyphylaxis is not caused by changes in drug pharmacokinetics but rather by rapid
26 desensitization of the target tissues to the drug.
27
28 (Choice E) A pharmacodynamic drug interaction occurs w hen one drug alters the pharmacological action of
29 another drug upon a target tissue. An example of a pharmacodynamic drug interaction is the noncompetitive
30 antagonism of w arfarin anticoagulation by vitamin K .
31
32 Educational objective:
33 Erythromycin and other macrolide antibiotics can slow the metabolism of w arfarin by inhibiting hepatic
34 cytochrome P450 microsomal enzymes. At a given w arfarin dosage, the initiation or discontinuation of
35 erythromycin w ill respectively increase or decrease INR through a pharmacokinetic drug interaction.
36
37 Time Spent: 171 seconds 68% answ ered correctly. Last updated: [3/27/201 O].
38 Copyright © USMLEWORLD,LLC. SimExam ver. 2011 .11.397425
1 A

2 Item: 17 of 46 II P Mark -<:I t>-


Q. Id : 6491 (388147] Previous Next
3
4
5
6 A 25-year-old Caucasian female presents to the emergency department with difficultyj?r.eatbir g. She
7 describes occasional f.1;1est tightness since she moved to a new house in the countryside two months ago.
8 Physical examination reveals prolonged exJ'.)ir.atio,n and scatter.eq_exRiretory wheeze§,.bilaterplly. Twenty
9 minutes after treatment is begun in the emergency department, the patient experiences significant relief.
10 Intracellular increase in which of the following is most likely responsible for the symptom relief in this patient?
11
12
13
A. MAP kinase [2%]
14 X @ B. cGMP [1 8%]
15 .; C. cAMP [66%]
16
D. DAG [3%]
18 E. hsp90 [1 %]
19 F. Phospholipase A2 [7%]
20
21 G. Ca 2 • [3%]
22
23
24 Explanation: User Id: 388147
25
26 This patient's recent history, symptoms, and physical findings are consistent with intermittent expiratory
27 airflow obstruction provoked by extrinsic allergic asthma, which is the most common type of
28 asthma. Emergency treatment of an acute asthmatic attack includes supplemental oxygen and
29 bronchodilation with high-dose nebulized 132 agonists or anticholinergic agents. Inhaled 132 agonists such as
30 albuterol can improve FEV1 or peak expiratory flow rates within five minutes of administration by nebulizer,
31 whereas the maximum bronchodilator effect of inhaled ipratropium (an anticholinergic) may require thirty
32 minutes. Therefore, an inhaled 132 agonist is most likely responsible for the symptom relief reported in this
33 patient. 132-adrenergic agonists bind to seven transmembrane G protein-coupled receptors that activate
34 Gs. Gs, in turn, activates adenyl cyclase, producing cAMP as an intracellular second messenger. cAMP
35 relaxes smooth muscle cells by enhancing the activity of cAMP-dependent protein kinase A, which inhibits
36 myosin phosphorylation. cAMP also lowers intracellular calcium concentrations.
37
38 (Choice A) The mitogen-activated protein (MAP) kinase cascade can be activated by ligand binding to
.,, . . ' . ,, . '
39
40
1 A

2 Item: 17 of 46 II P Mark -<:I t>-


Q. Id : 6491 (388147] Previous Next
3
4 (Choice A) The mitogen-activated protein (MAP) kinase cascade can be activated by ligand binding to
5 receptors w ith intrinsic tyrosine kinase activity. Active MAP kinases promote the synthesis and
6 phosphorylation of transcription factors such as Fos and Jun. These transcription factors stimulate the
7 production of growth factors and proteins that control the entry of cells into the cell division cycle.
8
9 (Choice B) An increase in bronchial smooth muscle cGMP could be produced by occupancy of a histamine
10 H1 receptor linked to certain G proteins. H1 receptor activation w ould cause bronchoconstriction, how ever.
11
12 (Choice D) DAG is a common second messenger in a G protein-coupled receptor signal transduction
13 pathw ay distinct from that activated by 132 agonists. In the DAG pathway, the first effector molecule activated
14 is phospholipase C-y (PLC), w hich catalyzes the breakdow n of membrane inositol phospholipids into tw o
15 second messenger products: inositol triphosphate (IP3) and diacylglycerol (DAG). IP3 releases stored
16 calcium from the endoplasmic or sarcoplasmic reticulum, w hich can lead to bronchoconstriction. DAG
activates protein kinase C (PKC), w hich phosphorylates regulatory proteins and transcription factors not
18 involved in acute bronchoconstriction .
19
20 (Choice E) Formation of molecular chaperone heat shock protein 90 (hsp90) is induced by heat and other cell
21 stressors. These chaperones interact w ith new ly synthesized proteins, aiding in their proper folding and
22 transport across the endoplasmic reticulum and Golgi complex. If the proteins are so damaged that they
23 cannot be rescued from misfolding, the chaperones facilitate their degradation. Hsp90 and other heat shock
24 proteins are not know n to acutely affect bronchial smooth muscle tone.
25
26 (Choice F) Increased phospholipase A2 activity in smooth muscle cells generates additional arachidonic acid
27 metabolites, including leukotrienes. Leukotrienes (particularly LTC4, LTD4, and LTE4) are potent mediators of
28 bronchoconstriction that contribute to the pathophysiology of acute severe asthma attacks.
29
30 (Choice G) One of the effects of increased cAMP inside cells is a decrease in intracellular calcium, thereby
31 favoring bronchodilation. Increased Ca++ inside smooth muscle cells causes pulmonary bronchoconstriction.
32
33 Educational objective:
34 Emergent relief of an acute severe asthma attack is most quickly achieved w ith inhaled 132-adrenergic
35 agonists. 132-adrenergic agonists bind to seven transmembrane G protein-coupled receptors that activate Gs
36 and then adenyl cyclase, producing cAMP as an intracellular second messenger. cAMP relaxes bronchial
37 smooth muscle cells through inhibition of m yosin phosphorylation and low ering of the intracellular calcium
38 concentration.
1 A

2 Item: 18 of 46 II P Mark -<:I t>-


Q. Id : 6495 (388147] Previous Next
3
4
5
6 An elderly patient w ho eats mostly j'.1rocessed foods and Sj:1ends little time outdo,o r.§.corr11lains_0Lbo[)~ Rain
7 ,a!ld_weak[Jess . f-jejlas,J3.recenlhistory_of.yertebral fracture. W p,ich of the follow ing bone deposits is most
8 likely responsible for this patient's symptoms?
9
10
11
A. Compact bone [11 %]
12
13 B. Spongy bone [36%]
14
15 C. Lamellar bone [19%]
16
17
~ @ D. Osteoid [33%]

19
20 E . Bony callus [2%]
21
22
23
24 Explanation: User Id: 388147
25
26 Osteomalacia is associated w ith markedly reduced bone density, and typically manifests in adults w ith
27 periosteal bone pain, tenderness, and muscle w eakness. Osteomalacia is most commonly caused by
28 vitamin D deficiency, w hich can arise secondary to poor diet, inadequate sunlight exposure, or gastrointestinal
29 malabsorption.
30
31 Pathophysiologically, the low levels of circulating 25-hydroxy vitamin D present in osteomalacia cause
32 secondary hyperparathyroidism . The increased parathyroid hormone then mobilizes calcium from bone,
33 leading to decreased bony mineralization and the deposition of excessive unmineralized osteoid matrix around
34 the more normally mineralized trabeculae. W hile the bone contours are generally not affected in adults w ith
35 osteomalacia, the new ly formed bone is w eak and prone to fracture.
36
37 (Choices A and B) The hard outer shell of most bones is comprised of compact bone. The innermost region
38 of most bones consists of spongy (cancellous) bone, w hich is a network of thin bony trabeculae separated by
. ,. ,,
39
40
1 A

2 Item: 18 of 46 II P Mark -<:I t>-


Q. Id : 6495 (388147] Previous Next
3
4
5 E. Bony callus [2%]
6
7
8
9 Explanation: User Id: 388147
10
11 Osteomalacia is associated w ith markedly reduced bone density, and typically manifests in adults w ith
12 periosteal bone pain, tenderness, and muscle w eakness. Osteomalacia is most commonly caused by
13 vitamin D deficiency, w hich can arise secondary to poor diet, inadequate sunlight exposure, or gastrointestinal
14 malabsorption.
15
16 Pathophysiologically, the low levels of circulating 25-hydroxy vitamin D present in osteomalacia cause
17 secondary hyperparathyroidism. The increased parathyroid hormone then mobilizes calcium from bone,
leading to decreased bony mineralization and the deposition of excessive unmineralized osteoid matrix around
19 the more normally mineralized trabeculae. While the bone contours are generally not affected in adults w ith
20 osteomalacia, the new ly formed bone is w eak and prone to fracture.
21
22 (Choices A and B) The hard outer shell of most bones is comprised of compact bone. The innermost region
23 of most bones consists of spongy (cancellous) bone, w hich is a network of thin bony trabeculae separated by
24 interconnecting spaces that contain bone marrow.
25
26 (Choice C) The two main forms of bone are w oven bone and lamellar bone; w hile w oven bone is immature
27 (and often pathologic in the adult), lamellar bone is stronger and contains regular, parallel collagen fibers in
28 sheets.
29
30 (Choice E) Arising after trauma, the bony callus consists of firm but flexible hyaline cartilage that is gradually
31 strengthened by calcium deposition and a surrounding meshw ork of w oven bone.
32
33 Educational objective:
34 Osteomalacia (vitamin D deficiency) is characterized by the deposition of excessive unmineralized osteoid
35 matrix in bone.
36
37 Time Spent: 132 seconds 33% answ ered correctly. Last updated: [3/27/201 O].
38 Copyright © USMLEWORLD,LLC. SimExam ver. 2011 .11.397425
1 A

2 Item: 19 of 46 II P Mark -<:I t>-


Q. Id : 6498 (388147] Previous Next
3
4
5
6 A 64-year-old Caucasian male with a history oL,diabetes mellitus_comi:ilains of cramping pain in his right
7 calf The pain is worse upon walking and during the night unless he sleeps in a chair. Physical examination
8 shows thioped skin over the rig!ll..toot and a small ulcer over the right toe. Which of the following most likely
9 accounts for this patient's symptoms?
10
11
12
A. Corticospinal tract degeneration [ 1%]
13 B. Deep vein thrombosis [16%]
14 .; @ C. Femoral artery atherosclerosis [55%]
15
16 D. Patchy nerve Elemyelinatien (27%•l
17 E. Spinal canal narrewing (1 %•l
18

20 Explanation: User Id: 388147


21
22 The diabetic individual described above appears to have peripheral arterial disease (PAD), as evidenced by
23 his thinning skin and calf claudication (with rest pain}. Other physical findings that suggest PAD include a
24 prolonged venous filling time, shiny colored skin, hair loss and nail changes. Femoral artery atherosclerosis is
25 most likely responsible for his symptoms.
26
27 (Choice A) Amyotrophic lateral sclerosis is associated with Wallerian degeneration of the corticospinal tracts
28 and muscle atrophy.
29
30 (Choice B) Thrombophilic states (eg, cancer, sepsis, antithrombin Ill deficiency, protein C or S deficiency,
31 Factor V Leiden mutation} can lead to the development of deep vein thrombosis (DVT}. Signs and symptoms
32 of DVT include leg pain, tenderness, edema, fever, Homans sign, and venous distension.
33
34 (Choice D) Multiple sclerosis is associated with patchy demyelination of the brain, spinal cord, and nerves.
35
36 (Choice E)Lumbar spinal stenosis is a narrowing of the spinal canal with compression of one or more of the
37 spinal roots. Most affected patients are over age 60. Patients experience back pain that radiates to the
38 buttocks and thighs. Symptoms often worsen with walking and lumbar extension and improve with lumbar
... . .. ' ' ' ,, .
39
40
1 A

2 Item: 19 of 46 II P Mark -<:I t>-


Q. Id : 6498 (388147] Previous Next
3
4 B. Deep vein thrombosis [ 16%]
5 ~ @ C. Femoral artery atherosclerosis [55%]
6
7
D. PatGhy nerve Elemyelination (27%•l
8 E. Spinal Ganal narro 1.1ing (1 %•l
1

9
10
11 Explanation: User Id: 388147
12
13 The diabetic individual described above appears to have peripheral arterial disease (PAD), as evidenced by
14 his thinning skin and calf claudication (with rest pain). Other physical findings that suggest PAD include a
15 prolonged venous filling time, shiny colored skin, hair loss and nail changes. Femoral artery atherosclerosis is
16 most likely responsible for his symptoms.
17
18 (Choice A) Amyotrophic lateral sclerosis is associated w ith W allerian degeneration of the corticospinal tracts
and muscle atrophy.
20
21 (Choice B) Thrombophilic states (eg, cancer, sepsis, antithrombin Ill deficiency, protein C or S deficiency,
22 Factor V Leiden mutation) can lead to the development of deep vein thrombosis (DVT). Signs and symptoms
23 of DVT include leg pain, tenderness, edema, fever, Homans sign, and venous distension.
24
25 (Choice D) Multiple sclerosis is associated w ith patchy demyelination of the brain, spinal cord, and nerves.
26
27 (Choice E)Lumbar spinal stenosis is a narrow ing of the spinal canal w ith compression of one or more of the
28 spinal roots. Most affected patients are over age 60. Patients experience back pain that radiates to the
29 buttocks and thighs. Symptoms often w orsen w ith w alking and lumbar extension and improve w ith lumbar
30 flexion. Numbness and paresthesias may occur.
31
32 Educational objective:
33 Diabetic patients w ith foot infections usually have peripheral arterial disease, as indicated by atherosclerotic
34 femoral arteries. Physical findings that suggest PAD include a prolonged venous filling time, shiny colored
35 skin, skin atrophy, hair loss, and nail changes.
36
37 Time Spent: 188 seconds 55% answ ered correctly. Last updated: [3/27/201 O].
38 Copyright © USMLEWORLD,LLC. SimExam ver. 2011 .11.397425
1 A

2 Item: 20 of 46 II P Mark -<:I t>-


Q. Id : 6504 (388147] Previous Next
3
4
5
6 A 21-j'ear-old Caucasian female has a strong family history of breast cancer and sarcomas. You strongly
7 suspect Li-Fraumeni syndrome. Blood cells from a serum sample would most likely demonstrate which of
8 the following?
9
10
A. One allele deletion [31 %]
11
12 B. Two allele deletion [33%]
13 C. Activating mutation of one allele [17%]
14
X @ D. Activating mutation of two alleles [ 17%]
15
16 E. Fermatien ef a fblsien gene [1 q~]
17
18
19 Explanation: User Id: 388147

Li-Fraumeni syndrome is a familial cancer syndrome associated primarily with early onset breast cancer and
21
soft tissue sarcomas. CNS malignancies, lymphoproliferative malignancies and adrenal tumors have also
22
been associated. Most cases of Li-Fraumeni syndrome result from a germline mutation in the p53 tumor
23
suppressor gene. Normally, p53 functions as a cell cycle regulator that promotes cell cycle arrest and
24
apoptosis of cells that have been stressed or damaged. In Li-Fraumeni syndrome, a germline defect in p53 is
25
present in every cell of the patient's body. By Knudson's "two-hit" hypothesis, a second "hit," or mutation, in a
26
single somatic cell that renders the remaining p53 gene nonfunctional would lead to unchecked cell
27
proliferation and tumor formation .
28
29
(Choices B & D) Activation of two oncogenes or deletion of two tumor suppressor genes will predispose to
30
malignancy.
31
32
(Choice C) In Li-Fraumeni syndrome, there is an inactivating mutation of one allele. An example of a single
33
34 allele activating mutation that predisposes to malignancy is activation of the RET protooncogene, which
35
occurs in the MEN2 syndrome.
36
(Choice E) Fusion protein formation is most notably associated with chronic myeloid leukemia (CML). In
37
CML, the Philadelphia chromosome, which is formed by a translocation between chromosomes 9 and 22,
38
1 A

2 Item: 20 of 46 II P Mark -<:I t>-


Q. Id : 6504 (388147] Previous Next
3
4
5
x @ D. Activating mutation of \\"lo alleles [1 7%]

6 E. Fermatien ef a fllsien gene [1 q~l


7
8
9 Explanation: User Id: 388147
10
11 Li-Fraumeni syndrome is a familial cancer syndrome associated primarily w ith early onset breast cancer and
12 soft tissue sarcomas. CNS malignancies, lymphoproliferative malignancies and adrenal tumors have also
13 been associated. Most cases of Li-Fraumeni syndrome result from a germline mutation in the p53 tumor
14 suppressor gene. Normally, p53 functions as a cell cycle regulator that promotes cell cycle arrest and
15 apoptosis of cells that have been stressed or damaged. In Li-Fraumeni syndrome, a germline defect in p53 is
16 present in every cell of the patient's body. By Knudson's "two-hit" hypothesis, a second "hit," or mutation, in a
17 single somatic cell that renders the remaining p53 gene nonfunctional w ould lead to unchecked cell
18 proliferation and tumor formation .
19
(Choices B & D) Activation of two oncogenes or deletion of two tumor suppressor genes w ill predispose to
21 malignancy.
22
23 (Choice C) In Li-Fraumeni syndrome, there is an inactivating mutation of one allele. An example of a single
24 allele activating mutation that predisposes to malignancy is activation of the RET protooncogene, w hich
25 occurs in the MEN2 syndrome.
26
27 (Choice E) Fusion protein formation is most notably associated w ith chronic m yeloid leukemia (CML}. In
28 CML, the Philadelphia chromosome, w hich is formed by a translocation between chromosomes 9 and 22,
29 results in formation of the BCR-ABL fusion gene and the bcr-abl fusion protein. This protein is an unregulated
30 tyrosine kinase that causes failure of cell cycle control.
31
32 Educational objective:
33 The Knudson "two hit" hypothesis states that, in a patient w ith an inherited inactivating mutation in a tumor
34 suppressor gene, malignant transformation can occur w hen any single somatic cell subsequently acquires a
35 "second hit" - i.e. develops a second mutation in the single remaining normal copy of said gene.
36
37 Time Spent: 76 seconds 31% answ ered correctly. Last updated: [3/27/201 O].
38 Copyright © USMLEWORLD,LLC. SimExam ver. 2011 .11.397425
1 A

2 Item: 21 of 46 II P Mark -<:I t>-


Q. Id : 6623 (388147] Previous Next
3
4
5
6 A 36-year-old w oman tells her therapist that being a single parent is extremely difficult. She describes a
7 recent incident w hen her 13-year-old daughter cut school to go out w ith her boyfriend. W hich of the follow ing
8 physician responses is the best example of an open-ended question?
9
10
A. "\'Vere yell hllrt ey yellr Elallghter's 9ehavier?" [1 %,j
11
12 B. "I imagine yellr Elallghter's eehavier maEle yell feel angry." [<l%•l
13 .; @ C. "How did that make you feel?" [84%]
14
D. "~as yellr Elallghter ever 9ehave8 like this 9efere?" [2%•l
15
16 E. '1A'elll8 it ee helpflll fer yell te talk a9ellt it?" [1 Q%,j
17
18
19 Explanation: User Id: 388147
20
Open-ended questions are vital to both information-gathering and establishing rapport w ith patients. Open-
ended questions invite the patient to respond freely, w ithout much direction from the physician. This
22
technique makes the patient feel as though their voice is heard and prevents the physician from missing
23
important parts of the history (as can occur w ith overly focused questioning).
24
25
Closed-ended questions are those w here one-w ord answ ers can suffice, for example, asking "Are you
26
upset?" A better question w ould be, "How do you feel about that?"
27
28
(Choice A) Asking the patient if she w as hurt by her daughter's behavior does not allow the patient to
29
completely express her feelings. It is a closed-ended question.
30
31
(Choice B) In this statement the physician assumes that this behavior made the patient feel angry. It does
32
not allow the patient to express her feelings freely.
33
34
35
(Choice D) This is a closed-ended, yes-or-no question.
36
(Choice E) An open-ended question is one that allow s the patient to talk freely about herself or her
37
feelings. Once again, 'W ould you like to talk about it?" is a closed-ended question that lends itself to a one-
38 ..........................
39
40
1 A

2 Item: 21 of 46 II P Mark -<:I t>-


Q. Id : 6623 (388147] Previous Next
3
4 19. "I 1mag1ne ye Yr ElaYghter·s eehav1er maae yey tee I angry." [<l~'•J
5 ., @ C. "How did that niake you f~I?" [84%]
6
D. "l=las yeyr ElaYghter ever 9ehave8 like this eefere?" [2%•]
7
8 E. ''A1eYIEl it ee helpfbil fer yey ta talk aeeblt it?" [1 Q%,]
9
10
11 Explanation: User Id: 388147
12
13
Open-ended questions are vital to both information-gathering and establishing rapport with patients. Open-
14 ended questions invite the patient to respond freely, without much direction from the physician. This
15
technique makes the patient feel as though their voice is heard and prevents the physician from missing
16
important parts of the history (as can occur with overly focused questioning).
17
18 Closed-ended questions are those where one-word answers can suffice, for example, asking "Are you
19
upset?" A better question would be, "How do you feel about that?"
20
(Choice A) Asking the patient if she was hurt by her daughter's behavior does not allow the patient to
22 completely express her feelings. It is a closed-ended question.
23
24
(Choice B) In this statement the physician assumes that this behavior made the patient feel angry. It does
25
not allow the patient to express her feelings freely.
26
27 (Choice D) This is a closed-ended, yes-or-no question.
28
29
(Choice E) An open-ended question is one that allows the patient to talk freely about herself or her
30
feelings. Once again, 'W ould you like to talk about it?" is a closed-ended question that lends itself to a one-
31
word reply.
32
33
Educational Objective:
34 Open-ended questions allow patients to talk about the emotions or issues they feel are most important. Open-
35
ended questions are vital for obtaining a detailed history and for establishing good rapport.
36
37 Time Spent: 78 seconds 84% answered correctly. Last updated: [3/27/201 O].
38 Copyright © USMLEWORLD,LLC. SimExam ver. 2011 .11.397425
1 A

2 Item: 22 of 46 II P Mark -<:I t>-


Q. Id: 6518 (388147] Previous Next
3
4
5
6 A 65-year-old man presents to your office reporting a one-w eek history of hoarseness. His past medical
7 history is significant for hyi:iertension diabetes mellitus and hyperlipidemia, all of w hich are being treated. He
8 also has a histo!)l of coronary artery stenting two years ago follow ing an episode of chest pain. Chest CT
9 scan today demonstrates dilation of the aortic arch. Involvement of w hich of the follow ing structures most
10 likely accounts for this patient's hoarseness?
11
12
A. Superior vena cava [3%]
13
14 x @ B. Trachea [4%]
15 C. Esophagus [3%]
16
D. Vagus nerve branch [88%]
17
18 E. Carotid arteries [1 %]
19
20
21 Explanation: User Id: 388147

The differential diagnosis for hoarseness is broad. Laryngitis is the most common cause. Other potential
23
causes include vocal cord paralysis, vocal cord nodules, laryngeal cancer, gastroesophageal reflux, thyroid
24
masses and simple overuse of the voice. Because the patient has aneurysmal dilation of the aortic arch on
25
chest CT in the absence of other symptoms, the most likely cause of his hoarseness is vocal cord paralysis
26
from damage to the left recurrent laryngeal nerve.
27
28
The right and left recurrent laryngeal nerves originate as branches off the bilateral vagus nerves w ithin the
29
thorax . The right recurrent laryngeal nerve loops below the right subclavian artery at its junction w ith the
30
brachiocephalic artery before ascending to the larynx. The left recurrent laryngeal nerve loops below the
31
aortic arch before coursing superiorly to the larynx. The recurrent laryngeal nerves innervate all of the intrinsic
32
muscles of the larynx except for the cricothyroid, w hich is innervated by the external laryngeal nerve.
33
34
(Choice A) Compression of the superior vena cava causes SVC syndrome, w hich is characterized by head
35
and neck swelling, cough and hoarseness. Malignancy is the most common cause.
36
37
(Choice B) Tracheal compression by an external mass may cause cough, w heezing and dyspnea.
38
1 A

2 Item: 22 of 46 II P Mark -<:I t>-


Q. Id: 6518 (388147] Previous Next
3
4
5
D. Vagus nerve branch [88%]
6 E. Carotid arteries [ 1%]
7
8
9 Explanation: User Id: 388147
10
11 The differential diagnosis for hoarseness is broad . Laryngitis is the most common cause. Other potential
12 causes include vocal cord paralysis, vocal cord nodules, laryngeal cancer, gastroesophageal reflux, thyroid
13 masses and simple overuse of the voice. Because the patient has aneurysmal dilation of the aortic arch on
14 chest CT in the absence of other symptoms, the most likely cause of his hoarseness is vocal cord paralysis
15 from damage to the left recurrent laryngeal nerve.
16
17 The right and left recurrent laryngeal nerves originate as branches off the bilateral vagus nerves w ithin the
18 thorax . The right recurrent laryngeal nerve loops below the right subclavian artery at its junction w ith the
19 brachiocephalic artery before ascending to the larynx. The left recurrent laryngeal nerve loops below the
20 aortic arch before coursing superiorly to the larynx. The recurrent laryngeal nerves innervate all of the intrinsic
21 muscles of the larynx except for the cricothyroid, w hich is innervated by the external laryngeal nerve.

23 (Choice A) Compression of the superior vena cava causes SVC syndrome, w hich is characterized by head
24 and neck sw elling, cough and hoarseness. Malignancy is the most common cause.
25
26 (Choice B) Tracheal compression by an external mass may cause cough, w heezing and dyspnea .
27
28 (Choice C) Esophageal compression w ould most likely cause dysphagia. Hoarseness w ould be unlikely
29 unless there w as concomitant GERO or laryngeal involvement.
30
31 (Choice E) Rapid carotid artery compression can cause cerebral ischemia.
32
33 Educational objective:
34 The right and left recurrent laryngeal nerves loop below the right subclavian artery and the aortic arch,
35 respectively. Compression of either of these nerves w ill cause hoarseness.
36
37 Time Spent: 214 seconds 88% answ ered correctly. Last updated: [3/27/201 O].
38 Copyright © USMLEWORLD,LLC. SimExam ver. 2011 .11.397425
1 A

2 Item: 23 of 46 II P Mark -<:I t>-


Q. Id : 6523 (388147] Previous Next
3
4
5
6 A virus isolated from human epithelial cells replicates in the nucleus using host enzymes. The organism most
7 likely belongs to w hich of the following virus families?
8
9
A. Togavirus [7%]
10
11 B. Paramyxovirus [ 17%]
12 C. Poxvirus [24%]
13
~ D. Papovavirus [43%]
14
15 x @ E. Picornavirus [1 0%]
16
17
18 Explanation: User Id: 388147
19
The papovaviridae are a family of DNA viruses that include the papillomaviruses and the polyomaviruses.
20
These viruses are small, nonenveloped, icosahedral viruses w ith circular double-stranded DNA genomes. Of
21
the papovaviridae, the papillomaviruses, such as human papillomavirus (HPV), are know n to cause disease
22
by infecting and replicating w ithin human squamous epithelial cells. These viruses rely on cellular molecular
machinery for protein synthesis and genome replication . Once the viral genetic material gains access to the
24
nucleus, mRNA is synthesized from the viral DNA template and is then translated into protein by cytosolic
25
ribosomes. Viral proteins inactivate the p53 and Rb proteins, causing the cell to enter the S-phase and
26
spurring replication of the viral genome by cellular DNA polymerases.
27
28
(Choice A) Togaviridae are enveloped, positive-sense, single-stranded RNA viruses that replicate their
29
genome w ithin the cytosol using a virally-encoded RNA-dependent RNA polymerase.
30
31
(Choice B) Paramyxoviruses are enveloped, negative-sense, single-stranded RNA viruses that replicate their
32
genome in the cytosol using a virally-encoded RNA polymerase that enters the cell w ith the infecting virion.
33
34
(Choice C) Poxviruses are the largest viruses. Their genome is a linear, double-stranded DNA. Poxviruses
35
are unique among DNA viruses in that their genome replication occurs completely in the cytosol utilizing
36
multiple virally-encoded enzymes.
37
38
1 A

2 Item: 23 of 46 II P Mark -<:I t>-


Q. Id : 6523 (388147] Previous Next
3
..- LI. 1 OJJuvav11 u~ L"''-' / u J
4
5 X @ E. Picornavirus [10%]
6
7
8 Explanation: User Id: 388147
9
10 The papovaviridae are a family of DNA viruses that include the papillomaviruses and the polyomaviruses.
11 These viruses are small, nonenveloped, icosahedral viruses w ith circular double-stranded DNA genomes. Of
12 the papovaviridae, the papillomaviruses, such as human papillomavirus (HPV}, are know n to cause disease
13 by infecting and replicating w ithin human squamous epithelial cells. These viruses rely on cellular molecular
14 machinery for protein synthesis and genome replication. Once the viral genetic material gains access to the
15 nucleus, mRNA is synthesized from the viral DNA template and is then translated into protein by cytosolic
16 ribosomes. Viral proteins inactivate the p53 and Rb proteins, causing the cell to enter the S-phase and
17 spurring replication of the viral genome by cellular DNA polymerases.
18
19 (Choice A) Togaviridae are enveloped, positive-sense, single-stranded RNA viruses that replicate their
20 genome w ithin the cytosol using a virally-encoded RNA-dependent RNA polymerase.
21
22 (Choice B) Paramyxoviruses are enveloped, negative-sense, single-stranded RNA viruses that replicate their
genome in the cytosol using a virally-encoded RNA polymerase that enters the cell w ith the infecting virion.
24
25 (Choice C) Poxviruses are the largest viruses. Their genome is a linear, double-stranded DNA. Poxviruses
26 are unique among DNA viruses in that their genome replication occurs completely in the cytosol utilizing
27 multiple virally-encoded enzymes.
28
29 (Choice E) Picornaviruses are positive-sense, single-stranded RNA viruses that replicate w ithin the
30 cytoplasm using a virally-encoded RNA polymerase.
31
32 Educational objective:
33 The Papovaviridae family of DNA viruses includes papillomaviruses and polyomaviruses. These viruses
34 replicate w ithin the host cell nucleus utilizing host cell DNA and RNA polymerases. Poxviruses, on the other
35 hand, are unique among DNA viruses in that they do not replicate in the host cell nucleus.
36
37 Time Spent: 48 seconds 43% answ ered correctly. Last updated: [3/27/201 O].
38 Copyright © USMLEWORLD,LLC. SimExam ver. 2011 .11.397425
1 A

2 Item: 24 of 46 II P Mark -<:I t>-


Q. Id : 6527 (388147] Previous Next
3
4
5
6 A 43-~ear-old male complains of low-grade fevers chest pain and cough productive of blood-streaked
7 sputum. He also notes the recent development of a skin rash on his arms. He has a history of recent
8 business travel to Mexico and South America. He smokes one pack of cigarettes per day and consumes
9 alcohol occasionall;t Chest x-ray shows a thin-walled cavitaiy lesion in the right middle lobe. Which of the
10 following organisms is most likely responsible?
11
12
A. Legioneffa pneumophifa [8%]
13
14 x @ B. Kfebsieffa pneumoniae [29%]
15 C. Pneumocystis jiroveci [3%]
16
D. Mycopfasma pneumoniae [9%]
17
18 E. Coccidioides immitis [51%]
19
20
21 Explanation: User Id: 388147
22
Coccidioides immitis is a dimorphic fungus that grows as a mold in the environment but converts to spherule
23
form within host organisms. It is classically contracted by people in the San Joaquin valley in California
("valley fever") though it is also endemic in southern Arizona, New Mexico, and Texas as well as parts of
25
Mexico and South America . Infection occurs when a host inhales arthroconidia from the soil. Once within the
26
host, the arthroconidia convert to multinucleate spherules (sporangia), which produce numerous endospores
27
capable of forming new spherules. Coccidioidomycosis most commonly causes asymptomatic infection or
28
mild pneumonia with pleural effusion. Rarely, C. immitis infection can cause formation of thin-walled cavities
29
in the lung. In these cases, patients may have hemoptysis in addition to fever and cough.
30
31
(Choice A) Legionella pneumophifa is a Gram negative bacillus that causes disease when aerosolized
32
organisms are inhaled from contaminated water sources. It can cause a self-limited flu-like illness known as
33
34 Pontiac fever or a severe life-threatening pneumonia known as Legionnaire's disease.
35
(Choice B) Kfebsiella pneumoniae is a Gram negative rod that classically causes a lobar pneumonia with
36
associated "currant jelly" hemoptysis in alcoholics.
37
38
1 A

2 Item: 24 of 46 II P Mark -<:I t>-


Q. Id : 6527 (388147] Previous Next
3
4
5
6 Explanation: User Id: 388147
7
Coccidioides immitis is a dimorphic fungus that grow s as a mold in the environment but converts to spherule
8
form w ithin host organisms. It is classically contracted by people in the San Joaquin valley in California
9
("valley fever") though it is also endemic in southern Arizona, New Mexico, and Texas as w ell as parts of
10
Mexico and South America . Infection occurs w hen a host inhales arthroconidia from the soil. Once w ithin the
11
host, the arthroconidia convert to multinucleate spherules (sporangia), w hich produce numerous endospores
12
capable of forming new spherules. Coccidioidomycosis most commonly causes asymptomatic infection or
13
14
mild pneumonia w ith pleural effusion. Rarely, C. immitis infection can cause formation of thin-w alled cavities
15
in the lung. In these cases, patients may have hemoptysis in addition to fever and cough .
16
(Choice A) Legionella pneumophi/a is a Gram negative bacillus that causes disease w hen aerosolized
17
organisms are inhaled from contaminated w ater sources. It can cause a self-limited flu-like illness know n as
18
Pontiac fever or a severe life-threatening pneumonia know n as Legionnaire's disease.
19
20
(Choice B) Klebsiella pneumoniae is a Gram negative rod that classically causes a lobar pneumonia w ith
21
associated "currant jelly" hemoptysis in alcoholics.
22
23
(Choice C) Pneumocystis jiroveci is an atypical fungal organism w ell know n for causing opportunistic
25 pneumonia in HIV-positive patients w ith CD4 cell counts less than 200/mL3 · Pneumonitis w ith w eakness and
26 dyspnea and a "ground glass" appearance on chest x-ray are the typical manifestations of infection.
27
28 (Choice D) Mycoplasma pneumoniae is a small atypical bacterium (no cell w all) that causes "w alking
29 pneumonia ." Affected individuals develop fever, malaise and a nonproductive cough that resolves slow ly over
30 the course of w eeks.
31
32 Educational objective:
33 Coccidioides immitis is a dimorphic soil fungus endemic to the southw est U.S. as w ell as to Mexico and
34 South America that most commonly causes asymptomatic infection or mild pneumonia. Rarely, it may cause
35 formation of a thin-w alled cavity in the lung leading to hemoptysis in addition to fever and cough.
36
37 Time Spent: 112 seconds 51% answ ered correctly. Last updated: [3/27/201 O].
38 Copyright © USMLEWORLD,LLC. SimExam ver. 2011 .11.397425
1 A

2 Item: 25 of 46 II P Mark -<:I t>-


Q. Id : 6532 (388147] Previous Next
3
4
5
6 A 45-year-old immigrant from Eastern Europe presents to clinic w ith progressive fatigability and sleep
7 disturbances. An enlarged thyroid gland is palpated on physical examination. Biopsy findings are show n on
8 the image below .
9
10
~
11
12
13
14
15
16
17
18
19 Dlsp/ay9d with permission lrom CulTf>flt Medldm> Group LLC
20
21 W hich of the following is the most likely diagnosis?
22
23
A. Graves disease [6%]
24
~ @ B. Hashimoto thyr0iditis [64% ]
26 C. Colloid gaiter [11 %]
27
D. Follicular adenoma [12%]
28
29 E. Follicular carcinoma [7%]
30
31
32 Explanation: User Id: 388147
33
34 Hashimoto thyroiditis is one of the most common causes of hypothyroidism worldwide, manifesting w ith
gradual thyroid failure and an atrophic or goitrous thyroid gland. The condition arises from an
35
autoimmune-mediated apoptosis of epithelial cells in the thyroid gland. An intense, diffuse lymphocytic
36
infiltration of the thyroid gland by thyroid-specific B and T cells is seen, accompanied by formation of lymphoid
37
germinal centers and destruction of thyroid follicles. High levels of serum thyroid autoantibodies (directed
38
1 A

2 Item: 25 of 46 II P Mark -<:I t>-


Q. Id : 6532 (388147] Previous Next
3
LI . I Vlll\..UIOI OUCI IVI 110 l I£. FUJ
4
5 E. Follicular carcinoma [7%]
6
7
8 Explanation: User Id: 388147
9
10 Hashimoto thyroiditis is one of the most common causes of hypothyroidism w orldwide, manifesting w ith
11 gradual thyroid failure and an atrophic or goitrous thyroid gland. The condition arises from an
12 autoimmune-mediated apoptosis of epithelial cells in the thyroid gland. An intense, diffuse lymphocytic
13 infiltration of the thyroid gland by thyroid-specific B and T cells is seen, accompanied by formation of lymphoid
14 germinal centers and destruction of thyroid follicles. High levels of serum thyroid autoantibodies (directed
15 against thyroglobulin and thyroid peroxidase in particular) correlate w ith the presence of a lymphocytic infiltrate.
16
17 (Choice A) Graves disease typically presents as a diffusely enlarged thyroid gland associated w ith
18 hyperthyroidism, exophthalmos, and pretibial m yxedema. The thyroid gland follicular epithelium is tall and
19 crowded, forming small pseudopapillae that project into the follicular lumen.
20
21 (Choice C) Colloid gaiter is a diffuse, nontoxic gaiter that can occur on an endemic or sporadic basis.
22 Flattening of the crowded columnar epithelium occurs in the thyroid follicles, allow ing the gland to become
23 enlarged and colloid rich.
24
(Choice D) Follicular adenoma tends to present as a painless, unilateral neck mass. Histologic findings
26 include an intact capsule that separates normal thyroid tissue from numerous large, w ell-differentiated,
27 colloid-containing follicles.
28
29 (Choice E) Follicular carcinoma typically presents as a single nodule in an older woman. Histologic features
30 include relatively uniform cells that form small follicles, nests, or sheets. Colloid may be present in the
31 follicles. Capsular invasion occurs, and areas of hemorrhage or vascular invasion are common.
32
33 Educational objective:
34 Hashimoto thyroiditis is a common cause of hypothyroidism, and is characterized by an intense lymphocytic
35 infiltration of the thyroid gland that results in follicular destruction and germinal center formation.
36
37 Time Spent: 68 seconds 64% answ ered correctly. Last updated: [11 / 10/2011].
38 Copyright © USMLEWORLD,LLC. SimExam ver. 2011 .11.397425
1 A

2 Item: 26 of 46 II P Mark -<:I t>-


Q. Id : 6539 (388147] Previous Next
3
4
5
6 A young patient is evaluated for poor exercise tolerance. P0 2 readings from different compartments of the
7 cardiovascular system are given below .
8
9
Superior vena cava 43 mm Hg
10 Pulmonary veins 98 mm Hg
11 Right atrium 65 mm Hg
12 Left atrium 97 mm Hg
13 Right ventricle 65 mm Hg
14 Left ventricle 97 mm Hg
15 Pulmonary artery 65 mm Hg
16 Ascending aorta 97 mm Hg
17 Descending aorta 97 mm Hg
18
19
The patient most likely has w hich of the follow ing findings on physical examination?
20
21
22 A. Decreased femoral-to-brachial blood pressure ratio [11 %]
23 B. Fixed splitting of S 2 on cardiac auscultation [48%]
24
25 C. Holosystolic murmur over the left sternal border [15%]
D. Mucosal cyanosis and finger clubbing [9%]
27
x @ E. Machinery-like murmur over the upper precordium [1 6%]
28
29
30 Explanation: User Id: 388147
31
32 In the normal individual, P0 2 levels are lowest in the deoxygenated blood of the superior vena cava, right
33
atrium, and right ventricle. After oxygenation in the pulmonary vasculature, blood P02 is high in the left atrium,
34
35
left ventricle, and aorta . In an individual w ith an atrial septal defect (ASD), the normally low P0 2 readings of
36 the right atrium (and right ventricle} are increased because of the shunting of oxygenated blood from the left
37 atrium to the right atrium . Based on the P0 2 readings provided above (including a low superior vena cava
38 PO,, moderate right atrial and right ventricular PO,, and high left atrial and left ventricular PO,), this individual
39
40
1 A

2 Item: 26 of 46 II P Mark -<:I t>-


Q. Id : 6539 (388147] Previous Next
3
4
5 In the normal individual, P0 2 levels are low est in the deoxygenated blood of the superior vena cava, right
6 atrium, and right ventricle. After oxygenation in the pulmonary vasculature, blood P0 2 is high in the left atrium,
7 left ventricle, and aorta . In an individual w ith an atrial septal defect (ASD), the normally low P0 2 readings of
8 the right atrium (and right ventricle) are increased because of the shunting of oxygenated blood from the left
9 atrium to the right atrium . Based on the P0 2 readings provided above (including a low superior vena cava
10
P0 2
, moderate right atrial and right ventricular P0 2
, and high left atrial and left ventricular P0 2
), this individual
11
12 has an ASD.
13
14 The left-to-right shunt of ASD causes right ventricle overload during diastole and an increase in pulmonary
15 blood flow . Larger left-to-right shunts tend to limit respiratory variation secondary to a delay in pulmonic valve
16 closure. As a consequence, S 2 (the sound of the semilunar valves closing) is w idely split and fixed. Because
17 the blood flow across the ASD is low velocity and minimally turbulent, it does not cause an audible murmur.
18 How ever, several other murmurs are associated w ith ASD, including a midsystolic pulmonary flow or ejection
19 murmur (secondary to increased flow across the pulmonic valve), a diastolic rumble (secondary to increased
20 flow across the tricuspid valve), and a low -pitched diastolic murmur (secondary to pulmonic regurgitation).
21
22 (Choices A & C) A decreased femoral-to-brachial blood pressure ratio is associated w ith coarctation of
23 aorta . Holosystolic murmur over the left sternal border is characteristic of ventricular septal defect (VSD). In
24 VSD right atrial blood oxygenation w ill not be increased.
25
(Choices D & E) Mucosal cyanosis and finger clubbing can be seen in Eisenmenger syndrome, w hich is
27 associated w ith a longstanding left-to-right shunt and pulmonary hypertension. Eventually, Eisenmenger
28 syndrome can result in the flow of deoxygenated blood from the right atrium to the left atrium (shunt reversal),
29 w ith the expected changes in P0 2 readings. Patent ductus arteriosus (PDA) is associated w ith a
30 machinery-like murmur over the upper precordium. The PDA allow s for oxygenated blood to flow from the
31 aorta to the pulmonary artery, causing a spike in the P0 2 reading of the pulmonary artery.
32
33
Educational objective:
34
Atrial septal defect is associated w ith a w idely split and fixed S 2 .
35
36
37 Time Spent: 186 seconds 48% answ ered correctly. Last updated: [11 /10/2011].
38 Copyright © USMLEWORLD,LLC. SimExam ver. 2011 .11.397425
1 A

2 Item: 27 of 46 II P Mark -<:I t>-


Q. Id : 6542 (388147] Previous Next
3
4
5
6 A Caucasian boy started crawling at 6 months of age and walking at 11 months of age. Novi, at age 5, he
7 needs support to walk and to rise from a sitting position. His 1-year-old brother can walk without assistance.
8 Their mother wonders what the chance is that the younger brother will suffer from the same condition (see
9 pedigree below).
10
11
12
13
14
15
16
17
18
19
20
21 .; @ A. 1/2 (70%]
22 B. 1/4 (23%]
23 C. 3/4 (3%]
24
25 D. 1/3 (4%]
26 E. 2/3 (1 %]

28
29 Explanation: User Id: 388147
30
31 Duchenne muscular dystrophy is an X-linked recessive disorder caused by a complete loss of function
32 mutation in the dystrophin gene. Patients with Duchenne muscular dystrophy develop normally at first,
33 crawling and walking at the appropriate age milestones. The disease typically presents between ages 3 and
34 5, with progressive muscle weakness, difficulty with physical play, and using Gowers maneuver to rise to a
35 standing position among the first signs. The weakness is progressive, and patients classically lose the ability
36 to walk by age 12. Since half of children born are male, 25% of the children of a mother who is a carrier for
37 Duchenne muscular dystrophy will be affected. In other words, half of male children will be affected while no
38 female children will be affected.
1 A

2 Item: 27 of 46 II P Mark -<:I t>-


Q. Id : 6542 (388147] Previous Next
3
4
5
6
7
8
9
10
11
12
~ @ A. 1/2 [70%]
13
14 B. 1/4 [23%]
15 C. 3/4 [3%]
16
D. 1/3 [4%]
17
18 E. 2/3 [1 %]
19
20
21 Explanation: User Id: 388147
22
23 Duchenne muscular dystrophy is an X-linked recessive disorder caused by a complete loss of function
24 mutation in the dystrophin gene. Patients w ith Duchenne muscular dystrophy develop normally at first,
25 crawling and w alking at the appropriate age milestones. The disease typically presents between ages 3 and
26 5, w ith progressive muscle w eakness, difficulty w ith physical play, and using Gowers maneuver to rise to a
standing position among the first signs. The w eakness is progressive, and patients classically lose the ability
28 to w alk by age 12. Since half of children born are male, 25% of the children of a mother w ho is a carrier for
29 Duchenne muscular dystrophy w ill be affected. In other words, half of male children w ill be affected w hile no
30 female children w ill be affected.
31
32 Educational objective:
33 Duchenne muscular dystrophy is an X-linked recessive disorder that classically affects males. Though most
34 cases are inherited, 1/3 of cases arise from new mutations w ith no family history of the disease. 50% of the
35 male children of a carrier mother w ill be affected.
36
37 Time Spent: 122 seconds 70% answ ered correctly. Last updated: [11 / 10/2011].
38 Copyright © USMLEWORLD,LLC. SimExam ver. 2011 .11.397425
1 A

2 Item: 28 of 46 II P Mark -<:I t>-


Q. Id : 6614 (388147] Previous Next
3
4
5
6 A 65-year-old man has a colonoscopy that reveals a suspicious lesion. Tissue biopsy confirms a diagnosis of
7 adenocarcinoma of the colon. At dinner that night, the man describes to his family at length the different
8 stages of colon cancer and the various treatments available. He insists that his family not worry too much
9 because he has learned about the disease and know s w hat to expect. W hat defense mechanism is this
10 patient using?
11
12
A. Denial [2%]
13
14 .; @ B. lntellectualization [83%]
15 C. Rationalization [12%]
16
D. Suppression [3%]
17
18 E. Repression [1 %]
19
20
21 Explanation: User Id: 388147
22
lntellectualization involves extensive, often academic-type thought about a topic in order to avoid emotional
23
issues. The man above intellectualizes his diagnosis by researching it extensively and talking about it in a
24
detached fashion . lntellectualization is considered an immature defense mechanism.
25
26
(Choice A) Denial is a defense mechanism in w hich reality is simply ignored, especially feelings that may be
27
too painful to acknow ledge. The patient above is not denying the existence of his illness.
29
(Choice C) Rationalization is an attempt to cover up unreasonable or unacceptable acts or ideas using
30
convincing reasoning. In rationalization, individuals find w ays to justify ideas, feelings, or actions that w ould
31
otherwise appear illogical or irrational.
32
33
34 (Choice D) Suppression is a mature defense mechanism that involves conscious or semi-conscious
attempts to temporarily w ithhold unpleasant information from one's aw areness to postpone dealing w ith it. An
35
example w ould a student deciding not to w orry about the results of an exam until they are released. This man
36
has certainly not postponed acknow ledgement of his new diagnosis.
37
38
1 A

2 Item: 28 of 46 II P Mark -<:I t>-


Q. Id : 6614 (388147] Previous Next
3
4 v @ B. lntellectualization [83%]
5 C. Rationalization [12%]
6
D. Suppression [3%]
7
8 E. Repression [1 %]
9
10
11 Explanation: User Id: 388147
12
13 lntellectualization involves extensive, often academic-type thought about a topic in order to avoid emotional
14 issues. The man above intellectualizes his diagnosis by researching it extensively and talking about it in a
15 detached fashion . lntellectualization is considered an immature defense mechanism.
16
17 (Choice A) Denial is a defense mechanism in w hich reality is simply ignored, especially feelings that may be
18 too painful to acknow ledge. The patient above is not denying the existence of his illness.
19
20 (Choice C) Rationalization is an attempt to cover up unreasonable or unacceptable acts or ideas using
21 convincing reasoning. In rationalization, individuals find w ays to justify ideas, feelings, or actions that would
22 otherwise appear illogical or irrational.
23
24 (Choice D) Suppression is a mature defense mechanism that involves conscious or semi-conscious
25 attempts to temporarily w ithhold unpleasant information from one's awareness to postpone dealing w ith it. An
26 example w ould a student deciding not to worry about the results of an exam until they are released. This man
27 has certainly not postponed acknow ledgement of his new diagnosis.

29 (Choice E) Repression is an immature defense mechanism w here unpleasant feelings are expelled or
30 w ithheld from one's consciousness. Unlike suppression, repressed thoughts are unconsciously pushed
31 away, not to be dealt w ith at a later point.
32
33 Educational objective:
34 lntellectualization is use of excessive thinking in order to avoid painful emotions or fears. It is particularly
35 common in patients recently diagnosed w ith serious medical illnesses.
36
37 Time Spent: 300 seconds 83% answ ered correctly. Last updated: [3/27/201 O].
38 Copyright © USMLEWORLD,LLC. SimExam ver. 2011 .11.397425
1 •
2 Item: 29 of 46 • \' Mark -<:J C>
Q. Id: 6552 (383147) Previous NeHt
3
4
5
6 A 40-year-old Caucasian male complains of food "sticking" in his throat. He also experiences nocturnal
7 regurgitation and cough. Barium study findings are shown in the image below.
8
9
10
11
12
13
14
15
16
17
18
19 '°
20
21
22
23
24
25
26
27
28

30
31
32
33
34
D/saployN wiltl pennlulon'""" c - t Medklne ~
35 LLCC 2000
36
37 Which of the following is the most likely cause of this patient's complaints?
38
1 A

2 Item: 29 of 46 II P Mark -<:I t>-


Q. Id : 6552 (388147] Previous Next
3
4
5
A. Gastroesophageal reflux [10%]
6
7 .; @ B. Esophageal aperistalsis [80%]
8 C. lntramblral leiemyema (2q~]
9
D. Saccbllar aertic aneblrysm (1 q~]
10
11 E. Metaplasia of the esophageal mucosa [6%]
12
13
14 Explanation: User Id: 388147
15
16 Achalasia (Greek for "does not relax") is an esophageal motility disorder characterized by the absence of
17 esophageal peristalsis in the distal esophagus and a hypertensive low er esophageal sphincter (LES) that is
18 unable to relax . These abnormalities are caused by reduced numbers of inhibitory ganglion cells in the
19 esophageal w all, w hich creates an imbalance favoring activity of excitatory ganglion cells. The
20 gastroesophageal junction becomes functionally obstructed as a result. Consequently, affected individuals
21 can experience dysphagia, regurgitation, chest pain, heartburn, and w eight loss. On barium sw allow , the
22 esophagus is dilated, w ith a narrow ing of the distal esophagus that has been likened to a "bird's beak" (as
23 seen in the image above).
24
25 (Choices A, C, D, and E) Gastroesophageal reflux is not easily detected on barium sw allow , unless the
26 patient has a fully open LES in the presence of free reflux; upper GI endoscopy or pH testing may be of more
27 benefit. Structural abnormalities (eg, large intramural leiomyomas) can be detected on barium sw allow ,
28 especially if the abnormality compresses the esophageal lumen. Strictures caused by Barrett metaplasia are
occasionally detected on barium sw allow , especially w hen the disease is advanced. Saccular aortic
30 aneurysms are not readily identified on barium swallow ; instead, ultrasonography is the preferred modality
31 w hen an asymptomatic, pulsatile abdominal mass is identified.
32
33 Educational objective:
34 Achalasia is characterized on barium sw allow by a dilated esophagus w ith distal narrow ing in the shape of a
35 "bird's beak."
36
37 Time Spent: 354 seconds 80% answ ered correctly. Last updated: [3/27/201 O].
38 Copyright © USMLEWORLD,LLC. SimEx am ver. 2011 .11.397425
1 A

2 Item: 30 of 46 II P Mark -<:I t>-


Q. Id : 6562 (388147] Previous Next
3
4
5
6 A 45-year-old African.;8merjcan male treated for uncomplicated prostatitis w ith trimethoprim/sulfamethoxazole
7 complains of sudden onset W:eaJsnes§..and exertional tach)'cardia His hemoglobin level is Z.2..rng/dl and
8 reticulocyte count is 8.0%. W hich of the following is the most likely cause of his symptoms?
9
10
11
A. Hemolysis due to RBC structural protein deficiency [12%]
12
13
14 B. Autoimmune hemolysis [8%]
15
16
17 C. Erythrocyte enzyme deficiency [69%]
18
19
20
21 D. Impaired hemoglobin synthesis [5%]
X @
22
23
24 E . Erythroid precursor DNA synthesis defect [7%]
25
26
27
28
29 Explanation: User Id: 388147

31 W hile being treated w ith trimethoprim/sulfamethoxazole, this patient develops signs and symptoms of acute
32 onset anemia . Based on his African-American heritage and recent usage of sulfamethoxazole (a sulfonamide
33 w ith oxidant properties), the diagnosis of glucose-6-phosphate dehydrogenase (G6PD) deficiency appears
34 most likely. This X-linked deficiency is most common among persons of African, Asian, or Mediterranean
35 descent. G6PD deficiency is characterized by inadequate production of reducing equivalents through the
36 pentose phosphate shunt pathw ay (the first step of w hich is catalyzed by G6PD). Consequently, oxidant
37 stress in an individual w ith G6PD deficiency can lead to methemoglobin formation, oxygen free radical
38 generation, and hemoglobin denaturation and precipitation w ithin RBCs. These changes can damage the
' ,, ' ' . . ,, ' ' '
39
40
1 A

2 Item: 30 of 46 II P Mark -<:I t>-


Q. Id : 6562 (388147] Previous Next
3
4
5 E. Erythroid precursor DNA synthesis defect [7%]
6
7
8
9
10 Explanation: User Id: 388147
11
12 While being treated w ith trimethoprim/sulfamethoxazole, this patient develops signs and symptoms of acute
13 onset anemia . Based on his African-American heritage and recent usage of sulfamethoxazole (a sulfonamide
14 w ith oxidant properties), the diagnosis of glucose-6-phosphate dehydrogenase (G6PD) deficiency appears
15 most likely. This X-linked deficiency is most common among persons of African, Asian, or Mediterranean
16 descent. G6PD deficiency is characterized by inadequate production of reducing equivalents through the
17 pentose phosphate shunt pathw ay (the first step of w hich is catalyzed by G6PD). Consequently, oxidant
18 stress in an individual w ith G6PD deficiency can lead to methemoglobin formation, oxygen free radical
19 generation, and hemoglobin denaturation and precipitation w ithin RBCs. These changes can damage the
20 RBC membrane, thereby acutely increasing the rate of intravascular and extravascular RBC hemolysis.
21
22 (Choice A) An example of a hemolytic anemia caused by an RBC structural protein defect is hereditary
23 spherocytosis. In this condition, the RBC structural components spectrin or ankyrin are abnormal.
24
25 (Choice B) W arm antibody autoimmune hemolytic anemia (AIHA) is the most common of the autoimmune
26 hemolytic diseases. It can be precipitated by drugs (eg, penicillin, cephalosporins) but is more often idiopathic.
27
28 (Choice D) An African-American might have a-thalassemia, w hich is characterized by impaired hemoglobin
29 synthesis. This condition is not commonly exacerbated by drug intake, how ever.

31 (Choice E) Reticulocyte count is not significantly elevated in megaloblastic anemia .


32
33 Educational objective:
34 Acute onset anemia in an African-American male being treated w ith a sulfonamide is most likely due to oxidant
35 stress hemolysis precipitated by glucose-6-phosphate dehydrogenase deficiency.
36
37 Time Spent: 244 seconds 69% answ ered correctly. Last updated: [3/27/201 O].
38 Copyright © USMLEWORLD,LLC. SimExam ver. 2011 .11.397425
1 •
2 Item: 31of46 • \ ' Mark -<:J C>
Q. Id: 6566 (3831471 Previous NeHt
3
4
5
6 A 63-year-old male is brought to the emergency room with difficulty speaking. He is only able to say simple
7 words like "yes" or "no." He can obey all spoken and written commands, but cannot repeat words spoken to
8 him. Which of the following areas labeled on the slide below is most likely damaged in this patient?
9
10
11
12
13
14
15
16
17
18
19 '°
20
21
22
23
24
25
26
27
28
29 Dl 1played with p•rml11Jon f rom Sprl n.1 •r He1lthc11r e Ltd.
30 CICopyrl1ht 1997 by Current M •dlcine

32
33 A. A [5%)
34
~ @ B. B [72%)
35
36 C. C [3%]
37 D. D (10%]
1 A

2 Item: 31of46 ll P Mark -<:I t>-


Q. Id : 6566 (388147] Previous Next
3
4 Explanation: User Id: 388147
5
6 This patient has Broca's (motor} aphasia, w hich results from a lesion in the inferior frontal gyrus of the frontal
7 lobe (Brodmann areas 44 & 45, Choice B}. This area is located near the head and upper limb regions of the
8 primary motor cortex, w hich is w hy right arm and face w eakness often accompany Broca's aphasia . Affected
9 patients demonstrate good comprehension but nonfluent speech and poor repetition.
10
11 Aphasias are classified based on the patient's capacity for comprehension, fluent speech, and ability to
12 repeat. The key forms of aphasia are summarized in the table below :
13 Aphasia Lesion Speech Comprehension Repetition
14
Broca (motorl lnferi or frontal nvrus Nonfluent Good Poor
15
Wernicke Superior temporal Fluent Poor Poor
16
(sensorvl avrus
17
18
Conduction Arcuate fasci culus Fluent Good Poor
(insulal
19 - Global Both Broca and Nonfluent Poor Poor
20
21 Wernicke areas
22 Transcortical Near Broca area Nonfluent Good Good
23 motor
24 Transcortical NearWernicke area Fluent Poor Good
25 sensorv
26
27 (Choice A) The frontal eye field (Brodmann area 8} is involved in control of eye movement. Destruction
28 causes ipsilateral deviation of the eyes.
29
30 (Choice C) The primary motor cortex houses the cell bodies of the upper motor neurons. Destruction of this
region causes contralateral spastic paresis.
32
33 (Choice D) The primary somatosensory cortex processes sensory input from the body. Lesions here cause
34 astereognosis (inability to identify objects by touch alone}, agraphesthesia (inability to identify characters
35 drawn on the skin by touch alone}, and hemihypesthesia (loss of sensation on one side of the body}.
36
37 (Choice E) This is the W ernicke area (Brodmann area 22}.
38
1 A

2 Item: 31of46 ll P Mark -<:I t>-


Q. Id : 6566 (388147] Previous Next
3
4
Aphasias are classified based on the patient's capacity for comprehension, fluent speech, and ability to
5
repeat. The key forms of aphasia are summarized in the table below:
6
7 Aphasia Lesion Speech Comprehension Repetition
8 Broca (motor) lnferi or frontal avrus Nonfluent Good Poor
9 Wernicke Superior temporal Fluent Poor Poor
10 tsensorvl avrus
11 Conduction Arcuate fasci culus Fluent Good Poor
12 (insulal
13 Global Both Broca and Nonfluent Poor Poor
14 Wernicke areas
15 Transcortical Near Broca area Nonfluent Good Good
16 motor
17 NearWernicke area
Transcortical Fluent Poor Good
18
sensorv
19 -
20
21 (Choice A) The frontal eye field (Brodmann area 8) is involved in control of eye movement. Destruction
22 causes ipsilateral deviation of the eyes.
23
24 (Choice C) The primary motor cortex houses the cell bodies of the upper motor neurons. Destruction of this
25 region causes contralateral spastic paresis.
26
27 (Choice D) The primary somatosensory cortex processes sensory input from the body. Lesions here cause
28 astereognosis (inability to identify objects by touch alone), agraphesthesia (inability to identify characters
29 drawn on the skin by touch alone), and hemihypesthesia (loss of sensation on one side of the body).
30
(Choice E) This is the W ernicke area (Brodmann area 22).
32
33 Educational Objective:
34 Motor (Broca's) aphasia is caused by lesions of the inferior frontal gyrus. It is a nonfluent aphasia w ith intact
35 comprehension and poor repetition. It is frequently associated w ith right-sided arm and face w eakness.
36
37 Time Spent: 60 seconds 72% answ ered correctly. Last updated: [10/3/2011].
38 Copyright © USMLEWORLD,LLC. SimExam ver. 2011 .11.397425
1 A

2 Item: 32 of 46 • \> Mark -<J C>-


Q. Id: 6571 (388147) Previous Next
3
4
5
6 A 31-year-old HIV-positive male complains of fever, pelvic pain, urinary frequency and dysuria. A tender mass
7 is palpated on rectal examination. Which of the following is most likely affected in this patient?
8
9
10
11
12
13
14
15
16
17
18
19 -
20
21
22
23
24 A. A [4%)
25
26
x ® B. B [20%)
27 ~ c. c [46%)
28 D. D [22%)
29
30
E. E [8%)
31
Explanation: User Id: 3881 47
33
34 This patient has prostatitis. Affected patients classically present with constitutional symptoms such as fever,
35 chills and malaise, in addition to urinary symptoms like dysuria and urinary frequency. Digital rectal exam
36 typically reveals a tender, warm prostate. The above MRI shows a cross-section of the male pelvis with the
37 prostate (C hoice C) visible just posterior to the pubic symphysis.
38
--
39
40 1ne Systems
41 ~
1 A

2 Item: 32 of 46 II P Mark -<:I t>-


Q. Id : 6571 (388147] Previous Next
3
4
5
6
7
8
9
10
11
A. A [4%]
12 X @ B. B [20%]
13 ~ C. C [46%]
14
15
D. D [22%]
16 E. E [8%]
17
18
19 Explanation: User Id: 388147
20
21 This patient has prostatitis. Affected patients classically present with constitutional symptoms such as fever,
22 chills and malaise, in addition to urinary symptoms like dysuria and urinary frequency. Digital rectal exam
23 typically reveals a tender, warm prostate. The above MRI shows a cross-section of the male pelvis with the
24 prostate (Choice C) visible just posterior to the pubic symphysis.
25
26 (Choices A & B) On MRI the pubic ramus and pubic symphysis (bone) appear dark, while the cartilage of the
27 symphyseal joint appears gray.
28
29 (Choices D & E) The anal canal and the levator ani muscle, respectively, are seen in the posterior midline of
30 the image provided.
31
Educational objective:
33 Prostatitis is characterized clinically by constitutional and urinary symptoms plus prostate gland tenderness
34 on digital rectal examination. On axial images of the male pelvis, the prostate is seen posterior to the pubic
35 symphysis.
36
37 Time Spent: 80 seconds 46% answered correctly. Last updated: [11 /10/2011].
38 Copyright © USMLEWORLD,LLC. SimExam ver. 2011 .11.397425
1 •
2 Item: 33 of 46 • \ ' Mark -<:J C>
Q. Id: 6576 (383147) Previous NeHt
3
4 In animal experiments, the ettects ot various substances on the glomerular t11trat1on rate (<:il-H) ano renal
5 plasma flow (RPF) are studied. Mean arterial pressure (MAP) is kept constant. In the figures below,
6 convergent arrowheads represent arteriolar constriction, while divergent arrowheads represent arteriolar
7 dilation. Which of the following figures signifies an increase in GFR and a decrease in RPF?
8
9
10
11
12 Afferent WEfferent
13
14
A. A [3%)
15
16
17
18
19 W Efferent
'° Afferent
20
21 B. B [3o/o)
22
23
24 ~
25
26
.; @
Afferent Do
()'t)
Efferent

27
28 C. c [87%)
29
30
31
Afferent
~Efferent
32

34 D
D. [4%]
35
1 A

2 Item: 33 of 46 II P Mark -<:I t>-


Q. Id : 6576 (388147] Previous Next
3

~
4
5
D. D [4%]
6
7
8 ~ /
-
'
9
10
11
Afferent Do
O'D
Efferent

12 E. E [3%]
13
14
15 Explanation: User Id: 388147
16
17 These experiments analyze the effect that changes in afferent or efferent arteriolar resistance w ill have on the
18 GFR and RPF. For a single nephron, RPF can be related to MAP and the total vascular resistance (TVR) in
19 the afferent and efferent arterioles as follow s:
20
21
RPF =MAP I TVR
22 In this case, TVR in the nephron equals the resistance in the afferent arteriole (Rafferent) plus the resistance in
23
24 the efferent arteriole (R•ffaenil· Therefore, for RPF to decrease w hen MAP is held constant, Rafferent and I or
25 Reffaent must increase. Likew ise, a decrease in either of these values w ill cause an increase in RPF.
26
27 The GFR can be estimated using the starling equation as follows:
28
29
GFR =(PGe - P ssl - (PGc - Pssl

30
This equation determines w hether the net hydrostatic pressure (P) and the net oncotic pressure (p) favors
31
flow of fluid from the glomerular capillaries (GC) to Bowman's space (BS). If all else is held equal, this
32
equation show s that an increase in hydrostatic pressure in the glomerular capillaries w ill favor increased
filtration and an increase in GFR. Constriction of the afferent arteriole w ill reduce steady state hydrostatic
34
pressure in the glomerular capillary (PGe) thereby decreasing the filtration of fluid from the plasma into
35
36 Bowman's space and decreasing GFR. Constriction of the efferent arteriole w ill increase Refferent, increase
37 the steady state PGC and increase the GFR. In summary, constriction of the efferent arteriole w ill increase
38 GFR and decrease RPF.
1 A

2 Item: 33 of 46 II P Mark -<:I t>-


Q. Id : 6576 (388147] Previous Next
3
4
5 In this case, TVR in the nephron equals the resistance in the afferent arteriole (R.n_..01} plus the resistance in
6 the efferent arteriole (R.n.,.01}. Therefore, for RPF to decrease w hen MAP is held constant, Rattaent and I or
7 Rettaent must increase. Likew ise, a decrease in either of these values w ill cause an increase in RPF .
8
9 The GFR can be estimated using the starling equation as follow s:
10
11
GFR =(PGe - Pesl - (PGc - Pesl
12
13 This equation determines w hether the net hydrostatic pressure (P) and the net oncotic pressure (p) favors
14 flow of fluid from the glomerular capillaries (GC) to Bowman's space (BS). If all else is held equal, this
15 equation show s that an increase in hydrostatic pressure in the glomerular capillaries w ill favor increased
16 filtration and an increase in GFR. Constriction of the afferent arteriole w ill reduce steady state hydrostatic
17 pressure in the glomerular capillary (P ocl thereby decreasing the filtration of fluid from the plasma into
18 Bowman's space and decreasing GFR. Constriction of the efferent arteriole w ill increase Refferent, increase
19 the steady state PGC and increase the GFR. In summary, constriction of the efferent arteriole w ill increase
20 GFR and decrease RPF.
21
22 (Choice A) Dilation of the afferent arteriole w ill cause an increase in RPF and GFR.
23
24 (Choice B) Dilation of the efferent arteriole w ill increase RPF and decrease GFR.
25
26 (Choice D) Constriction of the afferent arteriole w ill reduce the RPF and the GFR.
27
28 (Choice E) Dilation of both the afferent and efferent arterioles w ill increase RPF and decrease GFR.
29
30 Educational objective:
31 For RPF to decrease at a fixed mean arterial pressure, Rattaent and/or R•ffaent must increase. If other factors
32 affecting GFR are held constant, then an increase in R.n.,ent w ill increase GFR by increasing glomerular
capillary hydrostatic pressure, w hile an increase in Retterent w ill reduce GFR by decreasing the glomerular
34
35 capillary hydrostatic pressure.
36
37 Time Spent: 146 seconds 87% answ ered correctly. Last updated: [1 0/ 12/2011].
38 Copyright © USMLEWORLD,LLC. SimExam ver. 2011 .11.397425
1 A

2 Item: 34 of 46 II P Mark -<:I t>-


Q. Id : 6581 (388147] Previous Next
3
4
5
6 During an experiment, a researcher cuts u~)'elinatecloe!Y,e..fjb,ers. These fibers are most likely a
7 component of w hich of the follow ing?
8
9
10 A. Motor nerve to skeletal muscle [7%]
11
12 B. Sensory nerve from tendon [1 6%]
13
14 C. Sensory nerve from pacinian corpuscle [40%]
15
16
17
D. Autonomic preganglionic nerve [1 6%]
18
19 ~ @ E. Autonomic postganglionic nerve [22%]
20
21
22
23 Explanation: User Id: 388147
24
25 Of all the choices listed, only postganglionic autonomic nervous system axons are
26 unmyelinated. Unmyelinated neurons have a smaller diameter than m yelinated neurons and conduct action
27 potentials more slow ly (-2m/s). Other important unmyelinated axons include the afferent neurons that
28 conduct heat sensation and the afferent neurons that transmit slow-onset dull, burning or visceral pain. First
29 order bipolar sensory neurons of olfaction are also unmyelinated. Unmyelinated neurons are also referred to
30 as group C nerve fibers.
31
32 (Choices A, B, C & D) Motor neuron axons to skeletal muscles are m yelinated, allow ing for rapid conduction
33 velocity (Group A-alpha nerve fibers). Sensory axons that innervate Golgi tendon organs and conduct afferent
information on muscle tension contain A-beta m yelinated fibers. Pacinian corpuscles are rapidly adapting
mechanoreceptors located in the subcutaneous tissue of the skin as w ell as the mesentery, peritoneum and
joint capsules. They mediate touch and vibratory sensation and are innervated by m yelinated A-beta fibers.
37 Preganglionic autonomic nervous system fibers (both sympathetic and parasympathetic) are also m yelinated
38 (B fibers).
1 A

2 Item: 34 of 46 II P Mark -<:I t>-


Q. Id : 6581 (388147] Previous Next
3
4
5 C. Sensory nerve from pacinian corpuscle [40%]
6
7
D. Autonomic preganglionic nerve [16%]
8
9
10 .; @ E. Autonomic postganglionic nerve [22%]
11
12
13
14 Explanation: User Id: 388147
15
16 Of all the choices listed, only postganglionic autonomic nervous system axons are
17 unmyelinated. Unmyelinated neurons have a smaller diameter than myelinated neurons and conduct action
18 potentials more slow ly (-2m/s). Other important unmyelinated axons include the afferent neurons that
19 conduct heat sensation and the afferent neurons that transmit slow -onset dull, burning or visceral pain. First
20 order bipolar sensory neurons of olfaction are also unmyelinated. Unmyelinated neurons are also referred to
21 as group C nerve fibers.
22
23 (Choices A, 8, C & D) Motor neuron axons to skeletal muscles are myelinated, allow ing for rapid conduction
24 velocity (Group A-alpha nerve fibers). Sensory axons that innervate Golgi tendon organs and conduct afferent
25 information on muscle tension contain A-beta myelinated fibers. Pacinian corpuscles are rapidly adapting
26 mechanoreceptors located in the subcutaneous tissue of the skin as w ell as the mesentery, peritoneum and
27 joint capsules. They mediate touch and vibratory sensation and are innervated by myelinated A-beta fibers.
28 Preganglionic autonomic nervous system fibers (both sympathetic and parasympathetic) are also myelinated
29 (B fibers).
30
31 Educational objective:
32 Unmyelinated nerve fibers conduct action potentials more slow ly than do myelinated nerve fibers. Among
33 primary sensory (afferent) fibers, those responsible for slow pain, heat sensation and olfaction are
unmyelinated. Efferent neuron axons tend to be myelinated, w ith the notable exception of unmyelinated fibers
that originate from postganglionic autonomic neurons.

37 Time Spent: 118 seconds 22% answ ered correctly. Last updated: [3/27/201 O].
38 Copyright © USMLEWORLD,LLC. SimExam ver. 2011 .11.397425
1 A

2 Item: 35 of 46 II P Mark -<:I t>-


Q. Id : 6585 (388147] Previous Next
3
4
5
6 A 23-year-old woman tells her therapist that she and her boyfriend have recently developed a deeper level of
7 bonding because she has adopted his belief that God is protecting them w ith a green light that surrounds
8 them at all times. The light is visible only to the two of them . She most likely suffers from w hich of the
9 following?
10
11
A. Malingering [2%]
12
13 B. Schizotypal personality disorder [10%]
14 ~ C. Shared psychotic disorder [59%]
15
X @ D. Delusional disorder [26%]
16
17 E. Schizophrenia, undifferentiated [3%]
18
19
Explanation: User Id: 388147
20
21
This woman and her boyfriend share a common delusion . According to the DSM-IV, shared psychotic
22
disorder is characterized by the development of delusions in a person w ho is in a close relationship w ith a
23
delusional individual, w here the disturbance is not better explained by another psychotic disorder (e.g.
24
25 schizophrenia) or by the effects of a medical condition, medication, or drug of abuse.
26
(Choice A) Malingering is the intentional feigning of symptoms for secondary gain.
27
28
(Choice B) Schizotypal personality disorder is characterized by magical thinking and odd beliefs such as
29
telepathy, superstitions, bizarre fantasies or preoccupations. There may also be eccentric behavior, odd
30
thinking and ideas of reference. Individuals w ith schizotypal personality disorder often lack close relationships,
31
unlike the patient described here.
32
33
(Choice D) Delusional disorder is characterized by non-bizarre delusions present for at least one
34
month. "Non-bizarre" describes situations that, w hile unlikely, are possible. Examples include being followed,
cheated, or poisoned. Other features of delusional disorder include preserved occupational/social functioning
and failure to meet diagnostic criteria for schizophrenia . This patient could qualify for delusional disorder if
37
she did not share her boyfriend's delusion.
38
1 A

2 Item: 35 of 46 II P Mark -<:I t>-


Q. Id : 6585 (388147] Previous Next
3
4 .; C. Shared psychotic disorder [59%]
5
6
x @ D. Delusional disorder [26%]
7 E. Schizophrenia, undifferentiated [3%]
8
9
10 Explanation: User Id: 388147
11
12 This woman and her boyfriend share a common delusion. According to the DSM-IV, shared psychotic
13 disorder is characterized by the development of delusions in a person w ho is in a close relationship w ith a
14 delusional individual, w here the disturbance is not better explained by another psychotic disorder (e.g.
15 schizophrenia) or by the effects of a medical condition, medication, or drug of abuse.
16
17 (Choice A) Malingering is the intentional feigning of symptoms for secondary gain.
18
19 (Choice B) Schizotypal personality disorder is characterized by magical thinking and odd beliefs such as
20 telepathy, superstitions, bizarre fantasies or preoccupations. There may also be eccentric behavior, odd
21 thinking and ideas of reference. Individuals w ith schizotypal personality disorder often lack close relationships,
22 unlike the patient described here.
23
24 (Choice D) Delusional disorder is characterized by non-bizarre delusions present for at least one
25 month. "Non-bizarre" describes situations that, w hile unlikely, are possible. Examples include being follow ed,
26 cheated, or poisoned. Other features of delusional disorder include preserved occupational/social functioning
27 and failure to meet diagnostic criteria for schizophrenia . This patient could qualify for delusional disorder if
28 she did not share her boyfriend's delusion.
29
30 (Choice E) Although this patient harbors a delusion, she does not demonstrate other key features of
31 schizophrenia like hallucinations, disorganized speech, etc.
32
33 Educational Objective:
34 Shared psychotic disorder occurs w hen an individual in a close relationship w ith a person w ith an established
delusion develops that same delusion.

37 Time Spent: 78 seconds 59% answ ered correctly. Last updated: [3/27/201 O].
38 Copyright © USMLEWORLD,LLC. SimExam ver. 2011 .11.397425
1 A

2 Item: 36 of 46 II P Mark -<:I t>-


Q. Id : 6590 (388147] Previous Next
3
4
5
6 Two groups of investigators are studying the epidemiology of tuberculosis in rural Nepal. The first group uses
7 a sample of 1,000 rural residents and the second group uses a sample of 5,000 rural residents. Compared to
8 the first group, the second group is likely to report a lower:
9
10
11
A. Incidence [4%]
12 B. Prevalence [7%]
13 ~ C. Standard error [73%]
14
15
X @ D. Selection bias [ 15%]
16 E . Range [2%]
17
18
19 Explanation: User Id: 388147
20
21 Sample selection technique is an important component of study design. Proper technique minimizes
22 selection bias (choice D), making the sample as representative of the base population as
23 possible. Measures of disease occurrence like incidence (choice A) and prevalence (choice B) taken from
24 the representative sample should reflect the actual disease occurrence in the base population. Even w ith
25 good sample selection technique, how ever, there is alw ays random error. Standard error (SE) is a measure
26 of variability about the mean, calculated as:
27
28 SE =o/../n, w here o is the standard deviation and n is the sample size
29
30 As you can see from the formula, the SE decreases as the sample size is increased. Range is the difference
31 between extreme observations in the sample. It does not decrease w ith increased sample size (choice E).
32
33 Educational objective:
34 Standard error is a measure of variability around the mean. The larger the sample size, the smaller the SE.
35
Time Spent: 55 seconds 73% answered correctly. Last updated: [3/27/201 O].
37 Copyright © USMLEWORLD,LLC. SimExam ver. 2011 .11.397425
38
1 A

2 Item: 37 of 46 II P Mark -<:I t>-


Q. Id : 6593 (388147] Previous Next
3
4
5
6 An 8-year-old male is brought to the emergency department w ith a two day history of headaches, vomiting and
7 fever. He starts to cry w hen you turn on the examination room lights. Lumbar puncture is performed, and a
8 positive-sense single-sir nded RNA virus is isolated from the boy's cerebrospinal fluid. This virus most likely
9 belongs to w hich of the follow ing families?
10
11
.; @ A . Picomavirus [58%]
12
13 B. Reovirus [1 0%]
14 C. Coronavirus [11 %]
15
D. Arenavirus [7%]
16
17 E. Paramyxovirus [15%]
18
19
20 Explanation: User Id: 388147
21
22 This patient has aseptic meningitis caused by a positive-sense single-stranded RNA virus. Aseptic meningitis
23 causes acute fever, headache, photophobia, nausea and vomiting, plus nuchal rigidity on physical
24 examination. CSF analysis w ill reveal a predominately lymphocytic pleocytosis w ith normal glucose and
25 normal or slightly elevated protein levels. Enteroviruses are most commonly the culprit, accounting for
26 approximately 90% of cases w here an etiologic agent can be identified. The enteroviruses include poliovirus,
27 coxsackie A & B viruses, echovirus, enterovirus and hepatitis A . All of the aforementioned viruses except
28 hepatitis A have the potential to cause aseptic meningitis. The enterovirus genus belongs to the
29 picornaviridae family of viruses, w hich also includes rhinovirus, a cause of the common cold.
30
31 (Choice B) Members of the reoviridae family of viruses have double-stranded RNA genomes. Rotavirus, a
32 common cause of diarrhea in infants, belongs to this family.
33
34 (Choice C) Coronavirus is a positive-sense single-stranded RNA virus that, along w ith rhinovirus, is a major
35 cause of the common cold.
36
(Choice D) Arenaviruses are negative-sense single-stranded RNA viruses. Members of this family include
38 lymphocytic choriomeningitis virus, w hich can cause aseptic meningitis, and lassa virus, a hemorrhagic fever
. '. ' ' .
39
40
1 A

2 Item: 37 of 46 II P Mark -<:I t>-


Q. Id : 6593 (388147] Previous Next
3
4 E. Paramyxovirus [ 15%]
5
6
7 Explanation: User Id: 388147
8
9 This patient has aseptic meningitis caused by a positive-sense single-stranded RNA virus. Aseptic meningitis
10 causes acute fever, headache, photophobia, nausea and vomiting, plus nuchal rigidity on physical
11 examination . CSF analysis w ill reveal a predominately lymphocytic pleocytosis w ith normal glucose and
12 normal or slightly elevated protein levels. Enteroviruses are most commonly the culprit, accounting for
13 approximately 90% of cases w here an etiologic agent can be identified. The enteroviruses include poliovirus,
14 coxsackie A & B viruses, echovirus, enterovirus and hepatitis A. All of the aforementioned viruses except
15 hepatitis A have the potential to cause aseptic meningitis. The enterovirus genus belongs to the
16 picornaviridae family of viruses, w hich also includes rhinovirus, a cause of the common cold.
17
18 (Choice B) Members of the reoviridae family of viruses have double-stranded RNA genomes. Rotavirus, a
19 common cause of diarrhea in infants, belongs to this family.
20
21 (Choice C) Coronavirus is a positive-sense single-stranded RNA virus that, along w ith rhinovirus, is a major
22 cause of the common cold.
23
24 (Choice D) Arenaviruses are negative-sense single-stranded RNA viruses. Members of this family include
25 lymphocytic choriomeningitis virus, w hich can cause aseptic meningitis, and lassa virus, a hemorrhagic fever
26 virus passed to humans w hen dirt or dust contaminated by rodent urine is inhaled.
27
28 (Choice E) The paramyxoviridae family of negative-sense single-stranded RNA viruses includes the mumps
29 and measles viruses as w ell as respiratory syncytial virus (RSV) and parainfluenza virus. Both measles and
30 mumps may cause encephalitis. RSV is the most common cause of infant pneumonia and parainfluenza
31 virus causes croup.
32
33 Educational objective:
34 Enteroviruses are the most common cause of aseptic meningitis. These viruses are members of the
35 picornaviridae family, all of w hich have a positive-sense single-stranded RNA genome.
36
Time Spent: 61 seconds 58% answ ered correctly. Last updated: [3/27/201 O].
38 Copyright © USMLEWORLD,LLC. SimExam ver. 2011 .11.397425
1 A

2 Item: 38 of 46 II P Mark -<:I t>-


Q. Id : 6598 (388147] Previous Next
3
4
5
6 A researcher is studying macrophage antigen presentation to CD4+ T-lymphocytes. W hich of the follow ing
7 HLA class II components is digested by macrophages during antigen processing?
8
9
10 A. a chain [12%•]
11 X @ B. Transporter associated w ith antigen processing (TAP) [24%]
12 ~~ micregle9l!lin [25qfe]
13
14 D. Invariant chain [36%]
15 E. CD1 [2q~J
16
17
18 Explanation: User Id: 388147
19
20 Coded for by the HLA-D region, MHC class II molecules are primarily located on antigen-presenting cells
21 (macrophages, dendritic cells, and B-cells). Each class II molecule is a heterodimer composed of an a-chain
22 and a 13-chain. During antigen processing, the invariant chain brings the a- and 13-chains together to form a
23 stable complex in the rough endoplasmic reticulum (RER). The MHC-invariant complex then leaves the RER,
24 passes through the Golgi body, and enters an endocytic compartment in w hich the invariant chain is
25 proteolytic ally degraded and an external protein is inserted between the a- and 13-chains. This new ly formed
26 MHC-peptide complex travels to the cell surface, w here it is expressed and recognized by CD4+ helper T-ells.
27
28 (Choice A) The a-chain is part of the MHC-peptide complex but is not digested by macrophages during
29 antigen processing.
30
31 (Choices B and C) Viral proteins are degraded into peptides by proteasomes. The peptides are then
32 removed from the cytosol by transporters associated w ith antigen processing (TAPs), w hich stud the
33 endoplasmic reticulum (ER) membrane. Once inside the ER, the peptides assemble w ith transmembrane
34 polypeptide and 13 2 microglobulin. This complex then travels through the Golgi apparatus and inserts in the
35 plasma membrane for T-cell binding.
36
37 (Choice E) CD 1 molecules present lipid-containing foreign antigens (eg, m ycobacterial cell w all components)
to CD1 -specific T-cells.
39
40
1 A

2 Item: 38 of 46 II P Mark -<:I t>-


Q. Id : 6598 (388147] Previous Next
3
4 ~a m1sreg1eebl11n [Zb~"l

5 D. Invariant chain [36%]


6
7 E. CD1 [2%•]
8
9
Explanation: User Id: 388147
10
11
Coded for by the HLA-D region, MHC class II molecules are primarily located on antigen-presenting cells
12
(macrophages, dendritic cells, and B-cells). Each class II molecule is a heterodimer composed of an a-chain
13
and a 13-chain. During antigen processing, the invariant chain brings the a- and 13-chains together to form a
14
stable complex in the rough endoplasmic reticulum (RER). The MHC-invariant complex then leaves the RER,
15
passes through the Golgi body, and enters an endocytic compartment in w hich the invariant chain is
16
proteolytic ally degraded and an external protein is inserted between the a- and 13-chains. This new ly formed
17
MHC-peptide complex travels to the cell surface, w here it is expressed and recognized by CD4+ helper T-ells.
18
19
(Choice A) The a-chain is part of the MHC-peptide complex but is not digested by macrophages during
20
antigen processing.
21
22
(Choices B and C) Viral proteins are degraded into peptides by proteasomes. The peptides are then
23
removed from the cytosol by transporters associated w ith antigen processing (TAPs), w hich stud the
24
endoplasmic reticulum (ER) membrane. Once inside the ER, the peptides assemble w ith transmembrane
25
polypeptide and 13 2 microglobulin. This complex then travels through the Golgi apparatus and inserts in the
26
27 plasma membrane for T-cell binding.
28
29 (Choice E) CD1 molecules present lipid-containing foreign antigens (eg, m ycobacterial cell w all components)
30 to CD1-specific T-cells.
31
32 Educational objective:
33 During antigen processing in macrophages, the invariant chain is removed from the MHC-invariant complex
34 and replaced by an external protein . This MHC-peptide complex (containing an a-chain, 13-chain, and external
35 protein) is then expressed at the cell surface.
36
37 Time Spent: 107 seconds 36% answ ered correctly. Last updated: [3/27/201 O].
Copyright © USMLEWORLD,LLC. SimExam ver. 2011 .11.397425
1 A

2 Item: 39 of 46 II P Mark -<:I t>-


Q. Id : 6599 (388147] Previous Next
3
4
5
6 A 39-year-old female complains of reduced taste sensation, a burning sensation in her mouth, and lip
7 dryness. Laboratory findings are given belov1.
8 Hemoglobin 7.4 mg/dl
9 MCV 67fl
10
WBC count 5,500/mm 3
11
12 Platelet count 170,000/mm 3
13
14 Which of the follow ing is the most likely cause of her symptoms?
15
16 x @ A. f'erniGiebls anemia [16%•]
17
18
., B. lren ElefiGienGy [64%•]
19 C. Folate deficiency [4%]
20 D. Sideroblastic anemia [18%]
21
22 E. Hypothyroidism [1 %]
23
24
Explanation: User Id: 388147
25
26
Hypochromic, microcytic anemia is most commonly caused by iron deficiency secondary to blood loss. In
27
women of childbearing age, menstruation is the most common cause. In men and postmenopausal women,
28 occult blood loss in the gastrointestinal tract must be considered. Many iron-deficient individuals are
29
asymptomatic, w hile others present w ith typical symptoms of anemia (eg, w eakness, fatigue, headache,
30 irritability), as w ell as glossal pain, dry mouth, atrophy of the tongue papillae, and alopecia . When present,
31
pagophagia (a craving for ice) is specific for iron deficiency.
32
33 (Choice A) Pernicious anemia is a megaloblastic anemia that typically affects older women of northern
34 European descent. Patients may have lemon-yellov1 coloring, a smooth shiny tongue indicative of atrophic
35 glossitis, and a shuffling broad-based gait.
36
37 (Choice C) Folate deficiency causes megaloblastic anemia. Patients may present w ith sore tongue, pain on
38 sw allow ing, angular stomatitis, gastrointestinal upset, and hyperpigmentation in addition to typical symptoms
1 A

2 Item: 39 of 46 II P Mark -<:I t>-


Q. Id : 6599 (388147] Previous Next
3
4
5
6 Explanation: User Id: 388147
7
8 Hypochromic, microcytic anemia is most commonly caused by iron deficiency secondary to blood loss. In
9 w omen of childbearing age, menstruation is the most common cause. In men and postmenopausal w omen,
10 occult blood loss in the gastrointestinal tract must be considered . Many iron-deficient individuals are
11 asymptomatic, w hile others present w ith typical symptoms of anemia (eg, w eakness, fatigue, headache,
12 irritability), as w ell as glossal pain, dry mouth, atrophy of the tongue papillae, and alopecia . When present,
13 pagophagia (a craving for ice} is specific for iron deficiency.
14
15 (Choice A) Pernicious anemia is a megaloblastic anemia that typically affects older w omen of northern
16 European descent. Patients may have lemon-yellow coloring, a smooth shiny tongue indicative of atrophic
17 glossitis, and a shuffling broad-based gait.
18
19 (Choice C) Folate deficiency causes megaloblastic anemia . Patients may present w ith sore tongue, pain on
20 sw allow ing, angular stomatitis, gastrointestinal upset, and hyperpigmentation in addition to typical symptoms
21 of anemia .
22
23 (Choice D) Patients w ith sideroblastic anemia generally present w ith the typical anemia symptoms.
24 Peripheral blood smears often show basophilic stippling and microcytic, hypochromic red blood cells. Ring
25 sideroblasts may be seen in bone marrow aspirates. Alcoholism and myelodysplastic syndrome are
26 associated conditions.
27
28 (Choice E) Hypothyroidism is not associated w ith hematologic abnormalities. Signs and symptoms include
29 hypothermia, w eight gain, slow ed speech and movement, hoarseness, dry skin, pallor, brittle hair, coarse
30 facial features, macroglossia, bradycardia, and hyporeflexia.
31
32 Educational objective:
33 Iron deficiency causes a hypochromic, microcytic anemia . Affected individuals may present w ith typical
34 symptoms of anemia (eg, w eakness, fatigue, headache, irritability), as w ell as glossal pain, dry mouth,
35 atrophy of the tongue papillae, alopecia, and pagophagia .
36
37 Time Spent: 369 seconds 64% answ ered correctly. Last updated: [3/27/201 O].
38 Copyright © USMLEWORLD,LLC. SimExam ver. 2011 .11.397425
1 •
2 Item: 40 of 46 • \' Mark -<:J C>
Q. Id: 6602 (383147) Previous NeHt
3
4
5
6 A 43-year-old female who has been fasting for ten hours ingests 75 grams of glucose at the time indicated by
7 the black arrow below. The graph tracks her blood glucose and insulin levels over time.
8
9
10 120 Blood glucose
11 mg/di
12 80
13
14
15
16
Blood insulin
17
18
19 Normal baseline insulin level
20
21
22 Time
23
24 Which of the following is the best explanation for the observed findings?
25
26 A. RansreatiGeeta cell destrui;tien [2q~J
27
9. RansreatiGendecrine turner [13q~l
28
29 X @ C. Poor intestinal glucose absorption [19%]
30 ~ D. Insulin resistance in peripheral tissues [62%]
31
E. Impaired hepatic gluconeogenesis [4%]
32
33
34 Explanation: User Id: 388147
35
36
37
1 A

2 Item: 40 of 46 II P Mark -<:I t>-


Q. Id : 6602 (388147] Previous Next
3
4 OGTI}, this patient has normal blood glucose levels. She does not, therefore, have impaired glucose
5 tolerance or frank diabetes. Moreover, her insulin response to the OGTI - increased release follow ing
6 glucose administration and subsequent return to baseline - is relatively normal. The main abnormality seen
7 on the tracing above is baseline hyperinsulinemia, an early finding in insulin resistance.
8
9 In insulin resistance, the peripheral tissues (eg, adipose, skeletal muscle, liver) are unable to respond properly
10 to normal levels of insulin. The endocrine pancreas attempts to compensate by secreting larger amounts of
11 insulin to maintain blood glucose values w ithin normal range, as is seen above. In time, how ever, the
12 peripheral tissues become so resistant to insulin that serum glucose levels rise to abnormally high levels
13 despite maximal insulin production by the pancreas. This is know n as impaired glucose tolerance. Ultimately,
14 the pancreatic beta cells may fail, due in part to amyloid accumulation w ithin the islets, resulting in overt type II
15 diabetes mellitus.
16
17 (Choice A) In the setting of pancreatic beta cell destruction, insulin secretion rates w ould be inappropriately
18 decreased both at baseline and in response to a glucose challenge. Pancreatic beta cell destruction in
19 addition to peripheral insulin resistance defines type II diabetes.
20
21 (Choice B) An endocrine pancreatic tumor such as an insulinoma could raise baseline insulin levels but
22 w ould cause fasting hypoglycemia (fasting blood glucose < 40 mg/dl). Symptoms of hypoglycemia in
23 patients w ith this uncommon condition classically manifest follow ing exercise.
24
25 (Choice C) Intestinal glucose malabsorption w ould blunt the insulin secretary response to an oral glucose
26 challenge, w hereas this patient's plasma insulin response to the OGTI is relatively normal. Glucose-
27 galactose malabsorption can be inherited or acquired and primarily affects infants. Patients present w ith
28 osmotic diarrhea.
29
30 (Choice E) Impaired hepatic gluconeogenesis w ould low er fasting blood glucose levels and baseline insulin
31 levels during fasting . In patients w ith insulin resistance, the liver does not respond properly to insulin and
32 hepatic gluconeogenesis occurs for a longer period of time than it ought to.
33
34 Educational objective:
35 During fasting, a high baseline plasma insulin level in the setting of normoglycemia is a sign of insulin
36 resistance in the peripheral tissues.
37
38 Time Spent: 408 seconds 62% answ ered correctly.
1 A

2 Item: 41 of 46 II P Mark -<:I t>-


Q. Id : 6603 (388147] Previous Next
3
4
5
6 Marfan syndrome, a disease caused by a fibrillin gene mutation, and homocystinuria, a cystathionine
7 beta-synthase deficiency disease, cause patients to have a similar abnormal body habitus and set of skeletal
8 deformities. This phenomenon is know n as:
9
10
A. Pleiotropy [39%]
11
12 B. Variable expressivity [6%]
13 C. Incomplete penetrance [1 %]
14
D. Genetic heterogeneity [33%]
15
16 X @ E. Genetic linkage [ 10%]
17 F. Polyploidy [9%]
18
G. Anticipation [1 %]
19
20
21 Explanation: User Id: 388147
22
23 Genetic heterogeneity exists w hen mutations of different genes cause similar phenotypes. Patients affected
24 by both Marfan syndrome and homocystinuria are prone to have a Marfanoid body habitus, characterized by a
25 tall, thin stature with elongated extremities and arachnodactyly, as w ell as ectopia lentis. The muscular
26 dystrophies are another example of genetic heterogeneity, w here X-linked defects in dystrophin or autosomal
27 recessive and autosomal dominant defects in a variety of other cytoskeletal genes all cause similar
28 phenotypes involving muscular w eakness. Be careful not to confuse genetic heterogeneity w ith allelic
29 heterogeneity, w here different mutations in the same genetic locus cause similar phenotypes, or w ith
30 phenotypic heterogeneity, w here mutations of the same gene cause different phenotypes.
31
32 (Choice A) Pleiotropy is the ability of a single genetic defect to produce multiple simultaneous phenotypic
33 effects. An example is Marfan syndrome, w here a single defect in fibrillin causes skeletal, cardiac and
34 ophthalmologic effects.
35
36 (Choice B) Expressivity is variation in phenotypic severity among individuals w ith the same genetic mutation.
37
38
39
1 A

2 Item: 41 of 46 II P Mark -<:I t>-


Q. Id : 6603 (388147] Previous Next
3
4 cxp1anat1on: user 1a: .i1111141
5
6 Genetic heterogeneity exists w hen mutations of different genes cause similar phenotypes. Patients affected
7 by both Marfan syndrome and homocystinuria are prone to have a Marfanoid body habitus, characterized by a
8 tall, thin stature w ith elongated extremities and arachnodactyly, as w ell as ectopia lentis. The muscular
9 dystrophies are another example of genetic heterogeneity, w here X-linked defects in dystrophin or autosomal
10 recessive and autosomal dominant defects in a variety of other cytoskeletal genes all cause similar
11 phenotypes involving muscular w eakness. Be careful not to confuse genetic heterogeneity w ith allelic
12 heterogeneity, w here different mutations in the same genetic locus cause similar phenotypes, or w ith
13 phenotypic heterogeneity, w here mutations of the same gene cause different phenotypes.
14
15 (Choice A) Pleiotropy is the ability of a single genetic defect to produce multiple simultaneous phenotypic
16 effects. An example is Marfan syndrome, w here a single defect in fibrillin causes skeletal, cardiac and
17 ophthalmologic effects.
18
19 (Choice B) Expressivity is variation in phenotypic severity among individuals w ith the same genetic mutation.
20
21 (Choice C) Penetrance refers to the proportion of individuals w ith a given genotype that express the
22 associated phenotype. Incomplete penetrance occurs w hen less than 100% of individuals w ith a genotype
23 express its phenotype.
24
25 (Choice E) Genetic linkage occurs w hen alleles tend to be inherited jointly, usually because they are located
26 near one another on the same strand of DNA.
27
28 (Choice F) Polyploidy occurs w hen more than two complete sets of homologous chromosomes exist w ithin
29 an organism or cell. An example is triploidy (3n).
30
31 (Choice G) Anticipation occurs in trinucleotide repeat diseases like Huntington disease. It refers to an earlier
32 onset of disease w ith successive generations due to expansion of the repeat.
33
34 Educational objective:
35 Genetic heterogeneity is w hen mutations of different genes cause similar phenotypes.
36
37 Time Spent: 128 seconds 33% answ ered correctly. Last updated: [3/27/201 O].
38 Copyright © USMLEWORLD,LLC. SimExam ver. 2011 .11.397425
1 A

2 Item: 42 of 46 II P Mark -<:I t>-


Q. Id : 6609 (388147] Previous Next
3
4
5
6 Approximately w hat percentage of energy per glucose molecule metabolized is produced in erythrocytes as
7 compared w ith CNS neurons?
8
9
10 A. 5% [35%]
11 B. 10% [1 8%]
12 X @ C. 25% [14%]
13
14 D. 50% [9%]
15 E. 100% [23%]
16
17
18 Explanation: User Id: 388147
19
20 Because erythrocytes lack mitochondria, they are unable to produce energy by oxidative phosphorylation; they
21 utilize strictly anaerobic metabolism, w hich is also observed in muscle during strenuous exercise w hen
22 oxygen supplies are low and in tissues experiencing ischemia. Most other cells in the human body possess
23 mitochondria and an adequate supply of oxygen; therefore, they are able to utilize oxidative phosphorylation, a
24 much more efficient metabolic pathw ay for the production of energy (ATP).
25
26 The initial step in glucose metabolism in all cells is glycolysis. In glycolysis, glucose is metabolized to two
27 molecules of pyruvate yielding a net energy production of two molecules of ATP and two molecules of NADH.
28 Under anaerobic conditions, such as in a mature erythrocyte that lacks mitochondria, the NADH molecules
29 are used to convert pyruvate to lactate and are therefore unavailable to produce ATP. W hen oxidative
30 phosphorylation is available, such as in a CNS neuron, pyruvate is converted to acetyl CoA, w hich produces
31 one molecule of NADH. Acetyl CoA then enters the TCA cycle in the mitochondria and is used to generate
32 three more NADH, one FADH2 and one GTP
33
Anaerob ic Aerobic
34
ATP from glycolysis 2 2
35
36 NADH from glycolysis 0 2
37 NADH from pyruvate ~ Acetyl CoA 0 2
38 NADH from TCA cycle 0 6
39
40
1 A

2 Item: 42 of 46 II P Mark -<:I t>-


Q. Id : 6609 (388147] Previous Next
3
4 Because erythrocytes lack mitochondria, they are unable to produce energy by oxidative phosphorylation; they
5 utilize strictly anaerobic metabolism, w hich is also observed in muscle during strenuous exercise w hen
6 oxygen supplies are low and in tissues experiencing ischemia. Most other cells in the human body possess
7 mitochondria and an adequate supply of oxygen; therefore, they are able to utilize oxidative phosphorylation, a
8 much more efficient metabolic pathw ay for the production of energy (ATP).
9
10 The initial step in glucose metabolism in all cells is glycolysis. In glycolysis, glucose is metabolized to two
11 molecules of pyruvate yielding a net energy production of two molecules of ATP and two molecules of NADH.
12 Under anaerobic conditions, such as in a mature erythrocyte that lacks mitochondria, the NADH molecules
13 are used to convert pyruvate to lactate and are therefore unavailable to produce ATP . W hen oxidative
14 phosphorylation is available, such as in a CNS neuron, pyruvate is converted to acetyl CoA, w hich produces
15 one molecule of NADH. Acetyl CoA then enters the TCA cycle in the mitochondria and is used to generate
16 three more NADH, one FADH2 and one GTP
17
18 Anaerobic Aerobic
19 - ATP from glycolysis 2 2
20 NADH from glycolysis 0 2
21 NADH from pyruvate ~ Acetyl CoA 0 2
22 NADH from TCA cycle 0 6
23
24
FADH2 from TCA cycle 0 2
25 GTP from TCA cycle 0 2
26 Total ATP Produced 2 38
27 KEY
28 • 3 ATP produced for every NADH
29
30
• 2ATP produced for every FADH2
31
• 1 ATP produced for every GTP
32
33 Educational objective:
34 Anaerobic glycolysis and glycolysis in erythrocytes, w hich lack mitochondria, generates 2 ATP for every
35 glucose molecule metabolized. 38 ATP are generated by aerobic metabolism of one molecule of glucose by
36 the TCA cycle.
37
38 Time Spent: 92 seconds 35% ansv1ered correctly. Last updated: [11 /10/2011].
1 A

2 Item: 43 of 46 II P Mark -<:I t>-


Q. Id : 7232 (388147] Previous Next
3
4 A 62-year-old female comes to your office complaining of severe vulvar pruritus that keeps her awake at
5 night. Her past medical history is significant for diabetes mellitus and hypertension. She smokes one pack of
6 cigarettes per day. You prescribe itraconazole.
7
8 Item 1of2
9 W hich of the follow ing describes the mechanism of action of this agent?
10
11
A. Impairs peptidoglycan synthesis [1 6%]
12
13 ~ @ B. Impairs lipid synthesis [65%]
14 C. Disrupts nucleic acid synthesis [9%]
15
D. Interferes w ith protein synthesis [10%]
16
17 E. Impairs vitamin metabolism [0%]
18
19
20 Explanation: User Id: 388147
21
ltraconazole is a triazole antifungal agent. This class of drugs functions by inhibiting 14-alpha-sterol
22
demethylase, thus impairing ergosterol synthesis and leading to the accumulation of upstream products that
23
interfere w ith membrane function. Resistance to these agents may be acquired by mutations in the
24
14-alpha-sterol demethylase enzyme or increased drug efflux pumping.
25
26
(Choice A) Peptidoglycan is an essential cell w all component in bacterial organisms, not fungal organisms.
27
Agents such as beta-lactam antibiotics and vancomycin function by impairing peptidoglycan synthesis.
28
29 (Choice C) Flucytosine is an antifungal agent that functions by disrupting nucleic acid synthesis, specifically
30 by inhibiting thymidylate synthetase.
31
32 (Choices D & E) There are no antifungal agents in common use that function by interfering w ith protein
33 synthesis or impairing vitamin metabolism. Some antibacterial drugs do function by these mechanisms.
34 Macrolides and tetracyclines, for example, impair bacterial protein synthesis, w hile
35 trimethoprim-sulfamethoxazole impairs bacterial metabolism of the vitamin folic acid .
36
37 Educational objective:
38 ltraconazole and all other triazole antifungals function by inhibiting ergosterol synthesis. Ergosterol is an
1 A

2 Item: 43 of 46 II P Mark -<:I t>-


Q. Id : 7232 (388147] Previous Next
3
4 night. Her past medical history is significant for diabetes mellitus and hypertension. She smokes one pack of
5 cigarettes per day. You prescribe itraconazole.
6
7 Item 1of2
8 W hich of the follow ing describes the mechanism of action of this agent?
9
10
A. Impairs peptidoglycan synthesis [1 6%]
11
12 ~ @ B. Impairs lipid synthesis [65%]
13 C. Disrupts nucleic acid synthesis [9%]
14
D. Interferes w ith protein synthesis [10%]
15
16 E. Impairs vitamin metabolism [0%]
17
18
19 Explanation: User Id: 388147
20
21 ltraconazole is a triazole antifungal agent. This class of drugs functions by inhibiting 14-alpha-sterol
22 demethylase, thus impairing ergosterol synthesis and leading to the accumulation of upstream products that
23 interfere w ith membrane function. Resistance to these agents may be acquired by mutations in the
24 14-alpha-sterol demethylase enzyme or increased drug efflux pumping.
25
(Choice A) Peptidoglycan is an essential cell w all component in bacterial organisms, not fungal organisms.
26
Agents such as beta-lactam antibiotics and vancomycin function by impairing peptidoglycan synthesis.
27
28 (Choice C) Flucytosine is an antifungal agent that functions by disrupting nucleic acid synthesis, specifically
29 by inhibiting thymidylate synthetase.
30
31 (Choices D & E) There are no antifungal agents in common use that function by interfering w ith protein
32 synthesis or impairing vitamin metabolism. Some antibacterial drugs do function by these mechanisms.
33 Macrolides and tetracyclines, for example, impair bacterial protein synthesis, w hile
34 trimethoprim-sulfamethoxazole impairs bacterial metabolism of the vitamin folic acid .
35
36 Educational objective:
37 ltraconazole and all other triazole antifungals function by inhibiting ergosterol synthesis. Ergosterol is an
38 essential lipid in the fungal cell membrane.
1 A

2 Item: 44 of 46 II P Mark -<:I t>-


Q. Id : 7233 (388147] Previous Next
3
4
Item 2 of 2
5 The patient returns for a follov1-up appointment. Her pruritus has not improved w ith the treatment. On
6 physical examination, w hite patches of thinned skin are seen involving the labia majora. W hich of the
7 follow ing is the most likely diagnosis?
8
9
10 A. Leukoplakia [22%]
11 B. Candidiasis [26%]
12
C. Lichen sclerosus [48%]
13
14 X @ D. Bacterial vaginosis [3%]
15 E . Protozoa! infection [1 %]
16
17
18 Explanation: User Id: 388147
19
20 The patient described is suffering from lichen sclerosus et atrophicus (LS&A), an inflammatory condition
21 believed to have an underlying autoimmune pathophysiology w ith autoantibodies directed against the ECM1
22 protein. This condition can affect both males and females and may involve any part of the body, but the
23 female genital and perineal region is most commonly affected. The condition manifests w ith pruritus, dysuria,
24 pain w ith defecation and dyspareunia. The lesions begin as w hite, atrophic macules w ith a "cigarette paper"
25 quality, w hich then coalesce into plaques as the condition progresses. It is not uncommon for the entire
26 perivaginal and perianal area to be ultimately involved in females. Development of genital squamous cell
27 carcinoma and genital disfigurement are potential long-term sequelae of untreated disease. Ultra high
28 potency topical corticosteroids are the treatment of choice.
29
30 (Choice A) Leukoplakia occurs on mucosal surfaces such as the mouth or the genitals. The labia majora are
31 not considered mucosal. Leukoplakia simply refers to w hite thickening of a mucosal surface, of w hich there
32 are many potential causes. The most classic cause is EBV infection in the setting of immunosuppression .
33
(Choice B) Vaginal candidiasis manifests w ith a w hite, curd-like, malodorous discharge. In cutaneous
34
candidiasis, the skin folds are commonly involved, show ing erythema and satellite pustules. ltraconazole
35
should have alleviated this patient's symptoms if candidiasis w as the cause.
36
37 (Choice D) Bacterial vaginosis is a cause of vaginal pruritus and a foul "fishy" odor caused by Gardnere//a
38 vAninAlis It cinP.!': not r.nmmnnlv r.;i1 l!':P. r.11tAnP.n11!': IP.!':inn!':
39
40
1 A

2 Item: 44 of 46 II P Mark -<:I t>-


Q. Id : 7233 (388147] Previous Next
3
4 X @ D. Bacterial vaginosis [3%]
5 E. Protozoa! infection [1 %]
6
7
8 Explanation: User Id: 388147
9
10 The patient described is suffering from lichen sclerosus et atrophicus (LS&A), an inflammatory condition
11 believed to have an underlying autoimmune pathophysiology w ith autoantibodies directed against the ECM1
12 protein . This condition can affect both males and females and may involve any part of the body, but the
13 female genital and perineal region is most commonly affected. The condition manifests w ith pruritus, dysuria,
14 pain w ith defecation and dyspareunia . The lesions begin as w hite, atrophic macules w ith a "cigarette paper"
15 quality, w hich then coalesce into plaques as the condition progresses. It is not uncommon for the entire
16 perivaginal and perianal area to be ultimately involved in females. Development of genital squamous cell
17 carcinoma and genital disfigurement are potential long-term sequelae of untreated disease. Ultra high
18 potency topical corticosteroids are the treatment of choice.
19
20 (Choice A) Leukoplakia occurs on mucosal surfaces such as the mouth or the genitals. The labia majora are
21 not considered mucosa!. Leukoplakia simply refers to w hite thickening of a mucosal surface, of w hich there
22 are many potential causes. The most classic cause is EBV infection in the setting of immunosuppression.
23
24 (Choice B) Vaginal candidiasis manifests w ith a w hite, curd-like, malodorous discharge. In cutaneous
25 candidiasis, the skin folds are commonly involved, show ing erythema and satellite pustules. ltraconazole
26 should have alleviated this patient's symptoms if candidiasis w as the cause.
27
28 (Choice D) Bacterial vaginosis is a cause of vaginal pruritus and a foul "fishy" odor caused by Gardnerella
29 vagina/is. It does not commonly cause cutaneous lesions.
30
(Choice E) Trichomoniasis is a protozoa! infection that involves the vagina . This infection manifests w ith a
31
frothy, foul-smelling vaginal discharge as w ell as dyspareunia and vaginal pruritus. Cutaneous lesions do not
32
occur.
33
34 Educational objective:
35 Lichen sclerosus manifests w ith vaginal pain and pruritus as w ell as dyspareunia, dysuria and possibly pain
36 w ith defecation. Cutaneous lesions are atrophic w hite macules and patches. Genital obliteration and
37 degeneration into squamous cell carcinoma are possible complications.
38
1 •
2 Item: 45 of 46 . 1, Mark -<:J C>
Q. Id: 7242 (383147) Previous NeHt
3
4 A 72-year-old male nursing home resident is brought to the emergency department with a one day history of
5 confusion and fever. Past medical history is significant for moderate Alzheimer's dementia. His temperature
6 is 38.9°C (102°F), blood pressure is 75/56 mmHg, and heart rate is 11 2/min, regular. On examination, he is
7 lethargic but arousable. Coarse rhonchi are heard over the right lung base. A Foley catheter is inserted and
8 drains 50 ml of dark unne. Initial leukocyte count is 20 OOO/ m3 and serum creatinine is.1.2 .mgtdl
9 Treatment with intravenous fluids and antibiotics is initiated. On the third day of hospitalization, he is afebrile
10 and more responsive. His leukocyte count is now 12,000/mm3 and serum creatinine is 3.6 mg/dl. Renal
11 biopsy findings are shown on the slide below.
12
13
14
15
16
17
18
19
20 l

21 •
22
23
24
25
26
27
28
29
30
31
• • •

32
33
34
35
36
37
,
38
39
.. --~
• . •_,,
40 l/Unnary System
41 •
1 •
2 Item: 45 of 46 • \' Mark -<:J C>
Q. Id: 7242 (383147) Previous NeHt
3
4
5
6
7
8
9
10
11
,.

.
,..
..v.
;

12
13
14
15
16
17
18
19 '°
20
21
22 .. ~


23
24
25
26
27
28
29
30
31
32
33
34 Item 1 of 2
-
35 W hich of the following best accounts for the obseived findings?
36
37 x @ A. Interstitial nephritis (40%]
1 A

2 Item: 45 of 46 II P Mark -<:I t>-


Q. Id : 7242 (388147] Previous Next
3
4
5
Item 1of2
W hich of the follow ing best accounts for the observed findings?
6
7
8 X @ A. Interstitial nephritis [40%]
9 ~ B. Medullary ischemia [23%]
10
11
C. Cortical necrosis [19%]
12 D. Necrotizing vasculitis [8%]
13 E. Urinary tract obstruction [8%]
14
15
16 Explanation: User Id: 388147
17
18 This patient's initial presentation of hypotension and sepsis w ith a subsequent rise in the serum creatinine
19 level is characteristic of ischemic acute tubular necrosis (ATN). lschemic ATN typically occurs in the setting
20 of prolonged hypotension, dehydration, shock, or in postsurgical patients. The classic pathologic features
21 associated w ith ischemic ATN include detachment of tubular cells from the basement membrane and
22 occlusion of tubule lumens w ith casts composed of intact or degenerating epithelial cells, proteins, and
23 pigments. Cellular necrosis in ATN is limited to the outer medullary regions because the renal medulla is
24 particularly susceptible to ischemic injury due to relatively low medullary blood flow . The morphology of the
25 glomeruli, w hich are not observed on this slide, is typically normal. Usually, the decrease in renal function is
26 greater than the histologic findings w ould suggest in patients w ith ATN.
27
28 (Choice A) Interstitial nephritis consists of interstitial inflammation and edema w ith possible tubular cell
29 damage. It is often caused by medications (e.g. NSAIDs).
30
31 (Choice C) Rarely, severe sepsis can lead to cortical necrosis, w hich is a severe condition associated w ith
32 renal intravascular coagulation . Renal biopsy w ould show glomerular necrosis and thrombosis.
33
34 (Choice D) The histologic findings of necrotizing renal vasculitis consist of thickened vessel w alls w ith
intravascular fibrin and infiltrates of neutrophils and/or lymphocytes.
35
36 (Choice E) Urinary tract obstruction causes postrenal azotemia. Benign prostatic hypertrophy is one
37
potential cause. Catheterization in such patients w ould produce a massive diuresis.
38
1 A

2 Item: 45 of 46 II P Mark -<:I t>-


Q. Id : 7242 (388147] Previous Next
3
_. . ' ........... --~·-·· ·~ ....... __ ..... ~. ... l"" , "J
4
5 E. Urinary tract obstruction [8%]
6
7
8
Explanation: User Id: 388147
9
This patient's initial presentation of hypotension and sepsis w ith a subsequent rise in the serum creatinine
10
level is characteristic of ischemic acute tubular necrosis (ATN). lschemic ATN typically occurs in the setting
11
of prolonged hypotension, dehydration, shock, or in postsurgical patients. The classic pathologic features
12
associated w ith ischemic ATN include detachment of tubular cells from the basement membrane and
13
occlusion of tubule lumens w ith casts composed of intact or degenerating epithelial cells, proteins, and
14
pigments. Cellular necrosis in ATN is limited to the outer medullary regions because the renal medulla is
15
particularly susceptible to ischemic injury due to relatively low medullary blood flow. The morphology of the
16
glomeruli, w hich are not observed on this slide, is typically normal. Usually, the decrease in renal function is
17
greater than the histologic findings w ould suggest in patients w ith ATN.
18
19 (Choice A) Interstitial nephritis consists of interstitial inflammation and edema w ith possible tubular cell
20 damage. It is often caused by medications (e.g. NSAIDs).
21
22 (Choice C) Rarely, severe sepsis can lead to cortical necrosis, w hich is a severe condition associated w ith
23 renal intravascular coagulation. Renal biopsy w ould show glomerular necrosis and thrombosis.
24
25 (Choice D) The histologic findings of necrotizing renal vasculitis consist of thickened vessel w alls w ith
26 intravascular fibrin and infiltrates of neutrophils and/or lymphocytes.
27
28 (Choice E) Urinary tract obstruction causes postrenal azotemia . Benign prostatic hypertrophy is one
29 potential cause. Catheterization in such patients w ould produce a massive diuresis.
30
31 Educational objective:
32 Acute tubular necrosis can be caused by either ischemia or toxins. lschemic ATN often occurs in the setting
33 of dehydration or hypotension and is characterized pathologically by focal necrosis of the tubular epithelium in
34 the renal medulla w ith detachment of cells from the basement membrane and occlusion of tubule lumens w ith
35 casts.
36
37 Time Spent: 234 seconds 23% answ ered correctly. Last updated: [1 /8/2012].
38 Copyright © USMLEWORLD,LLC. SimExam ver. 2011.11 .397425
1 A

2 Item: 46 of 46 II P Mark -<:I t>-


Q. Id : 7243 (388147] Previous Next
3
4
5
6 Item 2 of 2
7 The patient recovers successfully from his illness. On the seventh day of hospitalization, he has no
8 complaints. His serum creatinine level is 1.1 mg/dl. W hich of the follow ing conditions show s a similar renal
9 pathology as that observed in this patient?
10
11
x @ A. Systemic lupus erythematosus [9%]
12
13 B. Minimal change disease [15%]
14 C. Aminoglycoside toxicity [62%]
15
16
D. Diabetes mellitus [7%]
17 E. Amyloidosis [7%]
18
19
20 Explanation: User Id: 388147
21
22 This patient's renal failure w as caused by ischemic acute tubular necrosis (ATN). In addition to ischemia,
23 ATN can also be caused by renal toxicity from certain nephrotoxins, w hich can either be extrinsic (from
24 outside the body) or intrinsic (from w ithin the body}. Extrinsic toxins include aminoglycosides, amphotericin B,
25 radiographic contrast dye, heavy metals, and others. Intrinsic toxins include m yoglobinuria, hyperuricemia,
26 and the presence of Bence-Jones proteins in multiple m yeloma .
27
(Choice A) Systemic lupus erythematosus causes lupus nephritis. This condition first manifests in
28
asymptomatic patients w ith microscopic hematuria and proteinuria, and can progress to diffuse proliferative
29
glomerulonephritis and end-stage renal disease.
30
31
(Choice B) Minimal change disease is a cause of nephrotic syndrome that most commonly occurs in
32 children. It is either self-limited or resolves w ith systemic corticosteroid therapy in most cases.
33
34 (Choice D) Diabetic nephropathy results in nodular glomerulosclerosis, w hich is caused by the deposition of
35 immunoglobulin chains and other proteins in the renal glomerular arterioles . Nephrotic syndrome is the typical
36 clinical presentation.
37
38 (Choice E) Amyloidosis results in the deposition of amyloid protein in the kidney. The type of protein
39
40
1 A

2 Item: 46 of 46 II P Mark -<:I t>-


Q. Id : 7243 (388147] Previous Next
3
4 C. Aminoglycoside toxicity [62%]
5
D. Diabetes mellitus [7%]
6
7 E. Amyloidosis [7%]
8
9
Explanation: User Id: 388147
10
11
This patient's renal failure w as caused by ischemic acute tubular necrosis (ATN). In addition to ischemia,
12
ATN can also be caused by renal toxicity from certain nephrotoxins, w hich can either be extrinsic (from
13
outside the body) or intrinsic (from w ithin the body). Extrinsic toxins include aminoglycosides, amphotericin B,
14
radiographic contrast dye, heavy metals, and others. Intrinsic toxins include myoglobinuria, hyperuricemia,
15
and the presence of Bence-Jones proteins in multiple myeloma .
16
17 (Choice A) Systemic lupus erythematosus causes lupus nephritis. This condition first manifests in
18 asymptomatic patients w ith microscopic hematuria and proteinuria, and can progress to diffuse proliferative
19 glomerulonephritis and end-stage renal disease.
20
21 (Choice B) Minimal change disease is a cause of nephrotic syndrome that most commonly occurs in
22 children. It is either self-limited or resolves w ith systemic corticosteroid therapy in most cases.
23
24 (Choice D) Diabetic nephropathy results in nodular glomerulosclerosis, w hich is caused by the deposition of
25 immunoglobulin chains and other proteins in the renal glomerular arterioles. Nephrotic syndrome is the typical
26 clinical presentation.
27
28 (Choice E) Amyloidosis results in the deposition of amyloid protein in the kidney. The type of protein
29 deposited is dependent on the cause of amyloidosis. Affected patients typically have either asymptomatic
30 proteinuria or nephrotic syndrome.
31
32 Educational objective:
33 Acute tubular necrosis can be caused by ischemia or nephrotoxins. Potential nephrotoxins responsible for
34 ATN include aminoglycosides, amphotericin B, radiocontrast dye, heavy metals, myoglobin, and Bence-Jones
35 proteins, among others.
36
37 Time Spent: 44 seconds 62% answ ered correctly. Last updated: [1 /8/2012].
38 Copyright © USMLEWORLD,LLC. SimExam ver. 2011 .11.397425
2 Item: 1 of 46 II P Mark -<:I t>-
Q. Id : 6594 (388147) Previous Next
3
4 A 5-year-old boy is brought to your office w ith facial edema and smoky-colored urine. His medical history is
5 significant for a mild skin infection on his face three w eeks ago. Urinalysis today is positive for protein . The
6 bacteria responsible for this patient's presentation have w hich of the follow ing characteristics?
7
8
9 x @ A. Catalase-positive [10%]
10 ., B. Bacitracin-sensitive [70%]
11 C. Coagulase-positive [11 %]
12
13 D. Non-hemolytic [2%]
14 E. Quellung reaction-positive [8%]
15
16
17 Explanation: User Id: 388147
18
19 - The most likely diagnosis here is acute post-streptococcal glomerulonephritis (APSGN), an immune
20 complex-mediated disease that occurs follow ing infection by lysogenized nephritogenic strains of
21 Streptococcus pyogenes (group A streptococcus, GAS). Symptoms typically include periorbital
22 (nondependent) edema and urine that is dark in color due to hematuria. Classically, the GAS infection that
23 precedes ASPGN w ill be either pharyngitis ("strep throat") or a cutaneous infection such as an abscess,
24 furuncle or impetigo. GAS is a catalase-negative, beta-hemolytic, Gram positive coccus sensitive to
25 bacitracin.
26
27 (Choice A) Staphylococci are catalase-positive, Gram positive cocci. S. aureus can cause multiple types of
28 infection, including cutaneous infections, septic arthritis, osteomyelitis, and w ound infections.
29
30 (Choice C) S. aureus is the only coagulase-positive, Gram positive coccus.
31
32 (Choice D) Non-hemolytic organisms are referred to as gamma-hemolytic. Some group D streptococci, such
33 as the enterococci, are gamma-hemolytic. These organisms may cause subacute bacterial endocarditis.
34
35 (Choice E) W hen Streptococcus pneumoniae is exposed to antibodies against its capsular antigens, sw elling
36 of the bacterial capsule is evident under the microscope. This sw elling defines a positive Quellung reaction.
37
38 Educational objective:
- . .
39
40
2 Item: 1 of 46 II P Mark -<:I t>-
Q. Id : 6594 (388147) Previous Next
3
4 v B. Bacitracin-sensitive [70%]
5 C. Coagulase-positive [11 %]
6
7 D. Non-hemolytic [2%]
8 E. Quellung reaction-positive [8%]
9
10
11 Explanation: User Id: 388147
12
13 The most likely diagnosis here is acute post-streptococcal glomerulonephritis (APSGN), an immune
14 complex-mediated disease that occurs follow ing infection by lysogenized nephritogenic strains of
15 Streptococcus pyogenes (group A streptococcus, GAS). Symptoms typically include periorbital
16 (nondependent) edema and urine that is dark in color due to hematuria. Classically, the GAS infection that
17 precedes ASPGN w ill be either pharyngitis ("strep throat") or a cutaneous infection such as an abscess,
18 furuncle or impetigo. GAS is a catalase-negative, beta-hemolytic, Gram positive coccus sensitive to
19 - bacitracin.
20
21 (Choice A) Staphylococci are catalase-positive, Gram positive cocci. S. aureus can cause multiple types of
22 infection, including cutaneous infections, septic arthritis, osteomyelitis, and w ound infections.
23
24 (Choice C) S. aureus is the only coagulase-positive, Gram positive coccus.
25
26 (Choice D) Non-hemolytic organisms are referred to as gamma-hemolytic. Some group D streptococci, such
27 as the enterococci, are gamma-hemolytic. These organisms may cause subacute bacterial endocarditis.
28
29 (Choice E) When Streptococcus pneumoniae is exposed to antibodies against its capsular antigens, sw elling
30 of the bacterial capsule is evident under the microscope. This sw elling defines a positive Quellung reaction.
31
32 Educational objective:
33 Group A streptococcus is a catalase-negative, beta-hemolytic, Gram positive coccus sensitive to bacitracin.
34 Immune complex-mediated diseases like rheumatic fever and acute post-streptococcal glomerulonephritis
35 may occur follow ing GAS infections.
36
37 Time Spent: 29 seconds 70% answ ered correctly. Last updated: [3/27/201 O].
38 Copyright © USMLEWORLD,LLC. SimExam ver. 2011.11.397425
Item: 2 of 46 II P Mark -<:I t>-
Q. Id : 6431 [388147) Previous Next
3
4
5
6 A 34-year-old Caucasian male treated two w eeks ago w ith antibiotics for urinary symptoms now returns to
7 clinic complaining of back pain and difficulty w alking. Physical examination reveals a swollen and tender right
8 knee and left-sided eye redness. W hich of the follow ing is the most likely diagnosis in this patient?
9
10
11 A. Hypersensitivity vasculitis [3%]
12 B. Disseminated gonococcal infection [29%]
13 ~ @ C. Reactive arthritis [61 %]
14
15 D. Ankylosing spondylitis [6%]
16 E . Rheumatoid arthritis [0%]
17 F. Viral arthritis [0%]
18
19 -
20 Explanation: User Id: 388147
21
22 Classically, reactive arthritis \ 'Reiter syndrome") occurs in males in their twenties and thirties. This condition
23 is characterized by the triad of conjunctivitis, urethritis, and seronegative arthritis of the appendicular skeleton
24 (knees, ankles, and feet in particular), an association remembered w ith the mnemonic of "can't see, can't pee,
25 can't climb a tree." Genitourinary infection or diarrhea caused by Chlamydia, Campylobacter, Salmonella,
26 Shigella, or Yersinia precedes the onset of rheumatic symptoms by two to six w eeks. In addition, more than
27 80% of affected individuals are positive for HLA-827.
28
29 (Choice A) Hypersensitivity syndrome is a severe reaction (typically to aromatic antiepileptic agents or
30 sulfonamide antibiotics) that results in rash, fever, hepatitis, arthralgias, lymphadenopathy, or hematologic
31 dysfunction approximately two to six w eeks after the drug is first used.
32
33 (Choice B) Disseminated gonococcal infection presents either as purulent arthritis (typically of the knee,
34 w rist, or ankle) or as the triad of tenosynovitis, dermatitis, and non-purulent polyarthralgias. Eye symptoms
35 are not seen.
36
37 (Choice D) Ankylosing spondylitis is a chronic, inflammatory disease of multiple articular and para-articular
38 structures in the axial skeleton. This form of seronegative arthritis presents w ith progressive spinal stiffness
Item: 2 of 46 II P Mark -<:I t>-
Q. Id : 6431 [388147) Previous Next
3
4 i-. v 1ra1 a11nrms l U"loJ
5
6
Explanation: User Id: 388147
7
8
Classically, reactive arthritis ("Reiter syndrome") occurs in males in their tw enties and thirties. This condition
9
is characterized by the triad of conjunctivitis, urethritis, and seronegative arthritis of the appendicular skeleton
10
(knees, ankles, and feet in particular), an association remembered w ith the mnemonic of "can't see, can't pee,
11
can't climb a tree." Genitourinary infection or diarrhea caused by Chlamy dia, Campylobacter, Salmonella,
12
Shigella, or Yersinia precedes the onset of rheumatic symptoms by tw o to six w eeks. In addition, more than
13
80% of affected individuals are positive for HLA-827.
14
15 (Choice A) Hypersensitivity syndrome is a severe reaction (typically to aromatic antiepileptic agents or
16 sulfonamide antibiotics) that results in rash, fever, hepatitis, arthralgias, lymphadenopathy, or hematologic
17 dysfunction approximately two to six w eeks after the drug is first used.
18
19 - (Choice B) Disseminated gonococcal infection presents either as purulent arthritis (typically of the knee,
20 w rist, or ankle) or as the triad of tenosynovitis, dermatitis, and non-purulent polyarthralgias. Eye symptoms
21 are not seen .
22
23 (Choice D) Ankylosing spondylitis is a chronic, inflammatory disease of multiple articular and para-articular
24 structures in the axial skeleton. This form of seronegative arthritis presents w ith progressive spinal stiffness
25 and back pain.
26
27 (Choice E) Rheumatoid arthritis is a chronic, inflammatory disease of multiple symmetric joints (typically
28 including the w rist, metacarpophalangeal, or proximal interphalangeal joints). Morning stiffness is common.
29
30 (Choice F) Viral arthritis is a symmetric, polyarticular disease associated w ith parvovirus, hepatitis B virus,
31 hepatitis C virus, rubella virus, and alphavirus infection.
32
33 Educational objective:
34 Reactive arthritis ("Reiter syndrome") is characterized by the triad of conjunctivitis, urethritis, and seronegative
35 arthritis of the appendicular skeleton.
36
37 Time Spent: 52 seconds 61 % answ ered correctly. Last updated: [1 /8/2012].
38 Copyright © USMLEWORLD,LLC. SimExam ver. 2011.11.397425
Item: 3 of 46 II P Mark -<:I t>-
Q. Id : 6604 (388147) Previous Next

4
5
6 A 3-year-old Caucasian male is being evaluated in the genetics clinic. His mother says that he seems not to
7 like active play, show s constant food-seeking behavior and has frequent temJJcer tantrums. Physical
8 examination demonstrates marked obesi!Y and small hands and feet. Chromosomal analysis show s a ~,XY
9 genotype w ith no abnormalities detected by fluorescent in situ hybridization EISH) W hich of the following
10 chromosomal abnormalities underlies his condition?
11
12
A. A deletion too small to be detected by FISH [11 %]
13
14 B. A reciprocal 14;15 translocation [3%]
15 C. Paracentric inversion involving chromosome 15 [7%]
16
D. Both chromosomes 15 are from the father [45%]
17
18 .; @ E. Both chromosomes 15 are from the mother [34%]
19 -
20
21 Explanation: User Id: 388147
22
This child has Prader-W illi syndrome (PWS), a condition characterized clinically by neonatal hypotonia,
23
extreme hyperphagia leading to morbid obesity and ultimately type II diabetes mellitus, short stature, small
24
25 hands and feet, hypogonadism and characteristic facies. Genetically, PWS is a disorder of genomic
imprinting resulting from a microdeletion on the paternal chromosome q11 region or from maternal uniparental
26
disomy 15, w here both chromosomes 15 are inherited from the mother and none from the father. In either of
27
these scenarios, information from the paternal chromosome 15 is lost. Though a paternal deletion is the most
28
common cause of PWS, it would be detected by FISH. The normal FISH described here indicates that the
29
cause of disease in this patient is maternal disomy 15.
30
31
(Choice A) FISH is a technique used to detect the presence or absence of specific DNA sequences on w hole
32
chromosomes. It can detect sequences as small as 100 kilobases and is commonly used to detect PWS
33
34 resulting from microdeletions in the paternal chromosome 15q11 region.
35
(Choice B) Reciprocal translocations involve the exchange of genetic material between two non-homologous
36
chromosomes. t(14;15) is not known to cause PWS.
37
38
Item: 3 of 46 II P Mark -<:I t>-
Q. Id : 6604 (388147) Previous Next

4
5
6 Explanation: User Id: 388147
7
8 This child has Prader-Willi syndrome (PWS), a condition characterized clinically by neonatal hypotonia,
9 extreme hyperphagia leading to morbid obesity and ultimately type II diabetes mellitus, short stature, small
10 hands and feet, hypogonadism and characteristic facies. Genetically, PWS is a disorder of genomic
11 imprinting resulting from a microdeletion on the paternal chromosome q11 region or from maternal uniparental
12 disomy 15, w here both chromosomes 15 are inherited from the mother and none from the father. In either of
13 these scenarios, information from the paternal chromosome 15 is lost. Though a paternal deletion is the most
14 common cause of PWS, it w ould be detected by FISH. The normal FISH described here indicates that the
15 cause of disease in this patient is maternal disomy 15.
16
17 (Choice A) FISH is a technique used to detect the presence or absence of specific DNA sequences on w hole
18 chromosomes. It can detect sequences as small as 100 kilobases and is commonly used to detect PWS
19 - resulting from microdeletions in the paternal chromosome 15q11 region .
20
21 (Choice B) Reciprocal translocations involve the exchange of genetic material between two non-homologous
22 chromosomes. t(14;15) is not know n to cause PWS.
23
24 (Choice C) Paracentric inversions are inversions of chromosomal DNA in one arm of the chromosome not
25 including the centromere. Paracentric inversions in chromosome 15 have been reported but are a very rare
26 cause of human disease.
27
28 (Choice D) Paternal uniparental disomy 15 causes Angelman syndrome, a disease characterized clinically by
29 mental retardation, epilepsy, tremors, poor motor and language development, and frequent laughter and
30 smiling.
31
32 Educational objective:
33 The key clinical features of Prader-Willi syndrome are neonatal hypotonia, extreme hyperphagia and morbid
34 obesity, short stature, small hands and feet, and characteristic facies. Prader-Willi results from maternal
35 uniparental disomy 15 or from a microdeletion in the 15q 11 region of the paternal chromosome 15.
36
37 Time Spent: 105 seconds 34% answ ered correctly. Last updated: [3/27/201 O].
38 Copyright © USMLEWORLD,LLC. SimExam ver. 2011 .11.397425
1 •
2 Item: 4 of 46 • \' Mark -<:J C>
Q. Id: 6437 (383141) Previous NeHt
3

5
6 A 7-year-old Caucasian boy suffers from an itchy rash involving the extensor surface of his arms (see the
7 image below).
8
9
10
11
12
13
14
15
16
17
18
19 '°
20
21
22
23
24
25
26
27 His mother has a history of prolonged diarrhea re uiring dietary correction . Intestinal biopsy is most likely to
28 reveal which of the following changes in this patient?
29
30
31 X @ A. Cobblestone mucosa and transn1ural inflammation [6%]
32 >I B. Atrophy of the intestinal villi [80%]
33
C. Mucosal erythema w ith neutrophil infiltration [6%]
34
35 D. Macrophages with accumulated PAS-positive granules [7%]
36 E. Multiple adenomatous polyps [1 %]
37
38
1 A

2 Item: 4 of 46 II P Mark -<:I t>-


Q. Id : 6437 (388147) Previous Next
3

Explanation: User Id: 388147


5
6
Celiac disease (also termed celiac sprue or gluten-sensitive enteropathy} is a chronic condition caused by
7
dietary exposure to w heat gliadins and related grains in susceptible individuals. This sensitivity to gluten can
8
trigger autoimmune reactions in the gastrointestinal tract and skin .
9
10 In the small bow el, celiac disease presents as marked villous atrophy in a background of diffuse
11 enteritis. CDS+ T-cell lymphocytes infiltrate the small intestine surface epithelium, w hile a w ide range of
12
inflammatory cells (CD4+ T-cell lymphocytes, plasma cells, macrophages, eosinophils, and mast cells) take
13 up residence in the lamina propria .
14
15 In the skin, celiac disease presents as a condition called "dermatitis herpetiformis." Groups of erythematous,
16 urticarial plaques develop on the extensor surfaces of the elbow s, knees, buttocks, and back. These
17 extremely pruritic lesions are caused by the cross-reaction of gliadin lgG and lgA antibodies w ith reticulin, a
18 component of the fibrils that anchor the epidermal basement membrane to the superficial dermis. Neutrophil
19 recruitment results in the creation of small microabscesses at the dermal papillary tips. Eventually, the
20 microabscesses coalesce to form subepidermal blisters.
21
22 (Choices A, C, and D) Cobblestone mucosa and transmural inflammation are characteristic of Crohn's
23 disease. Mucosal erythema and neutrophil infiltration of the small bow el are nonspecific findings associated
24 w ith a multitude of conditions, including infectious colitis and inflammatory bow el disease. Macrophages w ith
25 accumulated PAS-positive granules are consistent w ith Whipple's disease. Although all of these conditions
26 can present w ith diarrhea, they lack strong genetic components and are not associated w ith urticarial rash.
27
28 (Choice E) Multiple adenomatous polyps are often discovered on routine colonoscopy in otherwise healthy
29 patients. If numerous, adenomatous polyps can signify conditions such as familial adenomatous polyposis
30 (FAP), w hich has a strong genetic component but no association w ith urticarial rash.
31
32 Educational Objective:
33 In the small bowel, celiac disease presents as marked villous atrophy in a background of diffuse enteritis. In
34 the skin, celiac disease presents as "dermatitis herpetiformis," a condition in w hich small microabscesses at
35 the dermal papillary tips eventually coalesce to form subepidermal blisters.
36
37 Time Spent: 34 seconds 80% answ ered correctly. Last updated: [1 /22/2012].
38 Copyright © USMLEWORLD,LLC. SimExam ver. 2011 . 11.397425
1 •
2 Item: 5 of 46 • \' Mark -<:J C>
Q. Id: 6447 (383141) Previous NeHt
3
4
Autopsy findings in a 42-year-old Caucasian female are shown on the image below. She suffered from
6 exertional dyspnea and e)(~rienced sudden death

8
7

9
'
10
11
12
13
14
, '
15
16
17
18
19 '°
20
21
22
23
24
25
26
27
28
29
30
31
32
\
33
34
l"
35
36
37
38
1 A

2 Item: 5 of 46 II P Mark -<:I t>-


Q. Id : 6447 (388147) Previous Next
3
4
W hich of the follow ing best describes the changes in cardiac muscle cells responsible for the autopsy
findings?
6
7
8 A. High rate of mitotic activity [12%]
9 B. High rate of m yosin n1RNA synthesis [69%]
10
C. liflefblssin granblle assblmbllatien [11 %•]
11
12 X @ D. High rate of collagen mRNA synthesis [7%]
13 E. CarElias sell aJl9Jllesis [1 %•]
14
15
16 Explanation: User Id: 388147
17
18 The w all of the right ventricle (RV) is at least 2 cm in thickness, and is markedly thicker than that of the left
19 ventricle. In a normal heart, the RV thickness is 3-4 mm during diastole, and is significantly thinner than the
20 LV w all during diastole (around 1 cm). Thus, this specimen demonstrates right ventricular hypertrophy (RVH).
21
22 Cardiomyocytes are terminally differentiated cells that can no longer divide. Consequently, chronic
23 intensification of the heart's mechanical load results in increased dimension and mass of the individual
24 cardiomyocyte (ie, hypertrophy). The hypertrophic state is established through increases in the rate of protein
25 synthesis, the quantity of protein, and the quantity of sarcomeres and mitochondria w ithin each
26 cardiomyocyte. Gene expression is altered, w ith the genes responsible for fetal cardiac development
27 upregulated. As the heart makes larger quantities of numerous growth factors, m yosin mRNA is synthesized
28 at a faster rate.
29
(Choice A) A high rate of mitotic activity is seen w hen cell numbers are increasing (ie, during hyperplasia).
30
The heart undergoes hypertrophy, not hyperplasia, w hen responding to increased hemodynamic loads.
31
32 (Choice C) Lipofuscin granule accumulation is a benign finding associated w ith increasing cell age. It is not a
33
reflection of hyperplasia or hypertrophy.
34
35 (Choice D) A high rate of collagen mRNA synthesis and type I collagen deposition appears to be associated
36 w ith m yocardial fibrosis. The heart show n in the image above does not demonstrate significant fibrosis.
37
38 (Choice E) Cardiac cell apoptosis is associated w ith atrophy of the muscle tissue, w hich w ould grossly
39
40
1 A

2 Item: 5 of 46 II P Mark -<:I t>-


Q. Id : 6447 (388147) Previous Next
3
4
x @ D. High rate of collagen mRNA synthesis [7%]

6 E. CarElias sell apeptesis [1 %•]


7
8
9 Explanation: User Id: 388147
10
11 The wall of the right ventricle (RV) is at least 2 cm in thickness, and is markedly thicker than that of the left
12 ventricle. In a normal heart, the RV thickness is 3-4 mm during diastole, and is significantly thinner than the
13 LVwall during diastole (around 1 cm). Thus, this specimen demonstrates right ventricular hypertrophy (RVH).
14
Cardiomyocytes are terminally differentiated cells that can no longer divide. Consequently, chronic
15
intensification of the heart's mechanical load results in increased dimension and mass of the individual
16
cardiomyocyte (ie, hypertrophy). The hypertrophic state is established through increases in the rate of protein
17
synthesis, the quantity of protein, and the quantity of sarcomeres and mitochondria within each
18
cardiomyocyte. Gene expression is altered, with the genes responsible for fetal cardiac development
19
upregulated. As the heart makes larger quantities of numerous growth factors, myosin mRNA is synthesized
20
at a faster rate.
21
22 (Choice A) A high rate of mitotic activity is seen when cell numbers are increasing (ie, during hyperplasia).
23 The heart undergoes hypertrophy, not hyperplasia, when responding to increased hemodynamic loads.
24
25 (Choice C) Lipofuscin granule accumulation is a benign finding associated with increasing cell age. It is not a
26 reflection of hyperplasia or hypertrophy.
27
28 (Choice D) A high rate of collagen mRNA synthesis and type I collagen deposition appears to be associated
29 with myocardial fibrosis. The heart shown in the image above does not demonstrate significant fibrosis.
30
31 (Choice E) Cardiac cell apoptosis is associated with atrophy of the muscle tissue, which would grossly
32 manifest as a small heart.
33
34 Educational Objective:
35 Cardiac hypertrophy is characterized by an increased rate of myosin mRNA synthesis.
36
37 Time Spent: 48 seconds 69% answered correctly. Last updated: [11 /10/2011].
38 Copyright © USMLEWORLD,LLC. SimEx am ver. 2011 . 11.397425
1 •
2 Item: 6 of 46 • \ ' Mark -<:J C>
Q. Id: 6451 (383141) Previous NeHt
3
4
5
A 9-year-old male is brought to clinic with a skin lesion on his right big toe. He started wearing new shoes two
7 weeks ago. Physical examination reveals a callus Which of the following skin layers is most likely involved in
8 this pat1e.o!]
9
10
11
12
13
14
15
16 A ::::::::==::::::=====~~==:::::=:::::---~
17 B
18
19 c
20
21 D
22
23
24
25
26
27 A. A [50%]
28 B. B [28%]
29
30 C. C (11%]
31 X @ D. D [9%]
32 E. E [3%]
33
34
35 Explanation: User Id: 3881 47
36
37 The skin is composed of a superticial stratified squamous keratinized epithelium (commonly called the
38 epidermis) and a deeper connective tissue layer (commonly called the dermis). The epidermis and dermis
39
40
1 A

2 Item: 6 of 46 II P Mark -<:I t>-


Q. Id : 6451 [388147) Previous Next
3
4
5

7
8
9
A. A [50%]
10 B. B [28%]
11 C. C [11 %]
12
X @ D. D [9%]
13
14 E. E [3%]
15
16
17 Explanation: User Id: 388147
18
19 The skin is composed of a superficial stratified squamous keratinized epithelium (commonly called the
20 epidermis) and a deeper connective tissue layer (commonly called the dermis). The epidermis and dermis
21 interlock along a basement membrane boundary that separates the epidermal ridges from the dermal
22 papillae. The five layers of the epidermis, from the most superficial layer to the deepest layer, include the
23 stratum corneum, stratum lucidum, stratum granulosum, stratum spinosum, and stratum basale.
24
25 The stratum corneum is marked by "A" in the diagram above, and is typically composed of fifteen to twenty
26 layers of dead squamous cells that lack nuclei. As these cells are shed, they are replaced by the mitotic
27 activity of the stratum basale and stratum spinosum . The stratum corneum is generally thickest in areas of
28 the body commonly exposed to friction or trauma (eg, soles of the feet). A painless additional thickening of the
29 stratum corneum ("callus") may occur at locations of repeated external pressure or friction.
30
31 (Choices B, C, D, and E) The deeper layers of the epidermis ("B"), the basement membrane ("C"), the
32 dermis ("D"), and the hypodermis ("E") do not thicken to form a callus in response to friction or trauma .
33
34 Educational objective:
35 A callus is caused by a thickening of the most superficial skin layer, the stratum corneum .
36
37 Time Spent: 33 seconds 50% answ ered correctly. Last updated: [11 / 10/2011].
38 Copyright © USMLEWORLD,LLC. SimEx am ver. 2011 . 11.397425
1 •
2 Item: 7 of 46 • \ ' Mark -<:J C>
Q. Id: 6452 (383141) Previous NeHt
3
4
5
6 A 25-year-old mildly oveiweight female dies in a motor vehicle accident. Her ovaries are shown on the image
below.
8
9
10
11
12
13
14
15
16
17
..
18
19
...
20
21
22
23
24 Dl•pi.yed with ~aslon from Curront M- Group LLC
25
26
This patient's condition is most likely associated with decreased levels of which of the following?
27
28
29 X @ A. LH [27%]
30 ~ B. FSH [56%]
31
C. Prolactin [4o/o]
32
33 D. Androstenedione [7%]
34 E. Testosterone [2%]
35
F. Thyroid stimulating hormone [3%]
36
37
38
1 A

2 Item: 7 of 46 II P Mark -<:I t>-


Q. Id : 6452 (388147) Previous Next
3
4
5 X @ A. LH [27%]
6
~ B. FSH [56%]
8 C. Prolactin [4%]
9 D. Androstenedione [7%]
10
E. Testosterone [2%]
11
12 F. Thyroid stimulating hormone [3%]
13
14
15 Explanation: User Id: 388147
16
17 Polycystic ovary syndrome (PCOS) is an endocrine disorder associated w ith the abnormal production and
18 metabolism of androgens and estrogen in women. Characteristic features of the disease include
19 amenorrhea, obesity, hirsutism, virilization, peripheral insulin resistance, and dyslipidemia . Physical
20 examination of the affected ovaries demonstrates bilateral enlargement and smooth, thickened capsules. Cut
21 sections reveal subcapsular follicles in different stages of atresia . Hyperplastic theca stromal cells rim the
22 arrested follicles.
23
24 The anovulation and increased androgen levels seen in PCOS are thought to occur w hen ovarian theca cells
25 experience increased stimulation from high levels of circulating luteinizing hormone (LH). The ovarian theca
26 cells then produce larger quantities of androgens such as testosterone and androstenedione. Because
27 follicle-stimulating hormone (FSH) is decreased in proportion to LH, the ovarian granulosa cells are unable to
28 aromatize the androgens to estrogens. Anovulation results from the decreased estrogen levels.
29
30 (Choices A, D, and E) High levels of LH, androstenedione, and testosterone are characteristic of PCOS.
31
32 (Choice C and F) Prolactin and TSH levels are normal in PCOS.
33
34 Educational objective:
35 Low or low normal levels of FSH and a high LH/FSH ratio are typical findings of polycystic ovary syndrome.
36
37 Time Spent: 81 seconds 56% answ ered correctly. Last updated: [11 / 10/2011].
38 Copyright © USMLEWORLD,LLC. SimExam ver. 2011 .11.397425
1 A

2 Item: 8 of 46 II P Mark -<:I t>-


Q. Id : 6611 [388147) Previous Next
3
4
5
6 A 3-year-old Caucasian male suffers from mental retardation and involuntary movements. He fiercely bites
7 his lips and fingertips. Urinary uric acid secretion is high. W hich of the following enzymes is most likely
deficient in this patient?
9
10
11
A. Adenosine deaminase [3%]
12 B. Adenine phosphoribosyltransferase [2%]
13 ~ @ C. Hypoxanthine-guanine phosphoribosyltransferase [87%]
14
15
D. Purine nucleoside phosphorylase [3%]
16 E. Xanthine oxidase [5%]
17
18
19 Explanation: User Id: 388147
20
21 The clinical features described in the vignette are characteristic of Lesch-Nyhan syndrome, w hich is an X-
22 linked recessive defect in hypoxanthine-guanine phosphoribosyltransferase (HGPRT). This enzyme
23 participates in the purine salvage pathway by catalyzing the conversion of hypoxanthine and guanine to
24 inosine monophosphate and guanosine monophosphate, respectively. Deficiency of this enzyme causes
25 decreased purine salvage, increased purine synthesis, hyperuricemia and hyperuricuria . Characteristic
26 clinical features include self-mutilating behavior (classically finger and lip biting), mild mental retardation,
27 involuntary movements, delayed motor development, nephrolithiasis and gout. Parents may also report a
28 "frost" on the skin under their affected child's diaper due to hyperuricuria . Treatment w ith allopurinol
29 decreases uric acid levels and therefore relieves symptoms related to gout and urolithiasis, but this treatment
30 has little effect on neurological symptoms.
31
32 (Choices A & D) Deficiency of adenosine deaminase (ADA) causes autosomal recessive severe combined
33 immunodeficiency (SCID). ADA participates in purine catabolism. In the absence of ADA, high
34 concentrations of dATP accumulate resulting in inhibition of ribonucleotide reductase and therefore inhibition
35 of DNA synthesis and cell death. Purine nucleoside phosphorylase (PNP) is another enzyme in the purine
36 catabolism pathway; deficiency of this enzyme leads to an isolated T-lymphocyte defect; B-lymphocyte
37 function is intact.
38
1 A

2 Item: 8 of 46 II P Mark -<:I t>-


Q. Id : 6611 [388147) Previous Next
3
4
5 The clinical features described in the vignette are characteristic of Lesch-Nyhan syndrome, w hich is an X-
6 linked recessive defect in hypoxanthine-guanine phosphoribosyltransferase (HGPRT). This enzyme
7 participates in the purine salvage pathw ay by catalyzing the conversion of hypoxanthine and guanine to
inosine monophosphate and guanosine monophosphate, respectively. Deficiency of this enzyme causes
9 decreased purine salvage, increased purine synthesis, hyperuricemia and hyperuricuria. Characteristic
10 clinical features include self-mutilating behavior (classically finger and lip biting), mild mental retardation,
11 involuntary movements, delayed motor development, nephrolithiasis and gout. Parents may also report a
12 "frost" on the skin under their affected child's diaper due to hyperuricuria . Treatment w ith allopurinol
13 decreases uric acid levels and therefore relieves symptoms related to gout and urolithiasis, but this treatment
14 has little effect on neurological symptoms.
15
16 (Choices A & D) Deficiency of adenosine deaminase (ADA) causes autosomal recessive severe combined
17 immunodeficiency (SCID). ADA participates in purine catabolism. In the absence of ADA, high
18 concentrations of dATP accumulate resulting in inhibition of ribonucleotide reductase and therefore inhibition
19 of DNA synthesis and cell death. Purine nucleoside phosphorylase (PNP) is another enzyme in the purine
20 catabolism pathw ay; deficiency of this enzyme leads to an isolated T-lymphocyte defect; B-lymphocyte
21 function is intact.
22
23 (Choice B) Adenine phosphoribosyltransferase (APRT) is another enzyme involved in purine salvage; it is
24 responsible for the conversion of adenine to AMP. APRT deficiency results in hyperuricemia and adenine
25 stone u~olithiasis. APRT deficiency is transmitted in an autosomal recessive fashion and does not cause
26 neurological symptoms.
27
28 (Choice E) Xanthine oxidase converts hypoxanthine to xanthine and then xanthine to uric acid. Xanthine
29 oxidase deficiency results in hypouricemia, increased xanthine levels and formation of urinary xanthine stones.
30
31 Educational objective:
32 Lesch-Nyhan syndrome is an X-linked disorder caused by deficiency of hypoxanthine-guanine
33 phosphoribosyltransferase (HGPRT) resulting in hyperuricemia and hyperuricuria, w hich themselves cause
34 nephrolithiasis and gout as w ell as self-mutilating behavior (classically finger and lip biting), mild mental
35 retardation, involuntary movements and delayed motor development.
36
37 Time Spent: 29 seconds 87% answ ered correctly. Last updated: [3/27/201 O].
38 Copyright © USMLEWORLD,LLC. SimExam ver. 2011 .11.397425
1 A

2 Item: 9 of 46 II P Mark -<:I t>-


Q. Id : 6459 (388147) Previous Next
3
4
5
6 A new substance synthesized in the laboratory irreversibly binds to the 30S subunit of prokaryotic ribosomes.
7 As a result, incoming aminoacyl-tRNA access to mRNA is blocked and protein synthesis is inhibited. The
8 action of the new substance is most similar to w hich of the following?

10
x @ A. Erythromycin [1 6%]
11
12 B. Rifampin [4% ]
13 C. Doxycycline [64%]
14
D. Chloramphenicol [7%]
15
16 E. Puromycin [6%]
17 F. Diphtheria toxin [3%]
18
19
20 Explanation: User Id: 388147
21
22 Substances that inhibit protein synthesis by prokaryotic ribosomes are antibacterial antibiotics. Bacteria have
23 70S ribosomes w ith 50S and 30S subunits (whereas humans have SOS ribosomes w ith 60S and 40S
24 subunits). Those antibiotics that inhibit prokaryotic protein synthesis at the 30S ribosomal subunit include the
25 tetracyclines (eg, doxycycline) and the aminoglycosides (eg, streptomycin). Tetracyclines are bacteriostatic
26 antibiotics most useful in treating Mycoplasma pneumoniae as w ell as rickettsial and chlamydia! infections.
27 After binding to the 30S ribosomal subunit, tetracyclines block the aminoacyl tRNA from entering the
28 ribosomal acceptor site.
29
30 (Choices A and D) Erythromycin is a macrolide antibiotic that binds to the 50S prokaryotic ribosomal subunit
31 and blocks the translocation step by preventing tRNA release from the donor site after peptide bond
32 formation. Similarly, chloramphenicol binds to the 50S prokaryotic ribosomal subunit and blocks
33 peptidyltransferase action, thereby preventing new peptide bond formation.
34
35 (Choice B) Rifampin blocks prokaryotic mRNA synthesis by bacterial RNA polymerase. Typically used in
36 combination w ith other antibiotics to minimize bacterial resistance, rifampin treats active tuberculosis and
37 prevents meningitis caused by N. meningitidis or H. inf/uenzae.
38
1 A

2 Item: 9 of 46 II P Mark -<:I t>-


Q. Id : 6459 (388147) Previous Next
3
4 F. Diphtheria toxin [3%]
5
6
7 Explanation: User Id: 388147
8
Substances that inhibit protein synthesis by prokaryotic ribosomes are antibacterial antibiotics. Bacteria have
10 70S ribosomes w ith 50S and 30S subunits (whereas humans have BOS ribosomes w ith 60S and 40S
11 subunits). Those antibiotics that inhibit prokaryotic protein synthesis at the 30S ribosomal subunit include the
12 tetracyclines (eg, doxycycline) and the aminoglycosides (eg, streptomycin). Tetracyclines are bacteriostatic
13 antibiotics most useful in treating Mycoplasma pneumoniae as w ell as rickettsial and chlam ydia! infections.
14 After binding to the 30S ribosomal subunit, tetracyclines block the aminoacyl tRNA from entering the
15 ribosomal acceptor site.
16
17 (Choices A and D) Erythromycin is a macrolide antibiotic that binds to the 50S prokaryotic ribosomal subunit
18 and blocks the translocation step by preventing tRNA release from the donor site after peptide bond
19 formation. Similarly, chloramphenicol binds to the 50S prokaryotic ribosomal subunit and blocks
20 peptidyltransferase action, thereby preventing new peptide bond formation .
21
22 (Choice B) Rifampin blocks prokaryotic mRNA synthesis by bacterial RNA polymerase. Typically used in
23 combination w ith other antibiotics to minimize bacterial resistance, rifampin treats active tuberculosis and
24 prevents meningitis caused by N. meningitidis or H. influenzae.
25
26 (Choice E) Puromycin is an aminonucleoside antibiotic that has a structure analogous to aminoacyl-tRNA. In
27 the prokaryote, it leads to the premature release of unfinished polypeptide chains as polypeptidyl-puromycin
28 derivatives.
29
30 (Choice F) Diphtheria toxin inhibits protein synthesis in eukaryotic cells through ADP ribosylation of an
31 elongation factor (EF-2). EF-2 catalyzes the translocation of grow ing polypeptide chains along mRNA.
32
33 Educational objective:
34 Tetracyclines and aminoglycosides inhibit prokaryotic protein synthesis at the 30S ribosomal subunit,
35 w hereas erythromycin and chloramphenicol inhibit prokaryotic protein synthesis at the 50S ribosomal subunit.
36
37 Time Spent: 28 seconds 64% answ ered correctly. Last updated: [3/27/201 O].
38 Copyright © USMLEWORLD,LLC. SimExam ver. 2011 . 11.397425
1 A

2 Item: 10 of 46 II P M ark -<:I t>-


Q. Id : 6463 (388147] Previous Next
3
4
5
6 What type of regulatory protein is able to dictate differentiation of a progenitor cell into a dermal cell?
7
8
9
X @ A. Growth factor [29%]
B. Cytokine [15%]
11 C. Transcription factor [38%]
12
13
D. Growth factor receptor [ 17%]
14 E. Ribosome [2%]
15
16
17 Explanation: User Id: 388147
18
19 In embryonic development, the zygote begins as a single cell that divides to ultimately form all the cells in the
20 body. Thus, it is a totipotent stem cell. Each phase of embryonic maturation gives rise to progressively more
21 specialized tissues. As a given cell becomes more differentiated, it loses pluripotency. In the example
22 provided, differentiation progresses as follov1s:
23
24 Zygote __, Morula __, Embryoblast __, Epiblast __, Mesoderm __, Dermal Cell
25
26 The combination of a cell's differentiation history and various external signals create a transcription factor
27 milieu in the nucleus that favors expression of only those genes required for the cell to perform its function. All
28 nucleate cells in the human body possess the entire genome; transcription factors specific for each individual
29 tissue allow only those genes relevant to a cell's specific type and function to be expressed.
30
31 (Choice A) Growth factors are proteins that stimulate the growth and differentiation of various cell lines,
32 usually by regulating synthesis of transcription factors. Examples include GM-CSF and PDGF.
33
34 (Choice B) Cytokines are a diverse group of proteins used in cellular signaling and intercellular
35 communication. Growth factors are one type of cytokine.
36
37 (Choice D) Growth factor receptors transduce the signals delivered by their corresponding growth factors,
38 but do not directly regulate progenitor cell differentiation. A clinically relevant example of a growth factor
1 A

2 Item: 10 of 46 II P M ark -<:I t>-


Q. Id : 6463 (388147] Previous Next
3
4
5
Explanation: User Id: 388147
6
7
In embryonic development, the zygote begins as a single cell that divides to ultimately form all the cells in the
8
body. Thus, it is a totipotent stem cell. Each phase of embryonic maturation gives rise to progressively more
9
specialized tissues. As a given cell becomes more differentiated, it loses pluripotency. In the example
provided, differentiation progresses as follow s:
11
12
Zygote --+ Morula --+ Embryoblast --+ Epiblast --+ Mesoderm --+ Dermal Cell
13
14
15 The combination of a cell's differentiation history and various external signals create a transcription factor
16 milieu in the nucleus that favors expression of only those genes required for the cell to perform its function. All
17 nucleate cells in the human body possess the entire genome; transcription factors specific for each individual
18 tissue allow only those genes relevant to a cell's specific type and function to be expressed.
19
20 (Choice A) Growth factors are proteins that stimulate the growth and differentiation of various cell lines,
21 usually by regulating synthesis of transcription factors. Examples include GM-CSF and PDGF.
22
23 (Choice B) Cytokines are a diverse group of proteins used in cellular signaling and intercellular
24 communication . Growth factors are one type of cytokine.
25
26 (Choice D) Growth factor receptors transduce the signals delivered by their corresponding growth factors,
27 but do not directly regulate progenitor cell differentiation. A clinically relevant example of a growth factor
28 receptor is the epidermal growth factor receptor (EGFR, ErbB-1 , HER), w hich is overexpressed in certain
29 malignancies.
30
31 (Choice E) Ribosomes are composed of four different rRNA molecules in combination w ith ribosomal
32 proteins. They function to translate mRNA into protein but do not regulate cellular differentiation.
33
34 Educational objective:
35 Cellular differentiation is determined by the transcription factor milieu w ithin an individual cell.
36
37 Time Spent: 28 seconds 38% answ ered correctly. Last updated: [3/27/201 O].
38 Copyright © USMLEWORLD,LLC. SimExam ver. 2011 .11.397425
1 •
2 Item: 11 of 46 • \ ' Mark -<:J C>
Q. Id: 6468 (383147) Previous NeHt
3
4
5
6 An investigator is studying the effects of various drugs on the denervated eye. The experimental findings are
7 shown on the graph below.
8
9
10 O Control

12 0 light
13
• DrugX
14
15
16
17
18
19
20 Drug X is most likely to be which of the following?
21
22 A Atropine (14%]
23
24 x® B. Physostigmine (34%]
25 t1 C. Pilocarp1ne (39%]
26
D. Phenylephrine (7%]
27
28 E. Timolol (6%]
29
30
Explanation: User Id: 388147
31
32
The denervated eye demonstrates little pupillary response to light, resulting in minimal acetylcholine release
33
by the parasympathetic postganglionic neurons that innervate the pupillary sphincter of the medial iris. For
34
Drug X to cause pupillary constriction in the denervated eye, it must be a cholinomimetic agent that directly
35
activates the muscarinic receptors of the smooth muscles of the pupillary sphincter. Pilocarpine is one such
36
direct cholinergic muscarinic agonist.
37
38
1 A

2 Item: 11 of 46 II P Mark -<:I t>-


Q. Id : 6468 (388147] Previous Next
3
4
D. Phenylephrine [7%]
5
6 E. Timolol [6%]
7
8
9 Explanation: User Id: 388147
10
The denervated eye demonstrates little pupillary response to light, resulting in minimal acetylcholine release
12 by the parasympathetic postganglionic neurons that innervate the pupillary sphincter of the medial iris. For
13 Drug X to cause pupillary constriction in the denervated eye, it must be a cholinomimetic agent that directly
14 activates the muscarinic receptors of the smooth muscles of the pupillary sphincter. Pilocarpine is one such
15 direct cholinergic muscarinic agonist.
16
17 (Choice A) Atropine is a muscarinic receptor antagonist and therefore could not cause pupillary constriction in
18 a denervated eye.
19
20 (Choice B) Physostigmine is an indirect cholinergic agonist. To have an effect on the pupil, this
21 anticholinesterase inhibitor needs acetylcholine release from the parasympathetic postganglionic neurons to
22 counter. Denervation of the eye, how ever, minimizes any such release.
23
24 (Choice D) Phenylephrine is an alpha-adrenergic receptor agonist (a1 > a2). Consequently, phenylephrine
25 w ould activate a 1 receptors in the radial muscle of the iris, causing pupillary dilation instead of constriction.
26
27 (Choice E) Timolol is a non-selective beta blocker. Because pupillary constriction is mainly mediated by
28 muscarinic cholinergic receptors and pupillary dilation is mainly mediated by a 1-adrenergic receptors, a beta
29 blocker w ould not dramatically affect pupillary diameter. How ever, the beta blocker could inhibit 132 receptor-
30 mediated ciliary muscle relaxation for far vision and 132 receptor-mediated stimulation of aqueous humor
31 secretion.
32
33 Educational objective:
34 Direct muscarinic cholinergic agonists such as pilocarpine can cause pupillary constriction in the denervated
35 eye.
36
37 Time Spent: 22 seconds 39% answ ered correctly. Last updated: [11/ 10/2011].
38 Copyright © USMLEWORLD,LLC. SimExam ver. 2011 .11.397425
1 A

2 Item: 12 of 46 II P Mark -<:I t>-


3 a. Id : 6470 (388147] Previous Next

4
5
6 A 56-year-old Caucasian female diagnosed w ith severe anemia complains of numbness in her feet. Her
7 tongue is flat and shiny. She tends to fall w hen asked to stand w ith her feet together and eyes closed. W hich
8 of the follow ing is most likely overproduced in this patient?
9
10
11 A. Intrinsic factor [31% ]
B. Gastric acid [1 0%]
13 ~ C. Gastrin [30%]
14
15
D. Secretin [9%]
16 X @ E . Glucagon [5%]
17 F. Vasoactive intestinal peptide [15%]
18
19
20 Explanation: User Id: 388147
21
22 Pernicious anemia is a condition thought to arise from the immune-mediated destruction of gastric mucosa.
23 Over time, this damage results in chronic atrophic gastritis, a condition characterized by loss of the intrinsic
24 factor-secreting parietal cells. Profound hypochlorhydria develops, leading to increased serum gastrin levels
25 (antral G cell gastrin secretion is inhibited by hydrochloric acid}. Once the number of parietal cells is
26 sufficiently depleted and the stores of vitamin 8 12 are exhausted, pernicious anemia manifests w ith signs and
27 symptoms such as anemia, icterus, fatigue, atrophic glossitis, paresthesias, w eakness, and unsteady gait, as
28 seen in this patient.
29
30 (Choices A, B, and D) Levels of intrinsic factor, gastric acid, and secretin are low in individuals w ith
31 pernicious anemia-induced atrophic gastritis.
32
33 (Choices E and F) Glucagon and vasoactive intestinal peptide levels are typically unaffected by pernicious
34 anemia-induced atrophic gastritis.
35
36 Educational objective:
37 Pernicious anemia-induced atrophic gastritis is associated w ith profound hypochlorhydria and increased
38 serum aastrin levels.
39
40
1 A

2 Item: 12 of 46 II P Mark -<:I t>-


Q. Id : 6470 (388147] Previous Next
3
lU11yut ·~ llCll ClllU ~11111y . V Iit ltllU~ lU ICUI VVlltll Cl~l\tu lU ::iolCUIU VVILll llCI lt"tl lUYtll ltl CUIU ey e~ l.IU~tu . YV llll...11
4
5
of the follow ing is most likely overproduced in this patient?
6
7 A. Intrinsic factor [31 %]
8 B. Gastric acid [1 0%]
9
10 C. Gastrin [30%]
11 D. Secretin [9%]
X @ E. Glucagon [5%]
13
14 F. Vasoactive intestinal peptide [ 15%]
15
16
Explanation: User Id: 388147
17
18
Pernicious anemia is a condition thought to arise from the immune-mediated destruction of gastric mucosa.
19
Over time, this damage results in chronic atrophic gastritis, a condition characterized by loss of the intrinsic
20
factor-secreting parietal cells. Profound hypochlorhydria develops, leading to increased serum gastrin levels
21
(antral G cell gastrin secretion is inhibited by hydrochloric acid). Once the number of parietal cells is
22
sufficiently depleted and the stores of vitamin 8 12 are exhausted, pernicious anemia manifests w ith signs and
23
24 symptoms such as anemia, icterus, fatigue, atrophic glossitis, paresthesias, w eakness, and unsteady gait, as
25 seen in this patient.
26
27 (Choices A, 8, and D) Levels of intrinsic factor, gastric acid, and secretin are low in individuals w ith
28 pernicious anemia-induced atrophic gastritis.
29
30 (Choices E and F) Glucagon and vasoactive intestinal peptide levels are typically unaffected by pernicious
31 anemia-induced atrophic gastritis.
32
33 Educational objective:
34 Pernicious anemia-induced atrophic gastritis is associated w ith profound hypochlorhydria and increased
35 serum gastrin levels.
36
37 Time Spent: 23 seconds 30% answ ered correctly. Last updated: [3/27/201 O].
38 Copyright © USMLEWORLD,LLC. SimExam ver. 2011 .11.397425
1 A

2 Item: 13 of 46 II P Mark -<:I t>-


Q. Id : 6472 (388147] Previous Next
3
4
5
6 A 26-year-old male is found to have a left-sided scrotal mass during an annual physical examination. The
7 mass is palpable w hen the patient stands, but disappears w hen he lies dow n. Transillumination test is
8 negative. W hich of the follow ing is the most likely diagnosis?
9
10
A. Spermatocele [14%]
11
12 x @ B. Hydrocele [11 %]
C. Varicocele [60%]
14
D. Testicular cancer [10%]
15
16 E. Testicular torsion [4% ]
17
18
19 Explanation: User Id: 388147
20
Varicoceles are dilatations of the spermatic vein pampiniform plexus that develop in the left scrotum of
21
15-20% of adult males. Varicoceles tend to arise on the left side because the left renal vein is compressed
22
between the aorta and the superior mesenteric artery, thereby increasing the pressure w ithin the left renal
23
vein . In turn, the pressure in the left gonadal vein also increases, resulting in valve leaflet failure and
24
retrograde blood flow tow ard the testis on standing. W hen the recumbent position is assumed, much of the
25
blood returns to systemic circulation and the varicocele diminishes in size. The varicocele can present as a
26
nominal fullness or as a large, compressible mass likened to a "bag of w orms." Some individuals are
27
asymptomatic, w hile others experience aching pain relieved w ith recumbency, testicular atrophy (likely
28
induced by the minor increase in scrotal temperature), or reduced fertility.
29
30
(Choices A, B, D, and E) Spermatoceles, hydroceles, testicular cancers, and torsed testicles are not
31
diminished w ith recumbency. Spermatoceles are cystic masses that arise from the caput of the epididymis,
32
typically measuring between two and five centimeters in diameter. Normally, spermatoceles do not cause
33
34 symptoms. Hydroceles are peritoneal fluid collections that accumulate between the parietal and visceral
layers of the tunic a vaginalis. Hydroceles can be small or quite large, w ith pain and disability increasing w ith
35
the mass size. The fluid w ithin the hydrocele is easily transilluminated. Testicular cancer is the most
36
common solid tumor in young adult men, typically presenting as a firm, painless mass that does not
37
transilluminate. Testicular torsion is most often seen in infants and boys, presenting w ith acute pain after
38
1 A

2 Item: 13 of 46 II P Mark -<:I t>-


Q. Id : 6472 (388147] Previous Next
3
4 x @ B. Hydrocele [11 %]
5 ~ C. Varicocele [60%]
6
D. Testicular cancer [10%]
7
8 E. Testicular torsion [4%]
9
10
11 Explanation: User Id: 388147
12
Varicoceles are dilatations of the spermatic vein pampiniform plexus that develop in the left scrotum of
14 15-20% of adult males. Varicoceles tend to arise on the left side because the left renal vein is compressed
15 between the aorta and the superior mesenteric artery, thereby increasing the pressure w ithin the left renal
16 vein. In turn, the pressure in the left gonadal vein also increases, resulting in valve leaflet failure and
17 retrograde blood flow tow ard the testis on standing. When the recumbent position is assumed, much of the
18 blood returns to systemic circulation and the varicocele diminishes in size. The varicocele can present as a
19 nominal fullness or as a large, compressible mass likened to a "bag of w orms." Some individuals are
20 asymptomatic, w hile others experience aching pain relieved w ith recumbency, testicular atrophy (likely
21 induced by the minor increase in scrotal temperature), or reduced fertility.
22
23 (Choices A, 8, D, and E) Spermatoceles, hydroceles, testicular cancers, and torsed testicles are not
24 diminished w ith recumbency. Spermatoceles are cystic masses that arise from the caput of the epididymis,
25 typically measuring between two and five centimeters in diameter. Normally, spermatoceles do not cause
26 symptoms. Hydroceles are peritoneal fluid collections that accumulate between the parietal and visceral
27 layers of the tunic a vaginalis. Hydroceles can be small or quite large, w ith pain and disability increasing w ith
28 the mass size. The fluid w ithin the hydrocele is easily transilluminated. Testicular cancer is the most
29 common solid tumor in young adult men, typically presenting as a firm, painless mass that does not
30 transilluminate. Testicular torsion is most often seen in infants and boys, presenting w ith acute pain after
31 intense physical activity or minor trauma to the testis.
32
33 Educational objective:
34 Varicoceles are dilatations of the spermatic vein pampiniform plexus that enlarge w ith standing and regress
35 w ith lying dow n.
36
37 Time Spent: 26 seconds 60% answ ered correctly. Last updated: [3/27/201 O].
38 Copyright © USMLEWORLD,LLC. SimExam ver. 2011 .11.397425
1 A

2 Item: 14 of 46 ll P Mark -<:I t>-


Q. Id : 6473 (388147] Previous Next
3
4
5
6 A 57-year-old Caucasian male com(llains of poor appetite w eight loss and epigastric discomfort. Physical
7 examination reveals pale cQ!Jjunctivae, shiny< tongue, and axillary hyRerpigmentation. Gastric endoscopy w ith
8 biopsy reveals a high-grade malignancy. W hich of the follow ing is the most likely cause of his skin changes?
9
10
A. Peutz-Jeghers syndrome [ 11 % ]
11
12 B. Acanthosis nigricans [78%]
13 C. Metastatic melanoma [5%]
X @ D. Neurofibromatosis type 1 [3%]
15
16 E. Familial adenomatous polyposis [3%]
17
18
19 Explanation: User Id: 388147
20
Acanthosis nigricans is a reactive skin condition characterized by brow n to black, velvety, thickened plaques
21
that tend to arise in flexural regions such as the axillae or back of the neck. Acanthosis nigricans is most
22
commonly associated w ith benign conditions such as insulin resistance or obesity. How ever, this skin
23
condition can also signal the presence of malignancies w ithin the gastrointestinal tract (eg, stomach, liver} or
24
lungs. W hen acanthosis nigricans appears suddenly, quickly spreads, or involves the mucous membranes or
25
palms and soles, the likelihood of malignancy increases. It has been postulated that malignancy and
26
hyperinsulinemia cause acanthosis nigricans by influencing growth factor production.
27
28
(Choice A) Peutz-Jeghers syndrome is a rare autosomal dominant disease characterized by pigmented
29
mucocutaneous macules and hamartomatous polyps in the GI tract. Progressive growth can lead to
30
malignant transformation.
31
32
(Choice C) Melanomas are asymptomatic or pruritic lesions, typically measuring more than one centimeter in
33
34 diameter. Marked pigment variability may occur, w ith shades of black, brow n, red, navy blue, and/or gray
present. The borders are frequently irregular and notched. Melanomas most often appear on the skin, but can
35
also arise in the esophagus or metastasize to the liver.
36
37
(Choice D) Neurofibromatosis type 1 is characterized by neurofibromas, cafe-au-lait spots, and pigmented
38
1 A

2 Item: 14 of 46 ll P Mark -<:I t>-


Q. Id : 6473 (388147] Previous Next
3
4
5 Explanation: User Id: 388147
6
7 Acanthosis nigricans is a reactive skin condition characterized by brow n to black, velvety, thickened plaques
8 that tend to arise in flexural regions such as the axillae or back of the neck. Acanthosis nigricans is most
9 commonly associated w ith benign conditions such as insulin resistance or obesity. How ever, this skin
10 condition can also signal the presence of malignancies w ithin the gastrointestinal tract (eg, stomach, liver) or
11 lungs. When acanthosis nigricans appears suddenly, quickly spreads, or involves the mucous membranes or
12 palms and soles, the likelihood of malignancy increases. It has been postulated that malignancy and
13 hyperinsulinemia cause acanthosis nigricans by influencing growth factor production .

15 (Choice A) Peutz-Jeghers syndrome is a rare autosomal dominant disease characterized by pigmented


16 mucocutaneous macules and hamartomatous polyps in the GI tract. Progressive growth can lead to
17 malignant transformation .
18
19 (Choice C) Melanomas are asymptomatic or pruritic lesions, typically measuring more than one centimeter in
20 diameter. Marked pigment variability may occur, w ith shades of black, brow n, red, navy blue, and/or gray
21 present. The borders are frequently irregular and notched. Melanomas most often appear on the skin, but can
22 also arise in the esophagus or metastasize to the liver.
23
24 (Choice D) Neurofibromatosis type 1 is characterized by neurofibromas, cafe-au-lait spots, and pigmented
25 iris hamartomas. Neurofibromas that undergo malignant degeneration are called malignant peripheral nerve
26 sheath tumors.
27
28 (Choice E) Familial adenomatous polyposis is an autosomal dominant syndrome characterized by the growth
29 of dozens of colonic adenomatous polyps. Some patients develop benign extraintestinal manifestations (eg,
30 soft tissue tumors of the skin, hepatoblastomas). The polyps inevitably progress to colorectal cancer.
31
32 Educational objective:
33 Acanthosis nigricans is most commonly associated w ith benign conditions such as insulin resistance or
34 obesity. How ever, this skin condition can also signal the presence of malignancies w ithin the gastrointestinal
35 tract (eg, stomach, liver) or lungs.
36
37 Time Spent: 230 seconds 78% answ ered correctly. Last updated: [4/ 10/201 O].
38 Copyright © USMLEWORLD,LLC. SimExam ver. 2011 .11.397425
1 A

2 Item: 15 of 46 II P Mark -<:I t>-


Q. Id : 64n (388147] Previous Next
3
4
5
6 A 67-year-old male suffering from progressive exertional dysi:mea, a11ct.s y co~e is diagnosed w ith a.or;tic
7 stenosis. His aortic valve is replaced w ith a mechanical valve. W hich of the follow ing should be prescribed
8 for life long to prevent clot formation on the valve?
9
10
11
A. Thromboxane inhibitor [12%]
12 X @ B. Glycoprotein lib/Illa inhibitor [14%]
13 C. Antithrombin Ill agonist [10%]
14
D. Vitamin K antagonist [62%]
16 E . Tissue plasminogen activator [2%]
17
18
19 Explanation: User Id: 388147
20
21 Mechanical heart valves require chronic anticoagulation postoperatively. In patients w ith mechanical aortic
22 valves and no other risk factors for thromboembolism (eg, atrial fibrillation, left ventricular dysfunction,
23 hypercoagulable states), long-term anticoagulation w ith w arfarin to maintain an INR of 2.0-3.5 is a class I
24 recommendation. W arfarin operates by antagonizing normal y-carboxylation of vitamin K-dependent clotting
25 factors. Unlike unfractionated heparin, w arfarin can be taken orally on an outpatient basis, w hich is part of
26 w hy it is the anticoagulant of choice here.
27
(Choice A) Anti-platelet therapy alone is inadequate protection against the thromboembolic complications
28
associated w ith mechanical heart valves. The addition of low-dose aspirin (a thromboxane synthesis inhibitor)
29
to w arfarin therapy is a Class I recommendation. Although aspirin is not a substitute for anticoagulation w ith
30
w arfarin, it further decreases the risk of thromboembolism and reduces mortality from other cardiovascular
31
diseases.
32
33
(Choice B) Glycoprotein lib/Illa inhibitors are used in patients w ith acute coronary syndrome.
34
35 (Choice C) Heparin catalytically activates antithrombin Ill. Most experts recommend heparinization in the
36 immediate postoperative period until w arfarin takes effect. Because heparin is administered subcutaneously
37 or intravenously, it is not an ideal choice in the outpatient setting .
38
1 A

2 Item: 15 of 46 II P Mark -<:I t>-


Q. Id : 64n (388147] Previous Next
3
4
D. Vitamin K antagonist [62%]
5
6 E. Tissue plasminogen activator [2%]
7
8
Explanation: User Id: 388147
9
10
Mechanical heart valves require chronic anticoagulation postoperatively. In patients w ith mechanical aortic
11
valves and no other risk factors for thromboembolism (eg, atrial fibrillation, left ventricular dysfunction,
12
hypercoagulable states), long-term anticoagulation w ith w arfarin to maintain an INR of 2.0-3.5 is a class I
13
recommendation. W arfarin operates by antagonizing normal y-carboxylation of vitamin K-dependent clotting
14
factors. Unlike unfractionated heparin, w arfarin can be taken orally on an outpatient basis, w hich is part of
w hy it is the anticoagulant of choice here.
16
17 (Choice A) Anti-platelet therapy alone is inadequate protection against the thromboembolic complications
18 associated w ith mechanical heart valves. The addition of low-dose aspirin (a thromboxane synthesis inhibitor)
19 to w arfarin therapy is a Class I recommendation. Although aspirin is not a substitute for anticoagulation w ith
20 w arfarin, it further decreases the risk of thromboembolism and reduces mortality from other cardiovascular
21 diseases.
22
23 (Choice B) Glycoprotein lib/Illa inhibitors are used in patients w ith acute coronary syndrome.
24
25 (Choice C) Heparin catalytically activates antithrombin Ill. Most experts recommend heparinization in the
26 immediate postoperative period until w arfarin takes effect. Because heparin is administered subcutaneously
27 or intravenously, it is not an ideal choice in the outpatient setting .
28
29 (Choice E) Thrombolytics such as tissue plasminogen activator promote conversion of plasminogen to
30 plasmin, resulting in the cleaving of existing fibrin clots. Thrombolytics are not used to prevent clot formation.
31
32 Educational objective:
33 Mechanical heart valves require chronic anticoagulation postoperatively. W arfarin, an inhibitor of vitamin K-
34 dependent clotting factor synthesis, is the agent of choice in this patient population . Anti-platelet therapy w ith
35 aspirin may be used adjunctively.
36
37 Time Spent: 62 seconds 62% answ ered correctly. Last updated: [3/27/201 O].
38 Copyright © USMLEWORLD,LLC. SimExam ver. 2011 .11.397425
1 •
2 Item: 16 of 46 • \' Mark -<:J C>
Q. Id: 6432 (383147) Previous NeHt
3
4 A 64-year-old male with a firm, palpable nodule in his prostate complains of severe back pain. The pain is not
5 relieved by rest and wakes him during the night. Long-term treatment of this patient with leuprolide would
6 result in which of the following?
7
8
X ~ A. Increased pituitary hormone release (7%]
9
10 B. Impaired steroid synthesis (6%]
11 C. Impaired testosterone conversion in target tissues (15%)
12
D. Decreased Clfculatmg testosterone (52%)
13
14 E. Testosterone receptor down-regulation (19%)
15

17
Explanation: User Id: 388147
18
19 Hypothalamus
Gn·RH release
20
21
22 ~Gn·RH agonists
23
24
25 Anterior pituitary
26 LH synthesis
27
28
29
30
31 Leyd19 cells
32 Testosterone synthesis
33
34
35
36
37
38 Peripheral conversion of
6
Explanation: User Id: 388147
7
8
9 Hypothalamus
Gn-RH release
10

~Gn-RH agonists
11
12
13
14
15 Anterior pituitary
LH synthesis
17
18
19 '°
20
21 Leydtg cells µ-;..~
22 Testosterone synttiesis
23
24
25
26
27
28 Peripheral conversion of
29 Splronolactone testosterone to DHT
30
Finasteride
31
32
33
34
Androgen hormone-receptor
35 complex on target cells
36 ~ ....~lutamide
37 ~proterone
38
1 A

2 Item: 16 of 46 II P Mark -<:I t>-


Q. Id : 6482 (388147] Previous Next
3
}'\JIUroye 11 1 1 ur 1 110 1 1 e-rtt~µtur
4
complex on target cells
5 ~ .-:.lutamide
6 ~proterone
7
8
9 Back pain unrelieved by rest in a patient w ith a firm prostatic nodule is strongly suggestive of prostatic cancer,
10 w ith the back pain signaling the presence of metastatic, osteoblastic lesions in the vertebrae. Leuprolide is a
11 form of androgen deprivation therapy used in men w ith advanced prostate cancer. A GnRH analog, leuprolide
12 has agonist properties w hen administered in a pulsatile fashion and antagonist properties w hen administered
13 continuously. As an antagonist, leuprolide causes long-term reduction of circulating testosterone levels
14 (resulting in medical orchiectomy) by decreasing anterior pituitary gland release of luteinizing hormone
15 (LH). The testicular Leydig cells respond to the low er LH levels by producing less testosterone. Because
approximately 80% of new ly diagnosed prostate cancer cases are androgen dependent, leuprolide treatment
17 is useful in prolonging survival, relieving bone pain, and reducing serum PSA.
18
19 (Choice A) Continuous long-term administration of leuprolide inhibits pituitary gland release of FSH and LH,
20 resulting in reduced testosterone production and circulation .
21
22 (Choice B) Although some animal studies suggest that leuprolide directly inhibits enzymes involved in
23 testicular steroidogenesis, other studies favor different mechanisms.
24
25 (Choice C) Free testosterone is reduced by the cytoplasmic enzyme 5a-reductase to form the more potent
26 dihydrotestosterone in peripheral target tissues. Leuprolide does not inhibit 5a-reductase.
27
28 (Choice E) Leuprolide does not appear to dow n-regulate testosterone receptors in androgen-responsive
29 tissues. After administration for 10-14 days, how ever, leuprolide does dow n-regulate GnRH receptors on
30 gonadotropin cells in the pituitary gland. FSH and LH release is thereby prevented, resulting in gonadal
31 suppression.
32
33 Educational objective:
34 Long-term, continuous leuprolide administration in a male inhibits LH release and therefore testosterone
35 production, amounting to a "medical orchiectomy."
36
37 Time Spent: 92 seconds 52% answ ered correctly. Last updated: [11 / 10/2011].
38 Copyright © USMLEWORLD,LLC. SimEx am ver. 2011 . 11.397425
1 A

2 Item: 17 of 46 II P Mark -<:I t>-


Q. Id : 6434 (388147] Previous Next
3
4
5
6 In embryonic development, both melanocytes and germ cells undergo w hich of the follow ing?
7
8
9 A. Apeptesis [4 %.j
10 B. Metaplasia [4%]
11 .; @ C. Migration [90%]
12
13 D. ~ypertrephy [2%•l
14 E. Anaplasia [0%]
15
16
Explanation: User Id: 388147
18
19 During the first three months of embryonic development, precursor melanocytes migrate from the neural crest
20 to the dermis and then the epidermis. Once established in the epidermis, the precursor cells differentiate into
21 melanocytes. Identifiable melanosomes appear w ithin the melanocytes of the fetal epidermis by eight to ten
22 w eeks of gestational age.
23
24 Originating from the tissues of the posterior yolk sac, primordial germ cells similarly migrate to the gonadal
25 ridge during the fifth and sixth w eeks of embryonic development. (Like melanocytes, germ cells may stray
26 from the migration route and take up residence in different organs or tissues, w hich accounts for the
27 development of melanocytic and germ cell tumors in unusual locations.) As oogonia, the germ cells undergo
28 numerous mitoses and then meiosis, finally arresting as primary oocytes in meiotic prophase. As
29 undifferentiated spermatogonia, the germ cells proliferate in the basal compartment, w ith some giving rise to
30 primary spermatocytes.
31
32 (Choices A, B, D, and E) Melanocytes and germ cells do not typically undergo apoptosis, metaplasia,
33 hypertrophy, or anaplasia during normal embryonic development. How ever, these states may be observed in
34 other tissues. For instance, embryonic formation of fingers and toes requires the apoptosis of intervening
35 cells in these regions. Embryonic metaplasia is seen in the glucocorticoid-induced appearance of
36 hepatocytes w ithin the pancreas. Cardiac hypertrophy may develop in the fetus of a diabetic mother. Lastly,
37 w hile not a normal finding, anaplasia can arise in embryonic skeletal muscle, eventually leading to
38 rhabdomyosarcoma.
1 A

2 Item: 17 of 46 II P Mark -<:I t>-


Q. Id : 6434 (388147] Previous Next
3
4 .; @ C. Migration [90%]
5
6
D. ~ypertrephy [2%•l
7 E. Anaplasia [0%]
8
9
10 Explanation: User Id: 388147
11
12 During the first three months of embryonic development, precursor melanocytes migrate from the neural crest
13 to the dermis and then the epidermis. Once established in the epidermis, the precursor cells differentiate into
14 melanocytes. Identifiable melanosomes appear w ithin the melanocytes of the fetal epidermis by eight to ten
15 w eeks of gestational age.
16
Originating from the tissues of the posterior yolk sac, primordial germ cells similarly migrate to the gonadal
18 ridge during the fifth and sixth w eeks of embryonic development. (Like melanocytes, germ cells may stray
19 from the migration route and take up residence in different organs or tissues, w hich accounts for the
20 development of melanocytic and germ cell tumors in unusual locations.) As oogonia, the germ cells undergo
21 numerous mitoses and then meiosis, finally arresting as primary oocytes in meiotic prophase. As
22 undifferentiated spermatogonia, the germ cells proliferate in the basal compartment, w ith some giving rise to
23 primary spermatocytes.
24
25 (Choices A, B, D, and E) Melanocytes and germ cells do not typically undergo apoptosis, metaplasia,
26 hypertrophy, or anaplasia during normal embryonic development. How ever, these states may be observed in
27 other tissues. For instance, embryonic formation of fingers and toes requires the apoptosis of intervening
28 cells in these regions. Embryonic metaplasia is seen in the glucocorticoid-induced appearance of
29 hepatocytes w ithin the pancreas. Cardiac hypertrophy may develop in the fetus of a diabetic mother. Lastly,
30 w hile not a normal finding, anaplasia can arise in embryonic skeletal muscle, eventually leading to
31 rhabdomyosarcoma.
32
33 Educational objective:
34 During normal embryonic development, melanocytes migrate from the neural crest to the epidermis and germ
35 cells migrate from the yolk sac to the gonadal ridge.
36
37 Time Spent: 21 seconds 90% answ ered correctly. Last updated: [3/27/201 O].
38 Copyright © USMLEWORLD,LLC. SimExam ver. 2011 .11.397425
1 A

2 Item: 18 of 46 II P Mark -<:I t>-


Q. Id : 6496 (388147] Previous Next
3
4
5
6 An infant w ith facial abnormalities and thymic hyQOQlasia has a gene deletion mapped to chromosome
7 22q11 . W hich of this patient's cardiac structures w as most likely malformed in utero?
8
9
A. Primitive ventricle [9%]
10
11 B. Primitive atria [8%]
12 .; @ C. Truncus arteriosus [73%]
13
D. Sinus venosus [6%]
14
15 E. Cardinal veins [4%]
16
17
Explanation: User Id: 388147
19
DiGeorge syndrome (DGS) is typically caused by a deletion on chromosome 22q11 .2 that affects the
20
formation of the branchial arch and great vessels during the fourth w eek of embryogenesis. The clinical
21
features of this syndrome result from the failed development of the third and fourth pharyngeal pouches
22
23 (which are the predecessors of the thymus, parathyroid glands, and the ultimobranchial body). Consequently,
individuals w ith DGS have defective T cell-mediated immunity secondary to an absent or hypoplastic thymus,
24
tetany secondary to absent parathyroid glands, and defects in the heart and great vessels.
25
26
Conotruncal (outflow tract) defects are quite common in infants w ith DGS, and include persistent truncus
27
arteriosus, tetralogy of Fallot, and interrupted aortic arch. Persistent truncus arteriosus is a condition in w hich
28
a single great vessel arises from the heart, w ith the aorta, pulmonary arteries, and coronary arteries branching
29
off from the ascending segment of this vessel (normally, the truncus arteriosus divides in utero, separating
30
into the aorta and pulmonary artery).
31
32
(Choices A and B) W hile a ventricular septal defect is consistently associated w ith persistent truncus
33
34 arteriosus, the primitive ventricle and primitive atria are not typically directly affected in DGS.
35
(Choices D and E) In utero, the sinus venosus is a cavity that receives blood from the vitelline vein, the
36
umbilical vein, and the common cardinal vein. Eventually it forms the coronary sinus and oblique vein of the
37
left atrium and the sinus venarum of the right atrium. The cardinal veins return blood from the abdominal w alls
38
1 A

2 Item: 18 of 46 II P Mark -<:I t>-


Q. Id : 6496 (388147] Previous Next
3
4
D. Sinus venosus [6%]
5
6 E. Cardinal veins [4% ]
7
8
9 Explanation: User Id: 388147
10
11 DiGeorge syndrome (DGS) is typically caused by a deletion on chromosome 22q11 .2 that affects the
12 formation of the branchial arch and great vessels during the fourth w eek of embryogenesis. The clinical
13 features of this syndrome result from the failed development of the third and fourth pharyngeal pouches
14 (which are the predecessors of the thymus, parathyroid glands, and the ultimobranchial body). Consequently,
15 individuals w ith DGS have defective T cell-mediated immunity secondary to an absent or hypoplastic thymus,
16 tetany secondary to absent parathyroid glands, and defects in the heart and great vessels.
17
Conotruncal (outflow tract) defects are quite common in infants w ith DGS, and include persistent truncus
19 arteriosus, tetralogy of Fallot, and interrupted aortic arch. Persistent truncus arteriosus is a condition in w hich
20 a single great vessel arises from the heart, w ith the aorta, pulmonary arteries, and coronary arteries branching
21 off from the ascending segment of this vessel (normally, the truncus arteriosus divides in utero, separating
22 into the aorta and pulmonary artery).
23
24 (Choices A and B) W hile a ventricular septal defect is consistently associated w ith persistent truncus
25 arteriosus, the primitive ventricle and primitive atria are not typically directly affected in DGS.
26
27 (Choices D and E) In utero, the sinus venosus is a cavity that receives blood from the vitelline vein, the
28 umbilical vein, and the common cardinal vein. Eventually it forms the coronary sinus and oblique vein of the
29 left atrium and the sinus venarum of the right atrium. The cardinal veins return blood from the abdominal w alls
30 and the W olffian bodies. None of these structures is commonly involved in DGS.
31
32 Educational objective:
33 DiGeorge syndrome is characterized by thymic aplasia, parathyroid aplasia, and cardiac and great vessel
34 defects. The truncus arteriosus is commonly malformed in utero, resulting in persistent truncus arteriosus,
35 tetralogy of Fallot, or interrupted aortic arch.
36
37 Time Spent: 24 seconds 73% answ ered correctly. Last updated: [3/27/201 O].
38 Copyright © USMLEWORLD,LLC. SimExam ver. 2011 .11.397425
1 A

2 Item: 19 of 46 II P Mark -<:I t>-


Q. Id : 6500 (388147] Previous Next
3
4
5
6 An infant suffering from recurrent pyogenic;, joLec,tions has defective expression of 'c9lQJ As a result, his
7 antibody isotype sw itching jsJ o:iRaired. W hich of the following antibody levels are most likely to be low in this
8 patient?
9
10
11 A. lgA and lgM [6%]
12 B. lgG and lgM [9%]
13 X @ C. lgM and lgD [3%]
14
~ D. lgA and lgG [78%]
15
16 E. lgG and lgD [4%]
17
18
Explanation: User Id: 388147
20
21 lsotype sw itching is a mechanism by w hich a single B cell clone can express various different lg heavy-chain
22 isotypes that share an identical variable-diversity-joining (VDJ) region. The net result is the production of
23 antibodies w ith the same narrow antigenic specificity but w ith different effector functionality. Naive mature B
24 cells produce both lgM and lgD w ith identical VDJ (antigen-binding) regions. Once activated by antigen,
25 how ever, these B cells w ill isotype switch to allow for the generation of lgG, lgA, and lgE.
26
27 For isotype sw itching to occur, a two-signal process must take place. Signal 1 causes the activation of
28 transcription at a particular region in the lg locus, thereby determining the isotype specificity. Signal 2 is
29 dependent on the CD40 on B cells and causes DNA sw itch recombination. Therefore, if an individual had
30 defective expression of CD40, he would be unable to engage in isotype sw itching from lgM or lgD to lgG, lgA,
31 or lgE. Consequently, his lgG, lgA, and lgE antibody levels would be low.
32
33 Educational objective:
34 lsotype sw itching allow s for antigen-activated B cells to create lgG, lgA, and lgE from the original lgM and lgD
35 antibodies.
36
37 Time Spent: 41 seconds 78% answ ered correctly. Last updated: [3/27/201 O].
38 Copyright © USMLEWORLD,LLC. SimExam ver. 2011 .11.397425
1 A

2 Item: 20 of 46 II P Mark -<:I t>-


Q. Id : 6505 (388147] Previous Next
3
4
5
6 A researcher isolates chromosomal DNA and splits it into several large fragments. One of the fragments has
7 a high 5-methylcytosine content. Which of the following statements best describes this fragment?
8
9
10 /\. It is leesely asseciateEl 'Nith histenes [8%•]
11 B. It is a site for rRNA synthesis [9%]
12 ., @ C. It has low transcriptional activity [66%]
13
14 D. It is unwound for replication [7%]
15 E. It has eeen Elamagea ey UV raEliatien [1 Q%,]
16
17
18 Explanation: User Id: 388147
19 -
DNA methylation refers to the enzymatic addition of methyl groups to cytosine residues in the
21 genome. Cytosine-guanine dinucleotide repeat sequences are preferentially methylated. Genes w ith
22 methylated cytosine residues are effectively silenced. Approximately 1% of the human genome is methylated
23 at any given point in time. Methylation is believed to play a role in silencing genes not required by terminally
24 differentiated somatic cells. It can also be pathologic, for example in malignancies w here tumor suppressor
25 genes are dow nregulated by selective methylation.
26
27 DNA methyltransferases perform DNA methylation. These enzymes can maintain methylated regions during
28 DNA replication by using the parent strand as a template to carry out methylation of the daughter strand . DNA
29 methylation is considered a part of the epigenetic code - a heritable form of gene regulation not reliant on
30 changes in the genetic code itself.
31
32 (Choice A and D) Heterochromatin is tightly packed DNA associated w ith histones. The DNA w ithin
33 heterochromatin is not actively transcribed. Euchromatin is unwound DNA loosely associated w ith histones
34 that contains genes actively being transcribed . Genes packed into heterochromatin in a parent cell are
35 similarly packed into heterochromatin in daughter cells, making heterochromatin part of the epigenetic code
36 as w ell.
37
38 (Choice B) rRNA is synthesized w ithin the nucleolus by RNA polymerase I.
1 A

2 Item: 20 of 46 II P Mark -<:I t>-


Q. Id : 6505 (388147] Previous Next
3
LI . ll I~ UI l ~Y VU I IU IVI I CJ.1111...0llVI I L I / Uj
4
5 E. It has seen aamagea ey UV raaiatien [1 Q%,]
6
7
8 Explanation: User Id: 388147
9
10 DNA methylation refers to the enzymatic addition of methyl groups to cytosine residues in the
11 genome. Cytosine-guanine dinucleotide repeat sequences are preferentially methylated. Genes w ith
12 methylated cytosine residues are effectively silenced. Approximately 1% of the human genome is methylated
13 at any given point in time. Methylation is believed to play a role in silencing genes not required by terminally
14 differentiated somatic cells. It can also be pathologic, for example in malignancies w here tumor suppressor
15 genes are dow nregulated by selective methylation .
16
17 DNA methyltransferases perform DNA methylation. These enzymes can maintain methylated regions during
18 DNA replication by using the parent strand as a template to carry out methylation of the daughter strand. DNA
19 methylation is considered a part of the epigenetic code - a heritable form of gene regulation not reliant on
changes in the genetic code itself.
21
22 (Choice A and D) Heterochromatin is tightly packed DNA associated w ith histones. The DNA w ithin
23 heterochromatin is not actively transcribed. Euchromatin is unwound DNA loosely associated w ith histones
24 that contains genes actively being transcribed . Genes packed into heterochromatin in a parent cell are
25 similarly packed into heterochromatin in daughter cells, making heterochromatin part of the epigenetic code
26 as w ell.
27
28 (Choice B) rRNA is synthesized w ithin the nucleolus by RNA polymerase I.
29
30 (Choice E) UV radiation damages DNA by forming covalent bonds between adjacent pyrimidine residues
31 thereby forming cyclobutane pyrimidine dimers or 6,4 photoadducts.
32
33 Educational objective:
34 DNA methylation silences gene transcription and is part of the epigenetic code. Typically, methylated regions
35 are rich in cytosine-guanine dinucleotide repeat sequences.
36
37 Time Spent: 168 seconds 66% answ ered correctly. Last updated: [3/27/2010].
38 Copyright © USMLEWORLD,LLC. SimExam ver. 2011 .11.397425
1 •
2 Item: 21 of 46 • \' Mark -<:J C>
Q. Id: 6509 (383147) Previous NeHt
3
4
5
6 A 35-year-old Caucasian male is hospitalized with ·severe bone i:iain. He has been experiencing pain in his
7 extremities and back for several ~ears. Bone marrow aspirate findings are shown on the image below.
8
9
10
11
12
13
14
15
16
17
18
19 '°
20

22
23
24
25
26
27
28
29 Displayed v.ith pmnissiw ftan Springer Healthcare Ltd. © Copyright 2005 by Current Medicine
30
31
32 Which of the following enzymes is most likely deficient in this patient?
33
34 A. Hexosaminidase A [9%]
35
36 B. Glucocerebros1dase [45%]
37 C. a-galactosidase [8%]
38
1 A

2 Item: 21 of 46 II P Mark -<:I t>-


Q. Id : 6509 (388147] Previous Next
3
4
A. Hexosaminidase A [9%]
5
6 B. Glucocerebrosidase [45%]
7 C. a-galactosidase [8%]
8
D. Sphingomyelinase [8%]
9
10 E . Arylsulfatase A [16%]
11 X @ F. Glucose-6-phosphatase dehydrogenase [13%]
12
13
14 Explanation: User Id: 388147
15
16 Gaucher disease is the most common lysosomal storage disease and the most prevalent genetic disorder
17 among Ashkenazi Jew s. This condition is characterized by glucocerebroside deposition in the lysosomes of
18 macrophages. Normally, glucocerebroside (which is derived from the phagocytosis of senescent leukocytes
19 and erythrocytes) is degraded into its basic glucose and lipid elements by glucocerebrosidase. W hen the
20 glucocerebrosidase gene on chromosome 1q21 is mutated, how ever, a deficiency of glucocerebrosidase
develops. Macrophages called Gaucher cells fill w ith the lipid glucocerebroside and assume the appearance
22 of w rinkled tissue paper, as seen in the image above. Clinical manifestations of Gaucher disease include
23 hepatosplenomegaly, anemia, thrombocytopenia, bleeding, osteopenia, bone pain, and bone fractures.
24
25 (Choice A) Tay-Sachs disease is an autosomal recessive, neurodegenerative condition common in
26 Ashkenazi Jew s. It is caused by a deficiency of hexosaminidase A and is characterized by lysosomal
27 retention of Gm2 ganglioside. Weakness, loss of motor skills, and the characteristic cherry red macula
28 develop during infancy.
29
30 (Choice C) Fabry disease is an X-linked lysosomal storage disease caused by deficiency of a-galactosidase,
31 w hich results in lysosomal retention of glycosphingolipids. Clinical manifestations appear in childhood and
32 include severe extremity pain, decreased ability to sw eat, corneal and lenticular changes, gastrointestinal tract
33 dysfunction, and angiokeratomas.
34
35 (Choice D) Niemann-Pick disease type A is an autosomal recessive, lysosomal storage disorder caused by
36 sphingomyelinase deficiency. Sphingomyelin then accumulates in lysosomes, w ith affected individuals
37 developing profoundly abnormal neurologic function. Death results by two or three years of age.
38
1 A

2 Item: 21 of 46 II P Mark -<:I t>-


Q. Id : 6509 (388147] Previous Next
3
4 ar1u er ylrnucyLt::SJ 1s u~raut::u n1lu llS uas1c y1ucuse ar1u 11p1u t::1err1er1ls uy y1ucucereu1us1uase. vvr1er1lllt:-
5 glucocerebrosidase gene on chromosome 1q21 is mutated, how ever, a deficiency of glucocerebrosidase
6 develops. Macrophages called Gaucher cells fill w ith the lipid glucocerebroside and assume the appearance
7 of w rinkled tissue paper, as seen in the image above. Clinical manifestations of Gaucher disease include
8 hepatosplenomegaly, anemia, thrombocytopenia, bleeding, osteopenia, bone pain, and bone fractures.
9
10 (Choice A) Tay-Sachs disease is an autosomal recessive, neurodegenerative condition common in
11 Ashkenazi Jew s. It is caused by a deficiency of hexosaminidase A and is characterized by lysosomal
12 retention of Gm2 ganglioside. W eakness, loss of motor skills, and the characteristic cherry red macula
13 develop during infancy.
14
15 (Choice C) Fabry disease is an X-linked lysosomal storage disease caused by deficiency of a-galactosidase,
16 w hich results in lysosomal retention of glycosphingolipids. Clinical manifestations appear in childhood and
17 include severe extremity pain, decreased ability to sw eat, corneal and lenticular changes, gastrointestinal tract
18 dysfunction, and angiokeratomas.
19
20 (Choice D) Niemann-Pick disease type A is an autosomal recessive, lysosomal storage disorder caused by
sphingomyelinase deficiency. Sphingomyelin then accumulates in lysosomes, w ith affected individuals
22 developing profoundly abnormal neurologic function. Death results by two or three years of age.
23
24 (Choice E) Metachromatic leukodystrophy is an autosomal recessive, lysosomal storage disorder caused by
25 arylsulfatase A deficiency. Cerebroside sulfate accumulates as a result. Affected individuals demonstrate
26 peripheral neuropathy and motor skill difficulties. Death typically occurs by early adulthood.
27
28 (Choice F) Glucose-6-phosphatase dehydrogenase deficiency is an X-linked disorder characterized by low
29 levels of glutathione in its reduced form, leaving red blood cells more vulnerable to oxidative
30 stressors. Symptomatic individuals typically present w ith acute hemolytic anemia and neonatal jaundice.
31
32 Educational objective:
33 Gaucher disease is the most common lysosomal storage disease and is characterized by a deficiency of
34 glucocerebrosidase. Macrophages called Gaucher cells fill w ith the lipid glucocerebroside and assume the
35 appearance of w rinkled tissue paper.
36
37 Time Spent: 45 seconds 45% answ ered correctly. Last updated: [10/3/2011].
38 Copyright © USMLEWORLD,LLC. SimEx am ver. 2011 . 11.397425
1 A

2 Item: 22 of 46 • \> Mark -<J C>-


Q. Id: 6619 (388147) Previous Next
3
4
5
6 A 34-year-old male is brought to the ER with repeated episodes of coffee ground emesis. The patient is
7 frightened and complains of dizziness upon standing . His past medical history is not significant. His blood
8 pressure is 90160 mmHg supine and 80/55 m mHg sitting. Which of the following substances is most likely to
9 accumulate in this patient's vascular smooth muscle cells?
10
11
A. cAMP (14%)
12
13 B. cGMP (31%)
14 C. lnosrtol tnphosphate (IP3 ) (40%)
15
16 D. 2,3-bisphosphoglycerate (2,3-BPG) [9o/o)
17 X !!> E. S-adenosytmethionine (SAM) (6%)
18
19
20 Explanation: User Id: 388147
21 Epinephrine (01)
23
24
A
25
26
27
28
29
30
31
32 Cell Membrane
'
Phospholipase-c~
33 Phospholiplds
34 IP3 DAG
35
36
J.
0
E_R___c_a_·_~-1J---...
2
©
•ca><"---=:, 1
37
38 Protein Kinase-c
--
39
40 m
41 ~
1 A

2 Item: 22 of 46 II P Mark -<:I t>-


Q. Id : 6619 (388147] Previous Next
3
4
Explanation: User Id: 388147
5
6 Epinephrine (01)
7
8 A
9
10
11
12
13
14
15 \
16
Cell Membrane Phospholipase-c~
17
Phospholipids
18
IP3 DAG
19
20 J.0 0
21 _______,"----
ER __, ~ Ca,.~-,1

23 Protein Kinase-c
24
25 Phosphorylated proteins + - - - - - - - --'
26
27
28
l
Physiologic effects
29
30 The patient described in this vignette is most likely experiencing a brisk upper gastrointestinal hemorrhage as
31 evidenced by his coffee ground emesis and hypotension . In an effort to prevent circulatory failure due to
32 hypovolemia, the patient's sympathetic nervous system has increased its tone thereby causing
33 vasoconstriction and the sensation of anxiety. The two major mediators of the sympathetic nervous system
34 response are epinephrine and norepinephrine. These hormones I neurotransmitters function by stimulating
35 adrenergic cell surface receptors. The adrenergic receptor responsible for arteriolar contraction and
36 increasing blood pressure is the alpha-1 adrenergic receptor. The alpha-1 adrenergic receptor functions by
37 the inositol triphosphate (IP 3 ) second messenger system . In short, w hen a catecholamine binds the alpha-1
38
1 A

2 Item: 22 of 46 II P Mark -<:I t>-


Q. Id : 6619 (388147] Previous Next
3
4 The patient described in this vignette is most likely experiencing a brisk upper gastrointestinal hemorrhage as
5 evidenced by his coffee ground emesis and hypotension. In an effort to prevent circulatory failure due to
6 hypovolemia, the patient's sympathetic nervous system has increased its tone thereby causing
7 vasoconstriction and the sensation of anxiety. The two major mediators of the sympathetic nervous system
8 response are epinephrine and norepinephrine. These hormones I neurotransmitters function by stimulating
9 adrenergic cell surface receptors. The adrenergic receptor responsible for arteriolar contraction and
10 increasing blood pressure is the alpha-1 adrenergic receptor. The alpha-1 adrenergic receptor functions by
11 the inositol triphosphate (IP 3 ) second messenger system. In short, w hen a catecholamine binds the alpha-1
12
receptor, a Gq protein is stimulated that activates phospholipase C, w hich, in turn, releases IP 3 and
13
14 diacylglycerol (DAG) from membrane phospholipids. IP 3 liberates stored intracellular calcium resulting in
15 smooth muscle contraction w hile DAG activates protein kinase C.
16
17 (Choice A) Cyclic AMP is the second messenger formed follow ing stimulation of receptors linked to Gs
18 (stimulatory) proteins. Gs proteins activate adenylate cyclase. Adenylate cyclase cleaves ATP to form cyclic
19 AMP, w hich then activates protein kinase A for intracellular effects. This pathway is activated w ith stimulation
20 of beta-adrenergic receptors and several other hormones such as glucagon, PTH and others.
21
(Choice B) Cyclic GMP is the second messenger for atrial natriuretic peptide and nitric oxide (NO). These
23 ligands activate a specific G protein, w hich then activates guanylate cyclase resulting in formation of cGMP
24 from GTP. The intracellular effects are mediated via the activation of protein kinase G.
25
26 (Choice D) 2,3 BPG is a special byproduct of glycolysis produced in red blood cells, not in smooth muscle
27 cells. An increase in 2,3 BPG w ill increase the delivery of oxygen to the tissues (shift the hemoglobin oxygen
28 dissociation curve to the right).
29
30 (Choice E) SAM is produced by condensation of methionine w ith ATP. SAM functions as a methyl group
31 donor and is involved in several reactions requiring methyl transfer. In the adrenergic system, SAM is required
32 for the conversion of norepinephrine to epinephrine by the enzyme phenylethanolamine-N-methyltransferase
33 (PNMT). This enzymatic reaction predominantly occurs in adrenal medulla and is upregulated by cortisol.
34
35 Educational objective:
36 Stimulation of alpha-1 receptors by norepinephrine from the post-ganglionic sympathetic nerves w ill increase
37 IP 3 synthesis.
38
1 A

2 Item: 23 of 46 II P Mark -<:I t>-


Q. Id: 6513 (388147] Previous Next
3
4
5
6 A particular mouse lineage carries a human TNF-a transgene w ith a modified 3' region from a human globin
7 gene. As a result, unregulated human TNF-a expression is observed in various organs. Clinical
8 manifestations include symmetric polyarthritis and intestinal inflammation. The inflammatory condition in
9 these mice can be most specifically targeted by w hich of the follow ing?
10
11
A. Prednisone [9%]
12
13 ~ B. lnfliximab [79%]
14 x @ C. Tacrolimus [4%]
15
D. lmatinib [4%]
16
17 E. Methotrexate [4%]
18
19
Explanation: User Id: 388147
20
21
Tumor necrosis factor alpha (TNF-a) is an acute phase cytokine produced by activated macrophages. In low
22
concentrations, TNF-a mediates local inflammatory reactions by accelerating neutrophil migration into the
extravascular compartment, stimulating macrophage phagocytosis, and facilitating lymphocyte proliferation
24
25 and cytokine synthesis. In high concentrations, TNF-a is responsible for a range of systemic effects,
including fever, anorexia, circulation of corticotropin releasing hormone, acute phase response (with an
26
increase in serum C-reactive protein), septic shock, and cachexia.
27
28
Because TNF-a is a key mediator of inflammatory processes, it is an important treatment target for patients
29
w ith autoimmune conditions such as rheumatoid arthritis, Crohn's disease, ankylosing spondylitis, and
30
psoriasis. Anti-TNF-a agents include infliximab and etanercept. lnfliximab is a monoclonal antibody to TNF-a,
31
w hile etanercept is a recombinant TNF receptor fusion protein .
32
33
(Choice A) Prednisone reduces inflammation by suppressing polymorphonuclear leukocyte migration and
34
reversing increases in capillary permeability. It can be used as a nonspecific immunosuppressant in the
35
treatment of various conditions.
36
37
(Choice C) Tacrolimus suppresses T-lymphocyte activation by binding to FKBP-12, an intracellular protein . It
38
1 A

2 Item: 23 of 46 II P Mark -<:I t>-


Q. Id: 6513 (388147] Previous Next
3
4 D. lmatinib [4%]
5
E. Methotrexate [4%]
6
7
8 Explanation: User Id: 388147
9
10 Tumor necrosis factor alpha (TNF-a) is an acute phase cytokine produced by activated macrophages. In low
11 concentrations, TNF-a mediates local inflammatory reactions by accelerating neutrophil migration into the
12 extravascular compartment, stimulating macrophage phagocytosis, and facilitating lymphocyte proliferation
13 and cytokine synthesis. In high concentrations, TNF-a is responsible for a range of systemic effects,
14 including fever, anorexia, circulation of corticotropin releasing hormone, acute phase response (with an
15 increase in serum C-reactive protein), septic shock, and cachexia.
16
17 Because TNF-a is a key mediator of inflammatory processes, it is an important treatment target for patients
18 w ith autoimmune conditions such as rheumatoid arthritis, Crohn's disease, ankylosing spondylitis, and
19 psoriasis. Anti-TNF-a agents include infliximab and etanercept. lnfliximab is a monoclonal antibody to TNF-a,
20 w hile etanercept is a recombinant TNF receptor fusion protein .
21
22 (Choice A) Prednisone reduces inflammation by suppressing polymorphonuclear leukocyte migration and
reversing increases in capillary permeability. It can be used as a nonspecific immunosuppressant in the
24 treatment of various conditions.
25
26 (Choice C) Tacrolimus suppresses T-lymphocyte activation by binding to FKBP-12, an intracellular protein. It
27 is used in transplant patients and in the treatment of severe atopic dermatitis.
28
29 (Choice D) lmatinib inhibits Bcr-Abl tyrosine kinase, w hich is the abnormal gene product associated w ith
30 chronic m yeloid leukemia.
31
32 (Choice E) Methotrexate is a folate antimetabolite that prevents DNA synthesis, thereby targeting rapidly
33 proliferating cells.
34
35 Educational objective:
36 Anti-TNF-a agents include infliximab (a monoclonal antibody to TNF-a) and etanercept (a recombinant TNF
37 receptor fusion protein).
38
1 A

2 Item: 24 of 46 II P Mark -<:I t>-


Q. Id: 6515 (388147] Previous Next
3
4
5
6 A 63-year-old male complains of occasional squeezing chest pain w hile jogging or w alking uphill. The pain
7 subsides spontaneously w ithin one to two minutes after stopping. Prescribing low -dose aspirin (81 mg/day)
8 can reduce the risk of cardiovascular events in this patient by inhibiting w hich of the following?
9
10
A. Phospholipase ~ [11 %]
11
12 B. 5-lipoxygenase [3%]
13 C. 12-lipoxygenase [1 %]
14
15 D. Cyclooxygenase-1 [58%]
16 X @ E. Cyclooxygenase-2 [25%]
17
18
19 Explanation: User Id: 388147
20
21 The jogger described above appears to have stable angina, w hich is characterized by chest pain that
22 develops w ith exertion and resolves w ith rest. Because stable angina can progress to unstable angina or
23 m yocardial infarction (ie, acute coronary syndrome), management w ith medication such as aspirin may be
indicated.
25
26 Acute coronary syndrome is characterized by occlusion of the coronary vasculature, typically by a
27 thrombus. The thrombus arises secondary to increased platelet deposition and clotting factor activity at the
28 site of a disrupted atherosclerotic plaque. Aspirin is of benefit in countering thrombus formation because it
29 limits platelet aggregation. To accomplish this, aspirin inhibits prostaglandin synthesis by irreversibly
30 acetylating cyclooxygenase-1 (COX-1) and cyclooxygenase-2 (COX-2). Cyclooxygenase converts
31 arachidonic acid into PGG 2 and then PGH2 , w hich can then be metabolized into prostaglandins PGD 2 , PGE 2 ,
32 PGF 2 , PGl2 , or thromboxane ~- The latter is one of the more potent causes of platelet aggregation, w hich
33 explains aspirin's efficacy in reducing thrombus formation.
34
35 (Choices A, B, and C) In a step that can be inhibited by steroids, phospholipase ~converts membrane
36
phospholipids into arachidonic acid. Once formed, arachidonic acid can be transformed by 5-lipoxygenase
37
into 5-HPETE (the precursor to leukotrienes or lipoxins) or by cyclooxygenase into prostaglandins, as outlined
38
1 A

2 Item: 24 of 46 II P Mark -<:I t>-


Q. Id: 6515 (388147] Previous Next
3
4
5
Ix @ E. Cyclooxygenase-2 [25%]

6
7 Explanation: User Id: 388147
8
9 The jogger described above appears to have stable angina, w hich is characterized by chest pain that
10 develops w ith exertion and resolves w ith rest. Because stable angina can progress to unstable angina or
11 m yocardial infarction (ie, acute coronary syndrome), management w ith medication such as aspirin may be
12 indicated.
13
14 Acute coronary syndrome is characterized by occlusion of the coronary vasculature, typically by a
15 thrombus. The thrombus arises secondary to increased platelet deposition and clotting factor activity at the
16 site of a disrupted atherosclerotic plaque. Aspirin is of benefit in countering thrombus formation because it
17 limits platelet aggregation. To accomplish this, aspirin inhibits prostaglandin synthesis by irreversibly
18 acetylating cyclooxygenase-1 (COX-1) and cyclooxygenase-2 (COX-2). Cyclooxygenase converts
19 arachidonic acid into PGG 2 and then PGH 2 , w hich can then be metabolized into prostaglandins PGD 2 , PGE 2 ,
20 PGF 2 , PGl 2 , or thromboxane Ai· The latter is one of the more potent causes of platelet aggregation, w hich
21
explains aspirin's efficacy in reducing thrombus formation .
22
23
(Choices A, B, and C) In a step that can be inhibited by steroids, phospholipase A 2 converts membrane
25 phospholipids into arachidonic acid. Once formed, arachidonic acid can be transformed by 5-lipoxygenase
26
into 5-HPETE (the precursor to leukotrienes or lipoxins) or by cyclooxygenase into prostaglandins, as outlined
27
above. The enzyme 12-lipoxygenase mediates the formation of lipoxin A 4 and lipoxin 8 4 from 5-HPETE.
28
29 (Choice E) Present in platelets and most cells, cyclooxygenase-1 (COX-1) is a constitutive enzyme
30 responsible for the regulation of various cellular processes. In contrast, cyclooxygenase-2 (COX-2) is an
31 inducible enzyme formed in response to inflammatory stimuli. Traditional NSAIDs like aspirin are nonselective
32 COX inhibitors. COX-2 inhibitors like celecoxib, how ever, w ere designed to reduce the pain of inflammatory
33 conditions such as rheumatoid arthritis w hile causing minimal adverse effects (eg, gastroduodenal bleeding).
34
35 Educational objective:
36 Aspirin counters thrombus formation by irreversibly acetylating cyclooxygenase, thereby limiting production of
37 thromboxane Ai·
38
1 A

2 Item: 25 of 46 II P Mark -<:I t>-


Q. Id : 6519 (388147] Previous Next
3
4 A two-day-old neonate develops fever and irritability. Overnight, he has an episode of profound hypoglycemia.
5 As part of the urgent management, the physicians caring for him pass a catheter through the umbilical vein
6 into his right atrium. Through w hich structure must the catheter pass in order to reach the right atrium?
7
8
A. Ductus arteriosus [6%]
9
10 !l. Feramen evale [7q~l
11 .; C. 0bl€tbls venesbls [74q~]
12
13
x @ D. Portal vein [10%]
14 E. Azygous vein [3%]
15
16
17 Explanation: User Id: 388147
18
19 In utero, blood flow s from the placenta into the fetus via the umbilical vein . This blood then flow s to the fetal
20 liver w here it is shunted, in part, via the ductus venosus into the inferior vena cava . The ligamentum teres and
21 ligamentum venosum are the remnants of the umbilical vein and ductus venosus in the adult, respectively.
22
23 The procedure performed in this neonate is called umbilical vein catheterization . This procedure is a common
24 method of establishing central venous access in the neonate. The technique takes advantage of the fact that
complete closure of the ductus venosus does not occur until approximately one w eek of age, though this
26 structure is functionally closed w ithin hours of birth.
27
28 (Choice A) In the fetal circulation, the ductus arteriosus shunts blood from the pulmonary artery to the
29 proximal descending aorta . After birth, decreased local prostaglandin production in the endothelium of the
30 ductus arteriosus causes its closure.
31
32 (Choice B) In the fetus, the foramen ovale allow s blood to pass from the right atrium directly into the left
33 atrium, bypassing the lungs. Lung inflation at the time of birth decreases the right heart pressure relative to
34 the left heart pressure, causing closure of the foramen ovale. In most adults, the foramen ovale is totally
35 obliterated, w ith the fossa ovale as its remnant.
36
37 (Choice D) In the adult, the portal vein delivers blood from the gut to the hepatic sinusoids. The sinusoids
38 deliver blood to the hepatic vein, w hich drains into the IVC. In the fetus, the ductus venosus bypasses the
1 A

2 Item: 25 of 46 II P Mark -<:I t>-


Q. Id : 6519 (388147] Previous Next
3
4 E. Azygous vein [3%]
5
6
7 Explanation: User Id: 388147
8
9 In utero, blood flow s from the placenta into the fetus via the umbilical vein. This blood then flow s to the fetal
10 liver w here it is shunted, in part, via the ductus venosus into the inferior vena cava. The ligamentum teres and
11 ligamentum venosum are the remnants of the umbilical vein and ductus venosus in the adult, respectively.
12
13 The procedure performed in this neonate is called umbilical vein catheterization . This procedure is a common
14 method of establishing central venous access in the neonate. The technique takes advantage of the fact that
15 complete closure of the ductus venosus does not occur until approximately one w eek of age, though this
16 structure is functionally closed w ithin hours of birth .
17
18 (Choice A) In the fetal circulation, the ductus arteriosus shunts blood from the pulmonary artery to the
19 proximal descending aorta . After birth, decreased local prostaglandin production in the endothelium of the
20 ductus arteriosus causes its closure.
21
22 (Choice B) In the fetus, the foramen ovale allow s blood to pass from the right atrium directly into the left
23 atrium, bypassing the lungs. Lung inflation at the time of birth decreases the right heart pressure relative to
24 the left heart pressure, causing closure of the foramen ovale. In most adults, the foramen ovale is totally
obliterated, w ith the fossa ovale as its remnant.
26
27 (Choice D) In the adult, the portal vein delivers blood from the gut to the hepatic sinusoids. The sinusoids
28 deliver blood to the hepatic vein, w hich drains into the IVC. In the fetus, the ductus venosus bypasses the
29 sinusoids and drains directly into the IVC.
30
31 (Choice E) The azygous vein drains blood from the posterior w alls of the thorax and abdomen into the SVC.
32
33 Educational objective:
34 In the fetal circulation, blood from the placenta flow s through the umbilical vein into the liver. At the liver, blood
35 may either be delivered to the hepatic sinusoids or diverted to the IVC via the ductus venosus.
36
37 Time Spent: 58 seconds 74% answ ered correctly. Last updated: [3/27/2010].
38 Copyright © USMLEWORLD,LLC. SimExam ver. 2011 .11.397425
1 •
2 Item: 26 of 46 • \' Mark -<:J C>
Q. Id: 6525 (383147) Previous NeHt
3
4
A 53-year-old male who recently emigrated from Russia is being evaluated for persistent cough with
5
occasional hemoptysis. His past medical history is significant for tuberculosis and chronic bronchitis. He has
6
smoked cigarettes for the past 20 years. PPD testing yields 15 mm of indurated erythema. Sputum
7
microscopy stained with methenamine silver is shown belo\Y.
8
9
10
11
12
13
14
15
16
17
18
19 '°
20
21
22
23
24
25

27
28
29 Dl1plcyed witt. ptrmit1lon from Sprlqu H1altbcu. Ltd,
0 I OSIO, l 00'4 by Cunut Mtdicillt
30
31
32 Which of the following is the most likely cause of his current symptoms?
33
34
X r~ A. Mycobacterium tuberculosis [7%]
35
36 B. Candida albicans [6%J
37 ., C. Asperg1//us fumigatus (65%]
38
1 A

2 Item: 26 of 46 II P Mark -<:I t>-


Q. Id : 6525 (388147] Previous Next
3
4
5 D. Pneumocystis jiroveci (19%]
6 E. Actinomyces israefii [4%]
7
8
9 Explanation: User Id: 388147
10
11 Methenamine silver is a stain commonly used to identify fungal elements in histologic sections of tissue.
12 Aspergillus is fungus that is ubiquitous in the environment w orldwide. These organisms have a characteristic
13 microscopic morphology, w ith septate hyphae out of w hich asexual fruiting structures know n as
14 conidiophores project. Conidiophores each have a terminal vesicle w ith outwardly radiating phialides and
15 conidiospores attached, giving a "broom-like" appearance. Patients w ith a history of tuberculosis may develop
16 aspergillus colonization of lung cavities formed previously by tuberculosis infection. Multiplication of
17 aspergillus hyphal elements w ithin these cavities leads to formation of 'lungus balls," w hich can cause
18 occasional hemoptysis. Treatment is surgical excision of the fungal mass.
19
20 (Choice A) Reactivation of latent Mycobacterium tuberculosis infection may cause occasional hemoptysis,
21 but there would be acid-fast (pink) rods on histologic section.
22
23 (Choice B) Candida albicans is a yeast that exhibits pseudohyphae on microscopic examination . It may
24 cause cutaneous, oral or vaginal infections in immunocompetent hosts. W idespread disease occurs only in
25 the immunocompromised.

27 (Choice D) Pneumocystis jiroveci is a ubiquitous organism know n to cause opportunistic pneumonia in


28 HIV-positive patients w ith CD4 cell counts less than 200/ml 3 · On silver staining, cup- or disc-shaped
29 organisms are seen .
30
31 (Choice E) Actinomyces israefii is a Gram positive bacterium that classically causes mouth or GI tract
32 abscesses follow ing trauma . Microscopic examination of fluid drained from infected sites typically exhibits
33 yellow "sulfur" granules.
34
35 Educational objective:
36 Aspergillus is a ubiquitous fungal organism w hose reproductive forms classically exhibit a "broom-like"
37 structure on silver staining. Patients w ith a history of tuberculosis are prone to development of aspergillomas
38 ("fungus balls") in lung cavities.
1 A

2 Item: 27 of 46 II P Mark -<:I t>-


Q. Id : 6528 (388147] Previous Next
3
4
5
6 A colony of bacteria resistant to gentamicin and a colony of bacteria resistant to ampicillin are mixed on a
7 plate containing both antibiotics. Bacterial growth is noted. The same experiment is then repeated w ith the
8 addition of DNAse to the plate. Now no bacterial growth is observed . W hat is the most likely mechanism of
9 bacterial survival in the first experiment?
10
11
A. New mutation [4%]
12
13 ~ @ B. Transformation [46%]
14 C. Conjugation [33%]
15
D. Transduction [11 %]
16
17 E. Transposons [7%]
18
19
Explanation: User Id: 388147
20
21
There are three main mechanisms of genetic transfer between bacteria: transformation, conjugation and
22
transduction. Transformation is a process by w hich bacteria take up naked DNA from the environment and
23
incorporate it into their genomes. Bacteria that have the ability to take up foreign DNA and incorporate it into
24
their genome are referred to as competent. Examples of bacteria know n to have this ability in nature include
25
Haemophilus, Streptococcus, Bacillus and Neisseria. In the first experiment described here, transformation
26
occurs w hen genes for antibiotic resistance from one bacterial colony are taken up by the other. In the
second experiment, DNAse degrades naked DNA in the culture medium, preventing transformation.
28
29
(Choice A) W hile new mutations can theoretically lead to antibiotic resistance, the changes imparted by the
30
addition of DNAse above are more consistent w ith transformation as the mechanism of resistance here.
31
32
(Choice C) Conjugation is a process w here DNA is passed from one bacterium to another by direct cell-cell
33
34 interaction, such as w ith a sex pilus. Conjugation is an important method through w hich bacteria exchange
genetic material. How ever, the genetic material exchanged would not be subject to DNAse degradation.
35
36
(Choice D) Transduction is bacteriophage-mediated transfer of genetic information. During phage replication,
37
random bacterial DNA or bacterial DNA adjacent to the site of phage DNA insertion may be packaged into
38
1 A

2 Item: 27 of 46 II P Mark -<:I t>-


Q. Id : 6528 (388147] Previous Next
3
4 u. 1 rar1~uuc.uu11L'1 7oJ

5 E. Transposons [7%]
6
7
8 Explanation: User Id: 388147
9
10 There are three main mechanisms of genetic transfer between bacteria: transformation, conjugation and
11 transduction. Transformation is a process by w hich bacteria take up naked DNA from the environment and
12 incorporate it into their genomes. Bacteria that have the ability to take up foreign DNA and incorporate it into
13 their genome are referred to as competent. Examples of bacteria know n to have this ability in nature include
14 Haemophifus, Streptococcus, Bacillus and Neisseria. In the first experiment described here, transformation
15 occurs w hen genes for antibiotic resistance from one bacterial colony are taken up by the other. In the
16 second experiment, DNAse degrades naked DNA in the culture medium, preventing transformation.
17
18 (Choice A) W hile new mutations can theoretically lead to antibiotic resistance, the changes imparted by the
19 addition of DNAse above are more consistent w ith transformation as the mechanism of resistance here.
20
21 (Choice C) Conjugation is a process w here DNA is passed from one bacterium to another by direct cell-cell
22 interaction, such as w ith a sex pilus. Conjugation is an important method through w hich bacteria exchange
23 genetic material. How ever, the genetic material exchanged would not be subject to DNAse degradation.
24
25 (Choice D) Transduction is bacteriophage-mediated transfer of genetic information. During phage replication,
26 random bacterial DNA or bacterial DNA adjacent to the site of phage DNA insertion may be packaged into
daughter bacteriophages. Infection of other bacteria by these daughter phages can result in the transmission
28 of bacterial genes such as antibiotic resistance genes. DNAse would not inhibit this process.
29
30 (Choice E) Transposons mediate the transfer of genetic information w ithin a bacterium from one location in
31 the genome to another, for example, from a plasmid to chromosomal DNA or vice versa . This process is not
32 directly responsible for organism to organism genetic transfer, but moving a gene from chromosome to
33 plasmid readies it for conjugative transfer. Regardless, this process would not be subject to DNAse inhibition.
34
35 Educational objective:
36 Transformation is the process by w hich bacteria take up naked DNA from their environment and incorporate it
37 into their genomes.
38
1 A

2 Item: 28 of 46 II P Mark -<:I t>-


Q. Id : 6534 (388147] Previous Next
3
4 A 39-year-old male complains of facial r:iuffiness in the morning. He is HIV posifve. Laboratory evaluation
5 reveals proteinuria 3.5 g/day). Renal biopsy show s gl~rular e~helial cell enlargement and vacuolization,
6 as w ell as varying degrees of glomerular capillary w all collapse. Many renal tubules show cystic dilation and
7 are filled w ith r:iroteinaceous material This patient most likely suffers from w hich of the follow ing?
8
9
10 A. Anti-GBM disease [2%]
11 B. ANCA-associated rapidly progressive glomerulonephritis [4%]
12 C. MemeraneYs glemerYleneflhritis [1 1%•]
13
14 D. Memeranefjreliferative glemerYleneflhritis [17%•]
15 E. Postinfectious glomerulonephritis [4% ]
16 F. Berger disease [3%]
17
., @ G. Focal segmental glomerulosclerosis [58%]
18
19 H. Minimal change disease [1%]
20
21
22 Explanation: User Id: 388147
23
24 Focal segmental glomerulosclerosis (FSGS) is one of the more common causes of nephrotic syndrome in
25 adults and children. FSGS can occur in the setting of several different circumstances, and is quite often seen
26 in association w ith conditions such as HIV infection (HIV-associated nephropathy), heroin addiction, and sickle
27 cell disease.

29 In FSGS, only some glomeruli are collapsed and sclerotic; w ithin the affected glomeruli, only some segments
30 of the capillary tuft are involved . The sclerotic segments feature basement membrane collapse, increased
31 matrix production, and hyalinosis. One variant of FSGS w ith a particularly poor prognosis is collapsing
32 glomerulopathy, w hich is considered characteristic of HIV-associated nephropathy. In collapsing
33 glomerulopathy, the usual FSGS lesions are seen in addition to the collapse and sclerosis of the w hole
34 glomerular tuft. The glomerular epithelial cells tend to proliferate and hypertrophy, and marked tubular injury
35 w ith accompanying microcyst formation is seen (as in this patient).
36
37 (Choices A and B) Anti-GBM disease is a form of rapidly progressive glomerulonephritis (RPGN)
38 characterized by linear deposits of lgG (and often C3) in the glomerular basement membrane (GBM).
' . ' ' ,.
39
40
1 A

2 Item: 28 of 46 II P Mark -<:I t>-


Q. Id : 6534 (388147] Previous Next
3
4
5
6 In FSGS, only some glomeruli are collapsed and sclerotic; w ithin the affected glomeruli, only some segments
7
of the capillary tuft are involved . The sclerotic segments feature basement membrane collapse, increased
8
matrix production, and hyalinosis. One variant of FSGS w ith a particularly poor prognosis is collapsing
9
glomerulopathy, w hich is considered characteristic of HIV-associated nephropathy. In collapsing
10
glomerulopathy, the usual FSGS lesions are seen in addition to the collapse and sclerosis of the w hole
11
glomerular tuft. The glomerular epithelial cells tend to proliferate and hypertrophy, and marked tubular injury
12
w ith accompanying microcyst formation is seen (as in this patient).
13
14 (Choices A and B) Anti-GBM disease is a form of rapidly progressive glomerulonephritis (RPGN)
15
characterized by linear deposits of lgG (and often C3) in the glomerular basement membrane (GBM).
16
ANCA-associated rapidly progressive glomerulonephritis is a form of RPGN characterized by the presence of
17
serum antineutrophil cytoplasmic antibodies. In both cases, the glomerular tufts collapse and
18
crescent-shaped accumulations of proliferating cells and leukocytes appear w ithin the Bow man capsule.
19
20 (Choices C, D, and E) Membranous glomerulonephritis is characterized by diffuse glomerular capillary w all
21
thickening w ith spiky subepithelial lgG deposits. Membranoproliferative glomerulonephritis is characterized by
22 enlarged, hypercellular glomeruli accompanied by infiltration of neutrophils and monocytes. Increased
23 mesangial cellularity leads to capillary basement membrane thickening . Granular deposits of lgG and/or C3
24 are present. Postinfectious glomerulonephritis is characterized by diffuse hypercellularity of the glomeruli w ith
25 infiltration by polymorphonuclear leukocytes and monocytes. Diffuse granular deposits of lgG and C3 are
26
seen along the glomerular capillary w all and mesangium.
27
(Choice F) Berger disease (lgA nephropathy) is characterized by diffuse mesangial proliferation and
29 expansion of the extracellular matrix . Granular lgA deposits are present in the mesangium and capillary w alls.
30
31
(Choice H) Minimal change disease, by definition, is characterized by relatively normal glomeruli. In children
32 prone to relapsing minimal change disease, some small, sclerotic, involuted glomeruli may be identified.
33
34 Educational objective:
35
Collapsing glomerulopathy is a variant of focal segmental glomerulosclerosis characteristic of HIV-associated
36 nephropathy. The usual FSGS lesions are seen in addition to collapse and sclerosis of the w hole glomerular
37
tuft, glomerular epithelial cell proliferation and hypertrophy, and marked tubular injury w ith accompanying
38 microcyst formation .
1 •
2 Item: 29 of 46 • \' Mark -<:J C>
Q. Id: 6536 (383147) Previous NeHt
3
4
A 46-year-old hospitalized patient develops dyspnea. His chest x-ray is shown belo\Y.
5
6
7
8
9
10
11
12
13
14
15
16
17
18
19 '°
20
21
22
23
24
25
26
27
28

30 Which of the following is most likely to relieve this patient's symptoms?


31
32 A. Antibiotics [12o/o)
33 B. Diuretics and inotropes [33%]
34
35 X @ C. Bronchodilators [13%]
36 v D. Chest tube placement [28%)
37 E. Positive pressure ventilation [1 3%]
38
1 A

2 Item: 29 of 46 II P Mark -<:I t>-


Q. Id : 6536 (388147] Previous Next
3
4
5
6
7
8
9
10
11
12
13
14
15
16
17
18
19 -
20
21
22
23
The diagnosis of pneumothorax is confirmed by the presence of a w hite visceral pleural line on chest
24
radiograph (as seen on the patient's right in the image above}. This line is typically straight or convex tow ard
25
the chest w all, and indicates separation from the parietal pleura by a pocket of gas. Pulmonary vessels are
26
usually not visible beyond the visceral pleural boundary. Frequently, the heart and trachea w ill move aw ay
27
from the affected side.
28
Physical findings of pneumothorax include a decrease in chest excursion, diminished breath sounds, and
30
hyperresonant percussion on the affected side. Hypoxemia is also commonly present, as the poorly
31
ventilated regions of the lung remain \'lell perfused. Initial treatment must be focused on removing air from the
32
pleural space to allow the lung to re-inflate. Effective options for unstable patients include needle aspiration or
33
chest tube placement.
34
35
(Choice A, B, C, and E) Antibiotics would be appropriate treatment for bacterial pneumonia . Diuretics and
36
inotropes can be used to manage pulmonary edema secondary to congestive heart failure. Bronchodilators
37
can resolve acute exacerbations of pulmonary conditions such as asthma . Positive pressure ventilation is of
38
1 A

2 Item: 29 of 46 II P Mark -<:I t>-


Q. Id : 6536 (388147] Previous Next
3
4
5
6
7
8
9
10
11
12
13
14
15
16 The diagnosis of pneumothorax is confirmed by the presence of a w hite visceral pleural line on chest
17 radiograph (as seen on the patient's right in the image above). This line is typically straight or convex tow ard
18 the chest w all, and indicates separation from the parietal pleura by a pocket of gas. Pulmonary vessels are
19 usually not visible beyond the visceral pleural boundary. Frequently, the heart and trachea w ill move aw ay
20 from the affected side.
21
22 Physical findings of pneumothorax include a decrease in chest excursion, diminished breath sounds, and
23 hyperresonant percussion on the affected side. Hypoxemia is also commonly present, as the poorly
24 ventilated regions of the lung remain w ell perfused. Initial treatment must be focused on removing air from the
25 pleural space to allow the lung to re-inflate. Effective options for unstable patients include needle aspiration or
26 chest tube placement.
27
28 (Choice A, B, C, and E) Antibiotics would be appropriate treatment for bacterial pneumonia . Diuretics and
inotropes can be used to manage pulmonary edema secondary to congestive heart failure. Bronchodilators
30 can resolve acute exacerbations of pulmonary conditions such as asthma . Positive pressure ventilation is of
31 help in treating acute respiratory distress syndrome (ARDS). None of these therapeutic regimens is likely to
32 relieve this patient's symptoms; in fact, positive pressure ventilation is a know n cause of pneumothorax .
33
34 Educational objective:
35 Tension pneumothorax is treated w ith chest tube placement.
36
37 Time Spent: 38 seconds 28% answ ered correctly. Last updated: [11/ 10/2011].
38 Copyright © USMLEWORLD,LLC. SimExam ver. 2011 .11.397425
1 A

2 Item: 30 of 46 II P Mark -<:I t>-


Q. Id : 6543 (388147] Previous Next
3
4
5
6 A young female has severely hypoi:ilasti!; gre~es and thumbs, as w ell as a bicornuate uterus and ectopic
7 ureteral openings that w ere surgically corrected early in life. Mutation of w hich of the follow ing genes is most
8 likely responsible for these findings?
9
10
11 A. HoxA-13 [54%]
12 B. Pax-3 [26%]
13
14 x @ C. VHL [6%]
15 D. erbB2 [9%]
16
17 E. p53 [4%]
18
19
20 Explanation: User Id: 388147
21
22 Homeobox genes are a highly conserved set of genes found in animals, fungi and plants. These genes code
23 for protein transcription factors know n as homeodomain proteins. Homeodomain proteins modulate
24 expression of other genes in the cell. Via homeodomain proteins, homeobox genes guide development from
25 the earliest stages of embryogenesis to the final differentiation of cells. HoxA-13 is a homeobox gene mutated
26 in hand-foot-genital (HFG) syndrome, a condition characterized by clinodactyly, shortened thumbs, small feet,
27 short great toes and urinary-tract abnormalities. Genitourinary abnormalities include duplications of the
28 reproductive tract in women and hypospadias in men.
29
(Choice B) Pax-3 mutations are associated w ith W aardenburg syndrome, a condition characterized by
31 heterochromia irides, poliosis, dystopia canthorum and deafness.
32
33 (Choice C) The VHL gene is mutated in Von-Hippel-Lindau syndrome, w hich is characterized by a
34 predisposition to retinal, cerebellar, and spinal hemangioblastomas, as w ell as to renal cell carcinoma,
35 pheochromocytoma, and pancreatic tumors.
36
37 (Choice D) erbB2 (HER2/neu) is an epidermal growth factor receptor that is overexpressed in breast cancer.
38
1 A

2 Item: 30 of 46 II P Mark -<:I t>-


Q. Id : 6543 (388147] Previous Next
3
4
5
x @ C. VHL [6%]
6 D. erbB2 [9%]
7
E. p53 [4%]
8
9
10
11 Explanation: User Id: 388147
12
13 Homeobox genes are a highly conserved set of genes found in animals, fungi and plants. These genes code
14 for protein transcription factors know n as homeodomain proteins. Homeodomain proteins modulate
15 expression of other genes in the cell. Via homeodomain proteins, homeobox genes guide development from
16 the earliest stages of embryogenesis to the final differentiation of cells. HoxA-13 is a homeobox gene mutated
17 in hand-foot-genital (HFG) syndrome, a condition characterized by clinodactyly, shortened thumbs, small feet,
18 short great toes and urinary-tract abnormalities. Genitourinary abnormalities include duplications of the
19 reproductive tract in women and hypospadias in men.
20
21 (Choice B) Pax-3 mutations are associated w ith W aardenburg syndrome, a condition characterized by
22 heterochromia irides, poliosis, dystopia canthorum and deafness.
23
24 (Choice C) The VHL gene is mutated in Von-Hippel-Lindau syndrome, w hich is characterized by a
25 predisposition to retinal, cerebellar, and spinal hemangioblastomas, as w ell as to renal cell carcinoma,
26 pheochromocytoma, and pancreatic tumors.
27
28 (Choice D) erbB2 (HER2/neu) is an epidermal growth factor receptor that is overexpressed in breast cancer.
29
(Choice E) p53 is a tumor suppressor gene mutated in Li-Fraumeni syndrome. This syndrome is associated
31 w ith development of breast cancer and sarcomas at a young age.
32
33 Educational objective:
34 Homeobox genes function in guiding morphogenesis. Mutations in these genes cause structural defects. The
35 Hox genes are a family of homeobox genes. Mutations in HoxA-13 cause hand-foot-genital (HFG) syndrome.
36
37 Time Spent: 65 seconds 54% answ ered correctly. Last updated: [3/27/201 O].
38 Copyright © USMLEWORLD,LLC. SimExam ver. 2011 .11.397425
1 A

2 Item: 31of46 ll P Mark -<:I t>-


Q. Id : 6548 (388147] Previous Next
3
4
5 A young w oman complaining of pol~uria and polydil;)sia is diagnosed w ith nephrogenic diabetes insipidus.
6 W hich of the follow ing is the most likely reaction to w ater deprivation in this patient?
7
8 A. Increases blrine esmelarity [18%•]
9
.; B. Increases serblm Yase13ressin [6Q%,]
10
11 X @ C. Decreased serum osmolality [9%]
12 D. Decreased serum aldosterone [2%]
13
14
E. Decreased plasma renin [1 %]
15
16
Explanation: User Id: 388147
17
18 Vasopressin, or antidiuretic hormone (ADH), is the primary physiologic moderator of free w ater excretion in
19 humans. ADH is secreted secondary to hyperosmolality and depletion of the effective circulating volume (ie,
20 w ater deprivation). Normally, the w ater retention triggered by ADH causes a low ering of the plasma osmolality
21 and an increase in the volume of extracellular fluid.
22
23 ADH accomplishes the antidiuretic response by enhancing the w ater permeability of the principal cells along
24 the medullary portion of the collecting duct. The w ater is reabsorbed through new ly-created w ater channels
25 that span the luminal membrane to allow for osmotic equilibration w ith the surrounding hypertonic interstitium.
26 As a result, ADH quickly replenishes the w ater in systemic circulation . Once ADH w ears off, the w ater
27 channels accumulate w ithin pits along the luminal membrane to be returned to the cytoplasm through
28 endocytosis. If there is a defect at any step in this process (eg, poor w ater channel functionality or failure of
29 ADH to attach to the receptors), resistance to the high levels of circulating ADH occurs, follow ed by increased
30 urine output. This syndrome is called nephrogenic diabetes insipidus.

32 (Choices A, C, D, and E) In an individual w ith nephrogenic diabetes insipidus, urine osmolarity w ould fall and
33 serum osmolality w ould rise because of the increase in urinary excretion of w ater. Serum aldosterone and
34 plasma renin levels w ould also rise in an effort to increase w ater retention.
35
36 Educational objective:
37 Nephrogenic diabetes insipidus is associated w ith increased serum vasopressin levels and vasopressin
38 rP.~i~trinr:P. in thP. rP.nril r:ollP.r:tinn rh 1r:t~
39
40
1 A

2 Item: 32 of 46 II P Mark -<:I t>-


Q. Id : 6549 (388147] Previous Next
3
4
A new antidiabetic agent acts by activating iosulin-res110Dsiye genes in target tissues. It seems to increase
5
the synthesis and translocation of GLUT-4 in adipose tissue. The new agent is most similar to w hich of the
6
follow ing?
7
8
9 A. Repaglinide [8%]
10 X @ B. Glyburide [ 15%]
11
C. Acarbose [3%]
12
13 D. Pioglitazone [70%]
14 E . Glucagon-like peptide 1 [4%]
15
16
17 Explanation: User Id: 388147
18
19 The low ering of blood glucose through improvement of target cell response is accomplished by two classes of
20 oral hypoglycemic drugs: thiazolidinediones and biguanides. A prime example of the thiazolidinediones is
21 pioglitazone, w hich is an agonist for the peroxisome proliferator-activated receptor gamma (PPAR-y). PPAR-y
22 is primarily expressed in the liver, heart, skeletal muscle, and vessel w alls. Activation of the PPAR-y
23 receptors in adipose tissue cell nuclei is thought to increase the synthesis and translocation of GLUT-4, w hich
24 is an insulin-regulated glucose transporter. The rates of muscle glycogen synthesis and glucose oxidation
25 increase in response to the presence of GLUT-4.
26
27 (Choices A, B, C, and E) Repaglinide, glyburide, acarbose, and glucagon-like peptide 1 are not members of
28 the thiazolidinedione class. Repaglinide is a nonsulfonylurea hypoglycemic agent in the meglitinide class that
29 stimulates insulin release from beta cells. Glyburide is a sulfonylurea that stimulates insulin release from beta
30 cells. Acarbose is a competitive inhibitor of pancreatic a-amylase and intestinal brush border a-glucosidase,
31 thereby delaying carbohydrate hydrolysis and glucose absorption. Glucagon-like peptide 1 stimulates insulin
release from pancreatic islets, slow s gastric emptying, and prevents inappropriate glucagon release.
33
34 Educational objective:
35 Pioglitazone is an oral hypoglycemic agent and member of the thiazolidinedione class. It activates the
36 peroxisome proliferator-activated receptor gamma (PPAR-y), thereby increasing the synthesis and
37 translocation of the glucose transporter GLUT-4.
38
1 A

2 Item: 33 of 46 II P Mark -<:I t>-


Q. Id : 6558 (388147] Previous Next
3
4
A 45-year-old female follows a special weight-loss diet and exercises regularly. Her mother exj'.1erienced an
5
osteo[)Of,Otif hip fracture and was bedridden for some time. The patient takes !J.igtJ__doses_oLvitamin..Q, in an
6
effort to prevent fracturing her ow!J.bip·. Which of the following blood findings would be most expected in this
7
patient?
8
9 Calcium Phosphate Parathyroid Hormone
10
11 X @ A. t ! ! [18%]
12
13 B. t t ! [73%]
14
15 G. t t t f0%f
16
17 Q.. t t t f&%f
18
19 - €-. t t t f+%l-
20
21
22 Explanation: User Id: 388147
23
24 The major physiological effects of 1,25-dihydroxycholecaliferol (the most active form of vitamin D) include:
25
• increased intestinal absorption of orally ingested calcium and phosphate
26
• enhanced bone resorption and release of bone calcium and phosphate stores when the serum level of
27
one or both of these ions is deficient
28
29
30 In the setting of adequate oral calcium and phosphate intake, the primary effect of vitamin D consumption is to
31 increase intestinal absorption of calcium and phosphate, thereby raising serum calcium and phosphate
32 levels. An increase in serum calcium suppresses parathyroid hormone (PTH) release through feedback
inhibition.
34
35
Educational objective:
36 1,25-dihydroxycholecaliferol (active vitamin D) causes increased intestinal absorption of both calcium and
37
phosphate. The resulting rise in serum calcium levels suppresses parathyroid hormone release.
38
1 A

2 Item: 34 of 46 II P Mark -<:I t>-


Q. Id : 6559 (388147] Previous Next
3
4 A 56-year-old male is brought to the emergency department w ith recent onset lethargy, confusion, and
5 vomiting. He had been depressed for several months after the death of his w ife, but has never reported
6 suicidal ideation. Laboratory findings include the follow ing:
7
Arterial pH ~.22
8
9 p02 ~
95 mm Hg
pC02 mm Hg
10
Anion gap
11
12
Several hours later, he develops back pain, hematuria, and oliguria. Consumption of w hich of the follow ing
13
14
substances is most likely responsible for this patient's symptoms?
15
16 A. Amitriptyline [13%]
17 B. Ethanol [ 10%]
18
~ @ C. Ethylene glycol [75%]
19
20 D. Carbon monoxide [ 1% ]
21 E. Iron [1 %]
22
23
24 Explanation: User Id: 388147
25
26 The laboratory findings in this patient are demonstrative of an anion gap metabolic acidosis. (The anion gap
27 equals [Na•]- ([CI-] + [HC0 3 - ]) and normally measures 12-16 mEq/L; w hen the anion gap;:; 30 mEq/L,
28 unaccounted for acidic anions are in circulation.) The triad of sudden onset back pain, hematuria, and oliguria
29 in an individual w ith an anion gap metabolic acidosis is suggestive of ethylene glycol poisoning. Metabolites of
30 ethylene glycol are markedly toxic and know n to cause calcium oxalate crystal deposition in renal tubules
31
leading to renal failure.
32
33
(Choices A, B, D, and E) Tricyclic antidepressant toxicity is not classically associated w ith an anion gap
metabolic acidosis; rather, QRS prolongation, anticholinergic effects, agitation, seizures, and coma are
seen . Ethanol intoxication can cause ketoacidosis, w hich could elevate the anion gap. Other toxic effects of
ethanol use include CNS depression, nausea, and pancreatitis. Carbon monoxide poisoning can cause CNS
37
depression, nausea and vomiting, rhabdomyolysis, and m yalgia . Iron overdose can result in a non-anion gap
38 In fhp ::::ihc:::pnr P nf
39
40
1 A

2 Item: 34 of 46 II P Mark -<:I t>-


Q. Id : 6559 (388147] Previous Next
3
4 Several hours later, he develops back pain, hematuria, and oliguria. Consumption of w hich of the following
5 substances is most likely responsible for this patient's symptoms?
6
7
8 A. Amitriptyline [ 13%]
9 B. Ethanol [ 10%]
10 .; @ C. Ethylene glycol [75%]
11
12 D. Carbon monoxide [ 1%]
13 E. Iron [1 %]
14
15
16 Explanation: User Id: 388147
17
18 The laboratory findings in this patient are demonstrative of an anion gap metabolic acidosis. (The anion gap
19 equals [Na•] - ([Cl-] + [HC0 3 -]) and normally measures 12-16 mEq/L; w hen the anion gap ;:; 30 mEq/L,
20 unaccounted for acidic anions are in circulation.) The triad of sudden onset back pain, hematuria, and oliguria
21 in an individual w ith an anion gap metabolic acidosis is suggestive of ethylene glycol poisoning. Metabolites of
22 ethylene glycol are markedly toxic and know n to cause calcium oxalate crystal deposition in renal tubules
23 leading to renal failure.
24
25 (Choices A, B, D, and E) Tricyclic antidepressant toxicity is not classically associated w ith an anion gap
26 metabolic acidosis; rather, QRS prolongation, anticholinergic effects, agitation, seizures, and coma are
27 seen. Ethanol intoxication can cause ketoacidosis, w hich could elevate the anion gap. Other toxic effects of
28 ethanol use include CNS depression, nausea, and pancreatitis. Carbon monoxide poisoning can cause CNS
29 depression, nausea and vomiting, rhabdomyolysis, and m yalgia . Iron overdose can result in a non-anion gap
30 metabolic acidosis secondary to hydration of the non-bound ferric ion and proton release. In the absence of
31 cardiovascular collapse, oliguria would not result from ingestion of any of these substances.
32
33 Educational objective:
The triad of sudden onset back pain, hematuria, and oliguria in an individual w ith an anion gap metabolic
acidosis is suggestive of ethylene glycol poisoning.

37 Time Spent: 73 seconds Last updated: [3/27/201 OJ.


38
1 A

2 Item: 35 of 46 II P Mark -<:I t>-


Q. Id : 6563 (388147] Previous Next
3
4
Polymerase chain reaction is used to amplify Chlamydia trachomatis DNA fragments present in tissue
5
samples. DNA polymerase obtained from Thermus aquaticus is used in the reaction . The procedure involves
6
initial heating of the solution to 95°C (203°F) follow ed by cooling the solution to 55°C (131°F). Immediately
7
upon cooling, w hich of the following happens?
8
9
10 A. Double-strand DNA denaturation [5%]
11 B. Primer binding to single-strand DNA [57%]
12
13
C. Primer extension by DNA polymerase [7%]
14 D. DNA polymerase inactivation [5%]
15 x @ E. Double-strand DNA renaturation [26%]
16
17
18 Explanation: User Id: 388147
19
20 Most polymerase chain reaction (PCR) methods use thermal cycling, w hich amplifies DNA through the
21 alternate heating and cooling of a DNA sample. In the standard first step of thermal cycling PCR, heating of
22 the sample to 94-96°C denatures the DNA fragments present by disrupting the hydrogen bonds between
23 complementary bases of DNA strands. Two separate, single strands of the sample DNA are generated in the
24 denaturation process. The solution is then cooled to 50-65°C, allow ing the pre-made DNA primers (which
25 have been added in excess to the reaction mixture) to anneal to specific sequences on the single strands of
26 sample DNA. The reaction mixture is rew armed to 72°C to enable the heat-stable Taq (a name derived from
27 Thermus aquaticus) DNA polymerase to synthesize a new complementary DNA strand in the 5' to 3'
28 direction. Repetition of these steps exponentially amplifies the original tissue sample DNA.
29
30 (Choice A) Double-strand DNA denaturation occurs during the initial heating step of PCR.
31
32 (Choice C) Primer extension by DNA polymerase occurs during the rew arming step (after the primer binding
33 to single-strand DNA) of PCR.
34
(Choice D) Thermus aquaticus is a thermophilic bacterium that resides in hot springs and hydrothermal
vents. It produces an enzyme, Taq polymerase, that can tolerate the high temperatures used in PCR. Taq
37 polymerase functions best at 75-80°C, but is not inactivated by exposure to 55°C.
38
1 A

2 Item: 35 of 46 II P Mark -<:I t>-


Q. Id : 6563 (388147] Previous Next
3
1-\.. U UUUlt-~ll Cll IU U l 'lll-\ Utl ICllUI CIUUI I L\.J 70J
4
5 v B. Primer binding to single-strand DNA (57%]
6 C. Primer extension by DNA polymerase (7%]
7
8
D. DNA polymerase inactivation (5%]
9 X @ E. Double-strand DNA renaturation [26%]
10
11
12 Explanation: User Id: 388147
13
14 Most polymerase chain reaction (PCR) methods use thermal cycling, w hich amplifies DNA through the
15 alternate heating and cooling of a DNA sample. In the standard first step of thermal cycling PCR, heating of
16 the sample to 94-96°C denatures the DNA fragments present by disrupting the hydrogen bonds between
17 complementary bases of DNA strands. Two separate, single strands of the sample DNA are generated in the
18 denaturation process. The solution is then cooled to 50-65°C, allow ing the pre-made DNA primers (which
19 have been added in excess to the reaction mixture) to anneal to specific sequences on the single strands of
20 sample DNA. The reaction mixture is rew armed to 72°C to enable the heat-stable Taq (a name derived from
21 Thermus aquaticus) DNA polymerase to synthesize a new complementary DNA strand in the 5' to 3'
22 direction. Repetition of these steps exponentially amplifies the original tissue sample DNA.
23
24 (Choice A) Double-strand DNA denaturation occurs during the initial heating step of PCR.
25
26 (Choice C) Primer extension by DNA polymerase occurs during the rew arming step (after the primer binding
27 to single-strand DNA) of PCR .
28
29 (Choice D) Thermus aquaticus is a thermophilic bacterium that resides in hot springs and hydrothermal
30 vents. It produces an enzyme, Taq polymerase, that can tolerate the high temperatures used in PCR . Taq
31 polymerase functions best at 75-80°C, but is not inactivated by exposure to 55°C.
32
33 (Choice E) In the cooling step of PCR, primer annealing to separated DNA strands is more likely to occur
34 than is double-stranded DNA renaturation because excess DNA primers are present in the reaction mixture.

Educational objective:
37 Thermal cycling PCR involves heating for DNA strand denaturation, cooling for primer hybridization, and
38 rew arming for primer extension and DNA synthesis.
1 A

2 Item: 36 of 46 • \> Mark -<J C>-


Q. Id: 6567 (388147) Previous Next
3
4 A 21 -year-old male with a history of a recent upper respiratory infection complains of persistent fever, nasal
5 discharge and facial pain. CT scan findings are shown on the slide below.
6
7
8
9
10
11
12
13
14
15
16
17
18
19 -
20
21
22
23
24
25
26
27
28
29
30
31 Which of the following sinuses is most likely affected in this patient?
32
33
34 tl <@ A. Maxillary (85%I
35 B. Frontal [3%)
C. Ethmoid (7%)
37
D. Sphenoid [5%)
1 A

2 Item: 36 of 46 II P Mark -<:I t>-


Q. Id : 6567 (388147] Previous Next
3
,. 'I' I 111..I I VI ll ...... I VllVY•ll '::I Jll IUJl..J IJ ' I I V J l ' "' '"'' J UI I '-'l..l'-'U " ' ll llJ puu1..1 l l !
4
5
6 .; @ A. Maxillary [85%]
7 B. Frontal [3%]
8
C. Ethmoid [7%]
9
10 D. Sphenoid [5%]
11
12
13 Explanation: User Id: 388147
14
15 This coronal computed tomography (CT) image show s a section just posterior to the globes (the orbits are
16 large and the intraocular muscles are visualized w ithin them). Medial to the orbits, the ethmoid air sinuses are
17 seen arising from the nasopharynx. The curved structures inferior to the air sinuses are the nasal conchae
18 (turbinates). The maxillary air sinuses reside lateral to the inferior nasal conchae and inferior to the
19 orbits. Above w e see asymmetry between the right and left maxillary sinuses, secondary to an accumulation
20 of fluid w ithin the patient's left maxillary sinus (on the right side of the above image). The maxillary sinuses are
21 the sinuses most commonly affected during upper respiratory tract infections because their path of drainage
22 is located superior to the floor of the sinus; thus, gravity does not favor drainage of these sinuses.
23
24 (Choice B) The frontal sinuses are not visualized on the provided image. They reside w ithin the frontal bone
25 superior to the orbits and drain to the middle nasal meatus via the frontonasal duct.
26
27 (Choice C) The ethmoid air sinuses pictured here are unaffected in this patient. Specifically, the posterior
28 ethmoid air cells are imaged, and their drainage into the superior meatus is evident.
29
30 (Choice D) The sphenoid sinuses are not pictured. They are located posterior to the ethmoid air cells and
31 inferior to the sella turcica .
32
33 Educational objective:
34 The maxillary sinuses are located inferior to each orbit and are the sinuses most commonly affected during
35 upper respiratory tract infections due to their poor drainage.

37 Time Spent: 33 seconds 85% answ ered correctly. Last updated: [11 / 10/2011].
38 Copyright © USMLEWORLD,LLC. SimExam ver. 2011 .11.397425
1 •
2 Item: 37 of 46 • \ ' Mark -<:J C>
Q. Id: 6572 (383147) Previous NeHt
3
4
5
6 A 65-year-old male is brought to the emergency department w ith neurological complaints of recent onset.
7 Comprehensive evaluation suggests involvement of the artery noted with the arrow below:
8
9
10
11
12
13
14
15
16
17
18
19 '°
20
21
22
23
24 W hich of the following vision abnormalities is he most likely to have?
25
26 A. Bitemporal hemianopsia [6%]
27
28
B. Binasal hemianopsia [4%]
29 ~ @ C. Homonymous hemianopsia [58%]
30 D. Unilateral central scotoma [14%]
31
E. Monocular blindness [18%]
32
33
34 Explanation: User Id: 388147
35
36
1 A

2 Item: 37 of 46 II P Mark -<:I t>-


Q. Id : 6572 (388147] Previous Next
3
4
5 Which of the following vision abnormalities is he most likely to have?
6
7
A. Bitemporal hemianopsia [6%]
8
9 B. Binasal hemianopsia [4%]
10 .; @ C. Hom0nymous hemianopsia [58%]
11
D. Unilateral central scotoma [14%]
12
13 E. Monocular blindness [18%]
14
15
16 Explanation: User Id: 388147
17
18 The arrow above points to the middle cerebral artery. Infarction of the territory of cortex supplied by the middle
19 cerebral artery, if large, can damage the optic radiations. Unilateral visual pathway lesions distal to the optic
20 chiasm can cause unilateral homonymous hemianopsia.
21
22 (Choice A) Lesions of the midportion of the optic chiasm (as may occur with a pituitary tumor) cause
23 bitemporal hemianopsia.
24
25 (Choice B) Binasal hemianopsia is rare. It can result from lateral lesions of the optic chiasm, as has been
26 described when significant internal carotid atherosclerosis applies pressure on the chiasm bilaterally.
27
28 (Choice D) A unilateral central scotoma can result from unilateral macular disease or optic neuritis caused by
29 multiple sclerosis.
30
31 (Choice E) Monocular blindness is usually the result of damage to the globe, retina or optic nerve on one side.
32
33 Educational objective:
34 Any visual pathway lesion beyond the optic chiasm can cause homonymous hemianopsia. This may result
35 from infarction of the territories supplied by the middle, posterior cerebral or anterior choroidal arteries.
36
Time Spent: 34 seconds 58% answered correctly. Last updated: [11/10/2011].
38 Copyright © USMLEWORLD,LLC. SimExam ver. 2011 .11.397425
1 A

2 Item: 38 of 46 II P Mark -<:I t>-


Q. Id : 6573 (388147] Previous Next
3
4
5
6 A 56-year-old male is being evaluated for recent w eigll!.J2ss, an,or.exi9 and abdominal disteps,ion. You suspect
7 an abdom ipal malignancy. W hich of the follow ing lymph nodes is most likely to be involved?
8
9
10 /\. Deefl cervical [5%•]
11
12 B. Axillary [€i%•]
13
14
15
16 D. Deltef')ecteral [~%•]
17
18 E. Epitrochlear [3%]
19 -
20
21
22 Explanation: User Id: 388147
23
24 The left supraclavicular lymph node, located in the concavity between the clavicle and trapezius, is often
25 referred to as Virchow 's node. This node receives drainage from the thoracic duct, and thus samples lymph
26 from all of the abdominal viscera, the viscera of the left hemithorax, all tissues inferior to the umbilicus, and all
27 left-sided tissues superior to the umbilicus. Virchow 's node may become enlarged in the setting of abdominal
28 malignancies, breast cancer, lung cancer or lymphoproliferative disorders.
29
30 (Choice A) The deep cervical lymph nodes drain the head and neck.
31
32 (Choice B) The axillary lymph nodes are classically involved in metastatic breast cancer. They are palpable
33 in the axilla and are organized into four distinct groups along the axillary vein . Breast cancer can also
34 metastasize to the parasternal nodes.
35
36 (Choice D) The deltopectoral (infraclavicular) lymph nodes participate in the drainage of lymph from the arm .
37 They lie in the groove between the deltoid and pectoralis muscles.
1 A

2 Item: 38 of 46 II P Mark -<:I t>-


Q. Id : 6573 (388147] Previous Next
3
4 D. Deltepesteral (d%•l
5
6 E. Epitrochlear [3%]
7
8
9
10 Explanation: User Id: 388147
11
12 The left supraclavicular lymph node, located in the concavity between the clavicle and trapezius, is often
13 referred to as Virchow's node. This node receives drainage from the thoracic duct, and thus samples lymph
14 from all of the abdominal viscera, the viscera of the left hemithorax, all tissues inferior to the umbilicus, and all
15 left-sided tissues superior to the umbilicus. Virchow's node may become enlarged in the setting of abdominal
16 malignancies, breast cancer, lung cancer or lymphoproliferative disorders.
17
18 (Choice A) The deep cervical lymph nodes drain the head and neck.
19
20 (Choice B) The axillary lymph nodes are classically involved in metastatic breast cancer. They are palpable
21 in the axilla and are organized into four distinct groups along the axillary vein . Breast cancer can also
22 metastasize to the parasternal nodes.
23
24 (Choice D) The deltopectoral (infraclavicular) lymph nodes participate in the drainage of lymph from the arm.
25 They lie in the groove between the deltoid and pectoralis muscles.
26
27 (Choice E) The epitrochlear nodes lie near the elbow and drain lymph from the ulnar aspect of the forearm
28 and hand. Infection of the skin and/or soft tissue distal to the elbow is a common cause of epitrochlear node
29 enlargement.
30
31 Educational objective:
32 lntraabdominal malignancies can metastasize via lymphatic channels, w hich travel alongside the organ's
33 respective blood supply. The thoracic duct receives all lymphatic drainage from the abdominal viscera, and
34 lymph from this duct is sampled by the left supraclavicular lymph node. Enlargement of this node (Virchow's
35 node} may signify an occult abdominal malignancy.
36
37 Time Spent: 56 seconds 83% answ ered correctly. Last updated: [3/27/201 O].
Copyright © USMLEWORLD,LLC. SimExam ver. 2011 .11.397425
1 A

2 Item: 39 of 46 • \> Mark -<J C>-


Q. Id: 6577 (388147) Previous Next
3
4 A young patient with a hi.story of exertional dy.spnea and leg .swelling has his left ventricular myocardium
5 biopsied. Which of the following areas indicated on the image below represents the overlap between myosin
6 and actin filaments?
7
8
9
10
11
12
13
14
15
16
17
18
19 -
20
21
22
23
24
25
26
27
28
29
30
31
32 A. A (8%]
33
34 X @ B. 8 (17%]
35 >I C. C (61%]
36 D. 0 (14%]
37
38 E. E (0%]
--

& Connectwe Tissue


1 A

2 Item: 39 of 46 II P Mark -<:I t>-


Q. Id : 65n (388147] Previous Next
3
4 The electron micrograph (EM) of this patient's m yocardial biopsy demonstrates multiple sarcomeres in a
5 contracted state. The sarcomere can be divided into various bands and lines as pictured in the schematic
6 below . A sarcomere is defined as the distance from one Z line to the next. The Z line (Choice A) on electron
7 micrographs of muscle is typically distinctly darker than the remainder of the sarcomere. Thin filaments,
8 w hich are composed of actin, tropomyosin and troponin, anchor at the Z line. The I band (Choice B) contains
9 the Z line and only those sections of the thin (actin) filaments that do not overlap w ith thick (m yosin) filaments;
10 thus, the w idth of the I band can change depending on the contractile state of the sarcomere. It is notable that
11 transverse tubules (tubules in communication w ith the extracellular space) and terminal cisternae (extensions
12 of the sarcoplasmic reticulum) overlie the Z lines and I band. The H band (Choice D) is the inverse of the I
13 band in that it represents only the section of the sarcomere containing m yosin (thick) filaments that do not
14 overlap w ith actin (thin) filaments; it also changes in w idth depending on the contractile state of the
15 sarcomere. Both the I band and the H band w ill narrow as the sarcomere contracts. The M line lies at the
16 center of the H band; it is w here the m yosin filaments anchor in the center of the sarcomere. The region
17 w here m yosin (thick) filaments and actin (thin) filaments overlap is the segment of the sarcomere between the
18 H band {Choice D) and the I band (Choice B). Therefore, the correct answ er is {Choice C).
19 - Sarcomere
20 ZtoZ
21
22 Aband
J; J.
23 z disk H band z disk
24 J;
M line
J.
25
26
27
28
29
30 i ' i
Thick myofilament
31 Thin myofilament
32 I band
33
34 (Choice E) The structure labeled here is a mitochondrion, w hich is entirely distinct from the m yosin and actin
35 filaments and is not a part of the sarcomere. Mitochondria are plentiful in muscle. They can be distinguished
36 histologically by the presence of folded cristae in their intermembrane space.
37
38 Educational objective:
1 A

2 Item: 39 of 46 II P Mark -<:I t>-


Q. Id : 65n (388147] Previous Next
3
4
transverse tubules (tubules in communication w ith the extracellular space) and terminal cisternae (extensions
5
of the sarcoplasmic reticulum) overlie the Z lines and I band . The H band (Choice D) is the inverse of the I
6
band in that it represents only the section of the sarcomere containing m yosin (thick) filaments that do not
7
overlap w ith actin (thin) filaments; it also changes in w idth depending on the contractile state of the
8
sarcomere. Both the I band and the H band w ill narrow as the sarcomere contracts. The M line lies at the
9
center of the H band; it is w here the m yosin filaments anchor in the center of the sarcomere. The region
10
w here m yosin (thick) filaments and actin (thin) filaments overlap is the segment of the sarcomere between the
11
H band (Choice D) and the I band (Choice B). Therefore, the correct answer is (Choice C).
12
13 Sarcomere
14
ztoz
15 A band
16 H band
17
z disk
.j; :i.
z disk
18 M line -------+------
19 -
20
21
22
23
24 7 ' i
Thick myofilament
Thin myofilament
25
26 I band
27
28 (Choice E) The structure labeled here is a mitochondrion, w hich is entirely distinct from the m yosin and actin
29 filaments and is not a part of the sarcomere. Mitochondria are plentiful in muscle. They can be distinguished
30 histologically by the presence of folded cristae in their intermembrane space.
31
32 Educational objective:
33 The region of overlap of m yosin and actin filaments in the human skeletal muscle sarcomere is the segment
34 of the sarcomere between the H band and the I band. The A band is the only named band in the sarcomere
35 that contains an overlap of thick and thin filaments.
36
37 Time Spent: 29 seconds 61 % answ ered correctly. Last updated: [11 / 10/2011].
38 Copyright © USMLEWORLD,LLC. SimExam ver. 2011 .11.397425
1 A

2 Item: 40 of 46 II P Mark -<:I t>-


Q. Id : 6582 (388147] Previous Next
3
4
A mildl~ overweight, school-aged boy is brought to clinic by his mother. He has fallen asleep in class several
5
times. He goes to sleep at approximately 3:00 am and has difficulties w aking in the morning to go to
6
school. W hen he comes home his thinking is "fuzZ)'." and he does not w ant to do anything. On the
7
w eekends, he feels "normal" and l)la~s w ith his 11eers. W hich of the follow ing is the most likely cause of this
8
patient's complaints?
9
10
11 A. l>Jartelepsy [<l%•]
12 !l. Familial insemnia [2q~l
13
~ C. Circadian rhythm sleep disorder [38%]
14
15 X @ D. Obstructive sleep apnea [7%]
16 E. Separatien am~iety diserder [12q~l
17
F. Stheel pheeia [4Qq~l
18
19
20 Explanation: User Id: 388147
21
22 This child presents w ith an abnormal sleep-w ake cycle relative to his environmental light-dark cycle. If w illful
23 non-compliance w ith a normal sleep schedule, caffeine use late in the day and an underlying psychiatric
24 illness (i.e. depression) can be ruled out, then circadian rhythm sleep disorder is the most likely
25 diagnosis. The suprachiasmatic nucleus of this child's hypothalamus, w hich functions as the body's "internal
26 clock," has established the time of sleep onset to be approximately 3:00 AM. W hen social demands, such as
27 school, require w aking before the end of a normal sleep cycle, this child experiences excessive daytime
28 sleepiness. On w eekends, w hen he is presumably able to sleep until a later hour in the day, his alertness and
29 function after w aking are normal.
30
31 (Choice A) Classified as a primary hypersomnia, narcolepsy is a sleep disturbance characterized by daytime
32 REM sleep attacks and cataplexy (a sudden loss of antigravity muscle tone) in approximately 60% of affected
33 individuals. Auditory or visual hallucinations of a hypnagogic (upon falling asleep, more common) or
34 hypnopompic (upon w aking, less common) nature may occur in some cases. Sleep paralysis also occurs as
35 a component of this condition .
36
37 (Choice B) Although genetics may play a role in circadian rhythm sleep disorder, it is not classically
38 considered to be a familial disorder. There is. how ever. a condition know n as fatal familial insomnia. w hich is
1 A

2 Item: 40 of 46 II P Mark -<:I t>-


Q. Id : 6582 (388147] Previous Next
3
4
clock," has established the time of sleep onset to be approximately 3:00 AM. W hen social demands, such as
5 school, require w aking before the end of a normal sleep cycle, this child experiences excessive daytime
6 sleepiness. On w eekends, w hen he is presumably able to sleep until a later hour in the day, his alertness and
7 function after w aking are normal.
8
9 (Choice A) Classified as a primary hypersomnia, narcolepsy is a sleep disturbance characterized by daytime
10 REM sleep attacks and cataplexy (a sudden loss of antigravity muscle tone) in approximately 60% of affected
11 individuals. Auditory or visual hallucinations of a hypnagogic (upon falling asleep, more common) or
12 hypnopompic (upon w aking, less common) nature may occur in some cases. Sleep paralysis also occurs as
13 a component of this condition .
14
15 (Choice B) Although genetics may play a role in circadian rhythm sleep disorder, it is not classically
16 considered to be a familial disorder. There is, how ever, a condition know n as fatal familial insomnia, w hich is
17
an autosomal dominant "prion disease" that causes severe, intractable insomnia and death, as the name
18 implies.
19
20 (Choice D) Obstructive sleep apnea (OSA) is an obesity-associated sleep disorder characterized by periodic
21 upper airway obstruction during sleep. Clinical manifestations of OSA include loud snoring and w itnessed
22 apneic episodes during sleep. OSA causes daytime sleepiness but is actually a form of hypersomnia (rather
23 than insomnia) that does not involve an abnormal sleep-w ake cycle.
24
25 (Choice E) Separation anxiety disorder is not associated w ith insomnia or a disordered circadian
26 rhythm. Affected children may be reluctant to go to sleep unless they are at home or w ith a primary
27 attachment figure and they may have repeated nightmares about separation from loved ones, but neither of
28 these features need to be present to make the diagnosis.
29
30 (Choice F) A school phobia refers to a specific phobia of attending school and is unrelated to the circadian
31 sleep rhythm. The diagnostic criteria include those of any other specific phobia .
32
33 Educational objective:
34 Delayed sleep phase syndrome is a form of intrinsic circadian rhythm sleep disorder that is characterized by
35 a consistent delay in the time of sleep onset relative to the daily light-dark cycle. It results in excessive
36 daytime sleepiness during the school w eek in an otherwise healthy child or adolescent due to an inability to
37 attain the needed amount of sleep w hen social demands require premature w aking.
38
1 •
2 Item: 41 of 46 • \' Mark -<:J C>
Q. Id: 6583 (383147) Previous NeHt
3
4
A researcher conducts experiments on isolated skeletal muscles. Decreasing w hich of the following would
5
increase contraction velocity?
6
7
8 A. Preload [15%]
9 >1 @ B. After1oad (44%]
10
C. AGtiA aAEI myesiA evei:lap (14%1
11
12 D. e11ti:as ell1o1lar ealei1o1m (1 :!~4.1
13 E. Passive tension (13%)
14
15
16 Explanat ion: User Id: 388147
17
18 With respect to skeletal muscle, the after1oad refers to the force against which the muscle fiber must
19 contract. This is analogous to the afterload caused by diastolic pressure in cardiac ventricular contraction.
20 When a skeletal muscle fiber contracts, its maximum velocity of shortening is indirectly proportionate to the
21 afterload attached to the fiber, as illustrated in the graph below.
22
23
24
25
26
27
28
29
30
31
32
33
34
35
36 After1oad
37
38 Maximal contractile shortening velocity increases as the after1oad decreases.
39
1 A

2 Item: 41 of 46 II P Mark -<:I t>-


Q. Id : 6583 (388147] Previous Next
3
4
5
6
7
8
9
10
11
12
13 Afterload
14
15 Maximal contractile shortening velocity increases as the afterload decreases. The greatest velocity of
16 shortening occurs w hen there is no afterload.
17
18 (Choices A & C) Preload refers to the initial muscle fiber length prior to contraction . In skeletal muscle, the
19 preload refers to the amount of stretch placed on the fiber before contraction occurs. This is analogous to the
20 stretch produced during diastolic filling of the ventricles in the heart. Increasing preload increases contractile
21 velocity up to a maximum value. At preload levels beyond this maximum, actin/myosin overlap becomes
22 inefficient (decreased) and contractile velocity w ill decrease.
23
24
(Choice D) In skeletal muscle, decreasing extracellular [Ca++] tends to reduce contractility because
25
intracellular [Ca++] is low ered during contraction . Maximal contraction velocity w ould therefore decline if
26
27 extracellular [Ca++] w ere decreased.
28
29 (Choice E) The passive tension exerted by a skeletal myocyte is positively correlated w ith its resting length.
30 A decrease in passive tension generally implies a decrease in the fiber length and a decrease in maximal
31 contraction velocity.
32
33 Educational objective:
34 At constant contractility, the maximal velocity of skeletal muscle contraction increases as afterload
35 decreases. The highest contraction velocity is achieved w hen the afterload is zero.
36
37 Time Spent: 55 seconds 45% answ ered correctly. Last updated: [1 1/ 10/2011].
38 Copyright © USMLEWORLD,LLC. SimExam ver. 2011 .11.397425
1 A

2 Item: 42 of 46 II P Mark -<:I t>-


Q. Id : 6592 (388147] Previous Next
3
4
5 An in,testinaLesRirate from a young male w ho recently returned from a trip to Russia has the follow ing
6 microscopic findings:
7
8
9
10
11
12
13
14
15
16
Displayed t•tith pennission from Springe.- Heal1hc:are Ltd.© Copyrigbt 1997 by Current Medicine
17
18
19 The patient is most likely to experience w hich of the follow ing?
20
21 X @ A. Bloody diarrhea [30%]
22
23 B. Malabsorption [62%]
24 C. Liver abscess [7%]
25 D. Ascites [1 %]
26
27 E. Symmetric arthritis [0%]
28
29
Explanation: User Id: 388147
30
31
Giardia /amblia is a protozoan that exists in both cyst and trophozoite forms. The cyst form is the infectious
32
form, w hich can reside in sew age as w ell as in freshwater streams and lakes contaminated by animal or
33
human feces. After being ingested by the host animal, the cyst matures into a binucleate flagellated
34
trophozoite, as pictured above. The trophozoite is sometimes described as having a kite-like or ow l's eyes
35
appearance. The Giardia trophozoite attaches to the small bow el mucosa w ith a ventral sucking disk but
36
does not damage or invade the mucosa. The majority of infections are asymptomatic or minimally
37
symptomatic, though some patients may experience significant greasy, frothy, foul-smelling, nonbloody
38 Tho. C::\Jmntnmc:: nf ni::arrli::ac::ic:: ::ariQ. tho ro.c::11lt nf m::al::iihc::nrntinn
39
40
1 A

2 Item: 42 of 46 II P Mark -<:I t>-


Q. Id : 6592 (388147] Previous Next
3
4
5 The patient is most likely to experience w hich of the follow ing?
6
7
8
x @ A. Bloody diarrhea [30%]
9 B. Malabsorption [62%]
10 C. Liver abscess [7%]
11
D. Ascites [1%]
12
13 E. Symmetric arthritis [0%]
14
15
16 Explanation: User Id: 388147
17
18 Giardia /amblia is a protozoan that exists in both cyst and trophozoite forms. The cyst form is the infectious
19 form, w hich can reside in sew age as w ell as in freshwater streams and lakes contaminated by animal or
20 human feces. After being ingested by the host animal, the cyst matures into a binucleate flagellated
21 trophozoite, as pictured above. The trophozoite is sometimes described as having a kite-like or ow l's eyes
22 appearance. The Giardia trophozoite attaches to the small bow el mucosa w ith a ventral sucking disk but
23 does not damage or invade the mucosa. The majority of infections are asymptomatic or minimally
24 symptomatic, though some patients may experience significant greasy, frothy, foul-smelling, nonbloody
25 diarrhea accompanied by excessive flatulence. The symptoms of giardiasis are the result of malabsorption,
26 most likely secondary to dysfunction of the brush border enzymes. Patients classically have a history of
27 recent camping or travel abroad.
28
29 (Choices A & C) Though most Entamoeba histolytica infections are asymptomatic, there can be bloody
30 diarrhea and liver abscess formation. On microscopy, amoebae w ith ingested host red blood cells in the
31 cytoplasm (the trophozoite form of this organism) may be visualized. The cyst form has four nuclei.
32
33 Educational objective:
34 Giardia /amblia is a protozoan that causes a malabsorptive diarrhea follow ing consumption of contaminated
35 w ater. It does not cause invasive disease.
36
37 Time Spent: 28 seconds 62% answ ered correctly. Last updated: [1 0/3/2011].
38 Copyright © USMLEWORLD,LLC. SimExam ver. 2011 .11.397425
1 A

2 Item: 43 of 46 II P Mark -<:I t>-


Q. Id : 7222 (388147] Previous Next
3
4
A 32-year-old male w ith no significant past medical history com~ains_ot..tatigue. He says that his exercise
5 tolerance. has decreased dramatically over the last three w eeks. Physical examination reveals prominent
6
co[liunctival 11allor. W hite patches are present on the gingival and buccal mucosa. Fundoscopic examination
7
demonstrates hemo!Ihag~d occasional exudates bilateral!)' lfhere is no palpable lymphadenopathy or
8
organomegaly.
9
10 Item 1 of 2
11 W hich of the follow ing w ould be the most likely diagnostic finding on this patient's peripheral blood smear?
12
13
14
x @ A. Basophilic stippling [2 1% ]
15 .; B. Blast cells [37%]
16 C. Red cells w ithout central pallor [9%]
17
D. Atypical lymphocytes [27%]
18
19 E. Prominent eosinophilia [6%]
20
21
22 Explanation: User Id: 388147
23
24 This patient most likely has acute leukemia. Affected individuals typically present w ith symptoms of
25 pancytopenia . Thus, it is important to know that anemia causes fatigue, leukopenia is associated w ith
26 opportunistic infections (candidiasis, aspergillosis, bacterial pneumonia, perirectal infections, etc.), and
27 thrombocytopenia is associated w ith bleeding complications (gingival bleeding, retinal hemorrhages, etc.).
28 The w hite patches described in this patient's oral mucosa are most likely the result of oral thrush
29 (candidiasis). Hepatosplenomegaly and lymphadenopathy may or may not be found in these patients. Blast
30 cells, w hich are observed in over 90% of cases of acute leukemia, are likely to be found on this patient's
31 peripheral blood smear.
32
(Choice A) Basophilic stippling refers toblue granules (ribosomal precipitants) spread throughout the
33
cytoplasm of red blood cells. Basophilic stippling is often seen in the thalassemias, alcohol abuse, and lead
34
poisoning.
35
36 (Choice C) Patients w ith spherocytosis have red cells w ithout central pallor on peripheral blood smear.
37 Spherocytosis is associated w ith hemolytic anemia, but opportunistic infections and bleeding complications
38 <'! rP. not tvnir.<'!llv ~P.P.n
39
40
1 A

2 Item: 43 of 46 II P Mark -<:I t>-


Q. Id : 7222 (388147] Previous Next
3
...... . I ........ " '-'"'-' •O • ~I , ...,._. .. '"''-''OU._., .............. , l'-' ' '-'J
4
5 D. Atypical lymphocytes [27%]
6 E. Prominent eosinophilia [6%]
7
8
9 Explanation: User Id: 388147
10
11 This patient most likely has acute leukemia. Affected individuals typically present w ith symptoms of
12 pancytopenia . Thus, it is important to know that anemia causes fatigue, leukopenia is associated w ith
13 opportunistic infections (candidiasis, aspergillosis, bacterial pneumonia, perirectal infections, etc.), and
14 thrombocytopenia is associated w ith bleeding complications (gingival bleeding, retinal hemorrhages, etc.).
15 The w hite patches described in this patient's oral mucosa are most likely the result of oral thrush
16 (candidiasis). Hepatosplenomegaly and lymphadenopathy may or may not be found in these patients. Blast
17 cells, w hich are observed in over 90% of cases of acute leukemia, are likely to be found on this patient's
18 peripheral blood smear.
19
20 (Choice A) Basophilic stippling refers toblue granules (ribosomal precipitants) spread throughout the
21 cytoplasm of red blood cells. Basophilic stippling is often seen in the thalassemias, alcohol abuse, and lead
22 poisoning .
23
(Choice C) Patients w ith spherocytosis have red cells w ithout central pallor on peripheral blood smear.
24
Spherocytosis is associated w ith hemolytic anemia, but opportunistic infections and bleeding complications
25
are not typically seen .
26
27
(Choice D) Atypical lymphocytes (reactive lymphocytes) are commonly observed in viral infections but can be
28
seen in leukemias. How ever, they are not diagnostic of leukemia.
29
30 (Choice E) Prominent eosinophilia can be observed in a number of clinical settings including infection w ith a
31 parasite, atopic diseases, drug reactions and certain syndromes.
32
33 Educational objective:
34 Clinical features of acute leukemia are usually the result of pancytopenia . Blast cells are commonly observed
35 on peripheral blood smear.
36
37 Time Spent: 77 seconds 37% answ ered correctly. Last updated: [1/8/2012].
38 Coovriaht © USMLEWORLD .LLC. SimExam ver. 2011.11 .397425
39
40
9 The patient is diagnosed with a hematological malignancy and is started on chemotherapy. Four weeks later,
10 he complains of c,ough fever and chest discomfort. Chest x-ray demonstrates a patch~ infiltrate in the right
11 ui:>per lung lobe with a small area of cavitation. Bronchoalveolar fluid obtained during bronchoscopy shows
12 the following findings (see the slide below):
13
14
15
16
17
18
19
20
21
22
23
24
25
26
27
28
29
-
30
31
32
33 Displayed with peomissioo fran Springor Healtbcaie Ltd.
34 c eq,y,iglll 2005 by Q 11re111 Medicine
35
36 Which of the following is the virulence factor for the organism responsible for this patient's current condition?
37
38
39 X @ A. Large polysaccharide capsule (17%]
40 B . lipid-rich cell wall (35%]
41 C. lntracytoplasmic location (1 Oo/o]
42
D . Production of alveolar exudate [6%]
--

horetJcular Systems
6
7 Item: 44 of 46 II P Mark -<:I t>-
8 Q. Id : 7223 (388147] Previous Next
9
10 X @ A. Large polysaccharide capsule (17%]
11 B. Lipid-rich cell w all (35%]
12
C. lntracytoplasmic location (10%]
13
14 D. Production of alveolar exudate (6%]
15 E. Vascular invasion [32%]
16
17
18 Explanation: User Id: 388147
19
20 The patient described is suffering from invasive aspergillosis. Aspergiflus fumigatus can be recognized
21 histologically by the presence of septate hyphae w ith 45 degree branching. lmmunocompromised patients,
22 such as the patient described in the question stem, are at risk for developing a severe necrotizing pneumonia
23 due to aspergillus, w hich typically forms a "fungus ball" w ithin lung cavities from prior infections such as
24 resolved tuberculosis. These infections can disseminate to distant organs by hematogenous spread. Thus,
25 vascular invasion is a prominent feature of aspergillus infection that facilitates disease progression throughout
26 the body.
27
28 (Choice A) The polysaccharide capsule is the virulence factor of Streptococcus pneumoniae, Haemophifus
29 influenzae and Neisseria meningitidis.
30
(Choice B) A lipid-rich cell w all is the virulence factor of m ycobacterial organisms.
31
32 (Choice C) Many organisms avoid the immune system by residing in an intracytoplasmic location. Examples
33
include Mycoplasma pneumoniae, Penicil/ium marneffei, Neisseria gonorrhoeae and many others.
34
35 (Choice D) Production of an alveolar exudate can result from numerous bacterial pneumonias.
36
37 Educational objective:
38 Aspergillosis is an opportunistic infection commonly seen in patients w ith immunosuppression due to acute
39 leukemia. The organism exhibits septate hyphae w ith 45 degree branching and tends to form "fungus balls" in
40 the upper lobes at sites of prior tuberculosis activity. Vascular invasion is a prominent feature of aspergillus
41 infection that facilitates disease progression throughout the body.
42
Time Spent: 107 seconds 32% answered correctly. Last updated: (1 211/2011].
6
7 Item: 45 of 46 II P Mark -<:I t>-
8 Q. Id: 7214 (388147] Previous Next
9 A 54-year-old male comes to the emergency department after an episode of bloody vomiting. He denies
10 fever, chills, abdominal pain, diarrhea, or constipation. He has a history of alcohol abuse w ith multiple
11 previous hospital admissions due to alcohol w ithdrawal and generalized tonic-clonic seizures. He is also an
12 ex-intravenous drug abuser and is currently in a methadone program. On examination, he appears
13 comfortable but is quite dizzy w hen asked to sit up. His blood pressure is 96/62 mmHg and heart rate is
14 102/min. There is no jugular venous distention. The abdomen is distended with dullness to i;iercussion at
15 both flanks. The liver is enlarged on palpation, and the tip of the spleen is pali;iable. There is trace i;iedal
16
edema. Laboratory studies show:
17
18
Hemoglobin 9.7 g/dL
MCV 98fl
19
20 Leukocyte count 5,000/mm 3
21 Platelet count 78,000/mm 3
22
23 Item 1 of 2
24 A pathologic examination of this patient's spleen is most likely to show w hich of the following?
25
26 x @ A. Lymphoid follicle expansion [8%]
27
28 B. Extramedullary hematopoiesis [30%]
29 C. Red pulp expansion [50%]
30 D. Histiocytic proliferation [8%]
31
32 E. Metastatic deposits [4% ]
33
34
Explanation: User Id: 388147
35
36 This patient's splenomegaly is due to portal hypertension that has resulted from his alcoholic liver disease.
37
Because the splenic vein is part of the portal circulation, any condition resulting in portal hypertension w ill also
38 most likely result in congestive hypersplenism. This congestion of blood w ill result in an apparent expansion
39 of the red pulp of the spleen, w hich is composed of blood-filled sinuses and cords lined by
40
reticuloendothelial-type cells.
41
42 (Choice A) Lymphoid follicle expansion can cause hypersplenism either in the setting of systemic infections
such as infectious mononucleosis or as a result of immunologic diseases such as systemic lupus
6
7 Item: 45 of 46 II P Mark -<:I t>-
8 Q. Id: 7214 (388147] Previous Next
9
10 X @ A. Lymphoid follicle expansion [8%]
11
12 B. Extramedullary hematopoiesis [30%]
13 C. Red pulp expansion [50%]
14 D. Histiocytic proliferation [8%]
15
16 E. Metastatic deposits [4%]
17
18
Explanation: User Id: 388147
19
20
This patient's splenomegaly is due to portal hypertension that has resulted from his alcoholic liver disease.
21
Because the splenic vein is part of the portal circulation, any condition resulting in portal hypertension w ill also
22
most likely result in congestive hypersplenism. This congestion of blood w ill result in an apparent expansion
23
of the red pulp of the spleen, w hich is composed of blood-filled sinuses and cords lined by
24
reticuloendothelial-type cells.
25
26 (Choice A) Lymphoid follicle expansion can cause hypersplenism either in the setting of systemic infections
27 such as infectious mononucleosis or as a result of immunologic diseases such as systemic lupus
28 erythematosus or rheumatoid arthritis (Felty syndrome).
29
30 (Choice B) Extramedullary hematopoiesis results w hen the hematopoietic precursor cells are displaced from
31 the bone marrow and infiltrate the liver and spleen preferentially, resulting in splenomegaly. A common
32 condition in w hich this occurs is myelofibrosis.
33
34 (Choices D and E) Histiocytic proliferation can cause splenomegaly in the setting of histiocytosis X, and any
35 metastatic tumor can infiltrate the spleen and lead to hypersplenism. This patient's splenomegaly, how ever,
36 is clearly due to portal hypertension.
37
38 Educational objective:
39 Portal hypertension produces splenomegaly by causing congestion of blood w ithin the spleen, w hich
40 effectively produces an expansion of the red pulp.
41
42 Time Spent: 89 seconds 50% answ ered correctly. Last updated: [1 /8/2012].
Copyright © USMLEW ORLD,LLC. SimEx am ver. 2011 . 11.397425
6
7 Item: 46 of 46 II P Mark -<:I t>-
8 Q. Id: 7215 (388147] Previous Next
9 After initial stabilization w ith intravenous fluids and blood products, the patient develops another bout of bloody
10 emesis. A upper gastrointestinal endoscopy show s bleeding esophageal varices. W hich of the following
11 peptides is most likely to immediately decrease the portal venous pressure in this patient?
12
13
14 A. Pentagastrin [8%]
15
B. Somatostatin [74%]
16
17 C. Glucagon [7%]
18
19 D. Secretin [3%]
20
21 x @ E . Cholecystokinin [8%]
22
23
24 Explanation: User Id: 388147
25
26 The current medical treatment of choice for esophageal variceal hemorrhage is octreotide, w hich is the
27 synthetic version of the natural hormone somatostatin. Octreotide is able to reduce splanchnic blood flow and
28 divert blood from the portal system to the systemic circulation by indirectly inducing splanchnic vascular
29 constriction. This process is accomplished by the inhibition of the release of hormones that produce
30 splanchnic vasodilation such as glucagon and vasoactive intestinal peptide (VIP). An additional benefit of
31 octreotide over other medical therapies that have been used in the past for treating acute variceal hemorrhage
32 is that it does not cause systemic vasoconstriction .
33
34 (Choice A) Pentagastrin is a synthetic gastrin analog . Interestingly, this agent can be used to screen for
35 conditions such as carcinoid syndrome (pentagastrin-stimulated serotonin test) and medullary carcinoma of
36 the thyroid (pentagastrin-stimulated calcitonin test).
37
(Choice C) Glucagon would cause splanchnic vasodilation and worsen variceal bleeding. Glucagon
38
secretion is impaired by octreotide.
39
40 (Choice D) The primary function of secretin is to alkalinize the duodenal contents. This hormone increases
41 glucagon secretion and would therefore be counterproductive in this setting.
42
43 (Choice E) Cholecystokinin induces contraction of the gallbladder and expulsion of pancreatic exocrine
44
6
7 Item: 46 of 46 II P Mark -<:I t>-
Q. Id: 7215 (388147] Previous Next
8
9
., o . ..::iu111C1lU:>lC1u11 L1 <t 7 0 J

10
C. Glucagon [7%]
11
12 D. Secretin [3%]
13
14 x @ E. Cholecystokinin [8%]
15
16
17 Explanation: User Id: 388147
18
19 The current medical treatment of choice for esophageal variceal hemorrhage is octreotide, w hich is the
20 synthetic version of the natural hormone somatostatin. Octreotide is able to reduce splanchnic blood flow and
21 divert blood from the portal system to the systemic circulation by indirectly inducing splanchnic vascular
22 constriction. This process is accomplished by the inhibition of the release of hormones that produce
23 splanchnic vasodilation such as glucagon and vasoactive intestinal peptide (VIP). An additional benefit of
24 octreotide over other medical therapies that have been used in the past for treating acute variceal hemorrhage
25 is that it does not cause systemic vasoconstriction .
26
27 (Choice A) Pentagastrin is a synthetic gastrin analog . Interestingly, this agent can be used to screen for
28 conditions such as carcinoid syndrome (pentagastrin-stimulated serotonin test) and medullary carcinoma of
29 the thyroid (pentagastrin-stimulated calcitonin test).
30
31 (Choice C) Glucagon would cause splanchnic vasodilation and worsen variceal bleeding. Glucagon
32 secretion is impaired by octreotide.
33
(Choice D) The primary function of secretin is to alkalinize the duodenal contents. This hormone increases
34
35 glucagon secretion and would therefore be counterproductive in this setting.
36
(Choice E) Cholecystokinin induces contraction of the gallbladder and expulsion of pancreatic exocrine
37
secretions into the duodenum. This hormone also increases glucagon secretion and would therefore be
38
counterproductive in this setting.
39
40 Educational objective:
41 Somatostatin (octreotide) is the medical treatment of choice for acute variceal bleeding because it reduces
42 splanchnic blood flow by inhibiting the hormones responsible for splanchnic vasodilation.
43
44

You might also like